You are on page 1of 200

Exam Section 1: Item 1 of 50 National Board of Medical Examiners®

Comprehensive Basic Science Self-Assess ment

Y 1. A 12-year-old boy is admitted to the hosr:1ital because of lethargy, hip pain, and a temperature of 39.4 °C (103° F). He has been hospitalized several other times because of pneumonia. His neonatal period! was normal. Complete blood counts are within normal
limits, and a test for HIV antibody is negative. Blood cultures grow Staphylococcus aureus. Serum electrophoresis is most likely to show which of the following patterns?

AMA
A v

b
B

A)
8
C)

Correct Answer: D.

The electrophoresis in Choice D demonstrates a deficiency in immune globulins. This young patient witln S. aureus bacteremia and a history of multiple prior infections has a clinical syndrome consistent with hypogammaglobulinemia, an immunodeficiency reflected
by absence of the gamma fraction on protein electrophoresis. Protein electrophoresis separates the major proteins found in the serum by size and polarity. Larger proteins migrate slowly, and proteins with negative charge (eg, albumin) migrate toward the positive
pole just as those with neutral or positive charge (eg, weakly negative, neutral, or positive immune globulins) migrate toward the negative pole. The large peak on the left of each of these diagrams is representative of albumin. The alpha fraction is next and is
comprised primarily of a1-antitrypsin, thyroid-binding globulin, and transcortin. The beta fraction contains transferrin and 13-lipoproteins in addition to complement. The gamma fraction contains the immunoglobulins, including lgM, lgG, lgD, and lgE. A reduced
number of circulating immunoglobulins as is seen in hypogammaglobulinemia, results in a marked reduction or absence of this spike.

Incorrect Answers: A. B, C, and E.

Choice A shows a narrow monoclonal spike in the gamma region. In the appropriate clinical context, this is suggestive of multiple myeloma, although other syndromes such as Waldenstrom macroglobulinemia, smoldering myeloma, monoclonal gammopathy of
undetermined significance, and amyloidosis can demonstrate spikes in this region.

Choice B shows a blurring of the beta-2 and gamma regions, sometimes referred to as beta/gamma fusion. This is accounted for either by increased C-reactive protein or by increased lgA, which can be seen in patients with severe respiratory tract infections,
rheumatoid arthritis, or cirrhosis.

Choice C shows a large but wide spike in the gamma region consistent with hypergammaglobulinemia. In this instance, there is no monoclonal spike but rather a wide array of immunoglobulins present. Conditions that present with nonspecific
hypergammaglobulinemia include chronic liver disease, connective tissue disease, and inflammatory conditions.

Choice E shows a nonnal protein electrophoresis.

Educational Objective: Serum electrophoresis separates proteins found in the blood based upon electric charge and size. Abnonnalities in the electrophoresis pattern can be indicative of certain disease states. In this question, the absence of the gamma region
spike indicates an immunodeficiency with hypogammaglobulinemia or agammaglobulinemia.
Exam Section 1: Item 2 of 50 National Board pf Mqdlcal Ex;1.mlners®
Comprehensive 1Sas1c science Serr-Assessment

✓ 2. Free purine and pyrimidine bases are reutilized in normal metabolism. In children with Lesch-Nyhan syndrome who have intellectual disability, poor muscle coordination, and self-mutilation tendencies, there is a defect in the salvage of which of the following
pairs of bases?

At-Adenine and thymine


B) Guanine and hypoxanthine
G)-GYanine andYris asid
Ct-Urasil and sytosine
e)-Xanthine aAd hypoxanlhine

Correct Answer: B.

Lesch-Nyhan syndrome presents with intellectual disability, aggressive behavior, self-mutilation, gout, and dystonia. The disorder is due to inactivating mutations of hypoxanthine-guanine phosphoribosyltransferase (HGPRT), a key enzyme in the purine salvage
pathway, and is inherited in an X-linked recessive fashion. HGPRT catalyzes the conversion of guanine to guanosine monophosphate and hypoxanthine to inosine monophosphate. Patients with deficient activity of HGPRT are unable to salvage guanine and
hypoxanthine and develop resultant increased levels of xanthine and uric acid. Hyperuricemia in Lesch-Nyhan syndrome is treated with xanthine oxidase inhibitors, such as allopurinol or febuxostat, in order to reduce the synthesis of uric acid.

Incorrect Answers: A, C, D and E.

Adenine and thymine (Choice A) are purine and pyrimidine bases, respectively. Purine and pyrimidine salvage are handled through two distinct pathways that are not commonly involved in a single disease process.

Guanine and uric acid (Choice C) accumulation may occur as part of Lesch-Nyhan syndrome, however, the accumulation of uric acid is also secondary to accumulation of hypoxanthine. Choice B more accurately describes defective salvage of guanine and
hypoxanthine as the fundamental effect of HGPRT dysfunction. The accumulation of uric acid is secondary.

Uracil and cytosine (Choice D) are pyrimidine nucleotides. Pyrimidine salvage is not affected by mutations of HGPRT.

Defects of xanthine and hypoxanthine (Choice E) metabolism may result from defects in HGPRT. However, HGPRT dysfunction results in impaired hypoxanthine salvage with resultant excessive production of xanthine, rather than impaired xanthine salvage.

Educational Objective: Lesch-Nyhan syndrome presents with intellectual disability, aggressive behavior, self-mutilation, gout, and dystonia. Mutations in HGPRT cause this disorder by impairing salvage of the purines guanine and hypoxanthine, which leads to
increased levels of xanthine and uric acid. Hyperuricemia in Lesch-Nyhan syndrome is treated with xanthine oxidase inhibitors, such as allopurinol or febuxostat. in order to reduce synthesis of uric acid.

r
Previous
,.
Next
,
Score Report
rr5
Lab Values
F
Calculator
r
Hel
"'
Pause
Exam Section 1: Item 3 of 50 National Board pf Medical Exii.mlners®
Comprehensive Basic Science Self-Assessment

✓ 3. A 42-year-old man is struck by a motor vehicle. His only injury is a closed fracture of the proximal tibia. Initial neurovascular examination shows no deficits. Twenty-four hours later, he has increased leg pain and paresthesias in the dorsal space between his
first and second toes. The patient begins to pass dark red urine and becomes oliguric. Urinalysis is positive for blood but no erythrocytes are seen on microscopic examination. Which of the following acute disorders is the most likely cause of the renal
failure?

A ) Glomerulonephritis
S}. Hemolytic 1Jremic syndrome
C) Interstitial nephritis
D) Nephrotic syndrome
E ) Tubular necrosis

Correct Answer: E.

Tibial fractures present a high risk for compartment syndrome. The fracture results in blood vessel injury and muscle injury, inflammation, and edema. Because the fascia containing the anterior compartment of the leg does not stretch, bleeding and swelling can
cause increased pressure in the compartment. This increased pressure in turn inhibits venous drainage, further increasing pressure in the compartment. Eventually the nerve supply and associated arteries are compromised, leading to the classic signs and
symptoms of compartment syndrome. Signs and symptoms of compartment syndrome include pain out of proportion to examination findings, pain with passive movement of the muscles, paresthesia, pallor, pulselessness, and paralysis. Compromised blood supply
deprives muscle and tissue of oxygen and glucose, leading to tissue ischemia and necrosis. Muscle necrosis leads to rhabdomyolysis, myoglobinuria, and acute renal failure. Evaluation of rhabdomyolysis reveals red or brown urine and urinalysis is typically positive
for blood due to the presence of myoglobinuria without microscopic evidence of red blood cells. A complication of rhabdomyolysis is acute kidney injury from acute tubular necrosis secondary to the release of nephrotoxic myoglobin and nonprotein heme pigments.
Acute tubular necrosis typically occurs following an ischemic or nephrotoxic insult to the kidneys, which results in loss of the tubular epithelium. Granular, muddy brown casts are common on urinalysis. Compartment syndrome is treated by immediate fasciotomy to
decrease compartment pressure and support tissue perfusion.

Incorrect Answers: A, B, C, and D.

Glomerulonephrilis (Choice A) refers to a variety of glomerular diseases, including nephritic and nephrotic syndromes. Nephritic syndromes typically present with acute renal failure associated with hematuria, red blood cell urine casts, and hypertension. Nephrotic
syndrome typically presents with excessive proteinuria (greater than 3 g/day), hyperlipidemia, hypoalbuminemia, and edema.

Hemolytic-uremic syndrome (Choice B) is classically associated with infection from Escherichia coli 0157:H7, a Gram-negative rod. It often presents in children with bloody diarrhea, microangiopathic hemolytic anemia, thrombocytopenia, and renal failure. It is not
associated with trauma or compartment syndrome.

Interstitial nephritis (Choice C) is usually caused by a hypersensitivity reaction to offending drugs (eg, nonsteroidal anti-inflammatory medications, diuretics, and sulfonamides) and is characterized by a rash, acute kidney injury, and eosino;philuria.

Nephrotic syndrome (Choice D) is characterized by proteinuria (greater than 3 g/day), edema, hypoalbuminemia, and hyperlipidemia. Complications include hypercoagulability due to urinary loss of antithrombin-3 and infection due to urinary loss of gamma
globulins.

Educational Objective: Tibial fractures can be complicated by compartment syndrome due to increased compartment pressure in the anterior leg. Compartment syndrome leading to muscle necrosis and rhabdomyolysis can be complicated by acute tubular necrosis
due to the nephrotoxicity of myoglobin and its byproducts.

r
PrevloU&
r
Next
,-
Score Report
fc5
Lab Values
F
Calculator
r
Help
If'
Pause
Exam Section 1: Item 4 of 50 National Board pf Medical Examiners®
Comprehensive Basic Science Serr-Assessment

✓ 4. A male newborn is found to have a defect in anchoring fibrils. Which of the following skin findings is most likely in this patient?

A ) Blisters
B ) Easy bruising
C) Eczematous rash
D) Inability to sweat
E ) Thickened skin
F ) Ulcers
Correct Answer: A.

The hemidesmosome is an intricate complex of proteins whose ultimate function is to anchor the basal keratinocytes of the epidermis to the dermis at the dermal-epidermal junction. Any impairment of the hemidesmosome will cause the basal keratinocytes to
separate from the dermis, causing a blister to form. Because the hemidesmosomes of neighboring skin are still intact, these will be tense blisters. Anchoring fibrils are made of type VII collagen and are a component of the hemidesmosome. A mutation or antibody tc
collagen type VII, as seen in epidermolysis bullosa, will cause blistering to occur. Bullous pemphigoid is another disease which affects the hemidesmosome. In contrast, pemphigus vulgaris is caused by antibodies to desmosomes, the protein complex which
maintains cell to cell adhesion in the epidermis. Because the target of pemphigus vulgaris is more superficial, in the epidermis, those blisters will be fragile and flaccid.

Incorrect Answers: B, C, D, E, and F.

Easy bruising (Choice B) and petechiae may be seen in Vitamin C deficiency, or scurvy. Vitamin C is necessary to produce collagen, as it is a cofactor in the hydroxylation of proline and lysine residues, which is a key step in the conversion of preprocollagen to
procollagen.

Eczematous rashes (Choice C) may be seen in many genetic disorders including atopic dermatitis, autosomal dominant hyper-lgE syndrome (Job syndrome), and Wiskott-Aldrich syndrome. While acute eczematous reactions can have small vesicles due to edema
within the epidermis, blisters are not seen. Itchy, erythematous patches are a classic finding.

Inability to sweat (Choice D) is seen in a group of inherited disorders called ectodermal dysplasias. In these disorders, ectoderm-derived structures including the sweat glands, hair, teeth, and nails are abnormal. The anchoring fibrils are intact, and blisters are not
typically seen.

Thickened skin (Choice E) is a feature of a group of disorders called keratodermas, which can be inherited or acquired. The thick, scaly skin in these disorders is due to thickening of the stratum corneum, the top layer of the epidermis. The hemidesmosome is not
affected and blisters are not seen.

An ulcer (Choice F) is a defect in the epidermis that leaves the dermis exposed. Ulcers can be a feature of autoimmune diseases such as systemic lupus erythematosus and Beh�t disease, however they are not due to a defect in anchoring fibrils or the
hemidesmosome.

Educational Objective: A defect in any component of the hemidesmosome, including the anchoring fibrils, will result in blister formation. Epidermolysis bullosa is characterized by antibodies against or a mutation of the anchoring fibrils.

r
Previous
r
Next
,-
Score Re ort

Lab Values
F
Calculator

Hel
tft
Pause
Exam Section1: Item 5of 50 National Board pf Medical Examiners®
Comprehensive B asic Science Serr-Assessment

✓ 5. A 65-year-old woman has ascites. Which of the following additional findings indicates a diagnosis of constrictive pericarditis rather than cirrhosis?

A� Edema of thelower exm1mities


g� Esophageal varises
G� Hypoalbuminemia
D) Hyponatremia
E ) Increased jugular venous pressure
� � Splenomegaly
Correct Answer: E .

Increased jugular venous pressure (JVP) is an expected finding in constrictive pericarditis (CP) and would not be found in patients with volume overload secondary to cirrhosis. CP describes a pathologic state whereby the pericardium, which encases the entirety of
the heart and the origins of the great vessels, loses its elasticity. This can occur in patients with viral infections, connective tissue disease, tuberculosis, or as a result of cardiac surgery or radiation. During the normal cardiac cycle, increased venous return to the
right atrium (RA) and right ventricle (RV) during inspiration leads to transient expansion of the RV with slight bowing of the interventricular septum into the left ventricle (LV). This increased RV preload does not impair LV filling as the pericardia! sac expands to
accommodate the increased RV volume. In constrictive pericarditis, the pericardia! sac loses its elasticity. When venous return to the right heart increases, the pericardia! sac is unable to expand, which exacerbates movement of the interventricular septum into the
LV. This impairs diastolic filling of the LV and reduces cardiac output. LV diastolic filling is further reduced by a reduction in preload from the pulmonary veins. The constricted pericardium does not respond to normal changes in intrathoracic pressure during
inspiration, but the pulmonary venous system, which lies outside of the pericardium, experiences a normal drop in pressure during inspiration. This difference creates an abnormal pressure gradient that reduces LV preload and leads to reduced cardiac output. The
RV and LV develop interventricular dependence whereby increased pressure in each ventricle begins to affect the other ventricle. CP ultimately results in equalizatidn of pressures in all four chambers, with clinical evidence of right heart failure exhibited by an
increased JVP on physical examination, along with peripheral edema, and occasionally congestive hepatopathy or ascites. While cirrhosis can lead to hypervolemia, this is often manifested by peripheral edema, pulmonary edema, pleural effusions, or ascites, rarely
increased JVP pressure, unless concomitant right heart failure is also present.

Incorrect Answers: A, B, C, D, and F.

Edema of the lower extremities (Choice A) is seen frequently in cirrhosis and is caused by hypoalbuminemia and systemic vasodilation with capillary leak. In CP, extremity edema is secondary to right heart failure.

Esophageal varices (Choice B) are a consequence of portal hypertension and associated retrograde flow across the portosystemic venous anastomosis of the lower esophagus. This causes dilation of the submucosal veins in the esophagus (varices), which are a
risk factor for gastrointestinal bleeding. It is encountered in cirrhosis but is not a feature of CP.

Hypoalbuminemia (Choice C) is seen in cirrhosis but not in CP. Albumin is synthesized in the liver, which is impaired in patients with cirrhosis.

Hyponatremia (Choice D) is common in cirrhosis with ascites. Portal hypertension results in systemic vasodilation and decreased effective circulating volume. This activates the renin-angiotensin-aldosterone system and induces release of antidiuretic hormone. The
net effect is an increase in resorption of free water resulting in hyponatremia.

Splenomegaly (Choice F) in cirrhosis is a consequence of portal hypertension and is typically not seen in CP.

Educational Objective: Constrictive pericarditis (CP) results from the loss of the norn:ial elasticity of the pericardium, which causes interventricular interdependence and equalization of pressures throughout the heart with impaired diastolic filling. Increased jugular
venous pressure (JVP) is often seen on physical examination and is a result of elevated filling pressures and right heart failure.

r
Previous
r
Next
,-
Score Rep _o_________________________________________
rt _L_
_____ab_Va_ u_es_C_alcul�p
l_ _
� F �
_
tft
u_s__________________
P_a_ e _
Exam Section 1: Item 6 of 50 Natlonal Board of Medic al Examiners®
Comprehensive 1:1as1c Science Sen-Assessment

✓ 6. A 62-year-old man is being evaluated for rectal bleeding. An x-ray of the gastrointestinal tract with contrast material is shown. Which of the following is the most likely explanation for the feathery appearance
in the portion of the gastrointestinal tract indicated by X when compared with the portfion indicated by Y?

A ) Absence of circular muscle


B ) Fewer villi

--- -------------------------------------------------------------
C ) Greater bowel motility
0) Greater mucosa I surface area
E ) Less circular and longitudinal smooth muscle

Correct Answer: D.

The jejunum (X) can be distinguished from the ileum (Y) by several characteristic features. The inner mucosal folds, or plicae circulares, are more prominent, more numerous, and taller in the jejunum compared to the ileum. This leads to an increased mucosa!
surface area and an associated feathered appearance after the administration of oral contrast material. The jejunum also demonstrates a larger caliber, thicker muscular walls, longer vasa rectae, and fewer arcades. The ileum is notable for the presence of Peyer
patches and mucosa! lymphoid follicles, which can be identified histologically.

Incorrect Answers: A, B, C, and E.

Absence of circular muscle (Choice A) is not a feature of either the jejunum or the ileum. The muscularis extema, which is organized into circular and longitudinal layers, extends through all segments of the small intestine.

Fewer villi (Choice B) is not a feature of the jejunum. Both the jejunum and the ileum contain villi. However, the plicae circulares are much more prominent and numerous in the jejunum, leading to an increased overall surface area and an associated increased
number of mucosa! villi compared to the ileum.

Greater bowel motility (Choice C) is not a distinguishing characteristic of the jejunum as compared to the ileum. Bowel motility is controlled by peristaltic contractions and migrating motor complexes that are under the control of the enteric and central nervous
systems.

Less circular and longitudinal smooth muscle (Choice E) is a feature of the ileum (Y) rather than the jejunum (X). The jejunum has a thicker muscular wall in comparison to the ileum, but its feathered appearance with oral contrast material is primarily due to its
prominent plicae circulares and increased mucosal surface area.

Educational Objective: The jejunum possesses several characteristic features, including prominent plicae circulares, which lends the jejunum an increased mucosal surface area and a feathered appearance with oral contrast material. The jejunum also
demonstrates a larger caliber, thicker muscular walls, longer vasa rectae, and fewer arcades.

,, ,. , frE r- ,,, If+


Previo us Next Score Rep_o_rt__________________________________________ _
_L_ab_V_ a_l_ue_s_C_a_lc_ u _la_tor_ _ _ _ e lp
_ H_ :Pa use
Exam Section 1: Item 7 of 50 National Board pf M11dlcal Exftllllners®
Comprehensive 1:1a111c: science Serr-Assessment

✓ 7. A 65-year-old man who has chronic lymphocytic leukemia has the sudden onset of fatigue and shortness of breath. Laboratory studies show:
Hemoglobin 6.5 g/dL
Hematocrit 19%
Leukocyte count 50,000/mmJ
Platelet count 170,000/mmJ
Reticulocyte count 8%


Serum bilirubin
Total 4 mg/dL
Conjugated 0.5 mg/dL

A peripheral blood smear is shown. Which of the following is the most likely cause of the anemia?

A ) Aplastic anemia
B) Autoimmune hemolysis
G}- Hereditary spherocytosis
C}- Microangiopathic hemolysis
et MonGClonal gammopatny
Correct Answer: B.

Autoimmune hemolysis explains this patient's anemia, unconjugated hyperbilirubinemia, reticulocytosis (large blue-colored erythrocytes), and spherocytes on the peripheral smear. Autoimmune hemolytic anemia (AIHA) occurs due to production of antibodies
targeting circulating red blood cell (RBC) surface antigens, marking them for removal in the reticuloendothelial system (RES) or fixing complement leading to intravascular hemolysis. lgG antibodies are active at the physiologic temperature of the human body
(called "'warm AIHA'"); these do not routinely activate complement. Instead, they mark the cells for removal. Phagocytosis of the lgG antibody along with a component of the erythrocyte membrane creates sphere-shaped RBCs seen on peripheral smear. In
contrast, lgM antibodies bind and fix complement at lower temperatures and result in intravascular hemolysis (called "cold AIHA"). lgG antibodies are known as "warm" antibodies, and lgM antibodies are known as "cold" antibodies. Regardless of the type of
antibody, laboratory findings demonstrate a normocytic anemia, an increased reticulocyte count indicating hematopoiesis, and unconjugated hyperbilirubinemia from hemoglobin released from lysed erythrocytes. Diagnosis is made by laboratory analysis, a
compatible clinical history, and positive direct antibody test (DAT, Coombs). In the DAT, the patient's erythrocytes are washed free of plasma, and incubated with Coombs reagent, an anti-lgG and anti-complement antibody. If an autoantibody or complement is
bound to the surface of the erythrocytes, the Coombs reagent will bind to it and cause agglutination, a positive test. Chronic lymphocytic leukemia (CLL) predisposes to the development of warm AIHA because malignant CLL cells produce autoantibodies.

Incorrect Answers: A, C, D, and E.

Aplastic anemia (Choice A) results from bone marrow destruction of erythrocyte precursors and can be a primary autoimmune process, secondary to viral infections, medications, myelotoxin exposures (eg, heavy metals), or acquired clonal abnormalities. It
manifests with anemia and inappropriately low reticulocyte count.

Hereditary spherocytosis (Choice C) is caused by mutations in spectrin or ankyrin cytoskeletal anchoring genes that leads to fragile erythrocyte membranes and poor distensibility. Patients present with chronic hemolytic anemia (and family history of anemia) and
spherocytes on peripheral smear. While this patient does have spherocytes, his clinical history of CLL makes AIHA much more likely.

Microangiopathic hemolysis (MAHA) (Choice D) presents with anemia, increased serum free hemoglobin and lactate dehydrogenase, decreased haptoglobin, and increased unconjugated bilirubin, however, schistocytes (fragmented RBCs) would be seen on
peripheral smear. MAHA often results from an infectious or inflammatory insult damaging endothelium. Subsequent microthrombi form and erythrocyte destruction occurs from shearing across them.

Monoclonal gammopathy (Choice E) is seen in multiple myeloma, Waldenstrom macroglobulinemia, and monoclonal gammopathy of unknown significance. While anemia can occur, it is most commonly a result of bone marrow infiltration by malignant cells (eg,
myeloma). Peripheral smear may show RBC aggregation in rouleaux formation but would not demonstrate spherocytes or reticulocytes.

Educational Objective: AIHA results from the binding of an autoantibody to RBC surface antigens, leading to removal via the RES or to intravascular hemolysis. Diagnosis is made by a positive DAT (Coombs), unconjugated hyperbilirubinemia, low haptoglobin,
and the presence of spherocytes and reticulocytes on the peripheral smear.

r
Previous
r
Next
,,,
Score Re ort

Lab Values
r-·
Calculator

Hel
"'
Pause
Exam Section 1: Item 8 of 50 National Board pf Medical Examiners®
Comprehensive Basic Science Serr-Assessment

✓ 8. A 47-year-old man with a history of rheumatic valvular disease comes to the physician because of chest pain and difficulty breathing for the past 2 days. An ECG shows an acute myocardial infarction. Examination
shows a systolic murmur. Pressure tracings from the aorta, left ventricle (LV), and left atrium (LA) are shown. Which of the following is the most likely diagnosis?

A}Aortic obstruction � Aortic


B}Aortic regurgitation I
E
G} Left ventricular aneurysm
E
D) MitraI obstruction
E ) Rupture of the chordae tendineae
...
:::,
LV
1/)
1/)

Q.

Time
Correct Answer: E.

Rupture of the chordae tendineae with development of mitral regurgitation accounts for the findings depicted in the graph. The diagram depicts changes in pressure within the aorta, LV, and LA over a single cardiac cycle. Normally the LV will undergo
isovolumetric contraction with expulsion of blood through the aortic valve once the pressure inside the LV exceeds the aortic pressure. When pressure in the aorta (diastolic pressure) exceeds or equals the LV pressure, the aortic valve closes and the LV is able
to relax, corresponding to a rapid decrease in pressure. This will allow for the ventricle to fill during diastole. During ventricular systole, the LA should demonstrate only a minimal increase in pressure because a competent mitral valve prevents regurgitation of
blood back into the LA. However, if the mitral valve is incompetent (mitral regurgitation), blood from the LV will enter the LA through the mitral valve. This is depicted in the diagram as an increase in pressure in the LA during LV systole, which is abnormal.
Following a myocardial infarction, a common cause of mitral valve regurgitation is rupture of a papillary muscle and associated chordae tendineae, which typically anchor the leaflets of the mitral valve to the ventricles and prevent regurgitation during ventricular
systole. When ruptured, there is no support structure to keep the mitral valve leaflets closed, so the increased LV pressure forces them back into the LA with consequent development of mitral regurgitation. Depending upon the severity, this can result in acutely
decompensated heart failure, and surgical repair may be required.

Incorrect Answers: A, 8, C, and D.

Aortic obstruction (Choice A) from aortic stenosis means that the LV must generate a much higher pressure to overcome the stenotic valve and effectively eject blood into the aorta. This would be depicted by an LV pressure tracing that exceeds (taller than) the
aortic pressure tracing during systole.

Aortic regurgitation (Choice 8) is the result of an incompetent aortic valve. After closure of the aortic valve, blood will continue to flow backwards into the LV, which would appear on the graph as a reduced and protracted decrease in pressure on the LV tracing
after systole. There will also be a decrease in aortic diastolic pressure because of the regurgitation.

Left ventricular aneurysm (Choice C) results in paradoxical outward motion of the myocardium during systole with reduced stroke volume. It would not cause increased left atrial pressure during systole, as in mitral regurgitation.

Mitral obstruction (Choice D) from mitral stenosis results in an abnormally increased LA pressure due to the impaired flow of blood from the LA into the LV. Additionally, the LV stroke volume may also be reduced from impaired diastolic filling.

Educational Objective: Rupture of the chordae tendineae and papillary muscle is a potential complication of myocardial infarction and results in acute mitral regurgitation. On a pressure-time graph, this manifests as an increased left atrial pressure during
ventricular systole from regurgitant blood.

r
Previous
r
Next
,-
Score Re ort

Lab Values
F
Calculator

Hel
tft
Pause
- -

Exam Section 1: Item 9 of 50 Nallonal Board pf M11dlcal Examiners®


Comprehensive 1:1as1c Science Self-Assessment

✓ 9. After being given an infusion of mannitol (400 mM), a healthy person is most likely to have which of the following changes in plasma osmolality and plasma ADH (vasopressin) concentration?

10

(/)
(/)
<l)
a.
O ..J
"' E
"' ---- 5
�Ol
J: a.

"'
E
(/)
.!!! 0
a.
280 290 300 310
Plasma osmolality (mOsmol/L)

O Control before infusion


A)
B)
C)
D)
E)
Correct Answer: A.

Mannitol, an osmotically active carbohydrate monomer, raises serum osmolality when given as an infusion. Mannitol, when infused, is a hyperosmolar solution. Following infusion, the net serum osmolality will be increased as mannitol molecules remain within
serum. In tum, the increased osmolality of serum will draw fluid into the intravascular space from the interstitium. This principle is used therapeutically in the treatment of acute increases in intracranial and intraocular pressure, for example in cases of acute cerebral
edema, impending brain herniation, or glaucoma. Osmoreceptors detect the change in osmolality, and trigger production of antidiuretic hormone (ADH) by the hypothalamus with release by the posterior pituitary gland. ADH, in tum, increases insertion of aquaporin
channels in the -collecting duct of the nephron. Free water is reclaimed by the aquaporins, which dilutes the serum and in turn normalizes osmolality. In this graph, person A has serum studies that reflect increased osmolality and increased production of ADH, which
reflects serum cilanges following the administration of mannitol.

Incorrect Answers: B, C, D, and E.

Choice B reflects increased osmolality without associated increased serum ADH. This set of values may be obtained in the immediate moments followin,g a rapid bolus of mannitol, but ADH levels would quickly increase to compensate for increased serum
osmolality.

Choice C reflects increased osmolality and decreased ADH, which is not consistent with an appropriate physiologic response to increased osmolality.

Choice D reflects decreased osmolality, not increased as would occur with the administration of mannitol. In a case of physiologically decreased serum osmolality, as in psychogenic polydipsia, low serum ADH would be expected as a normal physiologic response.

Choice E reflects decreased serum osmolality and increased serum ADH, which would be consistent with a diagnosis of syndrome of inappropriate ADH, a frequent cause of euvolemic, hypoosmolar hyponatremia.

Educational Objective: Mannitol, an osmotically active carbohydrate monomer, raises serum osmolality when given as an infusion. Osmoreceptors detect the change in osmolality and trigger the production of antidiuretic hormone by the hypothalamus.

r
Previous
r
Next
,
Score Report
tr5
Lab Values
F
Calculator

Help

Pause
Exam Section 1: Item 10 of SO National Board pf M11dlcal Exftllllners®
Comprehensive 1:1a111c: science Serr-Assessment

✓ 10. A 37-year-old man comes to the physician because of a 1-week history of nausea, vomiting, and abdominal pain. He ate raw oysters while vacationing in Mexico 2 weeks ago. He has smoked 1 pack of cigarettes daily for the past 15 years but has now lost
the desire to smoke. Physical examination shows scleral icterus and slight enlargement and tenderness of the liver. Which of the following pathogens is most likely responsible for the development of these symptoms in this patient?

A ) Epstein-Barr virus
B) Hepatitis A virus
C) Hepatitis B virus
D) Hepatitis C virus
E) Vibrio vutnmcus
Correct Answer: B.

This patient's presenting findings of nausea, vomiting, abdominal pain, icterus, hepatomegaly and right upper quadrant tenderness two weeks after eating raw shellfish is consistent with an acute hepatitis A virus (HAV) infection. HAV is a non-enveloped, single­
stranded RNA virus that is spread through fecal-oral transmission. It is commonly acquired through ingestion of poorly cooked, improperly handled, or raw foods, including shellfish. HAV has an incubation period of several weeks. Presenting signs and symptoms
include fever, nausea, vomiting, poor appetite, abdominal pain, jaundice, al'ld hepatomegaly, although many patients are asymptomatic. Laboratory findings include transaminitis, increased alkaline phosphatase, hyperbilirubinemia, and bilirubinuria. The diagnosis is
confirmed by the serologic detection of anti-HAV lgM antibodies. HAV infection is typically a self-limited illness and does not progress to chronic infection.

Incorrect Answers: A, C, D, and E.

Epstein-Barr virus (Choice A) causes infectious mononucleosis, which presents with fever, hepatosplenomegaly, pharyngitis, and posterior cervical lymphadenopathy. It is transmitted primarily through saliva and respiratory secretions and is not transmitted through
uncooked food.

Hepatitis B (Choice C) and C virus (Choice D) are transmitted through bodily fluids. Hepatitis B is commonly transmitted through blood, sexual contact, or perinatally. Hepatitis C is commonly transmitted through blood and is especially common in the setting of
intravenous drug use. The patient's history is suggestive of oral transmission through raw shellfish.

Vibrio vulnificus (Choice E) is a Gram-negative, primarily water-borne bacterium that causes sepsis or bacteremia and is associated with the consumption of raw shellfish. V. vulnificus sepsis presents acutely after the ingestion of shellfish with fever, hypotension,
shock, bullous skin lesions, gastrointestinal hemorrhage, and coagulopathy, which may progress to disseminated intravascular coagulation. It is distinguished from HAV infection by its rapid onset, absence of jaundice, and generally severe clinical presentation.

Educational Objective: Hepatitis A virus (HAV) causes a self-limited infection that presents with nausea, vomiting, abdominal pain, icterus, hepatomegaly, and right upper quadrant tenderness that evolves over several weeks following the il'lgestion of poorly cooked,
improperly handled, or raw foods, including shellfish. HAV is diagnosed by the serologic detection of anti-HAV lgM antibodies.

r
Previous
r
Next
,,,
Score Re ort

Lab Values
r-·
Calculator

Hel
"'
Pause
Exam Section 1: Item 11 of SO National Board pf Medical Examiners®
Comprehensive Basic Science Serr-Assessment

✓ 11. An 8-year-old boy is brought to the physician by his parents because of disruptive behavior. They say, "He is easily distracted, constantly interrupt:s us while we are talking, and seems to be always moving. His teacher says he is always talking with his
neighbors, has trouble completing tasks, and refuses to wait his turn when playing games." Physical examination shows no abnormalities. If drug therapy is indicated, administration of a drug with which of the following mechanisms of action is most
appropriate?

A}Antagonism at�adrenergis receptors


B ) Blockade of voltage-gated Na + channels
C) Enhanced action of y-aminobutyric acid (GABA) at GABA A receptors
D) Increased release of dopamine and norepinephrine
E}Selesli•Je inllitiition of uptake of serotonin al nerve terminals
Correct Answer: D.

The first-line treatment for attention-deficit/hyperactivity disorder (ADHD) is stimulant medication such as amphetamine salt:s, which increase presynaptic release of dopamine and norepinephrine. ADHD presents with chronic symptoms of hyperactivity/impulsivity
and/or inattention that occur in more than one setting and impair academic, social, or emotional function. Children with predominant hyperactivity symptoms are unable to sit still and may have difficulty taking turns (postulated to result from a dysregulated reward
pathway), whereas children with predominant inattentive symptoms tend to daydream, process information slowly, and demonstrate difficulty in completing tasks (likely related to norepinephrine dysregulation). This patient is experiencing symptoms of both
inattention and hyperactivity. Combination treatment with medications and psychotherapy may be the most effective option, though medication monotherapy is also appropriate for children older tha111 6 years. Stimulants, which include amphetamine salts and
methylphenidate, are first-line agents. Both types of stimulants increase synaptic dopamine and norepinephrine, which improve reward processing and attention, respectively.

Incorrect Answers: A, B, C, and E.

Antagonism at 13-adrenergic receptors (Choice A) is the mechanism of 13-adrenergic blocker medications. Some 13-adrenergic blocker medications, such as propranolol, are centrally acting and can be used for performance anxiety or traumatic brain injury-related
agitation. These medications are unhelpful in ADHD.

Blockade of voltage-gated Na+ channels (Choice B) is the mechanism utilized by several mood stabilizers and antiepileptics including carbamazepine and lamotrigine. These medications are used for bipolar disorder, not ADHD.

Enhanced action of y-aminobutyric acid (GABA) at GABAA receptors (Choice C) is the mechanism of sedative agents such as benzodiazepines and barbiturates. Benzodiazepines can be used for panic disorder. However, these medications are not useful for ADHD
symptoms.

Selective inhibition of uptake of serotonin at nerve terminals ( Choice E) is the mechanism of first-line treatments for major depressive disorder, which belong to the class selective serotonin reuptake inhibitors. These medications do not target ADHD.

Educational Objective: The first-line treatment for attention-deficit/hyperactivity disorder is stimulant medication such as amphetamine salts and methylphenidate. These medications increase presynaptic release of dopamine and norepinephrin.e, which improve
hyperactivity and inattention, respectively.

r
Previous
r
Next
,.
Score Report

Lab Values
F
Calculator

Help
tft
Pause
Exam Section 1: Item 12 of 50 National Board pf M11dlcal Examiners®
Comprehensive isaslc S•Clence Serr-ASsessment

✓ 12. A previously healthy 35-year-old woman develops hypoxemia 35 minutes after ingesting a near-lethal dose of barbiturates. She has not aspirated. Which of the following sets of arterial blood gas values (in mm Hg) is most likely in this patient?

Po2 Pc02 (A-a)02


A) 40 50 35
B) 40 60 40
G}- 50 2& 4-0
D) 50 80 10
€)- eO � 23
Correct Answer: D.

Barbiturates are central nervous system depressants which act on the chloride pore of the GABA receptor, increasing the time the receptor is opened. Symptoms of overdose include enceplhalopathy and respiratory depression. Respiratory depression leads to
hypoventilation as a decreased volume of air is moved to the alveolar-capillary interface for gas exchange. Hypoventilation can be identified by decreased PO2, increased co-. 2, and a normal alveolar-arterial (A-a) gradient on arterial blood gas analysis. The
alveolar-arterial ( A-a) gradient, also written as (A-a)O 2, is the differe111ce between the partial pressure of oxygen in the alveoli (PAOi) and the arterial blood (PaOi). The PaO 2 is measured directly on the blood gas analysis. The PAO 2 must be estimated using the
alveolar gas equation, which is a function of the fraction of inspired oxygen (FiO 2), th_e partial pressure of carbon dioxide (PaCO 2), and the respiratory quotient (Ra), which relates the volume of CO 2 produced by tissue to the volume of 0 2 consumed. If there is
no disruption of the alveolar-capillary interface or impaired perfusion to ventilated lung, the PAO2 and PaO 2 will equalize as oxygen diffuses readily across the membrane. A normal A-a gradient of ~10 mm Hg exists based on physiologic shunt, which is a volume
of blood that does not interact with ventilated lung. This occurs due to a normal ventilation-perfusion mismatch that exists due to gravity (increased ventilation in the apices of the lung, increased perfusion in the bases) as well as circulation that bypasses the
alveolar-capillary interface entirely (eg, bronchial arteries to pulmonary veins, smallest cardiac veins that drain directly into the left ventricle). This deoxygenated blood mixes with freshly oxygenated blood returning from the lungs, leading to a decreased PaO 2
relative to the PAO-.2. Hypoventilation results in decreased gas exchange, with no disruption of diffusion or perfusion. A normal A-a gradient is expected.

Incorrect Answers: A, B, C, and E.

Choices A, B, and E are incorrect as they reveal an increased A-a gradient. l:he causes of hypoxemia can be divided into those that affect the A-a gradient and those that do not. A normal A-a gradient is seen in decreased environmental Fi02 (eg, high altitude)
and hypoventilation. An increased A-a gradient is seen in ventilation-perfusion (V/Q) mismatch (ventilation of lung that is not perfused, eg, pulmonary embolism), right-to-left shunt (perfusion in regions of lung that are not ventilated, eg, airway obstruction), and
diffusion limitation (destruction or impairment of the alveolar-capillary interface, eg, interstitial lung disease, pulmonary edema). These conditions disrupt normal gas exchange and increase the gradient between oxygen pressure in the alveoli and the arterial
blood.

Choice C is incorrect as it represents a respiratory alkalosis (decreased PCOi). This is caused by hyperventilation, which may be secondary to central nervous system lesions, hypoxemia, a prima . ry lung process, or iatrogenic causes. Central lesions include
traumatic brain injury, stroke, hyperthyroidism, medications (eg, salicylates), anxiety, pain, and fear. Primary lung processes include pneumothorax, pulmonary embolism, pneumonia, and acute asthma exacerbation. Iatrogenic hyperventilation is primarily due to
overven1ilalion of intubated patients, which may be intentional in cases of acute elevations in intracranial pressure. In this case, the patient has respiratory depression which causes co-. 2 retention and a respiratory acidosis.

Educational Objective: Respiratory depression results in hypoventila1ion, which is characterized by hypoxemia, respiratory acidosis (due to CO2 retention), and a normal alveolar-arterial gradient on arterial blood gas analysis. An increased A-a gradient is seen in
conditions that cause V/Q mismatch, shunt, and/or diffusion limitatio111.

r
Previous
r
Next
,
Score Report
rc5
Lab Values Ca�lp
p r, "'
Pause
Exam Section 1: Item 13 of SO National Board pf M11dlcal Exftllllners®
Comprehensive 1:1a111c: science Serr-Assessment

✓ 13. A 64-year-old woman develops pain and stiffness of the proximal interphalangeal joints and the right knee. The pain is made worse by activity and is relieved by rest X-rays of the knee show a narrowed joint space with radiodense subchondral bone and
cyst formation. Knee fluid aspirate shows:
Appearance clear, yellowish
Leukocyte count 250/mm 3 (N<200)
Neutrophils 5% (N<25%)
Glucose 101 mg/dl (N=B0-100)

Which of the following is the most likely cause of her condition?

A ) Acute gouty arthritis


B) Osteoarthritis
G}- Rhe1,1matgid arthritis
D) Septic arthritis
e}- Ngmial ago related changes in jgints
Correct Answer: B.

Osteoarthritis is the most common disease of joints in humans. It affects 60% to 80% of adults over the age of 65. It is characterized by global degeneration of a joint with fraying, fibrillation, and degradation of cartilage, thickening of the subchondral plate with
sclerotic bone, and subchondral cysts due to infiltration of synovial fluid into the bone. On radiographs, the disease is demonstrated by joint space narrowing, articular sclerosis, radiolucent subchondral cysts, and sharpened corners of the joints with new bony
protrusions (osteophytosis). If occurring before age 65 years, patients may have a history of prior traumatic injury to a joint that incited an inflammatory cascade and accelerated the onset of osteoarthritis. This presentation is known as posttraumatic arthritis. Joint
aspiration is characterized by normal appearing aspirate on Gram stain and culture. However, this synovial fluid typically contains cartilage and bone degradation particles as well as inflammatory cytokines and metalloproteases that are not detectable on traditional
aspiration. Osteoarthritis may occur concomitantly with inflammatory or septic arthritis. The diagnosis of osteoarthritis does not preclude the existence or occurrence of more aggressive arthropathies.

Incorrect Answers: A, C, D, and E.

Acute gouty arthritis (Choice A) is an inflammatory arthropathy incited by hyperuricemia, with deposition of uric acid crystals inside the joint and surrounding tissues. This type of arthritis presents following consumption of a purine-rich meal or alcohol (which
competes with renal excretion of uric acid). A knee aspirate would show negatively birefringent needle-shaped crystals and a leukocyte count of typically less than 50,000 cells/mm3.

Rheumatoid arthritis (Choice C) is a polyarticular inflammatory arthritis that is characterized by increased serum levels of rheumatoid factor. It most commonly affects the metacarpophalangeal and proximal interphalangeal joints and improves with use during the
day. Increased serum antibodies against cyclic citrullinated peptides are specific for rheumatoid arthritis.

Septic arthritis (Choice D) is most commonly bacterial, and presents with acute joint swelling, erythema, pain, and tenderness with range of motion. Aspirate typically reveals bacteria on Gram stain, most often Gram-positive organisms, and leukocytes of greater
than 50,000 cells/mm3 with a neutrophilic predominance.

Normal age-related changes in joints (Choice E) include increased stiffness and decreased strength and water content of cartilage. The thickness of the subchondral bone plate decreases as does the capacity of joint tissues to heal and regrow. Normal joint aging
does not cause pain or joint space narrowing.

Educational Objective: Osteoarthritis is the most common disease of joints in humans. It affects 60% to 80% of adults over the age of 65. It is characterized by global degeneration of a joint with fraying, fibrillation, and degradation of cartilage, thickening of the
subchondral plate with sclerotic bone, and subchondral cysts due to infiltration of synovial fluid into the bone. Arthrocentesis typically demonstrates few leukocytes, normal appearance, and normal glucose.

r
Previous
r
Next
,,,
Score Re ort

Lab Values
r-·
Calculator

Hel
"'
Pause
Exam Section 1: Item 14 of 50 National Board of M11dlcal Examiners®
Comprehensive isaslc Science Serr-ASsessment

✓ 14. Which of the following labeled depolarizations is associated with the smallest stroke volume?

A B C D E
A)
B)
C)
D)
E)
Correct Answer:. B.
The electrical complex labeled B in the rhythm strip is characteristic of a premature ventricular contraction (PVC) and is associated with a lower stroke volume when compared to a normal sinus beat. Normal cardiac conduction starts with an electrical impulse in the
sinoatrial (SA) node located in the right atrium, which is depicted on the rhythm strip as the •p wave." The electrical impulse is conducted throughout the atria, causing coordinated contraction of the right and left atria, with consequent active filling of the right and left
ventricles. The electrical impulse also reaches the atrioventricular (AV) node located in the interatrial septum. Conduction is temporarily halted in the AV node to allow for complete active and passive filling of the ventricles, followed by conduction of the electrical
impulse down the His-Purkinje system in the interventricular septum, then to the right and left bundle, and finally to the myocardial cells of the ventricles. This results in contraction of the right and left ventricles (ORS complex) with expulsion of blood into the
pulmonary and systemic circulations, respectively. The ventricles subsequently repolarize (T wave) and relax to allow filling for the next cardiac cycle. Stroke volume is defined as the difference between the end-diastolic-volume and the end-systolic-volume. To
achieve an ideal stroke volume, diastolic filling time must be sufficiently long to allow the ventricles to fill completely before contraction occurs. This is achieved by delaying action potential conduction in the AV node. A premature ventricular contraction, which is
shown by letter "B," indicates premature ventricular systole. When this occurs, diastolic filling time is reduced, so the amount of blood in the ventricles prior to ventricular systole is less than would exist had filling time been sufficient. This results in a reduced stroke
volume.

Incorrect Answers: A, C, D, and E.

Each of these answers (Choices A, C, D, and E) depict normal sinus beats. There is a P-wave followed by a ORS complex and a T wave. Only Choice A lacks a T wave because of the superimposed, subsequent PVC. Normal cardiac conduction allows for
adequate filling of the ventricles before systole.

Educational Objective: Stroke volume is the difference between the ventricular end-diastolic volume and the end-systolic volume. PVCs shorten diastolic filling times leading to lower diastolic volumes with a consequent reduction in stroke volume during ventricular
systole.

r
Previous
r
Next
,
Score Re ort

Lab Values
e
Calculator
r,
Hel
"'
Pause
Exam Section 1: Item 15 of 50 National Board of Medical Examiners®
Comprehensive Basic Science Self-Assessment

✓ 15. A previously healthy 19-year-old woman comes to student health services because of a 3-day history of vulvar itching and vaginal discharge. She has a new sexual partner and uses no contraception. Speculum examination shows a profuse yellow-gray
discharge in the vagina. The pH of the discharge is 6. Microscopic examination of the discharge shows clue cells. Addition of KOH to the discharge produces a strong amine odor. Which of the following is the most likely cause?

A ) Bacterial vaginosis
B ) Bacteroides fragilis
C ) Candida albicans
D ) Haemophilus ducreyi
E ) Herpes simplex virus
F) HIV
G) Human papillomavirus
H ) Pneumocystis jirovecii (formerly P. carinil)
I ) Treponema pal/idum
J ) Trichomonas vagina/is
Correct Answer: A.

Bacterial vaginosis is a common gynecologic condition characterized by a shift of vaginal flora and overgrowth of particular bacterial species, most commonly Gardnerella vagina/is. G. vagina/is is a Gram-variable, facultative anaerobe. Bacterial vaginosis presents
with gray, thin, malodorous vaginal discharge and may cause vulvovaginal pruritus. Vaginal pH is typically above 4.5, and a fishy odor is detected upon KOH testing. The identification of clue cells on wet mount preparation is diagnostic. Metronidazole is the first-line
therapy. Bacterial vaginosis is not considered a sexually transmitted infection and treatment of asymptomatic sexual partners is not recommended.

Incorrect Answers: B, C, D, E, F, G, H, I, and J.

Bacteroides fragilis (Choice B) is an anaerobic, Gram-negative bacillus and a normal component of the gastrointestinal microbiota. It only results in infections when displaced from the colon such as following surgery, rupture, or trauma, where it has the potential to
cause bacteremia, intra-abdominal infections, pe ritonitis, and subcutaneous abscesses.

Candida a/bicans (Choice C) is a fungus that also forms part of the normal vaginal flora. Overgrowth of Candida species may produce vulvovaginitis. Candida! vulvovaginitis, in contrast to bacterial vaginosis. produces pruritus and a white discharge. Vaginal pH is
normal, and hyphae may be visualized on wet mount.

Haemophilus ducreyi (Choice D) is a Gram-negative organism that causes the sexually transmitted infection chancroid. Chancroid presents with painful genital ulceration and inguinal lymphadenopathy.

Herpes simplex virus (Choice E) is an enveloped, double-stranded DNA virus that causes sexually transmitted infections among other syndromes (eg, encephalitis, herpes labialis, herpetic Whitlow). Symptoms of genital herpes include pain, burning, and a vesicular
rash.

HIV (Choice F) is an enveloped, single-stranded RNA virus of the retrovirus family that causes acquired immunodeficiency syndrome. The acute viral syndrome resembles an influenza-like illness that transitions into a prolonged dormant period, followed by the
onset of immunodeficiency and opportunistic infections.

Human papillomavirus (HPV) (Choice G) is a non-eniveloped, double-stranded DNA virus of the papillomaviridae family that causes genital warts. Infection with HPV is a significant and preventable risk factor for cervical, penile, anal, and oropharyngeal cancer.

Pneumocystis jirovecii (formerly P. carini1) (Choice H) is an opportunistic fungal organism that can cause pneumonia in immunocompromised patients. Pneumonia secondary to P. jirovecii has become rare following the widespread use of prophylaxis with
trimethoprim-sulfamethoxazole in vulnerable patients.

Treponema pallidum (Choice I) is the causative spirochete bacterium of syphilis, which presents in multiple stages with varying symptoms, including primary with a painless chancre, secondary with fever, diffuse maculopapular rash, lymphadenopathy, and
condylomala laia, and iertiary with labes dorsalis, aortills, and gummas.

Trichomonas vagina/is (Choice J) is an anaerobic, flagellated protozoan that causes vaginal trichomoniasis. Trichomoniasis presents with pruritus, dysuria, dyspareunia, cervical erythema, and abnormal discharge that is described as green, frothy, and malodorous.
Vaginal pH is often above 4.5 and motile trichomonads can be visualized on wet mount preparation.

Educational Objective: Bacterial vaginosis is caused by the vaginal overgrowth of Gardnerella vagina/is. a Gram-variable, facultative anaerobe. It presents with gray, thin, malodorous vaginal discharge, a vaginal pH greater than 4.5, a fishy odor upon KOH testing,
and clue cells on microscopy.

r r ,,. r.§ F r r
Previous Next Score Report Lab Values Calculator Help Pause
Exam Section 1: Item 16 of 50 National Board of M11dlcal Examiners®
Comprehensive isas1c Science Serr-Assessment

✓ 16. A 9-year-old boy is brought to the physician because he told his teacher that his fingers felt "funny" after he swung on the monkey bars 2 hours ago. Neurologic examination shows numbness and tingling in the ring and small fingers of the left hand. The
physician concludes that the ulnar nerve may have been stretched while he was hanging from the bars. Which of the following other functions should be checked to assess the integrity of the ulnar nerve in the left limb?

A ) Abduction of the index, middle, ring, and small fingers


B ) Abduction of the thumb
G}- ExteRsieR ef the index, middle, ring, and small fingers
C} ExleRsien ef the lh1a1mb
E ) Opposition of the thumb
Correct Answer: A.

Peripheral nerve injuries of the upper extremity are commonly caused by compression, direct trauma, or nerve traction. In this patient, hanging from the monkey bars placed tension on the nerves that originate from the lower brachia! plexus. Roots CB and T1
contribute to the ulnar nerve. Injury to this nerve is causing the patient's distal motor and sensory symptoms. The ulnar nerve innervates the intrinsic muscles of the hand (eg, lumbricals, interossei, hypothenar, and palmaris brevis), and muscles of the forearm
(eg, flexor carpi ulnaris and the ulnar portion of flexor digitorum profundus). It also provides sensation to the volar aspects of the fourth and fifth digit. Injury to this nerve will cause weakness in abduction and adduction of the index, middle, ring, and small fingers,
as the dorsal interossei and the palmar interossei control these movements, respectively. To isolate ulnar nerve motor function, the patient's abduction and adduction of the digits can be evaluated with resistive strength testing. Sensation can be tested along the
volar aspect of the fourth and fifth fingers.

Incorrect Answers: B, C, D, and E.

Abduction of the thumb (Choice B) is primarily controlled by abductor pollicis brevis and abductor pollicis longus. Abductor pollicis brevis is innervated by the recurrent branch of the median nerve which can be injured during carpal tunnel surgery. The abductor
pollicis longus is innervated by the posterior interosseous nerve, a branch of the radial nerve. This nerve can be injured in a humerus fracture.

Extension of the index, middle, ring, and small fingers (Choice C) is provided by the extensor digitorum communis muscle, which is innervated by the radial nerve.

Extension of the thumb (Choice D) is controlled primarily by extensor pollicis brevis and extensor pollicis longus, both of which are innervated by the posterior interosseous nerve.

Opposition of the thumb (Choice E) is driven by oppon�ns pollicis, which is innervated by the recurrent branch of the median nerve.

Educational Objective: The ulnar nerve innervates the majority of the intrinsic muscles of the hand. It may be injured when traction is placed on the brachia! plexus. Extensors of the digits and wrist, as well as the abductors of the thumb, are innervated by the
radial nerve. The median nerve innervates the majority of the flexors of the wrist and fingefs as well as some intrinsic muscles of the thenar eminence.

r
Previous
r
Next
,
Score Report
frE
Lab Values
P
Calcu�lp�e
� ,,.
Exam Section 1: Item 17 of 50 National Board pf Medical Examiners®
Comprehensive Basic Science Sen-Assessment

✓ 17. A 22-year-old man is brought to the emergency department 20 minutes after sustaining a knife wound to the right side of his chest in a fight at a local bar. On arrival, he is short ot breath. His pulse is 108/min, respirations are 30/min, and blood pressure is
100/60 mm Hg. Physical examination shows that the trachea is deviated to the left. Further physical examination of the traumatized side is most likely to show which of the following pulmonary findings?

A ) Crackles
B ) Egophony
C) Hyperresonant percussion
D) Increased fremitus
E ) Strider
F ) Wheezing
G) Whispered pectoriloquy

Correct Answer: C.

Tension pneumothorax can be a complication of a penetrating thoracic trauma and occurs when air is able to enter the pleural space but is prevented from exiting. Increasing amounts of trapped air in the pleural space compress the lung parenchyma, creating an
inability for the lung to expand and causing it to collapse. As the pneumothorax progresses and continues to increase in size, it can result in tracheal and/or mediastinal deviation away from the affected hemithorax. It can also result in decreased venous return to the
heart, which manifests with hypotension and obstructive shock. Dullness in percussion occurs over solid structures such as bones or pleural effusions. Tension pneumothorax is characterized by absent breath sounds and hyperresonant percussion due to the
increased air in the intrathoracic space displacing normal lung parenchyma.

Incorrect Answers; A, B, D, E, F, and G.

Lesions that cause increased consolidation of the lung parenchyma, such as pneumonia, result in rales or crackles (Choice A), egophony (Choice 8), dullness to percussion, increased fremitus (Choice D), and whispered pectoriloquy (Choice G). Rales or crackles
refers to an abnormal rattling sound with respiration, which may be seen with pneumonia, but is more common in the setting of pulmonary edema, Dullness to percussion as well as decreased fremitus occurs when there is atelectasis or a pleural effusion present.
Egophony occurs when there is an increased resonance of voice sounds heard when auscultating the lungs with a stethoscope, which is due to a change in the frequency of sound passing through consolidated lung tissue. Fremitus refers to vibration and tremors
that can be palpated on the chest during patient phonation, which increases in lung consolidation and decreases when there is a pleural effusion that is separating the lung tissue from the chest wall. Whispered pectoriloquy refers to an increased sound of whispers
auscultated with a stethoscope over an area of lung consolidation. In a tension pneumothorax, these findings would not be present.

Strider (Choice E) is a high-pitched sound caused by turbulent airflow through obstructed or narrowed upper airway structures, such as the pharynx or trachea. It can be caused by illnesses such as croup or epiglottitis, or by structural lesions such as a laryngeal
tumor, foreign body, tracheomalacia, or laryngospasm.

Wheezing (Choice F) is a continuous low-pitched whistling sound on auscultation that occurs when there is narrowing of small airways, such as in asthma or chronic obstructive pulmonary disease. It would not likely be present in a patient with tension
pneumothorax.

Educational Objective: Tension pneumothorax occurs when air is able to enter the pleural space but is prevented from exiting. This leads to an increasing amount of air trapped in the pleural space with subsequent collapse of the lung parenchyma and mass effect
on the mediastinum. It is characterized by respiratory distress, tracheal deviation, hypotension, decreased breath sounds, and hyperresonant percussion on physical examination.

r
Previous
,.
Next
"
Score Report

Lab Values
r-:-
Calculator

Help
,,..
Pause
Exam Section 1: Item 18 of SO National Board of M11dlcal Exilllllners®
Comprehensive Basic Science Serr-Assessment

✓ 18. A 35-year-old man is brought to the emergency department because of multiple injuries sustained in a motor vehicle collision 45 minutes ago. He is lethargic and ,confused. The right pupil is sluggishly reactive to light and is 1 mm larger than the left pupil.
There is mild left hemiparesis. Further examination shows fracture of the right femur; ruptured spleen, and hemorrhagic pleural effusion. A CT scan of the head shows a subdural hematoma on the right. with mass effect and midline shift. He is intubated and
mechanically hyperventilated. Which of the following is the most likely rationale for hyperventilation in this patient?

A) Cause cerebral vasoconstriction


2} Deer-ease pulmonary atelectasis
C) Increase cerebral blood flow
O} lncfease lntr:acranial pfess1ire
Iii} hu:�ease oxygenation
Correct Answer: A.

The intracranial compartment is a fixed compartment surrounded by a rigid skull containing brain parenchyma, cerebrospinal fluid, and intravascular blood. Pathologic lesions that take up space in the intracranial compartment such as tumors, abscesses, and
extravascular blood such as subdural or epidural hematoma may displace normal intracranial structures, causing mass effect and midline shift. Cerebral blood flow (CBF) supplies intravascular blood in the intracranial space. Hypocapnia induces cerebral
vasoconstriction, decreasing the cerebral blood flow and the volume of intravascular, intracranial blood. Therapeutic mechanical hyperventilation induces hypocapnia and has been shown to reduce intracranial pressure in the acute setting. This method for
intracranial pressure reduction is not a long-term solution; rather, it is a temporizing measure prior to definitive treatment such as a craniotomy.

Incorrect Answers: B, C, D, and E.

Hyperventilation may decrease in pulmonary atelectasis (Choice B), however decrease in pulmonary atelectasis does not correlate with increased ventilation and hypocapnia directly. Decreasing atelectasis may be accomplished through incentive spirometry, or in
the case of a mechanically-ventilated patient, through increased positive end-expiratory pressure (PEEP).

Increased cerebral blood flow (Choice C) and increased intracranial pressure (Choice D) would not be helpful in this patient with a subdural hematoma displacing intracranial structures. The intracranial compartment is rigid, and increased blood flow or intracranial
pressure would cause further mass effect in this patient. Hyperventilation decreases cerebral blood flow and intracranial pressure through inducing cerebral vasoconstriction.

Hyperventilation has little effect on increasing oxygenation (Choice E). The oxygenation of a mechanically ventilated patient may be influenced by the fraction of inspired oxygen (Fi02) or by PEEP.

Educational Objective: The intracranial. compartment is a fixed, rigid space, and any lesions that occupy space in it may displace normal brain parenchyma, causing midline shift and mass effect. Therapeutic hyperventilation and resulting hypocapnia cause cerebral
vasoconstriction, which results in decreased cerebral blood flow and intracranial pressure.

r
Previous
r
Next
,
Score Report

Lab Values
F
Calcul�lp
r trt
Pause
Exam Section 1: Item 19 of 50 National Board pf M11dlca l Examiners®
Comprehensive isas1c Science Serr-Assessment

✓ 19. A 52-year-old woman with a long-standing history of generalized anxiety disorder comes to the physician for a follow-up examination. She has been taking an anxiolytic drug for 12 years, but she is not currently under a psychiatrist's care because she says
she is feeling better and that she prefers this physician to psychiatrists. She has hyperlipidemia, hypertension, and mild type 2 diabetes mellitus treated with pharmacotherapy. Following physical examination and laboratory studies, the physician makes
adjustments to the patient's medication regimen. Two days later, the physician receives 12 calls from the patient for clarification of treatment. recommendations; this pattern has occurred frequently in the past. In particular, the patient fixates om the potential
toxicities of her medications. She often asks questions that have specifically been answered during the recent office visit. Which of the following is the best approach to manage this patient's needs?

A) Advise the patient that constant questioning of recommendations suggests a lack of trust, and that she should seek care from a provider in whom she has confidence
B) Advise staff to ignore the patient's calls if they are too frequent
C) Contact one of the patient's family members to engage his or her help in enforcing more responsible use of the physician's time
D) Insist that the patient obtain routine psychiatric care
E) Set conditions and establish limits tor phone calls, and set up a regular schedule of office visits

Correct Answer: E.

The physician should manage this anxious patient's frequent phone calls by establishing limits and setting a regular appointment schedule. The physician-patient relationship has professional boundaries (also known as a therapeutic frame) that both parties should
respect. These boundaries include specific meeting times and relationship expectations. For instance, a physician should abstain from dominating the conversation with details about their personal life since their role is to be attentive and therapeutic to the patient.
When a patient transgresses these boundaries, such as making many phone calls outside of scheduled appointments, both parties will benefit from reestablishment of expectations. Many patients who are anxious are destabilized by poor interpersonal boundaries
and will respond well to structure.

Incorrect Answers: A, B, C, and D.

Referring the patient elsewhere (Choices A and D) without attempting to problem-solve with the patient would be premature. Boundary transgressions occur regularly in physician-patient relationships and should not warrant immediate referral, especially if the
transgressions are related to an anxiety disorder the physician is managing. Due to this patient's acute anxiety and attachment to this physician, te rminating the relationship would likely lead to significant distress.

Using other people to mediate the interpersonal relationship (Choices B and C) would prevent the direct communication and boundary setting that is necessary in this physician-patient relationship. One therapeutic role of the physician is to demonstrate that they
can handle challenges and difficult feelings that arise, which can assuage the patient's anxiety.

Educational Objective: The physician-patient relationship benefits from boundaries. When a patient transgresses these boundaries, the physician should utilize direct communication to reestablish expectations.

r
Previous
r
Next
, � p r, "'
Score Rep_ ort_ ____________________________________________ L_ab_Val_
_ ue_s_C_a_lc_u_ la_to_r_____H_elp Paus e
Exam Section 1: Item 20 of 50 National Boa.rd pf M11dlcal Examiners®
Comprehensive t1a&1c science serr-A1111eument

✓ 20. A healthy 35-year-old woman who wishes to become a regular platelet donor comes to a blood donation center. Blood is obtained by venous phlebotomy into sodium citrate and processed to yield platelet-rich plasma (PRP) for platelet function studies.
She has no personal or family history of bleeding disorders. Aliquots of the PRP are subsequently analyzed using a light-transmittance platelet aggregometer. Adding a 5 µM solution of which of the following reagents to the PRP is most likely to cause a
rapid and irreversible aggregation pattern in these studies?

A ) Adenosine diphosphate
B ) Collagen monomer
C) Immune globulin
D) Norepinephrine
la} Prostasyclln (PGI?;)
Correct Answer: A.

Adenosine diphosphate (ADP) addition to PRP will result in rapid and irreversible aggregation. ADP is critical in normal hemostasis. It acts via the receptors P2Y 1 and P2Y12 on the surface of platelets. Binding to these receptors results in G-protein coupled receptor
signaling, with consequent alteration of platelet shape and activation of GP lib/Illa via the Pl3K signaling pathway, which results in, rapid platelet aggregation. The P2Y12 receptor is the target of clopidogrel, an anti-platelet medication used commonly for the treatmen1
of acute coronary syndrome following percutaneous coronary intervention.

Incorrect Answers: B, C, D, and E.

Collagen monomers (Choice B) are essential building blocks of collagen superstructures that function to help adhere platelets at the site of clots, but they do not have a role in the process of rapid and irreversible platelet aggregation.

Immune globulin (Choice C) is given in intravenous form (IVIG) for various indications. It has been shown to cause platelet aggregation and thrombosis via the interaction of platelets with the Fe portion of lgG, but this reaction is considered a potential side effect of
!VIG and does not occur reliably.

Norepinephrine (Choice D) may induce platelet aggregation by binding to alpha-adrenergic receptors on the surface of platelets with a subsequent increased risk of microthrombi formation, but this is not a reliable effect.

Prostacyclin (PGl2) (Choice E) is produced through the cyclooxygenase pathway. It inhibits the activation and aggregation of platelets and it plays a role in vasodilation. Epoprostenol is a synthetic prostacyclin that can be used in the treatment of pulmonary arterial
hypertension.

Educational Objective: ADP rapidly and irreversibly causes platelet aggregation through its action on the P2Y 1 and P2Y12 receptors.

r
Previous
r
Next

Score Report
fr!i
Lab Values
P
Calculator

Help
tr'
Pause
Exam Section 1: Item 21 of SO National Board pf Medical Examiners®
Comprehensive aas1c Science Serr-Assessment

21. A 35-year-old woman, gravida 3, para 2, develops a massive hemorrihage after the vaginal delivery of a healthy female newborn at 38 weeks' gestation. She underwent a cesarean delivery 2 years ago because of cephalopelvic disproportion. A
hysterectomy is required at this time to control the bleeding. The gross and microscopic appearances of the uterus are shown. Which of the following is the most likely cause of the postpartum hemorrhage in this patient?

A ) Abruptio placentae
B ) Ectopic pregnancy
C) Endometriosis
D) Leiomyomata uteri
E ) Placenta accreta
F ) Placental s.ite trophoblastic tumor
Correct Answer: E.
Placenta accreta refers to the abnormal attachment of the placenta to the myometrium, rather than just the endometrium. This can be seen in the gross specimen and histology presented in this case. It most commonly presents with postpartum hemorrhage due to
retained products of conception but is also commonly recognized on routine prenatal ultrasound examination. Risk factors include prior cesarean delivery or uterine surgery, increased maternal age, placenta previa, and multiparity. Management of known placenta
accreta consists of elective cesarean delivery as ear1y as 34 weeks, potentially followed by a hysterectomy depending on the extent of the accreta and retained products of conception.
Incorrect answers: A, B, C, D, and F.
Abruptio placentae (Choice A) presents with vaginal bleeding, severe uterine pain, and tetanic contractions, typically in the third trimester prior to delivery. It commonly results from lower abdominal trauma, smoking, or cocaine use. It is a less likely cause of
postpartum hemorrhage in this patient with prior uterine instrumentation.
Ectopic pregnancy (Choice B) occurs when the fertilized embryo implants in a location outside of the uterine cavity. Rupture of an ectopic pregnancy can cause vaginal bleeding and abdominal pain. An ectopic pregnancy is an unlikely cause of postpartum
hemorrhage in this patient, as she delivered a healthy term baby.
Endometriosis (Choice C) is the development of endometrial tissue outside• of the uterine cavity. It can cause abnormal vaginal bleeding, pelvic pain, dyspareunia, and pain with defecation (dyschezia), but does not cause postpartum hemorrhage.
Leiomyomata uteri (fibroids) (Choice D) are benign tumors of the uterus, which can present with abnormal uterine bleeding and pelvic pain. However, they are more common in older women and are associated with an irregular1y shaped uterus on physical
examination. They do not typically cause postpartum hemorrhage.
Placental site trophoblastic tumor (Choice F ) is a form of gestational trophoblastic disease, which also includes invasive moles and choriocarcinoma. It may develop following a pregnancy, abortion, or molar pregnancy. It is not a typical cause of acute postpartum
hemorrhage.
Educational Objective: Placenta accreta is the abnormal attachment of the placenta directly to the myometrium, which most commonly presents with retained products of conception and postpartum hemorrhage. Risk factors include prior uterine instrumentation
(including cesarean delivery), increased maternal age, placenta previa, and multiparity. Given the high risk of morbidity and mortality, elective surgical delivery at 34 to 36 weeks is recommended.

r
Previous
r
Next
,-
Score Report

Lab Values

Calculator

Help
tft
Pause
- -

Exam Section 1: Item 22 of 50 National Board pf Medical Examiners®


Comprehensive 1:1as1c Science Self-Assessment

✓ 22. An 18-month-old boy in West Africa with perinatally acquired HIV infection is exposed to a child in his village with a fever and an erythematous, maculopapular rash. Three weeks later, the 18-month-old boy develops giant cell pneumonia, but there is no
evidence of a rash. Which of the following viruses is the most likely cause of the pneumonia?

A ) Coxsackievirus
B ) Influenza virus
C) Measles virus
D) Rubella virus
E ) West Nile virus
Correct Answer: C.

Measles (rubeola) virus is a highly contagious paramyxovirus that causes the disease measles, which is associated with acute febrile illness and potentially severe sequelae. It is more common in children than adults. It characteristically presents with prodromal
fever, cough, coryza, conjunctivitis, and a confluent maculopapular rash that starts at the head/111eck and spreads to the trunk, excluding the palms/soles. Physical examination may reveal bright red macules with a bluish-white center on the buccal mucosa and
lymphadenopathy. The virus replicates in epithelial cells of the respiratory tract and lymph node ,s causing lymphadenitis. Virulence factors include the F (fusion) protein which aids in viral fusion to host cells and can result in the formation of multi nucleated giant cells.
Hemagglutinin (HA) protein is involved in viral binding to host cells. Treatment is supportive. Vitamin A deficiency is associated with severe disease and complications, and supplementation should be provided, especially in children who are malnourished.
Complications include subacute sclerosing panencephalitis (a neurodegenerative disease associated with intellectual regression, personality changes, motor deterioration, seizures, and premature death), encephalitis, and giant cell pneumonia. Giant cell
pneumonia is rare, except in patients who are immunosuppressed. It is often fatal and characterized by the presence of inclusion bodies and multinucleated giant cells in the respiratory epithelium.

Incorrect Answers: A, B, D, and E.

Coxsackievirus (Choice A) is an RNA enterovirus which is transmitted via oral secretions or feces. Coxsackievirus type A causes hand, foot, and mouth disease, and herpangina. Coxsackievirus type B may cause myocarditis and pericarditis.

Influenza virus (Choice B) is an orthomyxovirus which is transmitted via respiratory droplets. Symptoms include high fever, chills, myalgias, malaise, headaches, nausea, and vomiting. Pneumonia may develop. Influenza types A and B are responsible for seasonal
pandemics. Complications include secondary bacterial infection of the respiratory tract or lungs, and myocarditis.

Rubella virus (Choice D) is a togavirus transmitted via respiratory secretions that causes the disease rubella. Symptoms include fever, arthralgias, and lymphadenopathy (most often postauricular) with a confluent rash that spreads from the head to the trunk and
then centrifugally to the extremities. Complications include vertical transmission to the fetus in women who are pregnant (resulting in congenital rubella syndrome), thrombocytopenic purpura, and Guillain-Barre syndrome.

West Nile virus (Choice E) is a flavivirus transmitted via the bite of an infected mosquito and may cause West Nile fever with progression to meningoencephalitis. West Nile fever presents with headache, myalgias, nausea, vomiting, and a non-specific
maculopapular rash distributed symmetrically on the extremities and trunk. Meningoencephalitis is associated with acute flaccid paralysis, myoclonus, ataxia, and seizures. Complications include rhabdomyolysis and Guillain-Barre syndrome.

Educational Objective: Giant cell pneumonia is a severe complication of measles that is rare in patients who are not immunosuppressed. Histology reveals inclusion bodies and multinucleated giant cells in the respiratory epithelium.

r
Previous
,.
Next

Score Report
frE
Lab Values
P
Calculator

Help

Pause .
Exam Section 1: Item 23 of SO National Board pf M11dlcal Exftllllners®
Comprehensive 1:1a111c: science Serr-Assessment


� Q)
Q) -

Ei
.0 Q.

::) Q.
z
Normal AB Type 2 Diabetes MeNitus
Fasting Serum Glucose

23. During an investigational study, fasting serum glucose concentrations are collected from a population of healthy individuals and a population of patients with type 2 diabetes mellitus. The results are shown in the graph. Switching from cut point A to cut
point B would most likely result in which of the following changes in test characteristics regarding identification of patients with type 2 diabetes mellitus?

A ) Decreased specificity
B ) Improved sensitivity
C) Increase in false-negatives
D ) Increase in, false-positives
E ) No change
Correct Answer: C.

Any population of patients will include those who have the disease and those who do not, and overlap may exist between these groups when considering testing for nonbinary diagnoses. For example, type 2 diabetes mellitus is understood to be an insensitivity of
peripheral tissues to insulin, marked by diminished insulin uptake and increased blood glucose. It is impossible to specifically measure the degree to which cellular receptors are resistant to insulin, therefore, altematjve diagnostic criteria are used reflecting
increased blood glucose and its effects. As of 2020, the American Diabetes Association criteria include two fasting serum blood glucose levels greater than or equal to 126 mg/dL, a random serum blood glucose level greater than or equal to 200 mg/dl with signs
and symptoms of diabetes, a hemoglobin-A1c level of greater than or equal to 6.5%, or a 2-hour oral glucose tolerance test with increased blood sugar beyond 200 mg/dL. Not all patients with a fasting serum blood glucose greater than or equal to 126 mg/dL will
have diabetes mellitus. For example, confounding factors such as chronic steroid use may result in increased blood glucose without impaired cellular uptake. These patients may receive positive tests, but do not have the underlying condition. Therefore, the set
points for diagnostic tests will result in some true positives (patients who have the disease and who the tests identifies as having the disease), true negatives (patients who do not have the disease who the test identifies as not having the disease), false positives
(patients who do not have the disease who the test identifies as having the disease), and fal.se negatives (patients who have the disease who the test identifies as not having the disease). The set point will determine the fraction of patients who are true or false
positive and true or false negative and should be chosen to optimize the sensitivity and specificity of the test. Point A would result. in the majority of persons with type 2 diabetes mellitus being identified as positive; therefore, the number of false negatives would be
low, and the number of true positives would be high. Moving the cut point from point A to point B (raising the threshold) would result in fewer people with the disease being identified, thereby increasing the number of false negatives and reducing the number of false
positives, which would decrease sensitivity and increase specificity.

Incorrect Answers: A, B, D, and E.

Decreased specificity (Choice A) and improved sensitivity (Choice B) would result from an increase in false positives (Choice D) and a decrease in false negatives, which would occur if the cut point were moved from B to A. Moving the cut point from A to B would
decrease the number of false positives and increase the number of false negatives. Specificity is calculated as true negatives / (true negatives + false positives). Sensitivity is calculated as true positives / (true positives + false negatives).

No change (Choice E) is incorrect as adjusting the cut point alters the fraction of true and false positives and negatives, which affects the sensitivity and specificity of the test.

Educational Objective: Raising the threshold for a positive test to diagnose patients with a disease decreases sensitivity and increases specificity of the test by increasing the number of false negatives and decreasing the number of false positives.

r
Previous
r
Next
,,,
Score Re ort

Lab Values
r-·
Calculator

Hel
"'
Pause
Exam Section 1: Item 24 of 50 National Board pf M11dlcal Examiners®
Comprehensive isaslc Science Serr-ASsessment

✓ 24. A 30-year-old man develops urinary incontinence 2 weeks after success·ful treatment of a fracture of the left pelvis that was sustained at work. He was pinned against a loading dock by a truck moving in reverse. Physical examination shows a distended
bladder. Cystometrography shows absence of micturition reflexes. After the bladder fills to capacity, overflow of urine occurs through the urethra a few drops at a time. This patient most likely sustained additional injury to which of the following during his
initial accident?

A)- Exlemal uriRary spl=liRcler


B ) Hypogastric nerve
C) Pelvic nerves
0)- PudeRdal R8P.186
Ii)- Skelelal motor fibers
Correct Answer: C.
Overflow incontinence is characterized by chronic urinary retention and a chronically distended bladder. When intravesical pressure exceeds outlet resistance, incontinence results as urine flows. It may be caused by physical barriers such as tumors or urethral
stricture, neurologic lesions, or as a side effect of some medications (eg, anticholinergics). After pelvic trauma, the pelvic nerve plexus can be damaged. Pelvic parasympathetic nerves in the pelvic plexus function to excite and contract the detrusor muscle of the
bladder via muscarinic acetylcholine receptors while sympathetic nerves mediate relaxation of the internal urethral sphincter via a1-adrenergic receptors, leading to normal urination. Damage to the pelvic nerve plexus can affect both bladder contraction and urethral
sphincter relaxation, leading to overflow incontinence. Overflow incontinence is treated with managing inciting conditions, timed voiding, or placement of a urethral catheter.
Incorrect Answers: A, B, D, and E.
The external urinary sphincter (Choice A) is innervated by the pudenda! nerve (Choice D). The pudenda! nerve supplies sensory neurons to the external genitalia along with somatic skeletal motor fibers (Choice E) to pelvic muscles, including the external urethral
sphincter. The pudenda! nerve does not carry any parasympathetic fibers. Motor fibers in the pudenda! nerve act on the external urethral sphincter via acetylcholine release on nicotinic receptors, causing contraction and urinary retention. Damage to these nerves
would result in loss of voluntary control over voiding.
The hypogastric nerve (Choice B) contains sympathetic fibers that release norepinephrine and result in relaxation of the detrusor muscle and contraction of the urethral smooth muscle to prevent voiding. Damage to these fibers would lead to total incontinence.
Educational Objective: Pelvic parasympathe1ic nerves in the pelvic nerve plexus function to excite and contract the bladder via muscarinic acetylcholine receptors while sympathetic nerves mediate relaxation of the internal urethral sphincter via a1-adrenergic
receptors, leading to normal urination. Damage to the pelvic nerve plexus can affect both bladder contraction and internal urethral sphincter relaxation, leading to overflow incontine:nce.

r
Previous
r
Next
,
Score Re ort

Lab Values
p
Calculator
r,
Hel
"'
Pause
Exam Section 1: Item 25 of SO National Board pf Medical Examiners®
Comprehensive aas1c Science Serr-Assessment

✓ 25. A 1-year-old boy is brought to the physician by his parents because of a 4-week history of progressive generalized tonic-clonic seizures and a strange odor to his urine. He has a history of delayed development. He was adopted from an orphanage in
Russia at the age of 6 months. Physical examination shows fair skin and blond hair. His phenylalanine hydroxy1ase gene is homozygous for a point mutation (GT->AT) in intron 12 of the affected gene that causes skipping of exon 12. Which of the following
is the most likely explanation for exon skipping in this patient's affected gene?

A ) Alternative polyadenylation site


B) Deletion of 5' untranslated region
C) Expansion of trinucleotide repeat
0) Gene duplication
E ) Nonhomologous recombination
F ) Nonsense mutation
G) RNA splice error
H) X inactivation
Correct Answer: G.

After DNA is transcribed into pre-messenger RNA (pre-mRNA), processing occurs to form the finished mRNA product. One element of this processing is the splicing out of intrans, non-coding portions of the genetic material which do not contribute to the final
protein, leaving behind only exons. From a single pre-mRNA, several unique proteins may be made if splicing variations occur. Correct splicing is dependent on the spliceosome recognizing a splice site at the start of an intron, which typically consists of base pairs
G-U-A-A-G. A point mutation in DNA changing a guanine to an adenine, as in this case, may mask a splice site and either cause an intron to remain in the final mRNA product erroneously or cause a larger region of pre-mRNA to be spliced, leading to the removal of
an exon. In this case, incorrect removal of exon 12 has created a defective version of phenylalanine hydroxylase and led to the condition phenylketonuria. Patients with phenylketonuria do not have the ability to convert phenylalanine to tyrosine. Accumulation of
phenylalanine leads to physical manifestations including a musty odor, impaired neurologic development, and seizures.

Incorrect Answers: A, B, C, D, E, F, and H.

Polyadeny1ation of the 3' end and capping of the 5' end with non-translated nucleotides are two additional processes completed during pre-mRNA processing. An alternative polyadenylation site (Choice A) would lead to an addition of numerous adenine residues.
Deletion of the 5' untranslated region (Choice B) would remove a series of nucleotides at the 5' end of the mRNA. Neither would cause the substitution at a splice site seen in this case.

Expansion of a trinucleotide repeat (Choice C) occurs when a series of three nucleotides in repetition is replicated incorrecUy and too many repeats are included in the replicated product. Conditions caused by trinucleotide repeat expansions include fragile X
syndrome, Huntington disease, Friedrich ataxia, and myotonic dystrophy.

Gene duplication (Choice D) occurs when a region of DNA containing a gene is erroneously duplicated. The nucleotide sequence of the gene remains intact, but there are now two copies of the same gene on the same chromosome.

Nonhomologous recombination (Choice E) is a repair mechanism by which double-stranded ONA breaks are repaired. The break is repaired without a need for a homologous strand to be used as a template, giving it the name nonhomologous recombination. This
process is error-prone and tends to add or delete genetic information rather than causing a single nucleotide substitution, as in this case.

A nonsense mutation (Choice F) occurs when a nucleotide is substituted in the replication or transcription process leading to an early stop codon to form. This leads to a truncated protein, which is usually nonfunctional. In this case, a single base pair substitution led
to the masking of a splice site rather than an early stop codon.

In females with two X chromosomes, X inactivation (Choice H) occurs randomly in each cell early in development. One of the two X chromosomes is chosen to be epigenetically silenced in that cell and its progeny. In an individual heterozygous for a mutated allele
on the X chromosome, the cell lines which randomly inactivate the X chromosome containing the wildtype allele will then express the dysfunctional protein. This is a mechanism of mosaicism in females.

Educational Objective: Before a strand of mRNA is mature, splicing of the intrans must occur, which requires specific splice sites to be recognized by the spliceosome. If a splice site is masked, an intron may be inappropriately retained, or an exon may be
erroneously removed.

r
Previous
r
Next
,-
Score Report

Lab Values

Calculator

Help
tft
Pause
Exam Section 1: Item 26 of SO National Board pf Medical Examiners®
Comprehensive Basic Science Serr-Assessment

✓ 26. A health care screening for diabetes mellitus is conducted at a weekend market with subjects including both men and women ranging in age from 25 to 95 years. Blood samples are obtained, and the blood glucose concentrations are determined. The
results are shown in the table:
Patient Glucose (mg/dl)
1 70
2 120
3 120
4 140
5 110
6 95
7 75
8 145
9 105
10 85
11 85
12 90
13 120
14 100
15 190

Which of the following best represents the median blood glucose concentration for this group (in mg/dL)?

A ) 95
B 105
C) 110
D) 120
E ) 125
Correct Answer: B.

Measures of central tendency within distributions of data include the mean, median, and mode. The median is the middle value of listed data points ordered from least to greatest. If the dataset contains an even number of values, the median is considered the mean
of the middle two values. The mode is the most frequently occurring value within a dataset. The mean is calculated by the sum of the values, divided by the total number of values, and is the most affected by the presence of extreme or outlier data points. In this
case, ordering the fifteen values of patient blood glucose (in mg/dL) in order yields the series 70, 75, 85, 85, 90, 95, 100, 105, 110, 120, 120, 120, 140, 145, 190. The middle value in the set is 105 mg/dL, which is the median of this dataset.

Incorrect Answers: A, C, D, and E.

95 mg/dL (Choice A) and 125 mg/dL (Choice E), are neither the mean, median, nor mode of the dataset.

110 mg/dl (Choice C), is the mean of the dataset and is computed by the sum bf all data points divided by the total number of data points. The sum is equal to 1,650 mg/dl. Dividing by 15 total data points yields a mean of 110 mg/dl.
120 mg/dL (Choice D), is the most frequently occurring value in the set, and is therefore the mode.

Educational Objective: Measures of central tendency within distributions of data include the mean, median, and mode. The median is the middle value of listed data points ordered from least to greatest. If the dataset contains an even number of values, the median
is considered the mean of the middle two values.

r
Previous
r
Next
,-
Score Report
fc'.i
Lab Values
F
Calculator

Hel
tft
Pause
Exam Section 1: Item 27 of SO National Board of M11dlcal Exilllllners®
Comprehensive Basic Science Serr-Assessment

✓ 27. A 32-year-old woman comes to the physician 1 day after she noticed a large red lesion on her back. One week ago, she returned from vacation in Cape Cod. After evaluation, a diagnosis of erythema chronicum migrans is made. Prior to prescribing an
antibiotic, the physician asks the patient if she could possibly be pregnant because an adverse effect of the antibiotic active against the causal organism is incorporation into fetal bones and teeth, causing yellow discoloration. The most likely mechanism of
this dnug is inhibition of which of the following?

--- A) Attachment of tRNA to ribosomes


el} CiJ:iydrofolate reductase
G} DNl>, repair by Dl>lA !)yrase
O} ElestrOA traASpGrt
E) Transpeptidation of peptidoglycan
Correct Answer: A.

Lyme disease is a tick-borne disease caused by Borrelia burgdorferi endemic in the Northeastern United States. In its early stages, it causes erythema chronicum migrans, which is an expanding rash that may have a central ring-like clearing. It can also be
associated with a flu-like illness. In the early stages of Lyme disease, it can be treated with doxycycline, which acts by preventing the attachment of aminoacyl-tRNA to ribosomes. Side effects of tetracycline antibiotics include gastrointestinal distress,
photosensitivity, and in children, discoloration of teeth and inhibition of bone growth, which makes it contraindicated in pregnancy. Later stages of Lyme disease can be characterized by carditis, atrioventricular heart block, facial nerve palsy (which is often bilateral),
migratory arthralgias, and if left untreated, encephalopathy and chronic arthritis. Later stages of Lyme disease that affect the heart or central nervous system are generally treated with parenteral ceftriaxone.

Incorrect Answers: B, C, D, and E.

Dihydrofolate reductase (Choice B) is an enzyme used in the synthesis of purines and thymidine, precursors of nucleic acids. Inhibition of dihydrofolate reductase by trimethoprim prevents folate synthesis and is bacteriostatic. It is avoided in the first trimester of
pregnancy but has no role in treating Lyme disease.

DNA gyrase (Choice C) functions in DNA synthesis and repair by relaxing supercoils during replication and transcription. Fluoroquinolones inhibit prokaryolic DNA gyrase (also known as topoisomerase), side effects of which include cartilage damage, tendonitis, or
tendon rupture.

Inhibitors of electron transport (Choice D) such as antimycin A inhibit the proton gradient and prevent normal synthesis of ATP. Doxycycline does not inhibit electron transport.

Transpeptidation of peptidoglycan (Choice E) provides strength to the cell wall. Monobactams inhibit transpeptidation by inhibiting penicillin-binding protein 3. They have no effect on Borrelia burgdorferi, a spirochete.

Educational Objective: Lyme disease is a tick-borne disease caused by Borrelia burgdorferi and characteristically causes erythema chronicum migrans, an expanding rash that may have a central ring-like clearing. Early Lyme disease is treated with doxycycline,
which prevents the attachment of aminoacyl-tRNA to ribosomes. Doxycycline can cause gastrointestinal distress, photosensitivity, and in children, discoloration of teeth and inhibition of bone growth, which makes it contraindicated in pregnancy.

r
Previous
r
Next
,
Score Report

Lab Values
F
Calcul�lp
r trt
Pause
Exam Section 1: Item 28 of SO National Board pf M11dlcal Exilllllners®
Comprehensive Basic Science Serr-Assessment

✓ 28. A female newborn is delivered vaginally at 37 weeks' gestation after an uncomplicated pregnancy, labor, and delivery. Physical examination shows a narrow cranium that is elongated in the anteroposterior dimension. Premature closure of which of the
following during embryogenesis is the most likely cause of this anomaly?

A ) Bregma
B ) Coronal suture
C) Lambda
D) Lambdoid suture
E ) Sagittal suture

Correct Answer: E.

Craniosynostosis refers to the premature ossification that occurs between the plates of the cranial bones leading to deformity of the skull. Most infants with this condition are asymptomatic. Genetic etiologies may involve abnormalities in expression of FGF and
TGF. Nonnally, the skull is composed of five independent bones. These include the two frontal bones, two parietal bones, and the occipital bone. The fontanelles are cartilaginous gaps that exist between these bony plates that, along with the cartilaginous sutures,
allow for movement, growth, and compression of the cranial bones particularly during childbirth when the skull passes through the pelvis and vaginal canal. Closure of one or more of these regions prematurely leads to abnormal growth and fusion of the skull. The
resulting deformity can be both cosmetic (abnormal head shape) and pathologic (limiting normal central nervous system growth). A number of patterns of deformity occur depending on which sutures prematurely fuse. In this patient, fusion of the sagittal suture
arrests the medial-lateral growth of the skull. This leads to continued anterior-posterior growth in disproportion to medial-lateral growth. A narrow, elongated anteroposterior cranium results, known as scaphocephaly.

Incorrect Answers: A, B, C, and D.

The bregma (Choice A) is the point at which the sagittal suture and the coronal suture meet. It is the anterior counterpart to the lambda.

Premature closure of the coronal suture (Choice B) leads to brachycephaly, which is a short, broad, flattened skull. This results in arrest of the anterior-posterior growth of the skull.

Lambda (Choice C) is the landmark where the sagittal suture intersects the lambdoid sutures, that is, the point at which the two parietal bones meet the occipital bone.

A unilateral lambdoid suture (Choice D) craniosynostosis will lead to a twisted appearance of the skull due to asymmetric growth.

Educational Objective: Craniosynostosis results from the premature fusion of cranial bone sutures, leading to asymmetric growth in the areas of remaining, unfused sutures. Skull shape deformity results, which is both cosmetic and physiologically problematic due tc
constriction of the central nervous system.

r
Previous
,,.
Next
,.
Score Report

Lab Values Ca�p
P � ,,.
Pause
Exam Section 1: Item 29 of SO National Board pf Medical Examiners®
Comprehensive aas1c Science Serr-Assessment

✓ 29. A 40-year-old woman comes to the physician for a follow-up examination 1 week after having a second increased blood pressure measurement. Her pulse is 72/min, and blood pressure is 168/104 mm Hg in both arms while sitting. Physical examination
shows no other abnormalities. Treatment with lisinopril and hydrochlorothiazide is initiated. One month later, her blood pressure is 120/80 mm Hg. Which of the following sets of hormonal changes is most likely in this patient?

Angiotensin I Angiotensin II Bradykinin


A� t t no change
B) t ! t
Gt- t t t
D) ! t t
Ii� t t ¼
Correct Answer: B.

Thiazide diuretics, such as hydrochlorothiazide (HCTZ), and ACE inhibitors, such as lisinopril, are first-line therapies for the treatment of hypertension. ACE inhibitors target the renin-angiotensin-aldosterone system which is one of the primary regulators of blood
pressure. The system is activated by the release of renin from the renal juxtaglomerular apparatus in response to low-flow states in the afferent arterioles or through the distal convoluted tubule (decreased afferent arteriole pressure and decreased delivery of
Na+ and Cl- ions to the macula densa both stimulate renin release). Renin acts on angiotensinogen via proteolytic cleavage to generate angiotensin I. Angiotensin I is then converted to angiotensin II by ACE, which is found in vascular endothelium (particularly
in the lungs). Angiotensin II has several important effects, including potent vasoconstriction leading to increased systemic vascular resistance, increased Na+ reabsorption in the renal tubules, aldosterone release by the adrenal cortex leading to increased Na+
and fluid retention, vasopressin release from the posterior pituitary leading to increased fluid retention, enhanced adrenergic function through inhibition of norepinephrine reuptake in sympathetic nerve endings, and stimulation of hypothalamic thirst centers. All of
these effects serve to increase blood pressure. ACE inhibitors decrease blood pressure by blocking the conversion of angiotensin I to angiotensin II. This results in increased angiotensin I, decreased angiotensin II, and increased bradykinin levels, as ACE is also
responsible for the degradation of bradykinin (a potent vasodilator). Thiazide diuretics act on the distal convoluted tubule to decrease Na+ and Cl- reabsorption and do not directly affect the renin-angiotensin-aldosterone pathway.

Incorrect Answers: A, C, D, and E.

Choices A and C are incorrect as inhibition of ACE affects the enzymatic degradation of bradykinin. Bradykinin levels are increased with use of ACE inhibitors. In addition to vasodilation, bradykinin is also a protussive agent, and persistent cough is one of the
main reasons ACE inhibitors are discontinued. Angioedema is also a common occurrence with ACE inhibitors due to the effects of bradykinin and can be life-threatening when the airway is involved.

Choices D and E are incorrect as levels of the ACE substrate angiotensin I increases when the enzyme is inhibited, while levels of the enzymatic product angiotensin II decrease.

Educational Objective: ACE inhibitors block the conversion of angiotensin I to angiotensin 11, with the downstream effect of decreasing blood pressure. ACE is also involved in degrading bradykinin.

r
Previous
r
Next
,-
Score Report

Lab Values

Calculator

Help
tft
Pause
Exam Section 1: Item 30 of SO National Board of M11dlcal Exilllllners®
Comprehensive Basic Science Serr-Assessment

✓ 30. A 55-year-old woman with systemic lupus erythematosus has an increased serum C-reactive protein (CRP) concentration. Which of the following is the most likely source of the increased serum CRP concentration in this patient?

A � Bone maml'N
g �BJ:ai:R
C) Kidney
D) Liver
ii+ LympJ:i ROG8
F ) Skin

Correct Answer: D.

C-reactive protein (CRP) is an acute phase reactant protein that is synthesized in the liver in response to the pro-inflammatory cytokine interieukin-6. It forms an important element of the innate immune response. CRP functions as an opsonin and aids in the
targeting of cells for phagocytosis and in the fixation of complement. Clinically, serum measurement of CRP is used as an indicator of active inflammation. Serum CRP is a non-specific inflammatory marker and can indicate inflammation due to infectious,
rheumatologic, or malignant etiologies.

Incorrect Answers: A, B, C, E, and F.

Bone marrow (Choice A) is the site of hematopoiesis and of the production of immune cells. B lymphocyte maturation occurs in the bone marrow. CRP is not synthesized in the bone marrow.

Brain (Choice B) is an immune-privileged site and does not play a role in the synthesis of acute phase reactants.

Kidney (Choice C) is the site of production of 1,25-dihydroxycholecalciferol, or vitamin D. There is evidence that vitamin D plays a role in regulation of the immune response. CRP is not synthesized in the kidney.

Lymph nodes (Choice E) are the site of mature B and T cells and function as the site of antigen presentation and activation of the acquired immune response.

Skin (Choice F) has a principal function in immunity, in addition to thermoregulation and fluid balance. Antigen-presenting cells within skin identify antigens and present them to other immune cells, which may produce a host of cytokines including interieukin-6, whict
triggers the synthesis of CRP by the liver.

Educational Objective: CRP is an acute phase reactant protein that is synthesized in the liver. CRP functions as an opsonin and aids in the targeting of cells for phagocytosis and in the fixation of complement. Serum CRP serves as a non-specific inflammatory
marker and can be useful in the diagnosis and monitoring of a variety of inflammatory diseases.

r
Previous
r
Next
,
Score Report

Lab Values
F
Calcul�lp
r trt
Pause
Exam Section 1: Item 31 of SO National Board pf M11dlcal Exilllllners®
Comprehensive Basic Science Serr-Assessment

✓ 31. A 45-year-old man comes to the physician because of a 2-month history of a dull pain on the left side of his scrotum. Examination of the left side of the scrotum shows a bluish appearance of the skin and a palpable mass that feels like a bag of worms.
Which of the following veins is most likely obstructed in this patient?

A}- IAferior meseAleric veiA


8}-IAferiorveAa cava
C) Inferior vesical vein
C�IAtemal iliac veiA
- -

E ) Left renal vein

Correct Answer: E.

A varicocele refers to dilated veins of the pampiniform plexus due to elevated venous pressure. Varicoceles are commonly located on the left side due to increased resistance to venous flow as the left gonadal vein drains into the left renal vein. On the right side, the
right gonadal vein drains directly into the inferior vena cava and is less likely to become obstructed. An obstruction of the left renal vein, such as may be caused by local invasion of a renal tumor, causes pressure to mount behind the obstruction which is transmitted
through the left gonadal vein to the pampiniform plexus surrounding the testicle. The dilation of the pampiniform plexus causes a bluish appearance of the overlying skin. Palpation of the dilated plexus demonstrates varicosities, which are often described as having
a vermiform {worm-like) feeling. Unlike a hydrocele, which is caused by failure of the processus vaginalis to close leading to accumulation of fluid around the testicle, a varicocele will not transilluminate with application of light to the scrotum. Infertility is a potential
complication of varicoceles, as the dilated veins result in increased temperature of the testicle, which impairs spermatogenesis. Treatment for varicoceles may require surgical embolization or ligation to manage infertility.

Incorrect Answers: A, B, C, and D.

The inferior mesenteric vein (Choice A) drains the large intestine into the splenic vein, which in turn merges with the superior mesenteric vein to form the hepatic portal vein. Obstruction of the inferior mesenteric vein would cause venous congestion of the large
intestine and superior rectum, not the pampiniform plexus in the scrotum.

The inferior vena cava (Choice B) is the large vessel formed by the merging of the bilateral common iliac veins which carries blood from the abdomen, pelvis, and lower extremities back to the heart. Its wide lumen is less likely to become completely obstructed than
that of a smaller vein. If occlusion did occur, widespread venous congestion of the bilateral lower extremities, pelvic organs, and gastrointestinal tract would occur.

The inferior vesical vein (Choice C) drains blood from the urinary bladder. Its obstruction would not lead to dilation of the pampiniform plexus.

The internal iliac vein (Choice D) and its tributaries drain blood from the pelvic organs and pelvic wall into the common iliac veins. The gonadal veins are not tributaries of the internal iliac vein and thus obstruction would not lead to varicocele formation.

Educational Objective: The testes are drained by the gonadal veins, which drain into the left renal vein on the left side and directly into the inferior vena cava on the right side. Obstruction of or increased pressure within the left gonadal vein or left renal vein leads to
a left-sided varicocele.

r
Previous
r
Next
,-
Score Re ort
� F·
Lab Values Calculator

Hel
tft
Pause
Exam Section 1: Item 32 of SO National Board pf M11dlcal Exilllllners®
Comprehensive Basic Science Serr-Assessment

✓ 32. A 55-year-old man comes to the physician because of a 2-month history of decreased appetite; he also has had a 9-kg (20-lb) weight loss and an intermittent rash during this period. Physical examination shows necrolytic migratory erythema over the
axillae and groin. Laboratory studies show a serum glucose concentration of 280 mg/dl and plasma glucagon concentration of 1500 pg/ml (N=20-100). A CT scan of the abdomen shows a mass at the head-of the pancreas. Which of the following
processes is most likely occurring in this patient's liver?

A ) Decreased activity of carnitine acetyltransferase


2} Glycggen synthesis
� Increased acti�•ity of acetyl Co/I. camox.ylase
D) Inhibition of ketogenesis
E ) Partial oxidation of fatty acids

Correct Answer: E.

Glucagonoma is a rare functional malignancy that secretes glucagon, which raises blood glucose. Occasionally, glucagonoma can present as a component of multiple endocrine neoplasia. Symptoms are nonspecific and include mild diabetes mellitus,
paraneoplastic rash (eg, necrolytic migratory erythema), and weight loss. Glucagon acts on hepatocytes via a cAMP pathway activating protein kinase A that ends in the activation of glycogen phosphorylase, with release of glucose monomers into serum. When a
glucagonoma is present, glucagon is continuously secreted, unlike in the physiologic state. When the glycogen stores are exhausted, the body then relies on the breakdown of fat through beta oxidation of fatty acids to provide energy. Beta oxidation occurs in the
mitochondria of the cell. In each cycle of beta oxidation, two carbon atoms are cleaved to form acetyl-CoA, which leaves the mitochondria via the camitine shuttle. In. addition to a molecule of acetyl-CoA, which then enters the citric acid cycle, a molecule of NADH is
garnered with each cycle. The ability to generate energy from the body's fat stores during a period of starvation is critical to maintaining the function of the vital organs, such as the brain.

Incorrect Answers: A, B, C, and D.

Carnitine acetyltransferase (Choice A) is a transmembrane protein found on the surface of mitochondria in liver, muscle, and brain, and catalyzes the bond between acyl-CoA and camitine, which permits the long-chain fatty acid to translocate into the mitochondrial
matrix where beta-oxidation (breakdown of the fatty acid) occurs. Its activity is increased in the presence of glucagon, not decreased.

Glycogen synthesis (Choice 8) occurs when there is a surplus of glucose in order to store energy for future use. In contrast, a glucagonoma secretes glucagon which signals a state of starvation to the liver, causing glycogen phosphorylase to be activated which
results in the degradation of glycogen stores for maintenance of serum glucose concentration.

Acetyl-CoA carboxylase (Choice C) regulates the metabolism of fatty acids and, when active, adds a carboxy group to acetyl-CoA to make malonyl-CoA. This is a key ingredient in the formation of new fatty acids. If the activity of this enzyme is increased, fatty acids
will be created rather than broken down. This would be seen in the fed state rather than the starvation state mimicked by increased glucagon levels.

Ketogenesis (Choice D) is the process by which ketone production occurs through fatty acid and amino acid degradation. Ketogenesis occurs in the starvation state in order to supply the brain, heart, and skeletal muscle with adequate nutrition. In the starvation
state mimicked by increased glucagon levels, ketogenesis is increased, not inhibited.

Educational Objective: Oxidation of fatty acids in the mitochondrial matrix occurs during a state of starvation, or a state with increased glucagon levels, in order to maintain nutrition when glycogen stores have been depleted.

r
Previous
,,.
Next
,.
Score Report

Lab Values Ca�p
P � ,,.
Pause
Exam Section 1: Item 33 of 50 National Board of Medical Examiners®
Comprehensive Basic Science Self-Assessment

✓ 33. A 45-year-old homeless man is brought to the emergency department by police 30 minutes after he was found unconscious. His breath and clothes smell of alcohol. His temperature is 36.8 °C (98.2 ° F), pulse is 68/min, respirations are 14/min, and blood
pressure is 110/55 mm Hg. Physical examination shows bronzed skin and spider angiomata on the chest. Laboratory studies show:
!Hemoglobin 10 g/dl
IHematocrit 30%
Mean corpuscular volume 110 µm l
Leukocyte count 9000/mm 3
Segmented neutrophils 70%
Lymphocytes 20%
Monocytes 10%
!Platelet count 160,000/mm l
Serum
Ferritin 200 ng/ml
Vitamin B12 (cobalamin) 500 pg/ml (N=160-950)
IRBC folate 20 ng/ml (N=166-640)

A peripheral blood smear shows occasional hypersegmented neutrophils and 3+ oval macrocytes. Serum studies are most likely to show which of the following sets of additional findings?

Methylmalonic Acid Homocysteine


A) Increased increased
Bt- IRGmased RGFmal
Gt- IRGreas::ed deGreased
D) Normal increased
E}- �"'" RO�mal
Ft- II.GrR1al decreased
G) Decreased increased
l=it- DeGreased RGrmal
I t- DeGreased deGreased
Correct Answer: D.

Folic acid, or vitamin B s, is converted in the body to tetrahydrofolic acid which functions as a coenzyme· in the synthesis of nucleic acids. Folate is contained in leafy vegetables and primarily absorbed in the jejunum. Folate supplementation is recommended in
pregnancy to decrease the risk of neural tube defects. It is also commonly warranted in patients with rapid cellular turnover, such as acute leukemia and sickle cell disease, to offset the consumption of folic acid in cell division. Folate deficiency is often seen in
patients with malnutrition, alcoholism, and patients taking anti-folate medications (eg, phenytoin, methotrexate). Megaloblastic anemia occurs in the setting of impaired DNA synthesis, most commonly related to folate or vitamin B 12 (cobalamin) deficiency and is
characterized by erythrocyte macrocytosis and hypersegmented neutrophils. Laboratory evaluation reveals anemia, increased mean cell volume, and normal white cell and platelet indices. The presence of hypersegmented neutrophils on peripheral smear is a
characteristic finding in megaloblastic anemia. Since folate and vitamin B 12 (cobalamin) are needed for the conversion of homocysteine to methionine, deficiency is associated with elevated homocysteine levels. However, vitamin B 12 (cobalamin) also acts as a
cofactor for the conversion of methylmalonyl-CoA to succinyl-CoA, and its deficiency is associated with increased methylmalonic acid levels. Therefore, fol ate deficiency would result in increased homocysteine levels but normal methylmalonic acid levels.

Incorrect Answers: A, B, C, E, F, G, H, and I.

Increased methylmalonic acid levels (Choices A, B, C, and D) would result from vitamin B 12 (cobalamin) deficiency, which would also cause increased homocysteine levels. Vitamin B 12 (cobalamin) deficiency presents with macrocytic, megaloblastic anemia, but
also with neurologic symptoms such as subacute combined degeneration of the spinal cord.

Normal (Choice E) or decreased (Choice F) homocysteine levels in the setting of normal methylmalonic acid levels would not be expected in the setting of folate deficiency, which would typically cause an increased homocysteine level.

Folate deficiency does not cause decreased methylmalonic acid levels (Choices G, H, and I), as it does not play a role in the metabolism of methylmalonic acid.
Educational Objective: Folate deficiency is often seen in patients with malnutrition, alcoholism, or high cell-turnover conditions such as acute leukemia and sickle cell disease. It is associated with megaloblastic anemia, hypersegmented neutrophils, erythrocyte
macrocytosis, and increased serum homocysteine levels with normal methylmalonic acid levels.

r r , tE F � r
Exam Section 1: Item 34 of SO National Board pf Medical Examiners®
Comprehensive Basic Science Self-Assessment

X 34 A 54-year-old woman with a 25-year history of rheumatoid arthritis that is refractory to therapy is admitted to the hospital for a joint replacement operation. Preoperative laboratory studies show:
.
Hemoglobin 10.5 g/dl
Mean corpuscular volume 79 µm3
Leukocyte count 8900/mm J
Platelet count 230,000/mm J
Serum
Iron 40 µg/dl
Transferrin 220 mg/dl (N=20�00)

A photomicrograph of a Prussian blue iron-stained smear of bone marrow aspirate is shown. Which of the following is the most likely cause of the anemia?

A}Autoimmune hemolysis
B ) Chronic disease
------------------------------------------------------------..
C ) Iron deficiency
g MaroN replacement by neoplastic tissue
E} Myelodysplas1a
F} p Thalassemia miRer

Correct Answer: B.

Chronic disease (anemia of chronic disease (ACD]) accounts for this patient's anemia. ACD is found in many chronic inflammatory, infectious, and malignant disease states including rheumatoid arthritis, lymphoma, and tuberculosis. This patient's rheumatoid
arthritis with microcytic anemia, normal iron levels, and bone marrow smear with Prussian blue-iron staining revealing increased iron storage are consistent with this diagnosis. Anemia is often categorized into macrocytic, normocytic, and microcytic etiologies.
Other causes of microcytic anemia include thalassemia, iron deficiency anemia, and sideroblastic anemia. To determine the specific cause of microcytic anemia (which guides therapy), diagnostic evaluation is necessary and includes red blood cell indices (eg, red
cell distribution width, mean cell volume, mean corpuscular hemoglobin concentration) and iron studies (eg, transferrin, serum iron level, total iron binding capacity, ferritin}. In ACD, iron studies will show normal or llow serum iron level, and normal or increased
ferrit in level. While ACD usually causes microcytosis, it can present with normal mean corpuscular volume when early. While not routinely performed, a bone rmarrow aspirate with Prussian blue staining for iron will reveal increased deposits of iron in the bone
marrow, as shown in the photomicrograph. The pathophysiology may relate to upregulated hepcidin, resulting in reduced intestinal absorption of iron and reduced release of iron from macrophages. While patients may not be iron deficient, their iron stores are
unavailable for hematopoiesis, leading to anemia.

Incorrect Answers: A, C, D, E, and F.

Autoimmune hemolysis (Choice A) occurs as a iresult of antibody production against circulating red blood cell surface antigens, leading to phagocytosis or activation of complement, both of which lead to destruction of the red cell in the spleen or intravascular
space. Diagnosis is made by a positive direct antibody test (Coombs). Laboratory analysis will also reveal unconjugated hyperbilirubinemia and low haptoglobin.

Iron deficiency anemia (Choice C) commonly results from occult blood loss through the gastrointestinal tract or via menstruation. Bone marrow biopsy, if performed, would demonstrate an absence of iron on Prussian blue staining. Serum ferritin level would be low.
In men or post-menopausal women with iron deficiency anemia, sites of potential blood loss should be evaluated with endoscopy and colonoscopy to assess for gastrointestinal causes (eg, colorectal carcinoma), along with a pelvic examination in women to
assess for abnormal uterine bleeding.

Marrow replacement by neoplastic tissue (Choice D) would present with systemic signs of malignancy such as fever, we ight loss, and malaise, and cytopenia on laboratory analysis as normal bone marrow is displaced by malignant cells. Bone marrow aspirate
would reveal malignant cells rather than a normal Prussian blue stain.
Myelodysplasia (Choice E) is frequently encountered in older individuals or in those who previously received cytotoxic chemotherapy. Findings on bone marrow aspirate include abnormal appearing cells of multiple lineages. This does not fit with the image shown
or the patient's presentation or marrow biopsy.

�-Thalassemia minor (Choice F) is caused by mutation of one of the two paired J3-globin chains. Patients will have an abnormal hemoglobin electrophoresis but are typically asymptomatic or only mildly symptomatic. Peripheral smear demonstrates microcytosis
and target cells, but iron levels are normal.

Educational Objective: Anemia of chronic disease is a common finding in autoimmune and inflammatoiry diseases, chronic infections, and malignancy. Increased hepcidin levels decrease intestinal iron absorption and increase iron storage in the bone marrow and
reticuloendothelial system, preventing use of iron by erythrocyte precursors, thereby resulting in anemia.

r r , � F � t"'
Previous Next Score Re ort Lab Values Calculator Hel Pause
Exam Section 1: Item 35 of 50 National Board of Medical Examiners®
Comprehensive tsaslc Science Self-Assessment

✓ 35. A newborn is delivered at 40 weeks' gestation to a 20-year-old woman after an uncomplicated pregnancy. Physical examination shows male-like external genitalia, with clitoral hypertrophy and labial fusion. The karyotype is 46,XX. The serum concentration
of cortisol is decreased and serum concentrations of 17-hydroxyprogesterone, androstenedione, and testosterone are increased. Which of the following is the most likely cause of the findings in this newborn?

A}-Andrngen receptGr mulatisn


B ) Defect in adrenal steroid bios nthesis
G}-Deficiency gf enz.yme activity In gsnadal steroid l:lsrmsne bissyntl:lesis
D}-Increased anti mi.illerian l:lsrmsne secretien
E }-Tr:anslscatisn sf the SRYgene Is ene sf theXchmmsssmes
Correct Answer: B.

Congenital adrenal hyperplasia (CAH) is a defect in adrenal steroid biosynthesis, particularly of cortisol, with variable effects on mineralocorticoids and androgens. The most common form of CAH is 21-hydroxylase deficiency, which presents with salt wasting in a
newborn or precocious puberty in a child, as well as virilization of infants with an XX karyotype (eg, clitoral hypertrophy, labial fusion), due to increased levels of androgens, as seen in this patient. Due to the enzyme deficiency, levels of 17-hydroxyprogesterone,
androstenedione, and testosterone are increased, while aldosterone and cortisol are decreased, leading to hypotension and hyper1<alemia. Treatment consists of exogenous glucocorticoid and mineralocorticoid supplementation.
Incorrect Answers: A, C, D, and E.

Androgen receptor mutations (Choice A) would lead to androgen insensitivity in genotypic males, which presents as normal-appearing female external genitalia, decreased or absent body hair, and lack of a uterus and ovaries. Patients will have functioning testes
and testosterone levels will be increased. However, as this biological female is presenting with virilization, androgen receptor mutations are an unlikely diagnosis.

Deficiency of enzyme activity in gonadal steroid hormone biosynthesis (Choice C) would lead to decreased levels of estradiol, progesterone, and testosterone, which is not seen in this patient.

Increased anti-mi.illerian hormone secretion (Choice D) or mi.illerian inhibitory factor would lead to the absence of development of the female internal reproductive organs in this patient but would not cause abnormalities in cortisol levels.

Translocation of the SRY gene to one of the X chromosomes (Choice E) would cause a fetus with an XX karyotype to develop as a biologic male, including external genitalia and testes. Hormone levels would be normal, making this an unlikely diagnosis in this
patient.

Educational Objective: Congenital adrenal hyperplasia results from a defect in adrenal steroid biosynthesis. Its most common form, 21-hydroxylase deficiency, presents with salt-wasting in infants or precocious puberty in children. It also causes the virilization of XX
fetuses, elevated levels of 17-hydroxyprogesterone, hypotension, and hyperkalemia. Treatment includes the exogenous administration of mineralocorticoids and glucocorlicoids.
Exam Section 1: Item 36 of 50 National Boa.rd of Medical Exi!Jlllners®
Comprehensive 1:1as1c Science Serr-Assessment

✓ 36. A 40-year-old womani with chronic paraplegia caused by multiple sclerosis is brought to the physician because of severe, debilitating muscle cramps for the past 3 weeks. Treatment with baclofen resolves her muscle cramps. Which of the following
receptors most likely decreased the muscle spasticity in this patient?

A } az-Adrenoreceptor
B } J3z-Adrenoreceptor
C} y-Aminobutyric acid a receptor
D} Calcium-sensing receptor
E } Ryanodine receptor
Correct Answer: C.

Baclofen is a v-aminobutyrie acid (GABA} analogue that acts as an agonist at GABA-a receptors. GABA-a receptors are metabotropic G-protein coupled receptors that increase efflux of potassium and thus hyperpolarize skeletal muscle cells and decrease action
potential frequency. As such, baclofen is a first-line treatment for muscle spasticity. GABA-a receptors are also found in the central nervous system (CNS}; therefore, baclofen can cause sedation, especially when used in conjunction with other CNS depressants.

Incorrect Answers: A, B, D, and E.

a2-Adrenoreceptors (Choice A) are G-protein coupled receptors localized to the CNS that decrease sympathetic outflow when activated. Examples of a2-adrenergic agonists include clonidine and guanfacine, which treat attention-deficit/hyperactivity disorder
(ADHD) and hypertensive urgency. Tizanidine is an ai-adrenergic agonist that is used as a muscle relaxant.

132-Adrenoreceptors (Choice B} are G-protein coupled receptors involved in smooth muscle relaxation. Albuterol, a bronchodilator used in asthma, is one example of a 132-Adrenergic agonist. 132-Adrenergic agonists do not act on skeletal muscle, though tremors can
be seen as a consequence of increased sympathetic tone.

Calcium-sensing receptors (Choice D} are G-protein coupled receptors located in the renal tubule and parathyroid gland that sense the extracellular concentration of calcium ions. These receptors influence calcium homeostasis by regulating renal calcium
reabsorption and parathyroid hormone release.

Ryanodine receptors (Choice E} are calcium channels located on the sarcoplasmic reticulum membranes in skeletal muscle. These channels mediate the intracellular release of calcium during the excitation-contraction coupling essential to muscle contraction.
Dantrolene antagonizes ryanodine receptors to treat muscle spasticity and malignant hypertherrnia.
Educational Objective: Baclofen is a GABA-8 receptor agonist, which hyperpolarizes muscle ceUs and relieves muscle spasticity.

r r , � r-:- r r
Previous Next Score Re ort Lab Values Calculator Hel Pause
Exam Section 1: Item 37 of 50 National Board pf M41dlcal Examiners®
Comprehensive Basic SClence Self-Assessment

✓ 37. A 20-month-old gin is brought to the physician by her mother because the mother is concerned about her daughter's development and behavior. The girl is able to walk and run and has a vocabulary of at least 100 words. However, when she attends
playdates with other children, she prefers to play by herself and will strike another child on the head if that child attempts to join her in playing with a particular toy. Which of the following best explains this patient's actions during play?

A) Attention-deficit/hyperactivity disorder
B) Conduct disorder
C) Early sign of autistic disorder
D ) Oppositional defiant disorder
E Normal behavior for a e

Correct Answer: E.

This toddler demonstrates nonnal behavior for age with age-appropriate gross motor, language, and social functioning. Children are expected to be able to walk up stairs by age 18 months and run by age 2 years, and the typically developing 18-month-old has a
vocabulary of approximately 100 words. Social skills may include pointing to show others something interesting, playing pretend, and showing affection to familiar others. Cooperative play is not expected to develop until age 4 years, so this child's inclination not to
share her toy is typical. Hitting and aggression are relatively common in toddlers. Typical fine motor skills at this age include stacking several blocks.

Incorrect Answers: A, B, C, and D .

Attention-deficit/hyperactivity disorder (Choice A) is a childhood disorder that presents with hyperactivity, impulsivity, and/or inattention. Though this child impulsively hits others, this behavior is common for her age. This child does not demonstrate hyperactivity or
inattention.

Conduct disorder (Choice B) refers to persistent emotional and behavioral problems such as purposeful aggression toward people and animals in children and adolescents. The behavioral problems can also include lying and serious rule violations. This child's
aggression is borne out of age-appropriate impulsivity rather than intentional bullying.

Early signs of autistic disorder (Choice C) may include deficits in social and language skills, sensory overstimulation, and frequent tantrums. This patient has age-appropriate social and language skills.

Oppositional defiant disorder (Choice D) manifests as a persistent pattern of angry mood, defiant behavior, and vindictiveness that causes distress and impaired functioning in the child or their caretakers. This child's aggression is borne out of age-appropriate
impulsivity rather than defiance.

Educational Objective: Developing 20-monlh old children can typically walk up stairs (and may be able to run), stack several blocks, use 100 or more vocabulary words, and play pretend. Aggressive, impulsive behavior is not uncommon, and cooperative play does
not typically develop until the age of 4 years.

r r , fr§ F r ff'
Previous Next Score Re ort Lab Values Calculator Hel Pause
Exam Section 1: Item 38 of 50 National Board of Medical Examiners®
Comprehensive tsaslc Science Self-Assessment

✓ 38. A 55-year-old woman who is a physician is admitted to the hospital because of a fractured femur sustained in a motor vehicle collision. Two days after admission, she develops tachycardia, restlessness, diaphoresis, and anxiety. She says that ''vague
shapes" are coming out of the walls. Which of the following is the most likely cause of this patient's current condition?

A ) Acute stress disorder


B ) Alcohol withdrawal
G} Qipelar dlserder
C}- Hisll'ienic persenality diserder
E}- Panic diserder
Correct Answer: B.
This patient is likely developing delirium tremens, a serious complication of alcohol withdrawal. Alcohol increases the central nervous system activity of gamma-aminobutyric acid (GABA), the main inhibitory neurotransmitter. Abrupt discontinuation of alcohol
therefore leads to sympathetic excess. Uncomplicated alcohol withdrawal typically presents with tremors, anxiety, restlessness, headache, nausea, and diaphoresis, which can occur hours after the cessation of alcohol use. Delirium tremens, the most concerning
and life-threatening complication of alcohol withdrawal, involves severe confusion and disorientation, fluctuations in consciousness, agitation, visual hallucinations, and autonomic instability (fluctuations in heart rate and blood pressure with hypertherrnia). As in this
patient, delirium tremens can begin 2 to 5 days after the patient's last alcoholic drink. Patients with delirium tremens may die from associated dysrhythmias, seizures, or respiratory failure. Delirium tremens is managed supportively with benzodiazepines to address
agitation and prevent the symptoms of withdrawal, fluid repletion, nutritional supplementation, and frequent assessment, including vital sign checks. Other complications of alcohol withdrawal include alcoholic hallucinosis (auditory or visual hallucinations without
confusion or autonomic instability).

Incorrect Answers: A, C, D, and E.

Acute stress disorder (Choice A) refers to symptoms of intrusive thoughts, hyperarousal, dissociation, avoidance, and/or depressed mood within 1 month of a traumatic event such as a life-threatening motor vehicle collision. This patient's tachycardia and
diaphoresis could represent symptoms of hyperarousal, and hallucinations do occasionally occur during trauma-related dissociative experiences. However, hallucinations occur less commonly in acute stress disorder than in delirium tremens, and the acute onset of
this patient's symptoms 48 hours after hospital admission is concerning for delirium tremens.

Bipolar disorder (Choice C) is characterized by discrete episodes of depression and mania, which can sometimes be accompanied by psychotic symptoms such as hallucinations and delusions. Tachycardia and diaphoresis would be uncommon, and primary
psychiatric disorders are more frequently associated with auditory, rather than visual, hallucinations.

Histrionic personality disorder (Choice D), a cluster B personality disorder, is characterized by theatrical, superficial expressions of emotion that unconsciously serve to gamer attention from others to fulfill emotional needs. Episodic tachycardia, diaphoresis, anxiety,
and visual hallucinations would be atypical.

Panic disorder (Choice E) features panic attacks that occur unexpectedly and are associated with changes in behavior to avoid panic attack triggers. Though panic attacks can include restlessness, tachycardia, and diaphoresis, visual hallucinations would be
atypical. Additionally, the timing of this patient's symptoms 48 hours after hospital admission is instead classic for delirium tremens.

Educational Objective: Delirium tremens is a life-threatening complication of alcohol withdrawal that presents with severe confusion, agitation, visual hallucinations, and autonomic instability. Delirium tremens typically begins 2 to 5 days after the patient's last
alcoholic drink.

r
Previous
r
Next
,
Score Report
fr!i
Lab Values
P
Calculator
r
Help
trt
Pause _
Exam Sectlon 1: Item 39 of 50 National Board of M11dlcal Examiners®
Comprehensive isaslc Sctence Self-Assessment

y 39. An investigator is studying a laboratory strain of HIV-1. During the experiment, point mutations are introduced in the long terminal repeat region of the virus. Infection of a human T-lymphocyte line with the mutant virus results in markedly decreased viral
replication compared with infection with the wild-type virus. A mutation most likely directly affected which of the following phases of the viral life cycle?

A ) Attachment of the virus to the cell surface


B ) Budding of the virus from the cell membrane
C) Fusion of the viral envelope and cell membrane
D) Integration of the provirus into the host cell genome
E ) RNA splicing to produce an mRNA encoding the envelope glycoproteins

Correct Answer: D.
Long terminal repeats (LTRs) are found at each of the 5' and 3' ends of retroviral DNA and transposons. In the case of retroviral DNA, they are formed by reverse transcription, which is required for the replication and virulence of HIV. LTRs are used to integrate the
viral genome with that of the host, and in this case, integration of the provirus into the host cell genome is most directly affected by mutations within the LTRs. The LTR contains regulators of gene expression. The integrated HIV provirus 5' LTR contains the
promoter sequence for transcription, while the 3' end contains the polyadenylation sequence. The viral integrase enzyme catalyzes the integration of proviral DNA with host DNA. Once viral DNA is integrated, the cell is permanently infected and carries the genome
for the remainder of its life. lntegrase inhibitors prevent the action of this enzyme, limiting the degree of reverse transcribed DNA that can fuse with the host genome.
Incorrect Answers: A, B, C, and E.
Attachment of the virus to the cell surface (Choice A) involves glycoprotein (eg, gp120), CD4, CXCR4, and CCR5 surface proteins. Mutations in host proteins may confer immunity.
Maraviroc, an entry inhibitor, blocks HIV attachment to CCR5. Budding of the virus from the cell membrane (Choice B) is the final step in the release of newly synthesized HIV virions. Proteases are involved in the final packaging and release of virions, and protease
inhibitors prevent successful completion of these steps.

Fusion of the viral envelope and cell membrane (Choice C) is required for viral penetration into the cytoplasm of the host cell, and involves interactions between surface glycoproteins (eg, gp120, gp41) and host membrane proteins (eg, CD4). Enfuvirtide inhibits the
binding of gp41 to prevent the creation of entry pores in the cell membrane.
RNA splicing to produce an mRNA encoding envelope glycoproteins (Choice E) would be affected by mutations at splice sites, which typically occur in the exon and intron segments of transcribed mRNA, not in the LTRs.
Educational Objective: Long terminal repeats (LTRs) are found at each of the 5' and 3' ends of retroviral DNA and transposons and are used to integrate the viral genome into the host genome. Mutations in these regions may prevent
integration.

r
Previous
r
}�ext
,-
Score �eport
tr§
Lab Values
F
C_alculator
r
Help
,,.
Pause
Exam Section 1: Item 40 of 50 National Boa.rd pf M·11dlcal Examiners®
Comprehensive saslc Science Self-Assessment

✓ 40. A 52-year-old woman undergoes surgical excision of a large multinodular goiter. During the operation, the superior thyroid artery is used as a landmark to avoid damage to a nerve proximal to that vessel. Which of the following best describes this nerve?

A ) External branch of the superior laryngeal nerve


B ) Inferior root of the ansa cervicalis
C) Nerve to the mylohyoid
D) Phrenic nerve
E) Transverse cervical nerve
Correct Answer: A.

During thyroid surgery, there are a number of important structures at risk. These structures include the parathyroid glands, the superior and inferior thyroid arteries, the recurrent laryngeal nerve, and the external branch of the superior laryngeal nerve. The superior
laryngeal nerve arises from the vagus nerve and courses along the superior thyroid artery, supplying innervation to the cricothyroid muscles.- Injury to this nerve will cause alterations in the pitch of the voice. The recurrent laryngeal nerve is also important for
vocalization, as it provides motor innervation for all of the intrinsic muscles of the larynx except for the cricothyroid muscle. It provides the sensory innervation for the laryngeal structures inferior to the vocal cords. The recurrent laryngeal nerves additionally have
unique anatomy as they branch from the vagus nerves distally in the thorax. The left recurrent laryngeal nerve loops inferior and posterior to the aortic arch, while the right recurrent laryngeal nerve loops inferior and posterior to the right subclavian artery. Both of
these nerves then progress cranially, rather than caudally.

Incorrect Answers: B, C, D, and E.

The ansa cervicalis (Choice B) is a loop of nerves in the anterior neck that originate from the spinal nerve roots of C1-C3. The inferior root is formed by the fibers of C2 and C3 and provides branches to the inferior portion of the omohyoid, stemothyroid, and
stemohyoid muscles.

The nerve to the mylohyoid (Choice C) arises from the mandibular division of the trigeminal nerve and supplies motor innervation for the mylohyoid muscle and the anterior belly of the digastric muscle.

The phrenic nerve (Choice D) arises from the cervical roots C3-C5 and provides motor innervation to the diaphragm. It also provides sensory innervation to the mediastinal pleura and the pericardium. Phrenic nerve injury can occur during cardiothoracic surgery or
thoracic or cervical trauma and can lead to diaphragmatic paralysis.

The transverse cervical nerve (Choice E) arises from the C1 and C2 cervical nerve roots. It provides sensory innervation to the skin on the anterior neck. It is anatomically distant from the thyroid.

Educational Objective: The superior laryngeal nerve and the recurrent laryngeal nerve have unique anatomic relationships. It is important that these nerves are not injured during anterior neck surgery (eg, thyroidectomy) as this will leave the patient with laryngeal
muscle paralysis, compromising phonation and airway protection.

r
Previous
r
Next
,
Score Report

Lab Values
P
Calc;:ulator
r-
Help
r
Pause
_ .
Exam Sectlon 1: Item 41 of 50 National Board pf Medical Examiners®
Comprehensive isaslc Science Self-Assessment

y 41. An investigator is studying patients with West Nile virus infection. During a 5-year period, data are collected on 25 patients diagnosed with this condition as confirmed by testing at the Centers for Disease Control and Prevention. Demographic information on
the patients is reported (age, gender, and ethnicity), as well as information about likely souirces of infection. Which of the following best describes this study design?

A ) Case series
B) Case contrnl study
C) Cohol:t srudy
D) Correlational study
E ) Cross-sectional study

Correct Answer: A.
A case series is a descriptive study design in which a number of consecutive or nonconsecutive cases of a disease and/or treatment are described in detail, with information about exposure, demographics, and comorbidities. Case series do not imply a cause-and­
effect relationship. They do not test a hypothesis nor are they randomized. They are useful in characterizing the natural history of a disease or response to treatment. They are also useful in describing rare diseases, as small amounts of patients may not permit
conduction of larger case-control, cohort, or randomized trials with sufficient power.
Incorrect Answers: B, C, D, and E.
A case-control study (Choice B) investigates an association between exposure and an outcome. In this study design, a group of patients with the disease (cases) are identified. A group of patients without the disease (controls) are matched on baseline
characteristics to the cases. Exposure data for the two groups is collected, and these data are compared to determine association with the outcome (disease) in question. An odds ratio may be calculated to compare exposures between groups.
A cohort study (Choice C) identifies a group of patients and follows them over time to identify whether an exposure is associated with an outcome of interest. Cohort studies may be retrospective or prospective in design. In a prospective design, the hypothesis and
analysis protocols are established prior to the start of the study period. In a retrospective design the hypothesis or question is designed after the study time period has passed. An example of a prospective cohort study would be following a group of 1,000 smokers
for a time period of 10 years and identifying the proportion of these patients who develop pancreatic cancer to identify the relative risk of pancreatic cancer in smokers.
Correlational studies (Choice D) are descriptive of research methods that do not imply causal relationships. Correlational studies can be subject to errors of interpretation. For example, a researcher may find a correlation between coffee drinking and pancreatic
cancer. There may be a confounding relationship between this correlation. Coffee drinkers may be more likely to be smokers, and smoking may have a causal relationship with pancreatic cancer instead. Thus, the study may be subject to confounding and causality
should not be inferred.
A cross-sectional study (Choice E) seeks to identify the prevalence of the condition at a particular point in time. An example of a cross-sectional study would be a si :ngle survey of a population inquiring whether patients have coronary artery disease and concurrently
inquiring about activity levels and diet. Thus, the risk factor and the outcomes are measured simultaneously. The study does not follow patients over time. All information is collected at a single time point.
Educational Objective: A case series includes a small number of patients and seeks to describe the course of a disease or treatment. It does not imply a causal relationship. It is observational in nature and is useful in describing a disease or treatment with limited
numbers of patients.

r
Previous
r
}�ext
,
Score �eport
tr§
Lab Values
F
C alculator
r
Help
,,.
Pause
_ .
Exam Section 1: Item 42 of 50 National Board pf Mqdlcal Examiners®
Comprehensive tsas1c science serr-ASsossmont

✓ 42. A previously healthy 20-year-old woman comes to the emergency department because of a 3-day history of fever, shaking chills, headaches, fatigue, and joint and muscle pain. She recently spent the summer working as a lifeguard on Long Island, New
York. She has never traveled outside the USA. She underwent splenectomy for injuries sustained in a motor vehicle collision at the age of 6 years. Her temperature is 39.1 •c (102.4 F). Physical examination shows no other abnormalities. A peripheral
°

blood smear shows small intraerythrocytic rings; the result of a polymerase chain reaction test for Plasmodium species is negative. Which of the following is the most likely causal organism?

A) Babesia microti
S}- 8atf0Re{kj baGJl!ifor:mis
G}- 8cmi9f61.'a brooGhisep#sa
D) Borrelia burgdotferi
E) Bruce/la melitensis
Correct Answer: A.

Babesiosis is a disease presenting similar to malaria that is caused by the protozoa Babesia microti. It is predominantly found in the northeastern United States and transmitted via the lxodes tick. It classically presents with fever, chills, myalgias, weakness,
headache, and an associated hemolytic anemia. Asplenia is a risk factor for severe disease. Diagnosis is made by direct visualization of organisms on a Giemsa-stained peripheral blood smear, which may appear in ring forms or in a tetrad formation resembling a
cross within erythrocytes. Treatment is with atovaquone plus azithromycin.

Incorrect Answers: B, C, D, and E.

Bartone/la bacilliformis (Choice B) is a Gram-negative, coccobacillus found in Peru, Ecuador, Colombia, and some parts of southern Florida that causes bartonellosis (Carrion disease) or Oroya fever in the acute phase of infection and verruga peruana in the chronic
phase. It is transmitted by sandflies.

Bordetella bronchiseptica (Choice C) is a Gram-negative bacillus and is a common cause of respiratory disease in dogs and cats, but rarely causes disease in humans.

Borrelia burgdotferi (Choice D) is a spirochete common in the northeastern United States that causes Lyme disease. It is transmitted by the lxodes tick. Babesia microti may be co-transmitted with Borrelia burgdotferi in this manner.

Bruce/la melitensis (Choice E) is one of the species of Gram-negative coccobacilli that cause brucellosis, also known as undulant fever. The other species that infect humans are B. abortus, B. canis, and B. suis. Brucellosis is commonly contracted by the
consumption of unpasteurized milk.

Educational Objective: Babesiosis is a malaria-like disease caused by the protozoa Babesia microti. It can be recognized by the presence of intraerythrocytic ring forms or a tetrad formation on a peripheral blood smear.

r
Previous
r
Next
,
Score Report
I'§
Lab Values
P
Calculator
r
Help
ttt
Pause
Exam Section 1: Item 43 of 50 National Board of Medical Examiners®
Comprehensive tsaslc Science Self-Assessment

✓ 43. A female newborn delivered at term develops jaundice, hepatomegaly, and rash shortly after delivery. Pregnancy was complicated by a 2-week febrile illness during the first trimester. The newborn is at the i25th percentile for length, 30th percentile for
weight, and 20th percentile for head circumference. One week later, a CT scan of the head shows intracranial calcifications. Two days later, she becomes increasingly lethargic and develops respiratory distress and seizures. She has a 2-year-old sister
who attends day care. Which of the following is the most likely causal virus?

A ) Cytomegalovirus
B ) Herpes simplex virus
C ) Measles virus
D) Mumps virus
E ) Varicella-zoster virus
Correct Answer: A.

Congenital infection with cytomegalovirus (CMV), also known as human herpesvirus 5 (HHV-5)., causes hearing loss, seizures, a petechial rash, and intracranial calcifications in the neonate when acquired in utero. Hepatomegaly, splenomegaly, and
lymphadenopathy are also commonly seen. The virus is transmitted via the placenta from the mother, who often contracts the virus after interacting closely with young children. Symptoms o( maternal primary infection typically include fever, myalgias, and laboratory
evidence of atypical lymphocytosis, often with abnormal liver enzymes. In an immunocompromi-sed host, CMV can also present with a variety of atypical syndromes including colitis, encephalitis, pneumonia, esophagitis, or retinitis. Histopathologic examination of
infected cells demonstrates prominent inclusion bodies. Unlike other viruses in the HHV family, CMV is not susceptible to guanosine analogs like acyclovir and valacyclovir as the virus does not possess the thymidine kinase necessary to activate this class of
medications.

Incorrect Answers: B, C, D, and E.

Herpes simplex virus (Choice B) infection can occur via the placenta or through direct inoculation as the neonate passes through the birth canal in a mother with active lesions. Infection with herpes simplex virus can cause both cutaneous herpetic lesions and
systemic manifestations like meningoencephalitis. It is not a cause of intracranial calcifications.

Measles virus (Choice C) may be acquired via placental transmission or at the time of delivery. Clinical manifestations include a prodrome stage of fever, malaise, and anorexia. This is followed by the classic triad of cough, coryza, and conjunctivitis. An
erythematous rash beginning on the face and spreading cephalocaudally is also characteristic. Neonates with measles infections may present with a spectrum of disease ranging from mild to severe.

Mumps virus (Choice D) infection is caused by a paramyxovirus and is more likely to be transmitted through respiratory droplets and direct contact than through the placenta. Clinical manifestations include fever, myalgias, and fatigue, which are then followed by
parotid gland swelling. Mumps orchitis, swelling of the testes, is a potential complication in males.

Varicella-zoster virus (Choice E) may be transmitted vertically through the placenta during pregnancy but is more likely to be transmitted after delivery from mother to neonate. Neonates with varicella infection present with fevers and a vesicular rash with individual
lesions classically in different stages of healing. Neonatal varicella has a mortality rate of up to 30%.

Educational Objective: Congenital infection with cytomegalovirus (CMV) results in hearing loss, seizures, petechial rash, intracranial calcifications, organomegaly, and lymphadenopathy. The virus is transmitted via the placenta from the mother.

r
Previous
r
Next
,,.
Score Report
fr!i
Lab Values
P
Calculator
r
Help
trt
Pause
Exam Section 1: Item 44 of 50 National Board of Medical Examiners®
Comprehensive tsaslc Science Self-Assessment

✓ 44. A 62-year-old man undergoes a physical examination on his arrival at the penitentiary to serve his prison sentence. A routine PPD skin test shows a 14-mm area of induration at 48 hours. A chest x-ray is
shown. Reciprocal activation of the two immune cell types involved in the pathogenesis of this patient's pulmonary lesions is mediated by interferon-gamma and which of the following cytokines?

A ) lnterleukin-4 (IL-4)
B ) IL-5
C) IL-7
D) IL-10
E ) IL-12

Correct Answer: E.

lnterleukin-12 (IL-12) and interferon-gamma activate the two primary cells responsible for granuloma formation in this patient with Mycobacterium tuberculosis (MTB) infection. IL-12 is an inflammatory cytokine secreted by macrophages that are infected with
MTB, an intracellular pathogen. IL-12 acts on both natural killer cells (NK cells) and cytotoxic T lymphocytes, causing them to release interferon-gamma, which has several roles including the promotion of Th1 T-cell differentiation, th.e augmentation of antigen
presentation by macrophages, and the differentiation of macrophages into multinucieated giant cells. Collectively, this activating pathway of IL-12 and interferon-gamma allow for granuloma formation, a histologic hallmark of infection with MTB, isolating the
invasive pathogen. Contrary to other diseases that result in granuloma formation, MTB infection often results in caseating (necrotic) granulomas.

Incorrect Answers: A, B, C, and D.

lnterleukin-4 (Choice A) induces differentiation of helper T cells to Th2 cells, not Th1 cells. It is important in the process of activating macrophages to assist with wound repair and fibrosis, not granuloma formation.

IL-5 (Choice B) is secretedi by Th2 helper T cells and mast cells. It activates B lymphocytes to secrete lgA.

IL-7 (Choice C) is secreted by numerous types of cells and is important in the maturation of both B and T lymphocytes but does not play a role in granuloma formation.

IL-10 (Choice D) is secreted by Th2 and regulatory T cells. It has myriad effects but overall decreases the inflammatory response. It downregulates the expression of MHC class II antigens and Th1 cytokines. Additionally, it inhibits the activity of activated
macrophages, which would decrease granuloma formation.

Educational Objective: MTB infects macrophages and results in granuloma formation within the lungs via the actions of IL-12 and interferon-gamma on macrophages, Th1 helper T cells, and NK cells.
Exam Section 1: Item 45 of 50 National Board of Medical Examiners®
Comprehensive tsaslc Science Self-Assessment

y 45. A 70-year-old man with poorly controlled hypertension is brought to the emergency department 30 minutes after the sudden onset of weakness of the left side of his face and body. He is alert and able to follow commands. His pulse is 80/min, and blood
pressure is 190/90 mm Hg. Physical examination shows mild flattening of the nasolabial fold on the left side and weakness and hyperreflexia of the left upper and lower extremities. There are no sensory deficits, but he has mild dysarthria. Six months later,
histologic examination of the area of the injury is most likely to show proliferation of which of the following cell types?

A ) Astrocytes
B ) Ependymal cells
C) Microglial cells
D) Neurons
E ) Oligodendrocytes

Correct Answer: A.

In the aftermath of a cerebral infarction (stroke), several histologic changes occur that culminate in a glial scar composed of proliferated astrocytes. One to 2 days after an ischemic event, neurons shrink and release lysosomal enzymes that lead to liquefactive
necrosis. Three to 5 days after the event, macrophages (called microglial cells in the central nervous system) transiently proliferate and phagocytose dead neurons. Within weeks, astrocytes migrate to the site of injury (as opposed to fibroblasts in the peripheral
nervous system) and begin to proliferate. After 2 weeks, the ischemic area becomes occupied by a permanent glial scar, which is composed of a dense network of neuroglial cells (astrocytes) and fluid-filled cystic spaces.

Incorrect Answers: B, C, D, and E.

Ependymal cells (Choice B) form the permeable lining of the ventricles that separates the CSF system and brain parenchyma. They do not migrate and are not involved in post-stroke histologic chan.ges.

Microglial cells (Choice C), derived from precursor monocytes, proliferate and phagocytose dead neurons days after an ischemic event and may remain in the region weeks after the initial injury. Microglial cells would not be present 6 months after the event.

Neurons (Choice D) undergo liquefactive necrosis after a stroke. They are permanently differentiated cells, similar to cardiac myocytes, and are unable to re-enter the cell cycle for regeneration. Therefore, few neurons would be seen on histologic examination after
6 months.

Oligodendrocytes (Choice E) are glial cells that produce myelin in the central nervous system. These cells are damaged in strokes. They slowly regenerate over months but would not be expected to multiply to the degree that scar-forming astrocytes proliferate.

Educational Objective: After a cerebral infarction, neurons undergo liquefactive necrosis and are replaced by a glial scar consisting of proliferated astrocytes. The glial scar forms approximately 2 weeks after the initial ischemic event.

r r , fr!i P r trt
Previous Next Score Report Lab Values Calculator Help Pause _
Exam Section 1: Item 46 of 50 National Board of Medical Examiners®
Comprehensive tsas1c science Self-ASsessment

✓ 46. A 35-year-old man comes to the physician because of a 6-month history of burning abdominal pain that occurs 1 to 2 hours after he eats. He also has had black stools for 2 days. Use of over-the-counter antacids and histamine-2 (Hi)-receptor blocking
agents has not been effective in relieving his symptoms. He is sweating profusely and has light-headedness when he stands. His blood pressure is 105/70 mm Hg while sitting. Physical examination shows epigastric tenderness. A CT scan of the abdomen
shows a 1-cm mass in the pancreas. lmmunohistochemical labeling of neoplastic cells in a biopsy specimen is most likely to involve the use of antibodies directed to which of the following substances?

A ) Amylase
B ) Gastrin
C ) Glucagon
D) Human pancreatic polypeptide
E ) Insulin
F ) Lipase
G) Serotonin
H ) Somatostatin
I ) Vasoactive intestinal polypeptide
Correct Answer: B.

This patient's findings of postprandial abdominal pain, melena, diaphoresis, and orthostatic hypotension in the setting of a pancreatic lesion are suggestive of a peptic ulcer secondary to Zollinger-Ellison syndrome (ZES). ZES stems from a pancreatic or duodenal
gastrin-secreting tumor. Gastrin is typically produced by gastric G cells and stimulates parietal cells to produce hydrochloric acid. Excessive production of acid in the setting of ZES leads to recurrent, chronic duodenal or jejuna! ulcers, which can present with
abdominal pain, diarrhea secondary to malabsorption, and possible hematemesis, melena, or hematochezia. lmmunohistochemical labeling of neoplastic cells using an anti-gastrin antibody reveals positive staining. Pancreatic gastrinoma and ZES may present as a
component of multiple endocrine neoplasia type 1 (MEN1).

Incorrect Answers: A, C, D, E, F, G, H, and I.

Amylase (Choice A) assists in the dige·stion of carbohydrates and is secreted in saliva and from the exocrine pancreas. Increased levels of amylase may indicate sialadenitis or pancreatitis.

Glucagon (Choice C) is produced by pancreatic alpha cells and primarily works to increase serum glucose. Positive glucagon staining on histology is characteristic of a glucagonoma, which presents with mild diabetes mellitus, necrolytic migratory erythema, and
weight loss.

Human pancreatic polypeptide (Choice D) regulates pancreatic endocrine and exocrine secretions. Its overexpression is nonspecific and may be observed in a variety of different hormonally active pancreatic tumors.

Insulin (Choice E) is produced by pancreatic beta cells and primarily works to lower serum glucose. Positive staining for insulin on histology is characteristic of an insulinoma, a rare neoplasm that produces insulin. Patients present with lethargy, weakness, fatigue,
diplopia, and hypoglycemia. Symptoms are exacerbated by fasting or exercise.

Lipase (Choice F) is a pancreatic enzyme that functions to digest triglycerides. Some pancreatic acinar cell carcinomas may demonstrate staining for lipase.

Serotonin (Choice G) is typically produced by carcinoid tumors. Carcinoid tumors of the gastrointestinal tract are neuroendocrine tumors that present with weight loss, flushing, diarrhea, bronchospasm, right sided cardiac valvular disease, gastrointestinal bleeding,
and abdominal pain.

Somatostatin (Choice H) is produced by a somatostatinoma, an uncommon neuroendocrine malignancy that presents with weight loss, mild diabetes mellitus, steatorrhea due to limited cholecystokinin release, and achlorhydria from the inhibition of gastrin.

Vasoactive intestinal polypeptide (VIP) (Choice I) can be produced by VIPomas. VIPoma is a rare tumor often associated with MEN1 that presents with profound watery diarrhea, electrolyte disturbances (eg, hypokalemia, hypercalcemia), achlorhydria, alkalosis,
flushing, and vasodilation.

Educational Objective: Zollinger-Ellison syndrome stems from a gastrin-secreting tumor located typically within the pancreas or duodenum that results in excessive production of acid by the gastric parietal cells. Patients develop recurrent, chronic duodenal or
jejuna! ulcers, which can present with abdominal pain, diarrhea secondary to malabsorption, and possible hematemesis, melena, or hematochezia. lmmunohistochemical labeling of neoplastic cells will reveal positive staining for gastrin.

r r , fr!i P r trt
Previous Next Score Re ort Lab Values Calculator Hel Pause
Exam Section 1: Item 47 of 50 National Board pf Medical Examiners®
Comprehensive l!as1c Sctence Serr-Assessment

)( 47. A 56-year-old man is brought to the emergency department 1 hour after having a generalized tonic-clonic seizure during his morning run. Vital signs are within normal limits. MR ls of the brain show a mass in the lateral aspect of the precentral gyrus of the
frontal lobe. Based on the MRI findings, which of the following additional deficits is most likely in this patient?

A) .A.trophy of the left upper extremity


B) Decroased proprioceptioR iR the Fight upper extremity
G) Decreased seRsatioR iR the left lower extremity
D) De•.<iatioA ofthe toRguetothe right
E ) Weakness of the left lower area of the face
F ) Weakness of the right lower extremity

Correct Answer: E.

The precentral gyrus of the frontal lobe contains the primary motor cortex that controls voluntary movements of the contralateral body. Medially, the precentral gyrus contains neurons that control the lower extremity. Laterally, the precentral gyrus controls the
contralateral face. The contralateral arm is controlled by neurons between the two areas. This distribution of control is known as somatotopy and permits localization of physical examination findings to a particular brain region. In this case, a lesion in the lateral
aspect of the right precentral gyrus would disrupt motor neurons controlling the left face, resulting in weakness, facial droop, and an upper motor neuron pattern of dysfunction (eg, hyperreflexia). Importantly, the forehead would be spared, as the facial nerve (cranial
nerve VII) nucleus that controls forehead musculature is dually innervated by upper motor neurons from bilateral precentral gyri. Because of this, the patient would be expected to symmetrically raise his eyebrows. Alternatively, lesions involving the lower motor
neurons of facial expression (eg, facial nerve inflammation in Bell palsy) affect the forehead and lower face together.

Incorrect Answers: A, B, C, D, and F.

Atrophy of the left upper extremity (Choice A) may be associated with an upper motor neuron lesion located in the right precentral gyrus that would be located medially relative to the location of this patient's mass. The contralateral arm is innervated by neurons
located in the parasagittal region of the precentral gyrus. As well, atrophy of the left upper extremity may be associated with lower motor neuron pathology in the ipsilateral spinal cord, spinal nerves, or peripheral nerves.

Decreased proprioception in the right upper extremity (Choice B) or decreased sensation in the left lower extremity (Choice C) could be associated with lesions of the contralateral thalamus, tracts leading to the postcentral gyrus, or within the postcentral gyrus, as
these areas are associated with position sense and touch sensation. Peripheral lesions associated with decreased proprioception or touch could involve the ipsilateral sensory neurons, dorsal root ganglia, or the dorsal column-medial lemniscus pathway within the
spinal cord.

Deviation of the tongue to the right (Choice D) is associated with loss of motor control from the right cranial nerve XII, which innervates most of the intrinsic and extrinsic muscles of the tongue. Absent tone on the right-side results in deviation toward the right. The
upper motor neurons for the tongue exist within the lateral contralateral precentral gyrus, but facial weakness would likely also be present.

Weakness of the right lower extremity (Choice F) may be associated with an upper motor neuron lesion in the left medial precentral gyrus in the region of the paracentral lobule. As well, right lower extremity weakness can be secondary to pathology involving the
ipsilateral lumbar spinal cord, cauda equina, or peripheral nerves.
Educational Objective: The precentral gyrus of the frontal lobe contains the primary motor cortex controlling the voluntary movement of the contralateral body. Laterally, the precentral gyrus controls the contralateral face. Pathology affecting the upper motor neuron
innervation to the contralateral face typically presents with only lower face weakness as the facial nerve nucleus that controls forehead musculature is dually innervated by upper motor neurons from bilateral precentral gyri.

r
Prevlou&
r
Next
,
Score Report
fr!i
Lab Value&
F
Calculator
r
Help
fl"
Pau&e .
Exam Section 1: Item 48 of 50 National Board of Medlcal Examiners®
Comprehensive 1:1as1c Science Serr-Assessment

✓ 48. A 23-year-old woman is brought to the emergency department because of shortness of breath for 2 weeks. Her respirations are 28/min. Physical examination shows no other abnormalities. Laboratory studies show:
Serum
Na + 135 mEq/L
K+ 4.0 mEq/L
c1- 110 mEq/L
HCO 3- 15 mEq/L
Arterial pH on room air 7.25
Urine
Sodium 20 mEq/L
Chlorine 30 mEq/L
Potassium 15 mEq/L

Which of the following is the most likely diagnosis?

A ) Alcoholic ketoacidosis
B) Crohn disease
G}- Diabetic ketoacidosis
Q}- Lactic acidosis
E ) Renal tubular acidosis
F ) Salicylate poisoning
Correct Answer: E.

Renal tubular acidosis (RTA) results in a hyperchloremic, non-anion gap metabolic acidosis from the dysregulation of the renal tubules causing the loss of bicarbonate or inability to excrete hydrogen ions. Metabolic acidosis can present with a compensatory
respiratory alkalosis, manifesting as tachypnea and shortness of breath. Types of RTA vary in etiology and pathophysiology, but all result in a non-anion gap metabolic acidosis. RTAs are not due to the accumulation of an unmeasured anion and do not present with
an increased anion gap. This patient demonstrates a low bicarbonate level, with a normal anion gap of 12, consistent with a non-anion gap acidosis. Distal RTA (type 1) results from a defect in the renal secretion of H+, which results in the lack of HC03- generation,
ultimately leading to a metabolic acidosis. It is characterized by a urine pH greater than 5.5. Some common causes of type 1 RTA include analgesic nephropathy, congenital obstructive urinary tract anomalies, medications, and autoimmune disease.

Incorrect Answers: A, B, C, 0, and F.

Alcoholic ketoacidosis (Choice A) refers to a starvation state induced by excess alcohol intake that results in increased ketone production and an anion gap metabolic acidosis due to alterations in the ratio of NAO+ to NADH.

Diabetic ketoacidosis (Choice C) typically occurs in patients with type 1 diabetes mellitus as a result of increased insulin requirements during times of physiologic stress (eg, infection) or insulin nonadherence. Diabetic ket.oacidosis results in a metabolic acidosis
with an increased anion gap.

Lactic acidosis (Choice 0) can result from any hypoperfusion state, hypoxia, deficiency in the citric acid cycle, impaired oxidative phosphorylation, or from vitamin deficiencies (eg, thiamine), and is associated with an increased anion gap metabolic acidosis.

Salicylate poisoning (Choice F) initially results in a respiratory alkalosis from the stimulation of respiratory centers causing hyperventilation. Subsequently, it results in the development of a delayed anion gap metabolic acidosis secondary to its decoupling effects on
the electron transport chain, resulting in increased anaerobic metabolism and the production of lactate and ketoacids.

Educational Objective: There �re three types of renal tubular acidosis (RTA), which vary in etiology and pathophysiology, but all result in a hyperchloremic, non-anion gap metabolic acidosis from underlying dysregulation of the renal tubules.

r
Previous
r
Next
,
Score Report
"3
Lab Values
F'
Calculator
r
Help
,,.
Pause
Exam Section 1: Item 49 of 50 National Boa.rd of Medi cal Examiners®
Comprehensive 1:1as1c Science Serr-Assessment

✓ 49. An investigator conducts a clinical study of 300 patients with stages Ill and IV colon carcinoma. It is found that over time, metastases show an increasing potential for growth and resistance to chemotherapy. Which of the following mechanisms of the cell
cycle best explains this observation?

A ) Decreased expression of cyclin B during GiM transition


B}- DeGr:eased phosphorylation of RB1 protein duringG 'I'5 transition
C) Genomic instability during S, G 2, and M phases
D}-O1,erexpression of p15, p16. a�ll during G4phase
Ii}-O\1erexpression of p21, p27, and p57 proteins during all phases
Correct Answer: C.

Genomic instability during S, G2, and M phases explains the increased potential for growth and resistance to chemotherapy agents used to treat stage Ill and IV colon carcinoma. The two primary forms of genomic instability that predispose to the formation of
malignant cells are microsatellite and chromosome instability. In the former, the failure of mismatch repair pathways (MMR) from mutations in MLH1, MSH2, APC, and other genes, results in the propagation of aberrant DNA and accumulation of mutations that
predispose to malignant transformation. This is the mechanism of oncogenesis found in familial cancer syndromes such as Lynch syndrome or familial adenomatous polyposis (FAP). In the latter instance of chromosome instability, specific activating mutations of
oncogenes or loss of function mutations of tumor suppressor genes results in oncogenic potential. This pathway accounts for the majority of sporadic colon cancers. Activating mutations of oncogenes such as KRAS and inactivating mutations of tumor suppressor
genes such as P53 predispose cells to abnormal growth through abnormal entry into the cell cycle, which results in cellular proliferation. Within the cell cycle, errors of replication further predispose to malignant growth and may account for resistance to
chemotherapeutic agents. The normal cell cycle in somatic cells involves several stages: G1, S, G2, and M phase. Go phase denotes cell cycle arrest. During G1 phase, the cellular contents, with the exception of the chromosomes, are replicated. In S phase,
chromosomal duplication occurs. In G2 phase, the fidelity of replication is checked. M phase involves the attachment of chromosomes to spindles followed by their separation and division into two identical cells, with the stages of M phase denoted as prophase,
metaphase, anaphase, and telophase. Derangement at any point during mitosis can predispose to the propagation of an abnormal and unstable genome, with the possible development of resistance to chemotherapy agents.

Incorrect Answers: A, B, D, and E.

Decreased expression of cyclin B during G2/M transition (Choice A) paradoxically reduces cell proliferation. Depleted cyclin B levels mean the cyclin B/CDK1 complex cannot form, which prevents cells from entering M phase.

Decreased phosphorylation of RB1 protein during G 1-S transition (Choice B) is not the correct answer. RB1 is a tumor suppressor gene and phosphorylation results in its inhibition, thereby allowing the cell to enter the cell cycle. Reduced phosphorylation would
prevent entry into the cell cycle and would inhibit growth.

Overexpression of p15, p16, and p18 during G 1 phase (Choice D) would result in the reduction of cellular proliferation as these genes are all tumor suppressors.

Overexpression of p21, p27, and p57 proteins during all phases (Choice E) of the cell cycle would result in decreased cellular proliferation as all of these encode cyclin-dependent kinases that halt progression of the cell cycle when upregulated .

Educational Objective: There are two primary mechanisms of genomic instability in the pathogenesis of colon cancer: microsatellite instability (15% of cases) and chromosome instability (85% of cases). Chromosome instability results initially in activating mutations
of oncogenes and/or loss of function mutations of tumor suppressor genes, resulting in the consequent aberrant entry into the cell cycle. Once cells enter mitosis, genomic instability occurs during chromosome replication (S phase), fidelity checks of the
chromosomes (G2 phase), and during separation of the chromosomes (M phase), which results in the accumulation of mutations, potentially causing increased proliferation and resistance to chemotherapy.

r
Previous
r
Next
,
Score Report

Lab Values
P
Calculator
r-
Help
r
Pause _
Exam Secllon 1: Item 50 of 50 National Board pf Mqdlcal Exftllllners®
Comprehensive aas1c: scienc:e Serr-Assessment

't 50. An investigator conducts a phase 1 clinical trial to test the efficacy of a systemic antagonist to lymphocyte function-associated antigen 1 (LFA-1). The drug is approved for topical administration to treat keratoconjunctivitis sicca in patients with multiple
sclerosis. During the trial, it is found that these patients are at an increased risk for bacterial infection as a result of this drug. The most likely explanation for this risk is drug-induced blockade of which of the following segmented neutrophil activities?

A} AGtiv-alioA of phagocytosis
B) Adhesion to the endothelium
G} Oiapedesis thmwgh I-he capillary ¥.•all
D) Migration to the site of injury
�} RolliAg al lhe ,,essel periphery
Correct Answer: B.

Neutrophil recruitment to sites of infection is an essential component of the innate immune response. Neutrophils directly eliminate pathogens by generation of reactive oxygen species and phagocytosis, recruitment of macrophages, and creation of a pro­
inflammatory environment. The stages of neutrophil recruitment are margination, rolling, adhesion, extravasation, and chemotaxis/migration. Lymphocyte function-associated antigen 1 (LFA-1) is an integrin protein involved in neutrophil adlhesion to the endothelium.
LFA-1 antagonism leads to decreased neutrophil.adhesion and an impaired innate immune response. The genetic disorder leukocyte adhesion deficiency type 1 is caused by a defect in LFA-1.
Incorrect Answers: A, C, D, and E.

Activation of phagocytosis (Choice A) is achieved by numerous inflammatory mediators, including bacterial proteins such as lipopolysaccharides, peptidoglycan, and teichoic acids, as well as complement proteins, inflammatory cytokines, and prostaglandins.
lntegrins are not involved in this process.

Diapedesis through the capillary wall (Choice C), also called extravasation, is the process by which neutrophils move from the vascular compartment into the extracellular space. It is mediated by the integrin platelet/endothelial cell adhesion molecule 1 (PECAM-1),
not LFA-1.

Migration to the site of injury (Choice D}, also called chemotaxis, is the movement of inflammatory cells through the extracellular space to the site of injury. It is mediated by numerous chemotactic agents, including IL-8, C5a, leukotriene 84, , 5-HETE, and formyl­
methiony1 peptides.

Rolling at the vessel periphery (Choice E) occurs following margination of the neutrophils to the periphery of the blood vessel and prior to neutrophil adhesion to the endothelial wall. It is mediated by selectins and glycoproteins (eg, Sialyl-Lewis X antigen).

Educational Objective: The recruitment of neutrophils from the peripheral circulation to sites of tissue injury and infection involves the distinct stages of margination, rolling, adhesion, extravasation, and chemotaxis/migration. Lymphocyte function-associated antigen
1 (LFA-1) is involved in the adhesion of neutrophils to the endothelium.

r
Previous
r
Next
,,,
Score Report

Lab Values
P
Calculator
r
Help
1ft'
Pause
Exam Section 2: Item 1 of 50 National Boa.rd pf M·11dlcal Exi\llllners®
Comprehensive tsas1c science Serr-Assessment

✓ 1. A 35-year-old woman and her 35-year-old husband come to the physician for genetic counseling after their son is diagnosed with a rare metabolic disease. Assuming Hardy-Weinberg equilibrium, the physician tells the parents about the incidence and
carrier frequency of this disorder. Which of the following is most likely to disturb the Hardy-Weinberg equilibrium of this disorder?

A ) Appreciable rate of gene mutation


B}- Random matings in thepopulation
G}- Relati'lely la,=ge population
D) No selection against a certain genotype
Ii}- No signifisant immigr:ant population
Correct Answer: A.

The Hardy-Weinberg principal, known as Hardy-Weinberg equilibrium, proposes that allele frequencies will remain constant across generations in the absence of evolutionary change. This model of population genetic stability requires a number of assumptions,
including that natural selection does not occur, that there are no random mutations, that no movement in or out of the population occurs in equilibrium, and that mating is completely random. With these assumptions, the frequencies of alleles can be calculated as
they will remain constant over time. If deviations from these assumptions occur, the allele frequencies within the population will change over time. The Hardy-Weinberg equilibrium equation is: p2 + 2pq + q2 = 1, where pis the frequency of the dominant allele and q
is the frequency of the recessive allele. 2pq is the probability of heterozygosity, p2 is the probability of dominant homozygote, and q2 is the probability of being a recessive homozygote. If there is an appreciable rate of gene mutation occurring within the population,
over time the allele frequency will change. Assuming eq�ivalent mutation rates in each gene, eventually all allele frequencies would change. Gene mutation prevents the stability or equilibrium of genotypes over time.

Incorrect Answers: B, C, D, and E.

Random matings in the population (Choice B) are required for Hardy-Weinberg equilibrium. Nonrandom mating promotes imbalances in the equilibrium with resultant selection for or against certain alleles.

A relatively large population (Choice C) is not necessary for Hardy-Weinberg equilibrium. However, small population sizes with small numbers of particular alleles are more subject to genetic drift.

No selection against a certain genotype (Choice D) is required for Hardy-Weinberg equilibrium. If a certain gene is selected for, that is, the organism with a specific genotype is more likely to breed and pass along the allele, then the prevalence of that particular
genotype will increase. Conversely, if a certain allele is selected against, the frequency of that allele will decrease over time.

Immigration transfers genetic material from one population to another. An influx of new genetic material would disrupt the Hardy-Weinberg equilibrium. Conversely, no significant immigration (Choice E) into the population would promote Hardy-Weinberg equilibrium.

Educational Objective: Hardy-Weinberg equilibrium models the frequency of alleles within a population from generation to generation, which considers alleles to be in stable equilibrium over time. Violation of its assumptions will cause deviation from this ideal
behavior and variation in the prevalence of alleles over time.

r
Previous
r
Next
,
Score Report

Lab Values
P
Calculator
r
Help
r
Pause
Exam Sectlon 2: Item 2 of 50 National Board pf Medical Examiners®
Comprehensive isaslc Science Self-Assessment

✓ 2. A 40-year-old man has moderate edema. A 24-hour urine collection contains 15 g of protein (Ns150 mg/24 h). Which of the following is the most likely mechanism of the edema?

A ) Decreased capillary hydrostatic pressure


B) Decreased plasma oncotic pressure
C) Increased plasma oncotic pressure
D) Decreased permeability of postcapillary venules
E) Increased permeability of postcapillary venules
Correct Answer: B.
Flow into the interstitial space is dependent on interactions between capillary hydrostatic pressure, interstitial fluid hydrostatic pressure, plasma colloid oncotic pressure, and interstitial fluid oncotic pressure. If capillary hydrostatic pressure is high relative to
interstitial fluid hydrostatic pressure, fluid will flow down a pressure gradient into the interstitial space. If the opposite is true (interstitial fluid pressure is higher than capillary pressure), fluid will flow into the capillaries. If plasma fluid oncotic pressure is high relative to
interstitial fluid oncotic pressure, fluid will be drawn back into the capillaries. Conversely, if interstitial fluid oncotic pressure exceeds plasma fluid oncotic pressure, fluid will move into the interstitial space. Typically, excess fluid in the interstitial space is removed by
the lymphatic system. However, when this system is overwhelmed, edema develops. Therefore, this patient has developed peripheral edema due to decreased plasma oncotic pressure secondary to the loss of protein through proteinuria.
Incorrect Answers: A, C, D, and E.
Decreased capillary hydrostatic pressure (Choice A) and increased plasma oncotic pressure (Choice C) would draw fluid into the capillaries from the interstitial space, decreasing edema.
Decreased or increased permeability of postcapillary venules (Choices D and E) would affect edema, as fluid would be prevented from moving into or out of the interstitial space despite the influence of plasma or interstitial fluid hydrostatic and oncotic pressures.
Any influence of venule permeability is a less likely underlying mechanism given this patient's profound proteinuria and subsequent decreased plasma oncotic pressure.
Educational Objective: Movement of fluid between capillaries and the interstitial space is dependent on the hydrostatic and oncotic pressures of both compartments. High capillary hydrostatic pressure and low plasma oncotic pressure will increase the movement of
fluid out of the capillaries and into the interstitial space. Conversely, high interstitial fluid hydrostatic pressure and low interstitial fluid oncotic pressure will drive fluid back into the capillaries and away from the interstitial space.

r
Previous
r
}�ext
,-
Score �eport
tr§ F
Lab Values C alculator
r
Help
,,.
Pause
_ .
Exam Section 2: Item 3 of 50 National Board of Medical Examiners®
Comprehensive tsaslc Science Self-Assessment

y 3. A 68-year-old man comes to the ph,ysician's office because of the sudden onset of loss of vision in the left eye. There is no pain. He has a 15-year history of well controlled type 2 diabetes mellitus and hypertension. He had a myocardial infarction 2 years
ago. Temperature is 36. 7 °C (98.1° F), pulse is 72/min, respirations are 16/min, and blood pressure is 200/100 mm Hg. Funduscopic examination shows a pale, white retina with retinal artery narrowing and decreased filling of the retinal veins. Which of the
following is the most likely cause of the loss of vision?

A ) Malignant hypertension
B ) Retinal artery embolism
C) Retinal detachment
• D) Retinal vein thrombosis
I;} Vitm<:>bls l!lem<:>rrhage sec<:>ndarytQ ne<:>1Jast:blla�ati<:>R
Correct Answer: B.

Central retinal artery occlusion (CRAO) presents with sudden, monocular vision loss, which is usually painless. Vision loss is typically profound, with severely reduced visual acuity, and occurs over seconds. There may be a history of preceding amaurosis fugax.
Physical examination shows a relative afferent pupillary defect, a pale fundus with a cherry red spot, and narrowing of the retinal arterioles with arteriolar boxcarring. The most common cause of CRAO is a retinal artery embolism from a carotid artery atheroma or
from a cardiac valvular vegetation. Embolic disease may also manifest with Hollenhorst plaques, which are smaller, retractile deposits that can be visualized at the bifurcations of retinal arterioles. CRAO is considered a form of embolic stroke affecting the eye, and
suspicion of CRAO should therefore prompt an immediate stroke risk factor evaluation. CRAO may also occur secondary to giant cell arteritis. There is no proven effective treatment for CRAO and the prognosis for visual recovery is poor. Patients who have been
affected by CRAO are at risk of secondary ocular neovascularization and glaucoma due to persistent retinal ischemia.

Incorrect Answers: A, C, D and E.

Malignant hypertension (Choice A) presents with optic disc edema, retinal arteriolar narrowing with arteriovenous nicking and silver wiring, retinal edema, cotton wool spots, and flame hemorrhages. It is typically a symmetric, bilateral process.

Retinal detachment (Choice C) presents with painless, monocular vision loss but is typically preceded by photopsia and bursts of floaters. The detachment is identifiable on dilated fundus examination as a distinct region of retinal elevation and is not usually
associated with retinal whitening, arteriolar narrowing, or reduced venous filling.

Retinal vein thrombosis (Choice D), presents with painless, monocular vision loss, and retinal venous dilatation and tortuosity, retinal hemorrhages, and nerve fiber layer infarcts (eg, cotton wool spots) on fundoscopic examination. Retinal vein thrombosis may affect
a single branch of the retinal veins, known as a branch retinal vein occlusion (BRVO) or may affect the central retinal vein (CRVO). Affected patients in both cases are at increased risk of subsequent neovascularization due to retinal ischemia.

Vitreous hemorrhage secondary to neovascularization (Choice E) presents with painless, monocular vision loss; patients may report seeing floaters and visual haze. Examination generally reveals hemorrhage in the vitreous cavity or in the subhyaloid space.
Although this patient has type 2 diabetes mellitus, which is a risk factor for neovascularization and vitreous hemorrhage, the contralateral eye will usually reveal a similar degree of retinopathy.

Educational Objective: Central retinal artery occlusion (CRAO) presents with sudden, painless, monocular vision loss, relative afferent pupillary defect, retinal whitening with a cherry red spot, and retinal arteriolar narrowing with boxcarring. The most common cause
of CRAO is a retinal artery embolism from a carotid artery atheroma or a cardiac Valvular vegetation. CRAO is considered a form of embolic stroke affecting the eye, and suspicion of CRAO should therefore prompt an immediate stroke risk factor evaluation.

r r , fr!f P r trt
Previous Next Score Report Lab Values Calculator Help Pause
Exam Section 2; Item 4 of 50 Nattonal Board of Medical Examiners®
Comprellenslve Basic Science Self-Assessment

✓ 4. A 22-year-old woman who has �-thalassemia has a �-hemoglobin allele that carries a single base substitution (A-. G) at the splice acceptor site of the second intron. Which of the following events in expression of the �-hemoglobin gene is most likely to be
impaired by this mutation?

A} SiRdiRg of RYCloosomal hiSWROS to tho mrnscripl


B} Clotwago of tho traRSCript by RNA ligaso
C) Modification of the transcript by small nuclear ribonucleoproteins
D) Recognition of the transcript by signal recognition particle
Ei} +1'3RSportofIAo traRscripl OYtofIAo RYCIOYS
Correct Answer: C.
Modification of the transcript by small nuclear ribonucleoproteins (snRNPs) is most likely to be affected by a single base substitution at the splice acceptor site of the second intron of the �-hemoglobin allele. Transcription is the process by which pre-messenger RN
(mRNA) is made from DNA, but pre-mRNA must undergo post-transcriptional modification to create mature mRNA, including the removal of intrans and connection of exons. Splice sites exist at the end of an intron and the beginning of an exon, and are recognized
by snRNPs, which are RNA-protein complexes that combine to form a spliceosome. These snRNPs remove the intrans, and exons are subsequently spliced together. A mutation in the splice acceptor site can result in the inclusion of intrans into the final mRNA, or
exclusion of exons, thereby rendering the mRNA and its translated protein non-functional. In the instance of 13-thalassemia, a single abnormal �-hemoglobin gene would result in 13-thalassemia minor.
Incorrect Answers: A, 8, D, and E.
Binding of nucleosomal histones to the transcript (Choice A) would not be affected by a mutation of the splice site. Histones are proteins with positively charged amino acids that bind to negatively charged DNA. A complex of histones is called a nucleosome.
Unwinding of the nucleosome is required for transcription and occurs via the action of histone acetylase.
Cleavage of the transcript by RNA ligase (Choice 8) is not affected by mutations in the splice acceptor site. RNA ligases are a class of enzymes that join RNA strands to one another.
Recognition of the transcript by signal recognition particle (SRP) (Choice D) occurs after transcription has already occurred. The SRP binds new peptides as they are being synthesized by the ribosome and results in a transient reduction in the pace of translation to
facilitate early initiation of protein translocation.
Transport of the transcript out of the nucleus (Choice E) occurs via a mechanism that involves assimilation of mRNA into ribonucleoprotein particles (RNP) with the subsequent recruitment of specific mRNA exporter proteins. These export proteins help move mRNA
out of the nucleus and into the cytoplasm. This would not be affected by a mutation in the splice acceptor site.
Educational Objective: The splice acceptor site resides between an intron and exon on pre-mRNA strands and functions to recruit snRNPs that remove intrans, with subsequent connection of exons via the spliceosome, forming fully mature mRNA. Mutations in the
acceptor site lead to abnormal inclusion of intrans or exclusion of exons and ultimately result in an abnormal, nonfunctional protein.

r
Previous
r
Next
,
Score Report
� P
Lab Values Calculator
r-
Help
r
Pause
Exam Section 2: Item 5 of SO National Board pf Medical Examiners®
Comprehensive l!as1c SClence Serr-Assessment

✓ 5. A 22-year-old woman comes to the physician because of a 6-month history of frequent headaches and fatigue. She tells the physician that she often overeats when she is anxious and has been making herself vomit and using laxatives for the past 2 years.
She is 168 cm (5 ft 6 in) tall and weighs 59 kg (130 lb); BMI is 21 kg/m2. Physical examination will most likely show which of the following conditions?

A ) Arthritis
B ) Decreased body hair
C) Dental caries
D) Galactorrhea
E ) Pituitary adenoma
F) Tachycardia
G) Thyroid gland enlargement
Correct Answer: C.

Patients with eating disorders such as anorexia nervosa and bulimia nervosa who purge by vomiting may demonstrate dental caries on physical examination. This patient likely has bulimia nervosa (purging type). Bulimia nervosa (purging type) involves cycles of
uncontrollable eating and compensatory behaviors such as vomiting, laxative, or diuretic overuse that occur at least once a week over 3 months or more. Unlike patients with anorexia nervosa, patients with bulimia nervosa typically have a normal BMI. Patients with
the binging/purging type of anorexia or bulimia nervosa can demonstrate dental caries from gastric hydrochloric acid erosion of enamel, parotid gland swelling, and scars on the knuckles secondary to abrasions from the incisors when inducing vomiting. The loss of
gastric hydrochloric acid leads to hypochloremia and metabolic alkalosis. In severe cases, signs of hypovolemia such as tachycardia and hypotension may be present. Treatment of bulimia nervosa is through a combined medical and psychiatric approach and
involves correcting fluid and electrolyte derangements alongside behavioral and pharrnacologic therapy.

Incorrect Answers: A, B, D, E, F, and G.

Arthritis (Choice A) is not a known eating disorder complication. Decreased bone mineral density and an increased risk for stress fractures are possible musculoskeletal sequelae of eating disorders, which are related to malnutrition and estrogen deficiency.

Decreased body hair (Choice B) is a common physical examination finding in anorexia nervosa resulting from decreased protein stores. The severe malnutrition and associated examination findings in anorexia nervosa are less common in bulimia nervosa.

Galactorrhea (Choice D) refers to abnormal, non-physiologic discharge of milk from the nipple. It typically results from hyperprolactinemia, which can occur due to medications (eg, dopamine antagonists), prolactinomas, and hypothyroidism (increased thyrotropin­
releasing hormone stimulates prolactin release). Patients with bulimia nervosa do not possess any of these risk factors for galactorrhea.

Pituitary adenoma (Choice E) may present with headaches, bitemporal hemianopsia (from optic chiasm compression), and hyper- or hypopituitarism. Bulimia nervosa may present with headaches and amenorrhea (though amenorrhea is more commonly seen in
anorexia nervosa). Bulimia nervosa and pituitary adenoma do not typically co-occur.

Tachycardia (Choice F) may occur in severe cases of the purging type of anorexia or bulimia nervosa due to hypovolemia or torsades de pointes from hypokalemia. However, dental caries are much more common and can occur in less severe cases.

Thyroid gland enlargement (Choice G) would not be expected in patients with bulimia nervosa. The binging phase of bulimia nervosa may stimulate thyroid activity, but thyroid enlargement would be atypical.

Educational Objective: Bulimia nervosa (purging type) involves cycles of uncontrollable eating and compensatory behaviors such as vomiting or overuse of laxatives or diuretics. Patients with the binging/purging type of bulimia nervosa can demonstrate dental
caries from gastric hydrochloric acid erosion of enamel.

r r , fr!i F· r fl"
Previous Next Score Report Lab Values Calculator Help Pause
Exam Secllon 2: Item 6 of 50 National Board pf M41dlcal Exftllllners®
Comprehensive Da&lt science Serr-Assessment

✓ 6. An investigator obtains a serum sample from an individual previously immunized with a vaccine that elicits immunity to C/ostridium tetani. Administration of this serum to a guinea pig prior to challenge with a potentially lethal dose of C. tetani is most likely to
mediate protection to the challenge dose through binding of antibody to which of the following products of the bacterial cells?

A ) Capsular polysaccharide
B ) Cell wall polysaccharide
C) Exotoxin
D ) Fimbria
E) Flagellum
F ) Peptidoglycan
Correct Answer: C.

C/ostridium tetani is 1he bacterial cause of tetanus, resulting in the development of spastic paralysis following bacterial inoculation via a penetrating traumatic wound. Physical examination findings include trismus, risus sardonicus, opisthotonos, and rigid abdominal
muscles. In neonates, a foul-smelling and erythematous umbilical stump that was cut at birth using non-sterile technique is associated with C. tetani infection. The major virulence factor is tetanospasmin, which is a protease exotoxin that cleaves the soluble NSF­
attachment protein receptor (SNARE) complex in Renshaw cells of the spinal cord, preventing the release of inhibitory GABA and glycine neurotransmitters by impairing the fusion of vesicles. II is carried from the site of inoculation to the spinal cord via retrograde
axonal transport. Infection can be prevented by immunization with a tetanus toxoid vaccine. The tetanus toxoid is a modified exotoxin that is no longer virulent but still stimulates an antigenic response in the host. The antibodies produced after immunization target
the exotoxin of C. tetani, disabling its function and preventing the onset of spastic paralysis.

Incorrect Answers: A, B, D, E, and F.

Capsular polysaccharide (Choice A) targets are used to make vaccines for pneumococcal disease, meningococcal disease, and Salmonella typhi. C/ostridium tetani does not have a capsule.

Cell wall polysaccharide (Choice B) is a bacterial virulence factor that forms a glycocalyx projecting from the bacterial cell wall. A glycocalyx protects bacteria from phagocytosis and aides in the adherence to other bacterial and inert materials.

Fimbria (Choice D) is a virulence factor that aides in the adherence and colonization of bacteria to host cells. It is composed of glycoproteins and is a common virulence factor seen in Gram-negative bacteria, especially E. coli.

Flagellum (Choice E) is a virulence factor that increases bacterial mobility and chemotaxis. It is composed of the protein flagellin. A flagellum is not a virulence factor present in the bacteria C. tetani.

Peptidoglycan (Choice F) is a polymer of sugars and amino acids that provides stability to the bacterial cell wall. 13-lactam antibiotics inhibit the transpeptidase cross-linking of peptidoglycan in the cell wall.

Educational Objective: Toxoid vaccines are composed of exotoxins that have been chemically modified to remove their virulence. They stimulate the host production of antibodies that target the virulent toxin, disabling its function. These toxin antibodies would
�revent the onset of spastic paralysis if inoculated with C. tetani.

r
Previous
,.
Next
,,.
Score Report
@
Lab Values
P
Calculator

Help
1ft'
Pause
Exam Secllon 2: Item 7 of 50 National Board pf Mqdlcal Examiners®
Comprehensive saslc Science Serr-Assessment

)( 7. A 7-year-old boy is about to undergo an appendectomy. An intravenous catheter needs to be inserted, but the patient is extremely fearful of being stuck with a needle while awake. The most appropriate anesthesia administered by mask to anesthetize this
patient quickly would have which of the following characteristics?

A ) High blood solubility


B ) High cerebrospinal fluid solubility
C) High lipid solubility
D) Low blood solubility
E ) Low lipid solubility
Correct Answer: D.

The rate of induction, or onset of anesthesia, with volatile anesthetics relies on blood solubility, cardiac output, and minute ventilation. Induction is primarily dependent on t�e partial pressure of inhaled volatile anesthetic present in the alveoli. Solubility in this case
can be equated with the blood gas partition coefficient, which describes the relative division between two phases that is typically taken by an anesthetic. Low blood solubility increases the speed of induction, as it allows the alveolar concentration to quickly come
into equilibrium with the concentration of anesthetic in the blood, preventing further anesthetic from being removed by the blood. Similarly, low cardiac output decreases the amount of blood delivered to the alveoli, and the anesthetic will more quickly equilibrate with
the blood that is present. Increases in minute ventilation also speed induction time by delivering more anesthetic to the alveoli, leading to a more rapid increase in the gas concentration. The converse of these statements is true, and induction speed decreases with
high blood solubility, high cardiac output, and low minute ventilation. Therefore, choosing a volatile anesthetic agent with low blood solubility will lead to rapid induction of anesthesia.

Incorrect Answers: A, B, C, and E.

High blood solubility (Choice A) will delay the speed of induction, as ii increases the amount of time it takes for the concentration of gas in the alveoli to reach equilibrium with the concentration of anesthetic in the blood.

High cerebrospinal fluid solubility (Choice B) would also increase the time required for induction, as anesthetic would be rapidly taken up into the cerebrospinal fluid from the bloodstream, increasing the time it takes for alveolar gas concentration to equilibrate with
blood anesthetic concentration.

Neither high lipid solubility (Choice C) or low lipid solubility (Choice E) would affect induction time. Potency of volatile anesthetic is dependent on lipid solubility, with those medications that are highly lipid soluble being much more potent.

Educational Objective: Induction time of anesthesia with volatile anesthetics depends on the blood solubility of the anesthetic, cardiac output, and minute ventilation. Low blood solubility decreased cardiac output, and high minute ventilation will increase the speed
of induction. Conversely, high blood solubility, increased cardiac output, and low minute ventilation will decrease the speed of induction.

r r , ta F r r-
Prev1ous Next Score R eport La_b Values Calculator Help Pause
_ .
Exam Section 2· Item 8 of SO National Bo,ard gf Medical Examiners®
ComprehensN"e Basic Science Sen-Assessment

'I( 8. A 56-year-old man dies 1 month after the onset of difficulty with short-term memory and anxiety. He underwent surgical excision of small cell carcinoma 1 year ago. Degenerative changes are most likely to be concentrated at which of the following labeled
sites on the normal MRI of the head?

A)
B)
C)
D)
E)
F)

Correct Answer: C.

Memory and emotional regulation are mediated by the limbic system, which includes the amygdala, mammllary bodies, stria medullarls, hlppocampus, clngulate gyrus, prefrontal cortex, parahlppocampal gyrus, ventral tegmenlum, and multiple deep nuclei. Label
·c· in the axial T2-weighted MRI image shown identifies the mesial temporal lobe, where the hippocampus and parahippocampal gyrus are located. The hippocampus is particularly sensitive to radiation, and degenerative changes or damage of the hippocampus
commonly resutt in shof1-term memory loss. Chemothera py has also been shown to injure hippocampal synapses. This patient's symptoms of short-term memOfY loss, anxiety. and a history of malignancy suggest damage to the limbic system with likely involvement
of the hippocampus and mesial temporal lobe. Mesial temporal lobe atrophy is also associated with Alzheime r disease and frontotemporal dementia.

Incorrect Answers: A, B, D. E. and F.

Choice A identifies the eye and superior orbil in which rectus muscles and the globe are visible. These structures are involved in vision and do not play a role in memory or emotional regulation.

Choice B identifies the region of the frontal pole. inferior frontal gyrus. straight gyrus, olfactory gyrus, and ol!acto,y sulcus. Olfactory sensations are closely related to emotional processing and memory. However, this region is less likely than the mesial temporal lobe
to result in short-term memo,y loss.

Choice D identifies the right substantia nigra. The substantia nigra is a nucleus for dopaminergic cells and is cntical for the modulation of movement in coordination with the basal ganglia. Pathology affec1ing this area is associated with Parkinson disease.

Choice E identiftes the superior cerebellar vermis. Chronic alcoholism is associated with atrophy of this region. Atrophy and lesions of this area are associated with truncal ataxia and gaH d11turt>anoes.

Choice F identifies the right occipital lobe. associated with visual processing. Lesions in this area would typically present wtth cont ralateral homonymous hemianopsia and IVOUld be unlikely to affect memory or emotional processing.

Educational Objective: Memory and emotional regulation are mediated by the limbic system. which includes the hippocampus. parahippocampal gyrus, and amygdala, located in the mesial temporal lobe. Cancer-related radiation and chemotherapy commonly
damage these mesial temporal lobe structures. leading to emotional dysregulation and the development of shof1-term memory loss.

r r , tr!: r- r r
Previous Next Seore Report Lab Values Caleulator Help Pause
Exam Section 2: Item 9 of 50 National Board pf Medical Examiners®
Comprehensive Basic Science SelT•ASsessment


✓ 9. A 45-year-old man comes to the physician because of progressively worsening, constant pain in his left thigh over the past 3 months. He is a long-distance runner. An x-ray of the femur shows thickening of 'I

the diaphysis and disruption of the cortex with focal areas of increased calcification. A glistening mass containing several cysts is surgically excised. A photomicrograph of tissue from the mass is shown.
Which of the following is the most likely diagnosis?

A)- A fr- ture



B ) Chondrosarcoma
Q


C ) Enchondroma

,,
Ct- fa, g sarcoma
Ii t- Giant eel carcinoma
J; t- Multilobulated

G-) Oste0saFooma
bone cyst

.,,
z.,
t

�"'
Correct Answer: B.

Chondrosarcoma is a malignant neoplasm that arises from the mesenchymal cells that produce cartilage. It commonly occurs in the axial skeleton, particularly in the pelvis, and often in patients in the fifth decadle of life. It is a slow-growing tumor that is initially
asymptomatic, permitting extensive growth before detection. Pelvic chondrosarcoma will often present with pelvic pain and compression of adjacent structures such as the lumbosacral plexus, leading to neurologic symptoms in the ipsilateral lower extremity.
Chondrosarcoma can be primary or arise secondary to Paget disease of the bone or prior radiation therapy. On histology, the tumor will demonstrate lobules of hyaline-like cartilage and nests of malignant chondrocyte-like cells. Radiographs will demonstrate
disorganized calcifications that appear in rings and arcs. Treatment is predominantly surgical resection. Chemotherapy and radiation have not been shown to be effective.

Incorrect Answers: A, C, D, E, F, and G.

Atypical stress fractures (Choice A) are pathologic fractures that occur secondary to long-term bisphosphonate use. Bisphosphonates inhibit osteoclasts thereby decreasing bone resorption and increasing bone mineral density, decreasing the risk of fracture in
osteoporosis. However, this also results in decreased bone remodeling in response to physiologic stress, which can lead to pathologic fractures, classically of the subtrochanteric region of the femur.

Enchondroma (Choice C) is a benign bone tumor that often occurs in the metacarpals and phalanges of the fingers. They are often painless and can be treated with observation alone. OIiier disease consists of multiple enchondromas that may present in a single
extremity. Maffucci syndrome presents with multiple enchondromas and soft tissue angiomas.Ewing sarcoma (Choice D) is a primary bone malignancy driven by 1(11;22) chromosomal translocation. On histology, Ewing sarcoma demonstrates multiple small,
round, blue cells. Radiographs may demonstrate elevated periosteum and layered neocortex formation ("onion skin" appearance). It often occurs in the long bones (eg, femur, tibia) and typically presents in adolescence.

Giant cell carcinoma (Choice E) is a rare carcinoma of the lung that contains pleiomorphic giant, multinucleated cells on histology. By contrast, giant cell tumors of bone are comprised of mononuclear stromal cells and multinucleated giant cells and are not a type
of carcinoma. Giant cell tumors of bone are generally nonmalignant but are aggressive and may invade local structures. They often arise in the distal femur.

Bone cysts (Choice F) are fluid-filled cavities within the bone, which are surrounded by a wall of fibrous tissue and may be mulmobulated. They typically occ,yr in children near the physis. They can cause a pathologic fracture due to thinned cortical bone, although
they are benign.

Osteosarcoma (Choice G) is an aggressive, malignant bone tumor that occurs mostly in adolescents. II is the most common primary malignancy of bone and arises from mesenchymal cells with osteoblast-like qualities. The tumor cells produce osteoid, that is,
the collagen matrix necessary for mineralization and bone formation. Clinical findings include pain, swelling, and erythema, with radiographs demonstrating 1Periosteal elevation adjacent to the lesion.

Educational Objective: Chondrosarcoma is a malignant neoplasm that arises from the mesenchymal cells that produce cartilage. It commonly arises in the axial skeleton, particularly in the pelvis. On histology, the tumor will show lobules of hyaline-like cartilage
and nests of malignant chondrocyte-like cells.

r r , � F � r
Previous Next Score Report Lab Values Calculator Help Pause
I Exam Section 2: 11em 10 of 50
NaUonal Board ol Medical Examiners®
Comprenenslve Basic Science Set1-ASsessmen1

✓ 10. A 34-year-old man has a herniated lumbar intervertebral disc. Laminectomy and removal of the herniated fragment are scheduled. Which of the following labeled sites on the normal lumbar axial CT scan shown is the most appropriate location for surgical
entrance to the neural canal?

A)
B)
C)
D)
E)
Correct Answer: D.
Disc herniations occur when the nucleus pulposus herniates through the outer fibrocartilaginous ring of the intervertebral disc, the annulus fibrosus. Disc herniations can occur from acute trauma, but often have an underlying degenerative component. Degeneration
of intervertebral discs is characterized by decreased water content arnd ftexibility of the spinal column. When the nucleus pulposus herniates through the annulus, it can cause direct compression of neural elements as well as irritation, cytokine release, and
inflammatory cell infiltration in the area of the affected nerve root. Often, acute disc herniation occurs laterally in the spinal canal due to the integrity of the posterior longitudinal ligament. This can cause acute radiculopathy, which classically presents with pain in a
dermatomal distribution, decreased sensation, and weakness in the corresponding myotome. Typically, the affected nerve root is that which exits the spinal canal one level below the disc herniation, because the nerve root is compressed as it traverses distally prior
to exiting the neural foramen. Compression of the distal aspect of the cord, known as conus medullaris syndrome, will present with bilateral upper motor neuron type {eg, hyperreflexia, spastic paresis) lower extremity weakness, perinea! numbness, and urinary
retention leading to overflow incontinence. Cauda equina syndrome is the compression of the bundle of nerve roots that dwell in the spinal canal distal to the conus medullaris. Compression of the cauda equina can cause similar symptoms to conus medullaris
syndrome, however, deficits are more likely to be asymmetric as the bilateral roots are seldom compressed in identical fashion, and it typically results in lower motor neuron type weakness (e.g, hyporeflexia). Along with anti-inftammatory medications, surgical
decompression may be indicated. During a disc decompression, portions of the posterior elements of the lumbar vertebrae are removed. Commonly, the spinal lamina (the bony arch connecting the pedicles, transverse, and spinous processes) is excised to permit
access to the herniated disc.
lncorrecti Answers: A, B, C, and E.

The vertebral body (Choice A) is the anterior portion of the spinal column. The intervertebral disc sits between each vertebral body and allows for shock absorption as well as relative motion between the vertebral bodies. Injuries to the vertebral body include
compression fractures in osteoporosis and burst fractures.

The pedicle (Choice B) is a stout bony pillar that connects the vertebral body to the posterior elements of the spinal column. Nerve roots exit above and below these pedicles through the neural foramina.
The transverse processes (Choice C) of the vertebrae serve as origination points for the psoas muscles, as well as longisslmus and multifidus muscles.
The spinous process (Choice E) is the posterior-most bony element of the vertebrae. The interspinous ligaments attach between these processes, serving to stabilize and resist forward flexion of the spine. These processes serve as an origin for the laUssimus dorsi
muscles.

Educa tional Objective: Injury to the spinal column involving bony, ligament, or disc disruption can lead to compression of neural el'ements. This presents as acute neurologic symptoms such as pain, weakness, and change in sensation in a dermatoma l and
myotomal distribution. The lamina serves as the most appropriate location for surgical entrance to the spinal canal to access the herniated disc.

r r , If: F � r
Previous Next Score Report lab Values Calculator Help Pause
Exam Section 2: Item 11 of 50 National Board pf Medical Examiners®
Comprehensive 1:1as1c Science Self-AsseBBment

✓ 11. A bronchial smooth muscle preparation is maintained in a tissue bath. Bioactive substances can be applied to the smooth muscle, and contraction and relaxation of the smooth muscle can be measured. The results .Q (I)
(/) "'

-u­
of administration of thromboxane A 2 (TXA 2) and prostaglandin E 2 (PGE 2) are shown. Which of the following is the most likely mechanism of the effect of application of the PGE 2? c<ll
Q) �

---..PGl:i
(.)
A }-Activation of phospholipase Ai Q) C:

(/) ---"'. •
B ) Closing K + channels ::, <I> TXA2
E :@
C) Increase in cAMP £�
0 (.)
D) Increase in inositol 1,4,5-trisphosphate 0 (I)
EQ
e} Inhibition of5 lipoxygonaso (/)

Correct Answer: C.

Smooth muscle cell contraction occurs when myosin light chains are phosphorylated, leading to cross-bridge formations between myosin heads and actin filaments. This reaction is performed by myosin light chain kinase (MLCK). This process is opposed by
myosin light chain phosphatase (MLCP), which dephosphorylates the myosin light chains resulting in decreased muscle tone. Prostaglandin E2 (PGE2 ) stimulates adenyly1 cyclase in smooth muscle cells, which leads to an increase in cAMP. cAMP inhibits the
activity of myosin light chain kinase, resulting in decreased phosphorylation of myosin light chains, which leads to decreased smooth muscle tension.

Incorrect Answers: A, B, D, and E.

Activation of phospholipase A2 (Choice A) occurs in proinflammatory states. Phospholipase A2 catalyzes the conversion of phospholipids into arachidonic acid, which is then modified into eicosanoids (such as thromboxane A2 ) to mediate inflammatory effects.

Closing K+ channels (Choice B) is not a direct action of PGE2 . K + channels are ubiquitous on cell membranes and important in regulating action potentials and membrane depolarization/repolarization.

Increase in inositol 1,4,5-triphosphate (Choice D) causes increased smooth muscle tension. Inositol 1,4,5-triphosphate (IP3) is a signaling molecule that increases intracellular calcium release from the sarcoplasmic reticulum of muscle cells. Calcium binds to
calmodulin, and the calcium-calmodulin complex activates MLCK, leading to increased muscle contraction.

Inhibition of 5-lipoxygenase (Choice E) decreases the production of leukotrienes from arachidonic acid. 5-lipooxygenase inhibitors are commonly used in the treatment of asthma. PGE2 does not affect the activity of 5-lipoxygenase.

Educational Objective: Prostaglandin E 2 is a potent bronchodilator that acts by stimulating adenylyl cyclase in smooth muscle cells to increase the intracellular concentration of cAMP. cAMP leads to smooth muscle relaxation by inhibiting myosin light chain
kinase.
Exam Section 2: Item 12 of 50 National Board pf Mqdlcal Examiners®
Comprehensive Basic Sctence Self-Assessment

✓ 12. A 25-year-old man comes to the physician because of sores on his penis for 2 days. Physical examination shows vesicular and ulcerative lesions on the penis. Drug therapy is ini:tiated. One week later, his symptoms resolve. Which of the following
enzymes is required to activate the most commonly prescribed drug?

A ) ATPase
B ) Cytochrome P450
C) Protease
D) Reverse transcriptase
E ) Th midine kinase

Correct Answer: E.

This patient's vesicular and ulcerative lesions on the penis are likely caused by herpes simplex virus in the form of genital herpes. Treatment for herpetic infections involves drugs that inhibit viral DNA polymerase, classically by guanosine analogs such as acyclovir,
valacyclovir, and farnciclovir. Prior to exerting their antiviral effects, guanosine analogs must be phosphorylated by the viral enzyme thymidine kinase. Phosphorylation is the process by which a phosphate group is added to the molecule. These molecules are then
able to inhibit the viral DNA polymerase by terminating the nascent DNA chain during replication. These drugs are effective against herpes simplex virus and varicella zoster virus, weakly effective against Epstein-Barr virus, and not effective against
cytomegalovirus. Development of a mutation in the viral thymidine kinase enzyme would prevent drug phosphorylation and confer resistance to guanosine analog medications.

Incorrect Answers: A, B, C, and D.

ATPases (Choice A) are a heterogenous group of enzymes that hydrolyze ATP into ADP and a molecule of inorganic phosphate. Guanosine analogs require phosphorylation, not dephosphory1ation, in order to be active.

Cytochrome P450 (Choice 8) enzymes are found in the liver and play a key role in the metabolism of many drugs. In most cases, this process alters a drug from its active form to an inactive metabolite. However, some drugs are converted from their prodrug form
into an active metabolite by the P450 enzymes, such as clopidogrel, but guanosine analogs are not among them.

Proteases (Choice C) cleave the initial polypeptides produced by the translation of viral RNA into smaller, functional parts. Protease inhibitors are used in the treatment of HIV and include darunavir, indinavir, ritonavir, and saquinavir.

Reverse transcriptase (Choice D) transcribes viral DNA from viral mRNA for incorporation into host DNA. This enzyme is inhibited by both nucleoside reverse transcriptase inhibitors, such as abacavir and didanosine, and non-nucleoside reverse transcriptase
inhibitors, such as efavirenz and nevirapine. These medications are used to treat HIV.

Educational Objective: Acyclovir, famciclovir, and valacyclovir are guanosine analogs that inhibit viral DNA polymerase and are used to treat herpes simplex virus infections. Guanosine analogs require phosphorylation by thymidine kinase to be activated.

r
Previous
r
Next
,
Score Report

Lab Values
F
Calculator
r
Help
,,.
Pause
Exam Section 2: Item 1J of 50 National Board of Mqdlcal Examiners®
Comprehensive aas1c Sctence Serr-Assessment

y 13. A 32-year-old woman comes to the physician because she is concerned about fertility. Menses occur at regular 28-day intervals. Physical examination shows no abnormalities. An endometrial biopsy specimen is obtained from tihe uterine wall. Histologic
sections show a well-develoQed stratum basale, intact spiral arteries, abundant endometrial stroma, and long, straight uterine glands with no secretions in their lumens. Based on this description, which of the following labeled options best corresponds to the
point in the patient's menstrual cycle at which the biopsy was done?

Menses
Ovulation

C)
D)

Correct Answer: B.

The first half of. the menstrual cycle, the follicular phase, begins with menses. After menses, follicle-stimulating hormone and luteinizing hormone (FSH and LH, respectively) levels begin to increase, which stimulates the development of a follicle. The follicle
produces estrogen, which leads to the proliferation of the endometrium. This proliferative endometrium consists of highly active endometrial epithelial cells, stromal cells, straight tubular endometrial glands, and extension of the uterine spiral arteries, as seen in this
patient. As estrogen rises, a surge occurs, which in turn stimulates a surge in LH that causes ovulation 14 days prior to menses. Immediately following ovulation, the luteal (secretory) phase begins as the corpus luteum forms. The corpus luteum secretes
progesterone to produce and maintain a secretory endometrial lining, which consists of tortuous glands full of secretions, full length spiral arteries, and edematous stromal cells. If no implantation occurs, the corpus luteum degrades to the corpus albicans, causing
estrogen and progesterone levels to decrease, resulting in menstruation. The loss of progesterone causes the endometrial lining to slough off due to constriction of the spiral arteries, and the follicular phase begins again.

Incorrect Answers: A, C, and D.

Immediately after menses (Choice A), the endometrium would be much less developed, as the entirety of the stratum functionalis (eg, the superficial layer) would have been sloughed off during menses. During menses, the spiral arteries retract into the stratum
basalis and the glands are partially lost. Well-developed spiral arteries and uterine glands, as well as abundant stroma, would not be expected immediately after menses.

The secretory phase (Choices C and D) consists of the time from ovulation to menstruation, which is typically 14 days in length. The endometrium undergoes differentiation during this time secondary to progesterone stimulation, with increases in the tortuosity of the
uterine glands and amount of secretions, further development of the stromal cell layer, and extension of the spiral arteries to cover the entire thickness of the endometrium. The histologic findings in this biopsy are more consistent with those found in the late
proliferative phase, prior to ovulation.

Educational Objective: Proliferative endometrium is present during the follicular phase of menstruation prior to ovulation. Histologically, it demonstrates straight uterine glands, extension of the spiral arteries, and abundant stromal cells.

r
Previous
r
Next
,
Score Re ort
fa
Lab Values
P
Calculator
r
Hel
"'
Pause
Exam Section 2: Item 14 of 50 National Board of Medical Examiners®
Comprehensive tsas1c science Self-ASsessment

✓ 14. An 18-month-old boy is brought to the emergency department because of lethargy for 3 hours. He has not eaten well for the past 24 hours. He had cardiorespiratory arrest associated with hypoglycemia after an episode of diarrhea 10 months ago. On
arrival, he is unresponsive. His temperature is 37°C (98.6° F), pulse is 140/min, respirations are 25/min, and blood pressure is 100/60 mm Hg. Physical examination shows hepatomegaly. Laboratory studies show:
Serum
Glucose decreased
Ketones decreased
Camitine decreased
Urine
Ketones decreased
Dicarboxylic acids present

Following intravenous administration of glucose, he becomes responsive. Medium-chain triglycerides are administered daily for 6 months. At a follow-up examination, his serum glucose concentration is within the reference range. A deficiency of which of
the following enzymes is the most likely cause of these findings?

A ) Glucose 6-phosphatase
B) HMG-CoA lyase
C) Hormone-sensitive lipase
D) Lipoprotein lipase
E ) Long-chain acyl-CoA dehydrogenase
F ) Phosphoenolpyruvate carboxykinase
Correct Answer: E.

Long-chain acyl-CoA dehydrogenase is an enzyme involved in the beta-oxidation of long-chain fatty acids. Deficiency of this enzyme is an autosomal recessive disorder and results in the accumulation of long-chain fatty acids in the mitochondrial matrix. It is
characterjzed by two forms: an early-onset form that results in severe, life-threatening cardiomyopathy, and a late-onset form that is characterized by episodes of symptomatic hypoglycemia. Signs and symptoms may include irritability, lethargy, hepatomegaly (due
to excess fat accumulation), and hypoglycemia. Symptomatic episodes can be triggered by illness, fasting, stress, or exercise. Hypoglycemia occurs without the production of ketones due to impaired fatty acid metabolism and ketone body production since there is
deficient production of acetyl-CoA from long-chain fatty acids. Immediate treatment involves correction of the patient's hypoglycemia and long-term management involves preventing acute episodes of hypoglycemia and dietary supplementation with medium-chain
fatty acids, which will utilize medium-chain acyl-CoA dehydrogenase for beta-oxidation, circumventing the patient's enzymatic deficiency for long-chain fatty acid beta-oxidation.

Incorrect Answers: A, B, C, D, and F.

Glucose 6-phosphatase (Choice A) is an enzyme located in the liver that functions in gluconeogenesis and converts glucose-6-phosphate to glucose for its release into the blood. Impaired gluconeogenesis can lead to severe hypoglycemia during periods of fasting.
Glycogen storage disease type I (von Gierke disease) is secondary to glucose 6-phosphatase deficiency and is characterized by excess glycogen in the liver, fasting hypoglycemia, increased levels of triglycerides, uric acid, and hepatomegaly.

HMG-CoA lyase (Choice B) is an enzyme that converts HMG-CoA into acetoacetate in the mitochondria and is key in ketogenesis during periods of fasting. Deficient ketogenesis can present with vomiting, diarrhea, dehydration, lethargy, and hypotonia.

Hormone-sensitive lipase (Choice C) functions in the metabolic degradation of stored triglycerides in adipocytes, resulting in the release of free fatty acids into the bloodstream. It is inhibited by insulin during the fed-state. Lipoprotein lipase (Choice 0) functions in
the degradation of triglycerides and release of free fatty acids in both chylomicrons and very low-density lipoprotein (VLDL). Deficiency can lead to accumulation of triglycerides in the body with resultant xanthomata, hepatosplenomegaly, and pancreatitis.

Phosphoenolpyruvate carboxykinase (Choice F) is an enzyme that catalyzes the irreversible conversion of oxaloacetate to phosphoenolpyruvate, a key regulatory step in gluconeogenesis. Deficiency can lead to hypoglycemia, failure to thrive, hepatomegaly, and
lactic acidosis. Treatment includes a diet with simple carbohydrates, especially during periods of fasting, illness, or exercise.

Educational Objective: Long-chain acyl-CoA dehydrogenase deficiency is an auiosomal recessive disorder of fatty acid metabolism. Signs and symptoms include irritability, lethargy, hepatomegaly (due to excess fat accumulation), and hypoglycemia. Episodes can
be triggered by illness, fasting, stress, or exercise. Immediate treatment involves correction of hypoglycemia and long-term management involves preventing acute episodes of hypoglycemia and dietary supplementation with medium-chain fatty acids.

r r , fr!i P r trt
Previous Next Score Re ort Lab Values Calculator Hel Pause
Exam Section 2: Item 15 of 50 National Boa.rd pf Mt1dlcal Examiners®
Comprehensive saslc Science Self-Assessment

✓ 15. Histone acetyltransferases catalyze the acetylation of lysine residues in the amino-terminal tails of histones. Which of the following is the most likely effect of this covalent modification on chromatin structure?

A ) Decreases the affinity of histones for DNA


B}- Decreases the Allcleosome coAtent of the A1Jcle11s
G}- IAcreases the affiAiP/ of histoAes fQr DNA
D}-IAcreases the A1JcleosomesoAleAt of the A1Jcleus
Ii}- Removes hisloAe H1 from DN/1.
�}- Remo>Jes hisloAe H4 fl:om DNA
Correct Answer: A.

Histone acetyltransferases decrease the affinity of histones for DNA by adding acety1 groups to histones lysine residues. Histones are proteinaceous cores, rich in lysine and arginine (positively charged amino acids). Without modification (eg, acetylation), the
positively charged histones attract negatively charged DNA, which loops around it, forming a chromatin nucleosome. Condensed heterochromatin is transcriptionally inactive. Acetylation of a histone is important in gene transcription as it permits relaxation of a DNA­
histone complex by reducing the molecular affinity between DNA and histone. When DNA is tightly bound to a histone, gene transcription does not occur, since binding sites for RNA polymerase II (which transcribes messenger RNA) will be occupied by the
presence of the histone. When the affinity between the histone and DNA is reduced, the unpacked (euchromatin) DNA can be bound by transcriptional enzymes, promoting gene expression.

Incorrect Answers: B, C, D, E, and F.

Decreasing the nucleosome content of the nucleus (Choice B) implies dissolution of the nucleosome structure, a discrete length of DNA (usually 126 base pairs, or nearly two turns of DNA) that is wrapped around a set of eight histones (H2A, H2B, H3, and H4)
forming a histone octamer. During transcription or DNA replication for cell division, the nucieosome is transiently disassembled, but then rapidly reassembled. Conversely, increasing the nucleosome content of the nucleus (Choice D) implies the formation of more
nucleosomes. Adding additional nucleosomes to existing DNA would indicate winding the chromatin tighter, but the tight regulation of DNA folding prevents this from occurring. The addition of an extra chromosome, however, such as in trisomy 21, would technically
increase the nucleosome content of the nucleus.

Increasing the affinity of histones for DNA (Choice C) is accomplished by the enzyme histone deacetylase. It results in an increased affinity of the histone for DNA, causing the formation of heterochromatin.

Removing histone H1 from DNA (Choice E) is not correct because this occurs via the action of histone chaperone proteins. Removing H1, which normally links nucleosomes together, allows for DNA unwinding.

Removing histone H4 from DNA (Choice F) is not correct as histone acetyltransferase only modifies existing histones.

Educational Objective: Histone acetyltransferase serves to add acetyl groups to histone lysine residues, causing a decreased affinity of histone for DNA. This results in relaxation of the DNA-histone complex (euchromatin), which allows for gene transcription.

r
Previous
r
Next
,
Score Re ort
� P
Lab Values Calculator
r-
Hel
r
Pause
Exam Section 2: Item 16 of 50 National Board of Medical Examiners®
Comprehensive tsaslc Science Self-Assessment

y 16. A 57-year-old woman comes to the physician because of chronic shortness of breath and swollen legs for 2 months. Her respirations are 12/min. Physical examination shows edema of the lower, extremities. Auscultation of the chest shows each cardiac
systole associated with a thrusting impulse palpable along the left sternal border, most prominently in the third, fourth, and fifth intercostal spaces. Which of the following disorders is the most likely cause of this cardiac finding?

A} Aneurysm of the ascending aorta


B } Left atrial hypertrophy
C} Left ventricular hypertrophy
D} Pericardia! effusion
E } Right atrial hypertrophy
F } Right ventricular hypertrophy

Correct Answer: F.

The physical examination findings of right ventricular hypertrophy may include a parasternal heave (also called a precordial heave). which is an impulse observed or palpated along the left sternal border. The right ventricle is the most anterior of the four heart
chambers and lies deep to the third, fourth, and fifth intercostal spaces when in its usual anatomic location. Right ventricular hypertrophy is a consequence of increased right ventricle pressure, which most commonly occurs secondary to chronic lung disease
resulting in increased pulmonary vascular resistance and resultant pulmonary hypertension. Chronic lung disease is a risk factor for the development of pulmonary hypertension due to diffuse hypoxia-induced vasoconstriction and pulmonary vascular remodeling.
When an area of lung has low oxygen concentration, reflexive contraction of vascular smooth muscle diverts blood flow to a region with increased ventilation. This is called ventilation-perfusion matching. Chronic, severe lung disease leads to widespread pulmonary
vasoconstriction, and the increased resistance to flow results in increased right ventricle pressures and development of right ventricular hypertrophy. Other etiologies of right ventricular hypertrophy include primary pulmonary hypertension, severe tricuspid
regurgitation, high altitude, left to right cardiac shunts, atrial or ventricular septa! defects, anomalous pulmonary venous return, E isenmenger syndrome, pulmonic valve stenosis, and cardiomyopathy. Patients often present with signs and symptoms of right-sided
heart failure, including dyspnea, dependent edema, jugular venous distention, and hepatic congestion.

Incorrect Answers: A, B, C, D, and E.

Aneurysm of the ascending aorta (Choice A) is a dilatation of the three layers of the aortic wall proximal to the aortic arch. They are often asymptomatic unless ruptured or infected. Physical examination may reveal an aortic murmur or bruit. While abdominal aortic
aneurysms are often palpable on physical examination, thoracic lesions are not.

Left atrial hypertrophy (Choice 8) can result from mitral stenosis, which presents with a murmur classically heard as an opening snap followed by a diastolic rumble, loudest over the cardiac apex with radiation to the axilla. Severe left atrial hypertrophy may present
with a discernible parastemal heave, though this is much rarer than right ventricular hypertrophy.

Left ventricular hypertrophy (Choice C) is a common disorder that may be caused by chronic systemic hypertension, aortic stenosis, or hypertrophic cardiomyopathy. It is associated with an S 3 or S4 gallop, a crescendo-decrescendo systolic murmur (if aortic
stenosis is present} and may rarely cause a parastemal heave. Right ventricular hypertrophy is a much more common cause due to its anterior location.

Pericardia! effusion (Choice D} refers to the presence of an abnormal amount of fluid in the pericardium. If severe enough, it can potentially progress to cardiac tamponade with compression of the cardiac chambers and obstructive shock. II typically causes the
equalization of pressures amongst the four heart chambers. Physical examination findings include Beck triad when in tamponade (hypotension, distended jugular veins, and distant heart sounds}.

Right atrial hypertrophy (Choice E} is often a result of tricuspid regurgitation, tricuspid stenosis, or right ventricular hypertrophy, though a parasternal heave is less likely to be felt as the right atrium is retrosternal and has much weaker contractile force than the right
ventricle.

E ducational Objective: A parasternal heave on physical examination is most commonly associated with right ventricular hypertrophy, which is the most anterior chamber of the heart. Right ventricular hypertrophy results from pulmonary hypertension, which is
frequently a complication of chronic lung disease due to hypoxia-induced pulmonary vasoconstriction and vascular remodeling.

r r , '1!i P r trt
Previous Next Score Report Lab Values Calculator Help Pause
Exam Section 2: Item 17 of 50 National Board pf M41dlcal Examiners®
Comprehensive sas1c SClence Self-Assessment

✓ 17. To investigate the association between cellular telephone use and the development of brain tumors, 782 patients with brain tumors are compared with 799 age- and sex-matched patients who do not have a malignancy. The results of the study find that
there was no increase in the odds ratio for cellular telephone use in patients who had brain cancer compared with those who did not have cancer. Which of the following best describes this study design?

A ) Case-control study
B ) Case series
C) Cross-sectional study
D) Prospective cohort study
E ) Randomized clinical trial
F ) Retrospective cohort study
Correct Answer: A.

A case-control study investigates an association between exposure and an outcome. In this study design, a group of patients with the disease (cases) are identified. A group of patients without the disease (controls) are matched on baseline characteristics to the
cases. Exposure data for the two groups is collected, and these data are compared to determine association with the outcome (disease) in question. An odds ratio may be calculated to compare exposures between groups.

Incorrect Answers: B, C, D, E, and F.

A case series (Choice B) is a descriptive study design in which a number of consecutive or nonconsecutive cases of a disease and/or treatment are described in detail, with information about exposure, demographics, and comorbidities. Case series do not imply a
cause-and-effect relationship. They do not test a hypothesis nor are they randomized. They are useful in characterizing the natural history of a disease or response to treatment. They are also useful in describing rare diseases, as small amounts of patients may not
permit conduction of larger case-control, cohort, or randomized trials with sufficient power.

A cross-sectional study (Choice C) seeks to identify the prevalence of the condition at a particular point in time. An example of a cross-sectional study would be a single survey of a population inquiring whether patients have coronary artery disease and concurrently
inquiring about activity levels and diet. Thus, the risk factor and the outcomes are measured simultaneously. The study does not follow patients over time. All information is collected at a single time point.

A prospective cohort study (Choice D) identifies a group of patients and follows them over time seeking to identify whether an exposure is associated with an outcome of interest. In a prospective design, the hypothesis and analysis protocols are established prior to
the start of the study period. An example of a prospective cohort study would be following a group of 1,000 smokers for a time period of 10 years and identifying the proportion of these patients who develop pancreatic cancer to identify the risk of pancreatic cancer
in smokers.

A randomized clinical trial (Choice E) is an experimental study design. Patients are randomly allocated to two or more interventional arms or control arms, and these patients are followed over time to evaluate an outcome of interest. Randomized design minimizes
opportunity for bias; thus, a randomized interventional study can be used to imply causation. Common examples of randomized trials include therapeutic comparisons between a new drug and the previous standard of care.

A retrospective cohort study (Choice F ) identifies a group of patients and reviews them historically in previous time, seeking to identify whether an exposure is associated with an outcome of interest. In a retrospective design, the hypothesis or question is designed
after the study time period has passed.

Educational Objective: A case-control study compares individuals with a particular disease or outcome (cases) with a matched group selected from the same population (controls). This comparison allows for the identification of unique exposures in the case group.

r r , fr§ F r ff'
Previous Next Score Report Lab Values Calculator Help Pause
Exam Section 2: Item 18 of 50 National Board of Medical Examiners®
Comprehensive tsas1c Science Self-Assessment


!
Cell Membrane Phospholipids
18. The illustration shows the arachidonic cascade that is activated when skin is exposed to poison ivy. Within this cascade, which of the following is the major mechanism of analogs of corticosteroids?
Phospholipase A2
A }-Acceleration of the metabolism of arachidonic acid Arachi don ic Acid
Ii}- Inhibition of receptors mediating selected responses to prostaglandins and leukotrienes Cye looxygen ase Lipoxygenase

C) Nonselective inhibition of cyclooxygenase and lipoxygenase


Prostagland ins Leukotrien es
D) Regulation of phospholipase A 2
lit Selective iAl=libitioA oflipoxygeAase Tissue receptors Tissue receptors
Correct Answer: D.

Topical corticosteroids, such as triamcinolone, betamethasone, and clobetasol, are the treatment of choice for allergic contact dermatitis. These medications are analogues of the corticosteroids produced by the zona fasciculata of the adrenal gland. While
corticosteroids decrease levels of virtually all cytokines, in the arachidonic cascade they specifically inhibit phospholipase A2. They also inhibit NF-KB, a positive regulator of cyclooxygenase (COX) 2 and thus indirectly inhibit COX2. Allergic contact dermatitis is a
type IV hypersensitivity reaction caused by exposure of the skin to either an irritant or allergen causing an eczematous rash to occur in the distribution of the exposure. The role of topical corticosteroids in decreasing cytokines is helpful because in this type of
hypersensitivity, sensitized helper T cells rely on cytokines to cause inflammation and attract macrophages. By decreasing production of cytokines, the inflammatory response and therefore the degree of symptom severity is significantly reduced.

Incorrect .Answers: A, B, C, and E.

Acceleration of the metabolism of arachidonic acid (Choice A) would increase the rate of production of proslaglandins and leukotrienes. Prostaglandins and leukotrienes increase neutrophil chemotaxis and decrease vascular tone, respectively, which both
facilitate the inflammatory response to an insult. This would cause worsening of inflammation, not improvement.

Inhibition of receptors mediating selected responses to prostaglandins and leukotrienes (Choice 8) is a mechanism used by several medications, such as montelukast. Montelukast is an example of a leukotriene receptor antagonist that is selective for
leukotrienes C4, D4, and E4. This medication is used to prevent smooth muscle contraction and airway edema in asthma. However, inhibition of these or prostaglandin receptors would not decrease the cytokine milieu contributing to the type IV hypersensitivity
reaction to poison ivy.

Aspirin and non-steroidal anti-inflammatory drugs (NSAIDs), excluding celecoxib, are non.selective inhibitors of COX 1 and 2 (Choice C). Zileuton is a selective inhibitor of lipoxygenase (Choice E). Both of these targets reduce inflammation and the downstream
effects of prostaglandins and leukotrienes. However, steroids act upstream in the pathway when compared with COX and lipoxygenase inhibitors.

Educational Objective: Topical corticosteroids are first-line therapy for allergic contact dermatitis. They decrease the cytokine milieu by inhibiting phospholipase A2, thus preventing helper T cells from recruiting additional inflammatory cells to the site of exposure.

r r , fr!i P r trt
Previous Next Score Report Lab Values Calculator Help Pause
Exam Secllon 2: Item 19 of 50 Nallonal Board pf Medical Examiners®
Comprehensive Basic Science Serr-Assessment

✓ 19. A newborn delivered at 28 weeks' gestation is in severe respiratory distress. The immature alveoli of this newborn's lungs have a diminished ability to serve as sites of effective gas exchange. An increase in which of the following parameters best explains
this finding?

A)-Alveolar radii
S)- Lung compliance
C ) Lung elastic recoil
D) Pleural pressure
Iii)-Surfactant secretion
Correct Answer: C.

Neonatal respiratory distress syndrome presents with dyspnea and tachypnea during the first few hours of life, most often occurring in premature neonates. Physical examination reveals evidence of increased respiratory effort with nasal flaring, expiratory grunting,
and intercostal retractions. This typically occurs secondary to an insufficient quantity of pulmonary surfactant production related to immature type II pneumocytes, which occurs in the setting of prem.ature delivery or the decreased stimulation of surfactant production
from mature pneumocytes (eg, cesarean delivery with reduced fetal cortisol production in the absence of vaginal compressive stress). Pulmonary surfactant is a phospholipid mixture that lines the alveoli and reduces surface tension. Surface tension is directly
proportional to the collapsing pressure experienced by the alveolus. Impaired surfactant production leads to alveolar collapse with two important consequences. The first consequence is a substantially reduced area for gas exchange resulting in hypoxemia, while
the secondary consequence is decreased lung compliance, requiring greater force to expand the lungs.

Incorrect Answers: A, B, D, and E.

An increase in alveolar radii (Choice A) would result in increased lung compliance and decreased lung elastic recoil. This can be observed with the law of Laplace, which describes the pressure on a system (in this case, alveolar collapsing pressure) as a function of
surface tension divided by radius. As the radius of the alveolus increases, the collapsing pressure decreases.

Lung compliance (Choice B) is decreased in neonatal respiratory distress syndrome. Compliance is the change in volume of a system in response to a change in pressure. The increased lung elastic recoil from collapsed alveoli leads to a decreased lung
compliance, as it takes a greater change in pressure to expand the lungs than if the alveoli were not collapsed.

Pleural pressure (Choice D) is decreased in neonatal respiratory distress syndrome relative to healthy lungs. A greater negative pressure is required to expand the lungs due to the decreased lung compliance during inspiration, and less positive pleural pressure is
needed for expiration to occur given the increased lung elastic recoil.

Surfactant secretion (Choice E) is decreased, not increased, in neonatal respiratory distress syndrome. The decreased production surfactant by type II pneumocytes is the underlying pathophysiologic process of neonatal respiratory distress syndrome.

Educational Objective: Neonatal respiratory distress syndrome is characterized by the reduced production of pulmonary surfactant. Surfactant is critical for reducing alveolar surface tension and collapsing pressure; without it, alveolar collapse results in impaired gas
exchange, decreased lung compliance, and increased lung elastic recoil.

r
Prev1ous
r
Next
,
Score Report
� P
Lab Values Calculator
r
Help
r-
Pause
_
Exam Section 2: Item 20 of 50 National Board of M11dlcal Examiners®
Comprehensive saslc Science Serr-Assessment

✓ 20. A 55-year-old woman comes to the physician because of a 3-year history of weakness, fatigue, decreased appetite, and constipation. She says that she did not receive medical treatment for these symptoms because she did not have health insurance until
recently. She was admitted to the hospital 1 year ago because of renal calculi, but she could not afford to pay for follow-up care. X-rays at that time were indicative of osteitis fibrosa cystica. Vital signs are within normal limits. Physical examination shows
bony tenderness and muscle strength of 4/5 with decreased tone. Serum studies show an increased calcium concentration and a decreased phosphorus concentration. Which of the following is the most likely diagnosis?

A ) Adult T-lymphocyte leukemia


B ) Chief cell adenoma of the parathyroid gland
C) Familial (benign) hypocalciuric hypercalcemia
D) Medullary carcinoma of the thyroid gland
E ) Multiple myeloma
Correct Answer: B.

Primary hyperparathyroidism results in hypercalcemia and hypophosphatemia secondary to the aberrant increased production of parathyroid hormone (PTH). The most common cause of primary hyperparathyroidism is a chief cell adenoma of the parathyroid gland.
PTH is produced by parathyroid chief cells and stimulates 1a-hydroxylase in the kidney, which leads to the conversion of 25-hydroxycholecalciferol to 1,25-dihydroxycholecalciferol (active vitamin D). Vitamin D acts to increase the intestinal absorption of calcium and
phosphate. PTH also plays a role in the regulation of calcium and phosphate by stimulating osteoclastic bone reabsorption and distal convoluted tubular calcium reabsorption and phosphate excretion in the kidney. Common symptoms of primary
hyperparathyroidism are secondary to hypercalcemia and include recurrent nephrolithiasis, bone pain from osseous resorption (osteitis fibrosa cystica), polyuria, dehydration, constipation, and psychiatric disturbances.

Incorrect Answers: A, C, D, and E.

Adult T-lymphocyte leukemia (Choice A) is a rare neoplasm of T lymphocytes that results as a sequela of infection with human T-lymphotropic virus type 1 (HTLV-1). While this disease produces lytic bone lesions and hypercalcemia, it is also accompanied by other
findings including generalized lymphadenopathy, hepatosplenomegaly, and immunodeficiency.

Familial (benign) hypocalciuric hypercalcemia (Choice C) arises from an abnormal calcium sensing receptor on parathyroid cells. As a result, PTH is not suppressed by normal or increased serum calcium. This leads to a mild hypercalcemia and decrease urine
calcium concentration in the setting of increased PTH due to the normal mechanism of action of PTH on osteoclasts and renal tubules. Symptomatic hypercalcemia and renal calculi are uncommon in this condition.

Medullary carcinoma of the· thyroid gland (Choice D) can result in the excessive production of calcitonin by the parafollicular cells. Symptoms of hypocalcemia, rather than hypercalcemia, may be present. Medullary thyroid carcinoma is associated with multiple
endocrine neoplasia (MEN) types 2A and 2B.

Multiple myeloma (Choice E) is a plasma cell neoplasia that presents with multiple lytic bone lesions in the skull, pelvis, or vertebrae, and may present with hypercalcemia secondary to bone destruction. Symptoms also include anemia, renal failure, frequent
infections, and diffuse skeletal pain. The absence of characteristic radiographic findings makes this diagnosis unlikely.

Educational Objective: Parathyroid chief cell adenoma is the most common cause of primary hyperparalhyroidism, and presents with symptoms of hypercalcemia, i'ncluding recurrent nephrolithiasis, bone pain from osseous resorption, polyuria, dehydration,
constipation, and psychiatric disturbances.

r
Previous
r
Next
,,.
Score Report

Lab Values
P
Calculator
r
Help
r
Pause
Exam Section 2: Item 21 of 50 National Board pf M-qdlcal Examiners®
Comprehensive tsaslc Sctonce 58IT•ASS&ssment

✓ 21. A 3-year-old girl is brought to the physician because of a 2-week history of easy bruisability. She has had frequent, large, foul-smelling stools for 3 months. She was found to have cystic fibrosis at the age of 18 months. She is below the 3rd percentile for
height and weight. Physical examination shows several ecchymotic areas over the extremities. This patient most likely has a deficiency of which of the following?

A) Antithrombin Ill
8) Platelets
C) Protein C
D) Vitamin 812 (cobalamin)
E) Vitamin C
--F ) Vitamin K
G) von Willebrand factor
Correct Answer: F.
Cystic fibrosis is an autosomal recessive disorder due to a defect in the CFTR gene, leading to a deficiency in a chloride channel that secretes chloride in the lungs and gastrointestinal tract and reabsorbs chloride in sweat glands. This abnormal chloride transport
causes decreased chloride and water secretion and increased water reabsorption, leading to abnormally viscous mucous in the lungs and gastrointestinal tract. The thick mucus in the gastrointestinal tract contributes to pancreatic insufficiency, steatorrhea, and
rnalabsorption, leading to a deficiency of fat-soluble vitamins A, D, E, and K. Vitamin K plays a critical role in the synthesis of hepatic coagulation proteins; it is oxidized in the liver during carboxy1ation of glutamic acid residues on coagulation factors II, VII, IX, X, and
proteins C and S. A deficiency in vitamin K leads to coagulopathy with elevated prothrombin time (PT) and activated partial thromboplastin time (aPTT) and is characterized by easy bruising.
Incorrect Answers: A, 8, C, D, E, and G.
Antithrombin Ill (Choice A) deficiency can be inherited or acquired secondary to nephrotic syndrome. Anlithrombin Ill functions to inhibit coagulation factors II, VII, IX, X, XI, and XII. Deficiency leads to hypercoagulability.

Platelets (Choice 8) function in primary hemostasis in which they adhere to areas of endothelial damage and form a temporary platelet plug to stop bleeding prior to the activation of the coagulation cascade. Qualitative or quantitative platelet defects are all
characterized by increased bleeding time, and generally present with prolonged mucous membrane bleeding (eg, epistaxis, menorrhagia), petechiae, or purpura.
Protein C (Choice C) inactivates factors Va and VIiia in the coagulation cascade, and deficiency leads to hypercoagulability, not easy bruising.
Vitamin 8 12 (cobalamin) (Choice D) is classically characterized by erythrocyte macrocytosis and hypersegmented neutrophils. It is most often seen in patients with malnutrition, alcoholism, pernicious anemia, or Crohn disease.

Vitamin C (Choice E) is an essential nutrient that is found in fruits and vegetables and is necessary for collagen synthesis, iron absorption, immune function, and conversion of dopamine to norepinephrine. Deficiency causes scurvy, characterized by swollen gums,
bruising, poor wound healing, petechiae, perifollicular and subperi�steal hemorrhages, and short, fragile, cur1y hair.
Von Willebrand factor (Choice G) deficiency is one of the most common hereditary bleeding disorders and is due to a quantitative or qualitative abnormality of von Willebrand factor, which binds platelets and subendothelial collagen in primary hemostasis. It causes
a mild bleeding disorder with elevated bleeding time.
Educational Objective: Cystic fibrosis results in pancreatic insufficiency, which leads to the gastrointestinal malabsorption and defi'ciency of fat-soluble vitamins A, D, E, and K. Vitamin K plays a critical role in the synthesis of hepatic coagulation proteins, and a
deficiency in vitamin K leads to coagulopathy with elevated PT and aPTT.

r
Previous
r
Next
,-
Score Report
fr3
Lab Values
P
Calculator
r
Help
trt
Pause
Exam Section 2: Item 22 of 50 National Boa.rd pf Medical Examiners®
Comprehensive 1:1as1c Science Serr-Assessment

✓ 22. Receptors for all of the members of the steroid hormone family share which of the following features?

A} CalsiYm mediated intrasellular effeGts


i�GTP binding pmteins
C� Mltosl:londrial membrane assosiation
D) Plasma membrane association
E) Zinc-containing DNA-binding domains

Correct Answer: E.

Steroid hormones act on intracellular receptors and have the ability to traverse the cell membrane due to their lipophilic nature. Some of these hormones include cortisol, estrogen and progesterone, testosterone, and aldosterone. All of these molecules are derived
from cholesterol as a common precursor. Steroid hormone receptors are intracellular and reside either in the cytoplasm or the nucleus and regulate gene expression. Because of the effects mediated through gene regulation, the onset of action of steroid hormones
occurs over the course of hours to days. Intracellular steroid receptors have a number of different protein domains. The DNA-binding domain classically has a zinc finger motif as a tertiary structure. This domain has affinity with the regulatory (eg, promoter, silencer)
regions of DNA.

Incorrect Answers: A, B, C, and D.

Activation of Gq-protein coupled receptors leads to an increase in calcium-mediated intracellular effects (Choice A). The signaling cascade of Gq includes activation of phospholipase C, which cleaves membrane bound phospholipids, leading to the formation of
inositol triphosphate (IP3) and diacylglycerol (DAG). This results in the subsequent increase in intracellular calcium concentration, which then activates protein kinase C and/or smooth muscle contraction.

GTP-binding proteins (Choice B) are also known as G proteins which are components of many transmembrane receptors. G proteins enter an activated state when they are bound to GTP molecules. Once the GTP molecule is cleaved to GDP, the G protein
becomes inactive. G proteins are commonly involved in signaling pathways of peptide hormones.

Mitochondrial membrane association (Choice C) is a description of pathways of cellular apoptosis. During activation, caspases will cleave proteins causing permeability of the cell membrane. In addition, caspase 8 associates with the mitochondrial membrane and
alters its permeability.

Plasma membrane association (Choice D) is a component of many signal transduction pathways. G-protein coupled receptors have a transmembrane domain, which in turn connects to an intracellular G protein. Signaling molecules with receptor tyrosine kinases,
such as insulin, are also closely associated with the cell membrane.

Educational Objective: Steroid hormones function within the nucleus by regulating gene expression. Steroid receptor DNA-binding domains contain zinc motifs.

r
Previous
,.
Next
,,
Score Report

Lab Values
P
Calculator
,:-.
Help
,,.
Pause
Exam Section 2: Item 23 of 50 National Board of M11dlcal Examiners®
Comprehensive saslc Science Serr-Assessment

)(' 23. A homeless 45-year-old man comes to the clinic because of localized back pain for the past 3 months. He has not seen a health care provider in more than 20 years. Physical examination shows tenderness over the lower thoracic spine. There are no
obvious neurologic deficits. X-rays of the spine show narrowing of the disc space between T10 and T11 and destruction of the adjacent T10 vertebral body. Which of the following is the most likely cause of these findings?

A ) Hyperparathyroidism
B ) Multiple myeloma
G} Osteitis defGrmans (Pagetdisease)
D ) Sarcoidosis
E ) Tuberculosis
Correct Answer: E.

Tuberculous disease of the spine, also known as Pott disease, is a spinal osteomyelitis or osteodiscitis caused by Mycobacterium tuberculosis. M. tuberculosis is a slow-growing organism transmitted via respiratory secretions. The slow growth and evasion of
immune defenses result in the disease presenting with primary, latent, and reactivation patterns. In the primary infection, patients present with subacute fevers, weight loss, night sweats, cough, and malaise. Latent tuberculosis can occur as the organism
proliferates within macrophages and is maintained within caseating granulomas. Although tuberculosis is primarily a disease of the respiratory tract, it can have a number of extrapulmonary manifestations. These include meningitis, dermatologic tuberculosis, and
vertebral discitis or osteomyelitis. In this case, the vertebral body and intervertebral disc are involved. Radiographs or CT/MRI imaging will demonstrate localized destruction of bony anatomy, potentially with compression of the spinal nerve roots or spinal cord.
Because of its airborne transmission pattern, patients with tuberculosis must be isolated to prevent transmission to other persons. Tuberculosis remains prevalent in homeless, incarcerated, and immunocompromised patients. Active disease is treated primarily with
a combination of rifampin, isoniazid, pyrazinamide, and ethambutol.

Incorrect Answers: A, B, C, and D.

Hyperparathyroidism (Choice A) leads to bony resorption and hypercalcemia through increased production of parathyroid hormone. Primary hyperparathyroidism results from functionally active parathyroid adenoma, hyperplasia, or carcinoma. Secondary
hyperparathyroidism is commonly due to chronic hyperphosphatemia and hypocalcemia in the setting of end-stage renal disease. As parathyroid hormone activates osteoclasts to maintain calcium levels, bone resorption results. This does not cause focal
destruction of vertebrae or intervertebral discs.

Multiple myeloma (Choice 8) is a neoplasm of proliferating plasma cells in the bone marrow, triggering local osteoclast activation with resultant lytic bone lesions. Classically, multiple myeloma presents with hypercalcemia, renal failure, and anemia, in addition to
bone lesions.Osteitis deformans (Choice C), also known as Paget disease, is a disorder of osteoclast and osteoblast activity marked by the formation of disorganized, woven bone instead of lamellar bone. This disease may present with pathologic fractures, lytic or
sclerotic lesions on radiographs, and cranial nerve dysfunction from compression within the neural foramina of the skull base.

Sarcoidosis (Choice D) is an autoimmune disorder that affects multiple body systems. Classically, it presents with cough and dyspnea as well as bilateral hilar lymphadenopathy due to non-caseating granulomatous inflammation. It can also present with anterior
uveitis, cutaneous manifestations such as erythema nodosa, and arthralgias.

Educational Objective: Pott disease refers to tuberculous infection of the spinal column, which manifests as destruction of the vertebral bodies and/or intervertebral discs (osteomyelitis, discitis). It presents with the stigmata of chronic tuberculosis along with back
pain and possible kyphosis.

r
Previous
r
Next
,,. �
Lab Values
P
Cal�ulator
r-
Help
r
Pause
Score Report
_
Exam Section 2: Item 24 of 50 National Boa.rd of M·edlcal Examiners®
Comprehensive 1:1as1c Science Serr-Assessment

✓ 24. A 49-year-old man comes to the physician because of a 1-month history of lower abdominal pain and increased urge to defecate. Capability to prevent spontaneous defecation is intact. He sustained nondisplaced multiple fractures of the pelvis 2 months
ago in a motor vehicle collision. He initially was on bed rest with pelvic support, progressed to non-weight-bearing crutch walking, and now is partial weight bearing with crutches. Medications now include naprosyn three times daily. Physical examination
shows an anal wink. Neurologic examination shows intact rectal sphincter tone. This patient is able to sustain cortical control over defecation via which of the following nerves?

A) Conus medullaris
B ) Hypogastric nerves
C) Myenteric plexus
D) Pelvic nerves
E ) Pudenda! nerves

Correct Answer: E.

The pudenda! nerve is formed from the sacral plexus by the S2, S3, and S4 nerve roots. The nerve carries both sensory and motor fibers and innervates the genitalia and the pelvic floor musculature, including the levator ani muscle and the external anal and
urethral sphincters. Pudenda! neuropathy can occur in Alcock canal syndrome, often due to impingement of the pudenda! nerve between the sacrotuberous or sacrospinous ligaments. This pathology can occur in cyclists, due to prolonged compression in this area,
patients with pelvic trauma or surgery, and in childbirth. The pudenda! nerve courses just medially to the ischial tuberosity, making this an important landmark for regional anesthetic blocks used in vaginal delivery or anorectal surgery. Given this patient's history of
pelvic trauma, there is likely disruption involving the complex interactions between sympathetic and parasympathetic nerves that provide autonomic control over urination, ejaculation, and defecation. However, his ability to maintain voluntary (cortical) control over
defecation demonstrates an intact neural pathway involving the pudenda! nerve, a somatic nerve, even if mild injury to this nerve was sustained.

Incorrect Answers: A, B, C, and D.

The conus medullaris (Choice A) is the distal termination of the spinal cord. Compression of the conus will lead to saddle anesthesia, overflow incontinence, and weakness or paralysis of the bilateral lower extremities.

The hypogastric nerves (Choice B) arise from roots of T11-L4 and provide sympathetic innervation to portions of reproductive organs. They are responsible for emission of semen into the posterior urethra which is the first stage of male ejaculation.

The myenteric plexus (Choice C) is part of the enteric nervous system located between inner and outer layers of the muscularis extema. Absence of this plexus is responsible for Hirschsprung disease, which is a congenital failure in the distal development of the
myenteric plexus, leading to large bowel obstruction.

The pelvic nerves (Choice D) references pelvic splanchnic nerves, which form from the sacral and lumbosacral plexus. These nerves supply autonomic innervation to the rectum, bladder, and the genitalia. Injury to these nerves causes a loss of function of bladder
and bowel, however they are under visceral, not somatic, control.

Educational Objective: The pudenda! nerve provides somatic sensory and motor innervation to the genitals and the pelvic floor musculature, including the levator ani muscle and external anal sphincter. Injury to this nerve can occur with pelvic trauma or chronic
compression.

r
Previous
r
Next
,
Score Report

Lab Values
P
Calculator
r-
Help
r
Pause
Exam Section 2: Item 25 of 50 National Board pf Medical Exi1.mlners®
Comprellenslve Basic Science Sert-Assessment

✓ 25. In a healthy 25-year-old man with a normal diet and fluid intake, which of the following substances filtered by the kidney has the greatest fraction excreted in urine?

A}Gha1CG68
B } Magnesium
G}-Ssdillm
D} Urea
E } Water
Correct Answer: D.

The fractional excretion of a solute measures the percentage of the original filtered amount that is present in the excreted urine. The calculations are made using the plasma and urine concentrations of the solute in question. The fractional excretion of a molecule is
more reliable than the urine concentration of that molecule as urine concentration can vary with the amount of water present in the urine. Out of the molecules listed, urea has the highest fractional excretion. Urea is created by the urea cycle as a way to excrete
nitrogen and prevent the accumulation of ammonia, which is toxic. Urea is filtered by the glomerulus and then reabsorbed in the proximal convoluted tubule at a rate that is lower than that of water. It is also an integral component of the medullary countercurrent
system, allowing for the reabsorption of water in the loop of Henle. Approximately 40% of the initially filtered urea is found in the final excreted urine.

Incorrect Answers: A, B, C, and E.

Glucose (Choice A} has a fractional excretion of nearly 0% in healthy, nondiabetic patients with serum glucose levels less than 180 mg/dl. The proximal convoluted tubule is the primary site of glucose reabsorption. In diabetes mellitus, the filtered load of glucose
exceeds the resorptive capacity of the proximal convoluted tubule and glucosuria ensues.

Under physiologic conditions, the fractional excretion of magnesium (Choice B} is approximately 3% to 5%. Unlike most other solutes, the site of reabsorption of magnesium is the thick ascending loop of Henle, not the proximal convoluted tubule.

The fractional excretion of sodium (Choice C) differs based on volume status but is usually less than 1 %. When dehydration is present, increased sodium is reabsorbed to create an osmotic gradient for water to follow. In a well-hydrated patient, there is less
reabsorption of sodium, leading to an increased fractional excretion. The fractional excretion of sodium is often used clinically to determine the cause of an acute renal injury.

The specific gravity of urine is the measurement more often used to determine the amount of water (Choice E) in urine. Fractional excretion applies to solutes rather than water, which is a solvent.

Educational Objective: Fractional excretion is the percentage of a filtered solute that is excreted in the urine. Urea has a fractional excretion of approximately 40%, which is much higher than that of magnesium (3%-5%), sodium (less than 1 %}, and glucose (0%)
under physiologic conditions.

r
Previous
r
Next
,,.
Score Report

Lab Values
F
Calculator
r
Help
If'
Pause
Exam Section 2: Item 26 of 50 National Board of Medlcal Examiners®
Comprehensive 1:1as1c Scaence Self-Assessment

✓ 26. A 32-year-old woman comes to the physician because of a 3-month history of headaches. Two weeks ago, she moved to an area women's shelter because her husband had been hitting her. She takes no medications. Physical examination shows multiple
bruises in different stages of healing over the face, neck, and trunk. There is circumferential bruising on both sides of the neck. Which of the following statements is most appropriate to gather pertinent information from this patient?

A ) "Did you bring your records from the emergency room with you?"
B ) "I see some bruising on your neck. How did that happen?"
Gt- "Please demonstrate to me l=low tl=lis neck bruising occllm1d."
D) "This looks like you were choked. Is that true?"
et-"1/llttat sort of de�<ice did yollr Rllsband Yse on yollr neck?"
Correct Answer: B.

Physicians should ask direct, open-ended questions about physical examination findings indicative of abuse. Physicians should start by summarizing their objective observations and end with a question about the findings. To protect patients' well-being, physicians
have an obligation to investigate evidence of abuse. Additionally, abuse may cause physical or psychological injury that may require diagnosis and treatment. Physicians should treat the immediate symptoms, which may manifest as medical or psychiatric
symptoms, and continue to provide regular care to monitor for long-term consequences of abuse. Physicians are mandated by many states to report abuse of children and vulnerable adults to protective agencies.

Incorrect Answers: A, C, D, and E.

Asking about emergency room records (Choice A) would avoid the topic of ongoing abuse and be unlikely to inform the physician about the patient's current social situation. Physicians should directly ask about concerning physical examination findings.

Physicians should not ask patients to reenact possible abuse (Choice C), which may retraumatize patients. Patients should be invited to discuss abuse using open-ended questions instead.

The physician should not assume that the bruises came from the husband's abuse (Choices D and E) or that the patient is willing to discuss this sensitive topic. The patient may feel intruded upon or defensive, which would preclude effective discussion. Asking an
open-ended question based on an objective observation would likely increase the patient's comfort level with a discussion about the abuse.

Educational Objective: Physicians should ask direct, open-ended questions about physical examination findings indicative of abuse without verbalizing assumptions about the etiology of the examination findings.

r
Previous
r
Next
,
Score Report
fr!: P
Lab Values Calculator
r
Help
,,.
Pause
.
Exam Section 2: Item 27 of 50 National Board pf Medical Examiners®
Comprehensive l!aslc Se1ence Serr-Assessment

✓ 27. A 52-year-old man who is in the hospital with malignant hypertension, heart failure, and a mild renal impairment is being treated with captopril, nitroprusside, and furosemide. After 48 hours of therapy, his blood pressure is controlled, but he develops
metabolic acidosis, lethargy, and difficulty breathing. Which of the following substances most likely caused these adverse effects in this patient?

A) Cyanide
B ) Iron
C) Lead
D) Nitric oxide
E) Selenium

Correct Answer: A.

Sodium nitroprusside is an intravenous, titratable vasodilator that can be used for the treatment of hypertensive emergencies. It breaks down in circulation to release nitric oxide, which in tum activates guanylate cyclase in vascular smooth muscle to result in
vascular smooth muscle relaxation and vasodilation via a cyclic guanosine monophosphate (cGMP) pathway. A byproduct of sodium nitroprusside is cyanide, and in rare cases, infusion of nitroprusside can cause cyanide toxicity. Cyanide poisoning occurs due to its
action of inhibiting the electron transport chain, leading to metabolic lactic acidosis. Cyanide inhibits the mitochondrial enzyme cytochrome c oxidase, preventing the synthesis of adenosine triphosphate, leading to lactic acidosis. The presentation of cyanide
poisoning is characterized by tachypnea due to compensatory respiratory drive, hypotension, confusion, lethargy, and bradycardia. The management of cyanide poisoning includes supportive care with intravenous fluids and vasopressors, sodium thiosulfate which
increases the conversion of cyanide to thiocyanate for renal excretion, amyl nitrites to induce methemoglobinemia which binds with cyanide and prevents it from acting on the electron transport chain, and hydroxocobalamin which combines with cyanide to create
nontoxic cyanocobalamin.

Incorrect Answers: B, C, D, and E.

Iron toxicity (Choice B) is characterized by gastrointestinal irritation, and later, anion gap metabolic acidosis, hepatic toxicity, acute kidney injury, coagulopathy, and fulminant hepatic failure. Due to the severity of toxicity, earty management may include whole bowel
irrigation with polyethylene glycol or gastric lavage.

Lead toxicity (Choice C) can occur for example from workplace exposure, lead paint in a residence, or contaminated drinking water. Symptoms of lead exposure can be variable and nonspecific, including abdominal pain, constipation, anorexia, myalgias and
arlhralgias, fatigue, headache, difficulty concentrating, and microcytic anemia.

Nitric oxide (Choice D) is the expected byproduct of sodium nitroprusside that leads to vascular smooth muscle relaxation and vasodilation, which is helpful in the management of hypertensive emergencies. It does not cause metabolic acidosis or difficulty breathing.

Selenium (Choice E) is a naturally occurring mineral that is harmless at normal doses, but in acute or chronic excess, can cause nausea, vomiting, hair loss, fatigue, and irritability. It is not associated with metabolic acidosis or sodium nitroprusside administration.

Educational Objective: Sodium nitroprusside breaks down to release nitric oxide, which leads to vascular smooth muscle relaxation and vasodilation via a cGMP pathway. A potential adverse byproduct of nitroprusside includes cyanide, which causes uncoupling of
the electron transport chain, leading to lactic acidosis, compensatory respiratory alkalosis, hypotension, lethargy, and bradycardia.

r
Previous
r
Next
,
Score Report
fr§
Lab Values
F
Calculator
r
Help
fl"
Pause
Exam Sectlon 2: Item 28 of 50 National Board pf Medical Examiners®
Comprehensive Basic Science Self-Assessment

y 28. A 78-year-old man comes to the physician with his daughter because of low back pain for 3 months. He underwent a bilateral orchiectomy 2 years ago for prostate cancer. He speaks French fluently, but he knows only a few English words. His daughter
offers to serve as an interpreter. After thanking the daughter for her offer, it is most appropriate for the physician to state whiclh of the following?

A) "Have you interpreted for your father before in a medical capacity?"


B��eve it wot,1ld be awlw.•ard f-Oradaughter to interpret for her father."
C) "It is best that I use a professional French interpreter."
01 "Please ask yot,1r fatherifhe's raady to begin the examination.•
E} "That ¥,<ot,1ld be fine. I'll just askyoulo step out of the room f-Or the physisal examination.•
Correct Answer: C.
Professional interpreters can provide high-quality translations that maintain patient safety. Credentialed interpreters are trained in medical knowledge and vocabulary, ethics, and cultural sensitivity. Relying on a family member for interpreting increases the risk of
miscommunication about medical or culturally relevant details and can jeopardize informed consent for treatment and patient safe•ty.
Incorrect Answers: A, B, D, and E.
Having past experience translating for a family member (Choice A) is suboptimal compared to the rigorous training professional interpreters receive. Irrespective of interpersonal awkwardness or the physician's personal b{lliefs (Choice B), family members should
not act as interpreters for patients (Choices D and E). The nuances of a medical situation may be misunderstood by an untrained interpreter and therefore miscommunicated to the patient. The only situation in which interpretation by a family member is acceptable
is an emergency when no other option exists.
Educational Objective: Physicians should utilize professional interpreters instead of family members to translate for patients. Professional interpreters are trained to accurately communicate in medical settings and are therefore crucial for patient safety and informed
consent to treatment.

r
Previous
r
Next
,
Score Report

Lab Values
t=
Calculator
r
Help
,,.
Pause .
Exam Section 2: Item 29 of 50 National Board pf Medical Examiners®
Comprehensive l!aslc Se1ence Serr-Assessment

)( 29. A 15-year-old girl is brought to the physician by her mother because of a 1-year history of increasingly severe abdominal pain that occurs periodically, but not regularly. Physical examination shows abdominal tenderness that is most acute around the
umbilicus. The physician suspects�pic endometrial tissue, and an operation is scheduled. During the operation, an 8-cm i:1iece of tissue is removed from the ilea! region on the antimesenteric side of the bowel approximately 90 cm proximal to the
ileocecal junction. The patient's pain resolves postoperatively. The ectopic tissue was most likely located in which of the following structures?

A� Ascending colon
• B ) Falciform ligament
C) Ilea! polyp

�� \41rmiform appendi:x
Correct Answer: D.

Meckel diverticulum results from persistence of the vitelline duct and can demonstrate ectopic, ifunctional gastric, pancreatic, or, rarely, endometrial tissue. It can present with acute or chronic hematochezia or melena. It is often painless but may also present with
abdominal pain. It typically presents in childhood, most commonly before age 2 years, although because of its indolent nature, it may not be diagnosed until later in life. It is commonly located two feet proximal to the ileocecal valve and is approximately two inches
in length. In children, it may serve as a potential lead-point for intussusception. A technetium 99m pertechnetate scan demonstrates uptake in the right lower quadrant, corresponding with ectopic gastric mucosa within the Meckel diverticulum. Treatment of
symptomatic cases is through surgical resection.

Incorrect Answers: A, B, C, and E.

The ascending colon (Choice A) is a common site of colon polyps. The ascending colon is distal to the ileocecal valve and is not consistent with the location of the excised tissue.

The falciform ligament (Choice B) is a peritoneal fold that divides the liver; it is anatomically distant from the described location of resection in this case.

Heal polyps (Choice C) are less common than their counterparts in the colon, and may be associated with hereditary polyposis syndromes, such as Peutz-Jeghers syndrome or familial adenomatous polyposis. The length of the removed tissue and its presentation
with intermittent abdominal pain is more suggestive of a Meckel diverticulum. •

The vermiform appendix (Choice E) is the site of acute appendicitis, which presents with periumbilical pain that migrates to the right lower quadrant, fever, anorexia, nausea, vomiting, diarrhea, and leukocytosis.

Educational Objective: Meckel diverticulum results from persistence of the vitelline duct and presents as an approximately two-inch diverticulum located two feet proximal to the ileocecal valve. It can cause abdominal pain, hematochezia, or melena. Meckel
diverticulum frequently harbors ectopic tissue, which is most commonly gastric or pancreatic tissue. Rarely, Meckel diverticulum may contain ectopic endometrial tissue.

r
Previous
r
Next
,
Score Report
fr!i
Lab Values
F
Calculator
r
Help
fl"
Pause
Exam Section 2: Item 30 of 50 National Board pf Mqdlcal Examiners®
Comprehensive isaslc Science Self-Assessment

30. A 14-year-old girl is brought to the physician by her mother because of episodes of increasingly severe facial blemishes during the past 6 months. She says that she does not eat high-fat foods or use makeup. Use of an astringent soap has not resolved
her symptoms. Menarche occurred at the age of 12 years. A photograph of her face is shown. Which of the following best describes the pathologic mechanism of this patient's condition?

A ) Acceleration of the proliferation of cells in the dermis by increased estrogen production


2} Colonization of apocrine sweat glands by fklngi
G} Deci:eased blood flow lo skin capillaries by overprod1a1ction of elastic fibers
D) Follicular epidermal hyperproliferation with excess production of sebum
E ) Thickening of the basement membrane in response to increased pubertal serum estrogen concentrations
Correct Answer: D.
The initial lesion of acne vulgaris is a oomedo, a hair follicle that has been blocked by keratin debris. The inciting event in comedo formation is hyperproliferation of the epidermis and abnormal keratinization. This is compounded by androgenic stimulation of
sebaceous glands associated with the hair follicle, which are together called a pilosebaceous unit, leading to increased sebum production. This increased sebum provides a substrate for bacterial overgrowth of normal skin flora including Staphylococcus epidermidis
and Propionibacterium acnes. With the accumulation of keratin debris, increased sebum production, and bacterial overgrowth, the comedo becomes inflamed and ruptures, causing a papule or cyst to form. Clinically, areas with increased sebaceous activity
including the face, upper back, and chest are prone to acne. It is a common disorder that affects the vast majority of adolescents. Treatments are aimed at reducing comedo formation (retinoids), decreasing sebum production (anti-androgens), and mitigating
bacterial overgrowth (antibiotics).
Incorrect Answers: A, B, C, and E.
Androgens, not estrogen (Choice A), contribute to acne vulgaris. While the sebaceous glands reside in the dermis, it is not proliferation of the sebaceous cells but increased production of their secretions that contribute to acne.
Bacterial colonization of the pilosebaceous unit, not colonization of the apocrine glands by fungi (Choice B) leads to acne vulgaris. Apocrine glands are located in the axillae, areolae, and anogenital regions. These areas are not typically affected by acne
vulgaris.Derrnal collagen fibers, rather than elastic fibers (Choice C), increase as a result of scarring. While the majority of collagen fibers in healthy skin are type I collagen, scar is initially created by type Ill collagen.
The basement membrane (Choice E) is not thickened by estrogen nor does it play a role in the development of acne vulgaris.
Educational Objective: Comedo formation, the key inciting event in acne vulgaris, is initiated by proliferation and abnormal keralinization of the hair follicle epidermis. Excess production of sebum then provides a hospitable environment for bacterial overgrowth. Acne
treatments are aimed at addressing these aberrancies.

r
Previous
r
Next
,-
Score Report
� F
Lab Values Calculator
r
Help
,,.
Pause
Exam Section 2: Item 31 of 50 National Board of M11dlcal Examiners®
Comprehensive saslc Science Serr-Assessment

✓ 31. A strain of Escherichia coli produces a temperature-sensitive tryptophan synthase and therefore requires exogenously supplied tryptophan for growth at 42 ° C but not at 30°C. Which of the following mutations is most likely responsible for this phenotype?

A ) Deletion
B ) Frameshift
C) Insertion
D) Missense
E ) Nonsense
Correct Answer: D.
Missense mutations are a type of point mutation in which the change of a single base pair forms a codon that results in substitution of a single, different amino acid from that usually encoded. Substitution of a single amino acid may result in a wide range of changes
in protein function depending on the position of the substitution. The change in amino acid may result in no or minimal change to protein function or may lead to non-functional or misfolded proteins. The described change in the mutant tryptophan synthase, in which
the protein retains normal function except under particular temperature conditions, is a minor change in protein function that is most consistent with a missense mutation. Other types of mutation typically result in more severe changes in protein function.

Incorrect Answers: A, B, C, and E.

Deletion (Choice A) of a portion of the genetic sequence results in shifts of the reading frame if the deletion does not involve a number of base pairs that is divisible by three. Deletion mutations typically result in more severe alterations of protein function (generally
loss of function) compared to missense mutations.

Frameshift mutations (Choice B) result from changes to the reading frame after insertion or deletion of a number of base pairs that is not divisible by three. Frameslhift mutations typically result in severe loss of function.

Insertion mutations (Choice C) involve the addition of base pairs to the genetic sequence and may result in shift of the reading frame if the insertion does not involve a number of base pairs that is divisible by three. Insertion mutations typically result in more severe
alterations of protein function compared to missense mutations.

Nonsense mutations (Choice E) are point mutations that result in premature stop codons and protein truncation. Nonsense mutations typically result in loss of protein function.
Educational Objective: Missense mutations are a type of point mutation in which the change of a single base pair forms a codon that results in substitution of a single, different amino acid from that usually encoded. Substitution of a single amino acid may result in a
wide range of changes in protein function depending on the position of the substitution.

r
Previous
r
Next
,,.
Score Report

Lab Values
P
Cal�ulator
r-
Help
r
Pause
_
Exam Section 2: Item 32 of 50 National Board pf Medical Examiners®
Comprehensive Basic Science Self-Assessment

✓ 32. A 52-year-old man with chronic renal failure receives a cadaveric kidney transplant. Postoperatively, he is given cyclosporine for immunosuppressive therapy. Six weeks after the operation, he develops
hypoxemia with an arterial Po2 of 40 mm Hg. A photomicrograph of a transbronchial biopsy specimen is shown. Which of the following pathophysiologic mechanisms best explains this patient's hypoxemia?

A}-Alveolar exudation due to Crypler;er;r;us Reef-OrmaRs infection


B ) Cytomegalovirus pneumonitis with diffuse alveolar damage
C) Neutrophilic alveolar consolidation due to pneumococcal pneumonia
D ) Pneumocystis jirovecii (formerly P. carinil) infection with alveolar exudation
E ) Vascular invasion by Aspergil/us with pulmonary infarction

Correct Answer: B.

Cytomegalovirus (CMV), also known as human herpesvirus-5 (HHV-5), is an opportunistic infection that commonly occurs in immunocompromised patients in the setting of solid-organ or allogeneic stem cell transplantation, severe ulcerative colitis, or HIV/AIDS.
CMV pneumonitis -.yith diffuse alveolar damage is one of the many presentations that can occur in patients taking immunosuppressants to prevent organ rejection following solid organ transplant. It typically occurs in the first 3 to 6 months following
transplantation. Radiologic imaging features are nonspecific but may demonstrate an interstitiaH pneumonitis pattern with the potential for infiltrative consolidations and ground-glass opacities to develop. The classic histologic findings of CMV pneumonitis are
infected cells with prominent basophilic nuclear inclusions, as seen in this patient's lung biopsy. Other manifestations of CMV infection associated with immunosuppression include systemic infectious mononucleosis, retinitis, esophagitis, colitis, and encephalitis.

Incorrect Answers: A, C, D, and E.

Alveolar exudation due to Cryptococcus neoformans infection (Choice A) may be seen in pulmonary cryptococcosis. Cryptococcus neoformans is an encapsulated yeast and opportunistic pathogen in immunocompromised patients, especially patients with
HIV/AIDS with low CD4 counts. Disseminated disease can also cause cryptococcal meningitis and encephalitis. Histology reveals fungal cells with narrow-based budding and a bright red inner capsule on staining with mucicarmine.

Neutrophilic alveolar consolidation due to pneumococcal pneumonia (Choice C) is seen in lung infections due to Streptococcus pneumoniae. Pneumonia classically presents with fever, dyspnea, productive cough, and pleuritic chest pain. Gram-positive
diplococci would be seen on sputum cultures or bronchoalveolar lavage.

Pneumocystis jirovecii (formerly P. carinii) infection with alveolar exudation (Choice D) is present in pneumocystis pneumonia. Chest radiologic imaging typically reveals diffuse, bilateral infiltrates often occurring prominently about the bilateral hila. Methenamine
silver or toluidine blue stains selectively stain the cyst walls and are used to confirm the diagnosis.

Vascular invasion by Aspergil/us with pulmonary infarction (Choice E) is a feature of angioinvasive aspergillosis. lmmunosuppression is a risk factor for this disease. Patients present with persistent fever, cough, hemoptysis, and chest pain. Vascular invasion can
result in distal pulmonary infarctions. Histologic findings include invasive hyphae in the lung parenchyma with extension into the vasculature.

Educational Objective: Soli:d-organ transplantation and immunosuppression to prevent organ rejection is frequently complicated by opportunistic pathogens. CMV is a common cause of disease in these patients and can present with pneumonitis with diffuse
alveolar damage, colitis, retinitis, esophagitis, and encephalitis. Infected cells typically demonstrate a large, atypical appearance and feature the presence of basophilic nuclear inclusions.

r ,.
-------1'.f!l��us ________1i11_
xt
"'
�cpJ11�ep1>rt
_
fr!i P
l,ab \lal!Jes �<:_aJcula_tor

Help_ �
,,..
l'_ause
Exam Section 2: Item 33 of 50 National Board of Medica l Examiners®
Comprehensive aastc Science Self-Assessment

✓ 33. A 2-year-old boy is brought to the physician because of a 6-month history of failure to thrive. Cardiac examination shows a grade 4/6 systolic murmur caused by increased pulmonic flow, followed by a fixed, widely split s 2. Echocardiography shows
hypertrophy of the right atrium, right ventricle, and pulmonary arteries. This patient most likely has which of the following congenital cardiac anomalies?

A ) Atrial septal defect


B ) Patent ductus arteriosus
C ) Persistent truncus arteriosus
D) Tetralogy of Fallot
E ) Transposition of the great arteries
F ) Ventricular septal defect
Correct Answer: A.
Atrial septal defect is a common congenital malformation of the interatrial septum. The most common type is an ostium secundum defect, although ostium primum defects are commonly associated with trisomy 21. The atrial septal defect results in a left-to-right
shunt with abnormal flow of blood from the left atrium to the right atrium, resulting in relative volume overload of the right atrium and ventricle. This increased stroke volume of the right ventricle results in delayed closure of the pulmonic valve, which presents as a
fixed, split S2, and low-grade physiologic ejection murmur on cardiac auscultation. The increased right heart volumes also result in a prominent right ventricular impulse on physical examination and may present an increased risk for the development of a right
bundle branch block. Chest radiographs characteristically demonstrate increased caliber of the main pulmonary artery and increased pulmonary vascular markings. If the atrial septa! defect remains uncorrected, it can result in the development of Eisenmenger
syndrome secondary to prolonged pulmonary vasculature remodeling resulting in pulmonary arterial hypertension and shunt reversal leading to cyanosis.
Incorrect Answers: B, C, D, E, and F.
Patent ductus arteriosus (PDA) (Choice B) is a persistent conduit between the aorta and the pulmonary artery that has failed to obliterate after birth. It results in a continuous, machine-like murmur best heard in the left second intercostal space, radiating to the
clavicle.
Persistent truncus arteriosus (Choice C) results from failure of the aorticopulmonary septum to divide the aorta and pulmonary trunk. Most patients present with a concomitant ventricular septa! defect and cyanosis noted in the neonatal period.
Tetralogy of Fallot (Choice D) consists of pulmonary infundibular stenosis, overriding aorta, ventricular septa! defect, and right ventricular hypertrophy, resulting in a right-to-left shunt with cyanosis fitrSt noted in infancy. The murmur heard with tetralogy of Fallot
relates to the underlying ventricular septa! defect characterized by a holosystolic murmur best heard in the left lower sternal border.
Transposition of the great arteries (Choice E) results in aberrant reversal of the normal anatomical association of the aorta and pulmonary artery to the cardiac chambers secondary to failure of the aorticopulmonary septum to spiral. This results in separate systemic
and pulmonary circulations, with the aorta leaving the right ventricle and the pulmonary trunk leaving the left ventricle. This is incompatible with life unless an additional shunt allowing admixture of blood is present (eg, PDA, ventricular septa! defect).
Ventricular septa! defect (Choice F) refers to a defect in the interventricular septum, most commonly the membranous portion. It is characterized by a holosystolic murmur best heard in the left lower sternal border.
Educational Objective: A fixed, widely split S2 is characteristic of an atrial septal defect due to increased blood flow through the pulmonic valve. Severe defects can result in pulmonary hypertension and development of Eisenmenger syndrome over time, with
reversal of the left-to-right shunt.

r
Previous
,.
Next
"
Score Report

Lab Values
r-:·
Calculator

Help
,,..
Pause
Exam Section 2: Item 34 of 50 National B011.rd pf M11dical Exftll}lners®
Comprehen&lve 11111c: science sen-A&Sessment

✓ 34. A 63-year-old woman comes to the physician because of a 5-day history of shortness of breath and swollen legs. Her respirations are 25/min and labored, and blood pressure is 130/50 mm Hg. She has a large subclavian arteriovenous fistula caused by a
stab wound to the left supraclavicular area 15 years ago. Physical examination shows 2+ edema of the lower extremities. Which of the following findings is most likely in this patient?

A ) Decreased arterial oxygen saturation


a}- Decreased mixed •1enous oxygen saturation
G}- Decreased stroke volume
D) Increased resting cardiac output
€}- Increased systemic •1ascular resistance

Correct Answer: D.

Increased resting cardiac output is an expected finding in this patient with clinical signs and symptoms of high-output heart failure in the setting of a large subclavian arteriovenous fistula. While most cases of heart failure are divided into heart failure with preserved
(HFpEF) or reduced (HFrEF) ejection fraction, high-output cardiac failure is a unique variety of cardiac failure. Causes of high-output cardiac failure include endocrine abnormalities such as hyperthyroidism, chronic lung disease, sepsis, cirrhosis, and large
arteriovenous (AV) fistulas as seen in this patient. AV fistulas are either congenital or acquired and result in an abnormal connection between a high-pressure system (arterial) and a low-pressure system (venous), without the benefit of the stepwise reduction in
pressure that occurs with blood transmission through the peripheral arterioles and capillaries. AV fistulas effectively bypass the arteriolar and capillary systems leading to significant shunting of blood directly from the arterial to the venous system. This results in
increased venous return to the right side of the heart, with a subsequent increase in heart rate and stroke volume. As cardiac output (CO) is defined by the product of stroke volume and heart rate, this results in increased CO. Eventually, due to the persistent
increased myocardial oxygen demand of the heart in the setting of high-output, patients will decompensate and present with signs and symptoms of heart failure, including dyspnea, increased jugular venous pressure, and peripheral edema.

Incorrect Answers: A, B, C, and E.

Decreased arterial oxygen saturation (Choice A) is not a feature of AV fistulas. Arterial oxygen saturation will be normal.

Decreased mixed venous oxygen saturation (Choice 8) is not a feature of AV fistulas. Highly oxygenated arterial blood passes directly into the venous circulation via the AV fistula, so the mixed venous oxygen saturation will be higher than expected.

Decreased stroke volume (Choice C) is not found in cardiac failure from AV fistulas. As a result of increased preload and stretch on the right ventricle, the stroke volume will be higher than normal.

Increased systemic vascular resistance (SVR) (Choice E) is not an expected finding in high-output cardiac failure from an AV fistula. SVR is largely maintained by the action of the arterioles and bypassing this part of the arterial circuit via an AV fistula results in low
SVR.

Educational Objective: Large arteriovenous (AV) fistulas can result in high-output cardiac failure. The abnormal connection between an artery and vein results in decreased systemic vascular resistance and an increased venous return to the right heart, with a
consequent increase in stroke volume and cardiac output.

r
Previous
r
Next
tf:
Score Report
ta F·
Lab Values Calculator
,:,
Help
""
Pause .
Exam Secllon 2: Item 35 of 50 National Board pf M�dlcal Examiners®
Comprehensive 1:1as1c Setence Serr-Assessment

✓ 35. A study is conducted to assess 32 patients in a community of 1000 who have developed drug-resistant tuberculosis during a 1-year period. These patients are removed from the community for treatment. Assuming that the risk for infection and
susceptibility to the disease is constant, which of the following best represents the number of individuals most likely to develop subsequent drug-resistant tuberculosis during the next year?

A ) 27
B ) 29
C) 31
D ) 32
E ) 33
Correct Answer: C.

The incidence rate of disease in a population equals the number of new cases of the disease divided by the number of persons at risk for contracting the disease. Incidence is a measure of risk and is the probability of a given condition occurring within a specified
period of time. It is often expressed as a single number of new cases but is more appropriately expressed as a fraction of a population. Importantly, when individuals are added to or are removed from a population, the denominator must also change to reflect the
new number of persons at risk for contracting the condition. The incidence rate is mathematically expressed as (the number of new cases) / (the number of persons at risk). In this study, 32 patients contracted drug-resistant tuberculosis over a 1-year period of time.
1,000 persons were at risk over that same period of time. The incidence rate of contracting drug-resistant tuberculosis is therefore 3.2% (32/1000). In the following year, once the 32 affected persons were removed from the community, with risk of infection and
susceptibility remaining constant, 968 persons would be at risk (1000-32). The number of individuals likely to develop drug-resistant tuberculosis in the following year is therefore computed by: Incidence rate= number of new cases/number of persons at risk= 3.2%
= number of new cases/968 individuals= 3.2%.968 individuals= 30.97 individuals, or approximately 31 individuals.

Incorrect Answers: A, B, D, and E.

27 (Choice A) and 29 (Choice B) would be potentially correct if a larger number of persons than 32 were removed from the population at risk.

32 (Choice D) reflects the number of persons who would be expected to contract drug-resistant tuberculosis if the population at-risk remained 1,000 individuals. The number of persons added to or removed from a population must be considered.

33 (Choice E) reflects the number of persons who would be expected to contract drug-resistant tuberculosis if the population at-risk contained 1,032 patients, that is, if the number of people supposed to be removed from the at-risk population were instead
mistakenly added. This could be arrived at as follows: Incidence rate= number of new cases/number of persons at risk= 3.2%= number of new cases/1,032 individuals= 3.2%•1,032 individuals= 33.024 individuals, or approximately 33 individuals.

Educational Objective: The incidence rate of disease in a population equals the number of new cases of the disease divided by the number of persons at risk for contracting the disease. Incidence is a measure of risk and is the probability of a given condition
occurring within a specified period of time. Importantly, when individuals are added to or are removed from a population, the denominator must also change to reflect the new number of persons at risk for contracting the condition.

r r
Pr11ylou_!i _!l!_e__irt
_ _
,-
�cor�B_eport
fa F
Lab Value!! _g_alcu1aio__
r ____
r. tft'
f:l_�lp_ _P__a_1,1s.!!._
Exam Section 2: Item 36 of 50 National Boll.rd pf Mqdlcal Exi\llllners®
Comprehensive tsaslc Science Serr-Assessment

✓ 36. An 81-year-old woman is admitted to the hospital because of a massive pulmonary embolism from a deep venous thrombosis. Her platelet count is 160,000/mm3. Appropriate pharmacotherapy is started. One week later, her platelet count is 55,000/mm3.
The thrombocytopenia was most likely caused by a drug with which of the following mechanisms of action?

A ) Activates tissue plasminogen,


B ) Interferes with the carboxylation of coagulation factors
C ) Irreversibly inactivates cyclooxygenase

_______
D ) Potentiates the action of antithrombin 111 _,
E ) Selectively inhibits factor Xa

Correct Answer: D.

Heparin results in immediate anticoagulation through binding and potentiating the action of anti-thrombin 111, causing the inhibition of thrombin and factor Xa. It is commonly used for anticoagulation in the setting of deep vein thrombosis, pulmonary embolism, and
acute coronary syndrome. Heparin-induced thrombocytopenia (HIT) is a potential complication of heparin therapy. HIT is characterized by throinbocytopenia (a decrease of greater than 50% from baseline) that typically occurs between 5 to 10 days following heparin
exposure, along with evidence of arterial or venous thrombosis. Complications include organ ischemia, skin necrosis, and limb gangrene. HIT occurs secondary to the development of lgG antibodies against the complex of heparin and platelet factor 4. This leads to
the activation of platelets, which can cause thrombosis and consumption of platelets. This abnormal activation yields progressively decreasing platelet counts, which typically nadir at greater than 50% from their initial level. Treatment requires cessation of all
heparin-related products (eg, unfractionated heparin and low molecular weight heparin), and the initiation of a direct thrombin inhibitor or direct Xa inhibitor to ensure adequate anticoagulation without worsening the thrombocytopenia.

Incorrect Answers: A, B, C, and E.

Tissue plasminogen activator (TPA) activates tissue plasminogen (Choice A). TPA is a thrombolytic that cleaves plasminogen to form active plasmin. Plasmin degrades the cross-links between fibrin leading to the dissolution of blood clots. The major adverse effect
of IPA is bleeding.

Warfarin is an anticoagulant that interferes with the carboxylation of coagulation factors (Choice B). This occurs through the inhibition of vitamin K epoxide reductase, which is necessary for recycling vitamin K, an essential cofactor in the synthesis of coagulation
factors in the liver. Warfarin may introduce a transient hypercoagulable state when initiated, which increases the risk for skin necrosis. Typically, warfarin is begun with the concomitant administration of heparin to avoid this complication.

Aspirin is an antiplatelet agent that irreversibly inactivates cyclooxygenase (Choice C), which prevents the conversion of arachidonic acid to thromboxane A2. Thromboxane A2 is involved in platelet activation and its decreased synthesis results in diminished platelet
activation. Complications of aspirin use include bleeding, Reye syndrome, and a mixed respiratory alkalosis-metabolic acidosis (in the setting of salicylate toxicity).

Novel (or direct) oral anticoagulants include apixaban, rivaroxaban, and edoxaban, which selectively inhibit factor Xa (Choice E). The other major novel oral anticoagulant is dabigatran, which is a direct thrombin inhibitor. The major adverse effect of the direct oral
anticoagulants is bleeding.

Educational Objective: Heparin is a common anticoagulant that potentiates the action of anti-thrombin Ill to inhibit multiple coagulation factors. A potential complication of heparin therapy is HIT, which is an antibody-mediated reaction to heparin-platelet factor 4
complexes that results in the abnormal activation of platelets, with subsequent thrombocytopenia and arterial or venous thromboembolic disease.

r
Prevlou&
r
Next
,
Score Report
t1E
Lab Values
F
Calculator
r
Help
If'
Pause
Exam Section 2: Item 37 of 50 National Board pf Medical Examiners®
Comprehensive tsaslc Science Sen-Assessment

✓ 37. A 68-year-old woman comes to the physician because of a 1-year history of severe abdominal pain after meals; she also has had an unintentional 9-kg (20-lb) weight loss during this period. The pain is relieved when she decreases the amount of food that
she eats. She has a history of atherosclerosis and underwent triple coronary artery bypass grafting 2 years ago. Physical examination shows a soft, nontender abdomen and an abdominal bruit. Pedal pulses are diminished. The most likely cause of this
patient's symptoms is stenosis of which of the following arteries?

A ) Greater pancreatic
B) Hepatic
C) Right gastric
D) Superior mesenteric
E ) Supraduodenal
Correct Answer: D.

Mesenteric ischemia occurs when there is a reduction in blood flow to the small intestine. It is either acute or chronic, in which acute mesenteric ischemia occurs due to an embolic or thrombotic arterial obstruction, and chronic mesenteric ischemia is caused by
chronic atherosclerotic stenosis of the celiac or mesenteric arteries, with the superior mesenteric artery the most commonly affected. Chronic mesenteric ischemia is characterized by recurring episodes of abdominal pain after eating, which is hypothesized to be
secondary to mismatch between intestinal blood flow and metabolic demands, or secondary to small intestinal hypoperfusion when blood is shunted to the stomach during meals. Symptoms can be progressive and lead to the avoidance of eating and weight loss.
Clinicians should have a high index of suspicion for chronic mesenteric ischemia especially in patients who exhibit other sequelae of atherosclerotic disease, including peripheral arterial disease with diminished pedal pulses or coronary artery disease.

Incorrect Answers: A, B, C, and E.

The greater pancreatic artery (Choice A) is a branch of the splenic artery that supplies the pancreatic tail and body. The common hepatic artery (Choice B) supplies the liver, pylorus, duodenum, pancreas, and gallbladder. It branches into the proper hepatic artery,
gastroduodenal artery, and the right gastric artery (Choice C), which supplies the lesser curvature of the stomach. The supraduodenal artery (Choice E) is a branch of the gastroduodenal artery that supplies the first and second parts of the duodenum. None of these
arteries supply the small intestine and would be unlikely to contribute to the symptoms of chronic mesenteric ischemia.

Educational Objective: Mesenteric ischemia occurs when there is a reduction in blood flow to the small intestine. Chronic mesenteric ischemia is caused by chronic atherosclerotic disease of the celiac or mesenteric arteries, with the superior mesenteric artery the
most commonly affected, and is characterized by recurring episodes of abdominal pain after eating, avoidance of food, and weight loss.

r
Previous
r
Next
,:
Score Report

Lab Values
F
Calculator
,:,
Help
If'
Pause
Exam Section 2: Item 3B of 50 National Board of Medical ExilJlllners®
Comprehensive i:sas1c Sctence Serr-Assessment

✓ 38. A 52-year-old woman with type 2 diabetes mellitus and hypercholesterolemia comes to the physician for a follow-up examination. Since her diagnosis 14 months ago, she has been exercising regularly, eating healthier foods, and taking omega-3 fatty acid
supplements. She has had a 9-kg (20-lb) weight loss during this period. She is 168 cm (5 ft 6 in) tall and now weighs 62 kg (137 lb); BMI is 22 kg/m2. Physical examination shows no abnormalities. Serum studies show decreased glucose and cholesterol
concentrations since her last visit 2 months ago. She says, "I'm glad everything is fine, but I am worried that I won't be able to stay on this regimen and that my health will get worse." Which of the following best describes this patient's stage of change?

A ) Precontemplation
B ) Contemplation
C) Preparation
D) Action
E ) Maintenance

Correct Answer: E.

This patient has already made significant behavioral changes, resulting in improved health parameters, but is concerned she may revert to her previous unhealthy diet and exercise habits. This scenario is consistent with the maintenance stage of behavioral
change. The physician and patient may collaboratively strategize about how to avoid relapse, which may involve bolstering skills for coping with temptation or joining a diabetes support group. The stages of behavioral change are used to define a patient's readiness
to change a health-related behavior such as treatment adherence, substance use, diet, or exercise habits. In sequential order, the stages of behavioral change are precontemplation, contemplation, preparation, action, maintenance, and termination. Physicians aim
to move patients through these stages over time with an interview technique called motivational interviewing. Motivational interviewing involves using open-ended, non-judgmental questions to help the patient explore their reasons for wanting to change or maintain
the habit.

Incorrect Answers: A, B, C, and D.

Patients in the precontemplation stage (Choice A) are not interested in changing their habit and may not see the benefit in making a change. This patient is aware of the benefits of exercise and a healthy diet and has already changed her habits, which signifies she
has surpassed the precontemplation stage.

Patients in the contemplation stage (Choice B) are typically aware of the benefits of the behavior change but may be ambivalent or indecisive about change. This patient has already improved her diet and exercise habits, which signifies that she is beyond the
contemplation stage.

Patients in the preparation stage (Choice C) have committed to making a change and are ready to discuss strategies and resources to help them make the change. For example, a patient may research information about a low-carbohydrate diet to improve his
eating habits. This patient has already changed her habits and has therefore surpassed the preparation stage.

In the action stage (Choice 0), patients start to take steps to change their behavior. For example, a patient may begin a low-carbohydrate diet to improve his unhealthy eating habits. This patient has established a new routine that involves healthier diet and exercise
habits, signifying that she is beyond the action stage.
Educational Objective: In the maintenance stage of behavioral change, patients have sustained a behavioral change. Physicians and patients may strategize about how to prevent relapse.

r r ,- f.!i F tit If'>


Previous Next Score Report Lab Values Calculator Help Pause
Exam Section 2: Item 39 of SO National Board of Medical Examiners®
Comprehensive 1:1as1c Science Serr-Assessment

✓ 39. A 76-year-old woman comes to the physician because of a 2-day history of abdominal pain, nausea, and vomiting. Her temperature is 38.9° C (102 °F), pulse is 88/min, respirations are 26/min, and blood pressure is 117/79 mm Hg. Abdominal examination
shows tenderness of the left lower quadrant. Laboratory studies show a leukocyte count of 22,500/mm3 with 8% bands. A CT scan of the abdomen shows an abscess in the left lower quadrant. T he patient undergoes drainage of the abscess and a partial
sigmoidectomy for a ruptured diverticulum. Neutrophil activation in this patient is directed, in part, by bacterial lipopolysaccharides. As a result, these lipopolysaccharides most likely bind to which of the following neutrophil receptor types?

A ) Cytokine
B ) G protein-linked
C) Ion channel-linked
D) Nuclear
E ) Toll-like

Correct Answer: E.
Acute diverticulitis presents due to infection or inflammation of existing colonic diverticula, which are small outpouchings of the bowel wall, most commonly located in the sigmoid colon. Diverticulitis presents with acute left lower quadrant abdominal pain, sometimes
accompanied by diarrhea, fever, and tenderness to palpation in the left lower quadrant. Complications of acute diverticulitis include abscess or fistula fonnation, and perforation (rupture). The innate immu·ne response to acute diverticulitis includes neutrophil
activation by lipopolysaccharide binding to toll-like receptors on the neutrophil surface. Toll-like receptors bind to pathogen-associated molecular patterns (PAMPs) such as bacterial lipopolysaccharides and activate the NF-KB pathway. NF-KB is a transcription factor
that induces the expression of several proinflammatory genes, promoting the production of cytokines, chemokines, and other inflammatory mediators.
Incorrect Answers: A, B, C, and D.
Cytokine (Choice A) receptor binding modulates the response of neutrophils to infection. Proinflammatory cytokines include interleukin-1 (IL-1), 6, 8, 12 and 18, interferons, and tumor necrosis factor. Anti-inflammatory cytokines include IL-4, 10, 11, and 13.
Lipopolysaccharides bind to toll-like receptors on neutrophils, not cytokine receptors.
G protein-linked (Choice B) receptors are involved in neutrophil chemotaxis. Ligands include leukotriene 84, platelet-activating factor, and complement fragment C5a.

Ion channel-linked (Choice C) receptors are involved in modulating several stages of neutrophil recruitment T hey do not bind with pathogen-associated molecular patterns.
Nuclear (Choice D) receptors are ligand-related transcription factors that control gene expression in response to external signals. Lipopolysaccharides bind to extracellular toll-like receptors on neutrophils, not intracellular nuclear receptors.
Educational Objective: Neutrophil activation as part of the innate immune response to infection involves binding of pathogen-associated molecular patterns to toll-like receptors on the neutrophil surface, leading to NF-KB activation and production of proinflammatory
mediators.

r r ,: � F ,:, If'
Previous Next Score Report Lab Values Calculator Help Pause
Exam Section 2: Item 40 of SO National Board of Medical Examiners®
Comprehensive 1:1as1c Science Serr-Assessment

✓ 40. A 28-year-old white woman comes to the physician for a follow-up examination. Two days ago, she was discharged from the hospital after being treated for a hypertensive crisis with seizure. She has no other history of major medical illness. Current
medications include a 13-adrenergic blocker, dihydropyridine calcium channel blocker, and a centrally acting agent stimulating presynaptic a 2 receptors. Her temperature is 37°C (98.6° F), pulse is 54/min, and blood pressure is 182/100 mm Hg. Physical
examination shows trace ankle edema. Cardiac examination shows an S4 gallop. A soft abdominal bruit is heard left of the umbilicus. Abdominal ultrasonography shows normal-sized kidneys and a normal liver, spleen, pancreas, and gallbladder; a small
left ovarian cyst is noted. Laboratory studies show that complete blood count, serum electrolyte concentrations, and renal and liver function test results are within the reference ranges. Which of the following is the most likely cause of this patient's
secondary hypertension?

A ) Chronic kidney disease


B ) Coarctation of the aorta
C) Fibromuscular dysplasia of the left renal artery
D) Hyperthyroidism
E ) Primary hyperaldosteronism
Correct Answer: C.
Fibromuscular dysplasia is a non-inflammatory and non-atherosclerotic angiopathy of medium-sized arteries (eg, renal, carotid), that results in multifocai fibrous and muscular thickening of the arterial wall. The resultant stenosis causes secondary hypertension due
to abnormal stimulation of the juxtaglomerular apparatus from low afferent blood flow leading to excessive production of renin and angiotensin. It can lead to severe hypertension in an otherwise healthy, young patient with no medical comorbidities or laboratory
abnormalities. Examination findings often include signs of left ventricular hypertrophy such as an S4 gallop and a renal artery bruit auscultated lateral to the umbilicus on the affected side. Angiography may reveal a bead-like appearance of the renal artery.
Treatment involves angioplasty or stenting of the stenosed renal artery to improve flow. ACE inhibitors can be considered for unilateral stenosis but can lead to acute renal failure in the setting of significant bilateral renal artery stenosis.
Incorrect Answers: A, B, D, and E.
Chronic kidney disease (Choice A) can have variable etiology, such as with chronic prerenal disease in the setting of heart failure or cirrhosis with decreased renal perfusion, intrinsic renal disease such as nephrosclerosis or atherosclerotic renal artery disease,
chronic nephritic or nephrotic glomerular disease, or chronic postrenal obstructive disease. Chronic kidney disease is more common in older patients after years of underlying kidney injury.
Coarctation of the aorta (Choice 8) often presents in childhood with hypertension in an upper extremity and a systolic murmur. It is associated with weak lower extremity pulses and differentially lower blood pressure in the lower extremities.
Hyperthyroidism (Choice D) may cause hypertension, but would be associated with other symptoms of hyperthyroidism, including heat intolerance, sweating, hair loss, warm, flushed skin, amenorrhea, diarrhea, palpitations, and weight loss. Vital signs may also
reveal hyperthermia, tachycardia, and tachypnea.
Primary hyperaldosteronism (Choice E) may cause hypertension but would be associated with hypokalemia and metabolic alkalosis. This would be unlikely in a patient with normal serum electrolytes.
Educational Objective: Fibromuscular dysplasia is a non-inflammatory and non-atherosclerotic angiopathy of medium-sized arteries that can lead to renal stenosis and secondary hypertension due to excessive production of renin and angiotensin. It commonly
presents in a young patient with no medical comorbidities or abnormal laboratory findings.

Pni�t
r r
ous _______1!11_x:t�
,: � F
La_l>_Vallle_11 -��lculat�r
,:, If'
Help__ Pau_se
_ _ _ §.cor!I�epp�

Exam Section 2: Item 41 of 50 National Board pf M11dlcal Examiners®
Comprehensive isas1c Science Serr-Assessment

✓ 41. A 75-year-old woman with non-small cell carcinoma of the lung is brought to the physician because of a 2-day history of lethargy and cognitive impairment. One month ago, she completed a 1-year regimen of chemotherapy and cranial irradiation. One
week ago, she received the diagnosis of major depressive disorder and began therapy with a selective serotonin reuptake inhibitor. She speaks with a blunted affect. Physical examination shows no abnormalities. Her Mini-Mental State Examination score
is 20/30. Serum studies show a sodium concentration of 122 mEq/L. The patient appears confused and falls asleep during 1he examination. Which of the following is the most likely diagnosis?

--- A) Delirium
8 ) Dementia, Alzheimer type
C) Dementia caused by Pick disease
D) Serotonin syndrome
E ) Vascular dementia
Correct Answer: A.

Delirium is an acute confusional state typically associated with acute medical illness in older patients. The pathogenesis of delirium is poorly understood but may be related to deficient acetylcholine signaling in the brain. Delirium presents with acute disturbances in
awareness, attention, and baseline cognition that fluctuate in severity over the course of the day and appear related to a substance or acute medical condition. The cognitive changes can include memory or language deficits, disorientation, or perceptual
disturbances (eg, visual hallucinations or paranoia). This patient demonstrates clear disturbances in awareness and cognition, as well as severe hyponatremia (likely due to syndrome of inappropriate antidiuretic hormone secretion associated with brain irradiation
and selective serotonin reuptake inhibitor initiation). Delirium prevention with cognitive stimulation and avoidance of cognition-impairing medications is key. Delirium is managed by addressing the underlying medical problem and supportive management such as
frequent reorientation and promoting sleep only at night to re-establish the appropriate circadian rhythm. Antipsychotic medication can manage symptoms of agitation or distressing hallucinations but does not shorten the duration of delirium.

Incorrect Answers: B, C, D, and E.

Dementia, Alzheimer type (Choice 8) or caused by !Pick disease (Choice C) would present with insidious cognitive decline instead of acute confusion. Fluctuations in the severity of cognitive deficits over the course of the day would be atypical, unless a concomitant
delirium was also present. Baseline dementia is a risk factor for developing acute delirium. Pick disease (also known as frontotemporal dementia) typically develops when patients are in their 50s and features personality changes in addition to cognitive decline.

Serotonin syndrome (Choice D), the result of serotonin toxicity (eg, from serotonergic medications such as selective serotonin reuptake inhibitors), can lead to the triad of acute confusion, autonomic hyperactivity (hyperthermia, tachycardia, hypertension), and
neuromuscular excitability (tremor, myoclonus, hyperreflexia). Due to serotonin receptors in the gastrointestinal tract, vomiting and diarrhea can also occur. In this patient with severe hyponatremia and no evidence of neuromuscular excitability or autonomic
instability, delirium is more likely.

Vascular dementia (Choice E) features stepwise cognitive decline following cerebral infarction (stroke). This patient does not demonstrate focal neurological deficits indicative of a current or prior stroke, and fluctuations in the severity of cognitive deficits over the
course of the day would be atypical for vascular dementia.

Educational Objective: Delirium is an acute confusional state typically associated with acute medical illness in older patients. Delirium presents with acute disturbances in awareness, attention, and baseline cognition that fluctuate in severity over the course of the
day.

r
Previous
r
Next
,
Score Report
rc5
Lab Values
e
Calculator
r,
Help
"'
!Pause
'
Exam Section 2: Item 42 of 50 Natlonal Board pf Medical Examiners®
Comprehensive Basic Sctence Self-Assessment

42. An 18-year-old man is brought to the emergency department 30 minutes after sustaining injuries in a motor vehicle collision. He has abdominal pain. His pulse is 120/min, and blood pressure is 80/60 mm Hg. Physical examination shows multiple
contusions over the trunk and abdominal tenderness. A chest x-ray shows multiple rib fractures. A CT scan of the abdomen is shown. Which of the following organs is most likely injured in this patient?

A) Bladder
B ) Liver
C) Pancreas
D) Spleen
E ) Stomach
Correct Answer: D.

The spleen is one of the most common organs damaged during blunt abdominal trauma, sustaining injuries such as laceration and rupture. The CT image demonstrates the upper abdomen, with a nonnal-appearing liver in the right upper quadrant and a complex
high-grade laceration and rupture of the spleen in the left upper quadrant surrounded by extravasated blood. Splenic rupture can cause large amounts of blood to leak into the peritoneal space, which can result in hemodynamic instability characterized by
tachycardia and hypotension. Patients with splenic injuries may have left upper quadrant tenderness, peritonitis from blood products irritating the peritoneum, or referred pain to the left shoulder secondary to diaphragmatic irritation. In hemodynamically stable
patients, minor splenic injuries may be treated nonoperatively with observation and reassessment. In unstable patients, exploratory laparotomy is required for repair of the injury or splenectomy, depending on the nature and severity of the injury.

Incorrect Answers: A, B, C, and E.

The bladder (Choice A) is not shown in this CT image. Blunt abdominal trauma to the lower abdomen or pelvis can cause bladder rupture and urine leakage into the abdominopelvic cavity. Bladder injuries are commonly associated with pelvic fractures. Patients with
bladder injuries may present with gross hematuria, suprapubic tenderness, difficulty urinating, or peritonitis from irritation by intraperitoneal urine.

Liver (Choice B) injuries in blunt abdominal trauma may cause hemoperitoneum, resulting in hemodynamic instability. However, the CT image reveals a normal appearing liver in the right upper quadrant without any evidence of injury.

Pancreatic (Choice C) injuries in blunt trauma are less common. Clinical evaluation is nonspecific for pancreatic injury, and may present with abdominal tenderness, abdominal wall ecchymosis, emesis, or may be asymptomatic. Diagnosis can be made with CT
imaging to assess for discontinuity of the pancreatic parenchyma and duct, but the pancreas is not shown in this image.
The stomach (Choice E) is located in the left upper quadrant adjacent to the spleen, but rupture or perforation of the stomach following blunt trauma is uncommon. Findings on CT scan suggesting stomach or other gastrointestinal injury include pneumoperitoneum
and intra-abdominal fluid in the absence of a solid organ injury. There is intra-abdominal fluid seen on tine image, but it is related to blood products from the splenic rupture and not due to a stomach injury.

Educational Objective: The spleen is one of the most commonly injured organs in the setting of blunt abdominal trauma. It can present with abdominal tenderness and if severe, hemodynamic instability from large volume hemoperitoneum. Treatment is supportive
for stable patients but requires exploratory laparotomy with repair or resection of the spleen for unstable patients.

r r , � F � r
Previous Next Score Report Lab Values Calculator Help Pause
Exam Section 2: Item 43 of 50 National Board pf Medical Examiners®
Comprehensive 1:Jas1c Science Serr-Assessment

X 43. A 15-year-old girl is brought to the physician because of a 3-day history of fever, sore throat, and malaise. Her temperature is 39.2° C (102.6° F). Physical examination shows diffuse pharyngeal erythema, moderately enlarged tonsils, and tender anterior and
posterior cervical lymphadenopathy. A complete blood count shows:
Leukocyte count 19,500/mm� (N=3500-10,500)
Segmented neutrophils 30%
Bands 7%
Eosinophils 2%
Lymphocytes 25%
Lymphocytes, atypical 30%
Monocytes 6%

Incubation of this patient's serum with sheep erythrocytes results in agglutination. The atypical lymphocytes in this patient are most likely which of the following cell types?

A ) B lymphocytes
B ) CD4+ T lymphocytes
C) CDS+ T lymphocytes
D) FOXP3-expressing regulatory T lymphocytes
E ) Natural killer cells
Correct Answer: C.

Infectious mononucleosis is a viral illness caused often by Epstein-Barr virus (EBV). EBV is transmitted through respiratory secretions and saliva, causing the illness to be commonly acquired by teenagers and young adults. EBV infects B lymphocytes through
CD21. While B lymphocytes are the cell type infected, atypical lymphocytes seen on peripheral blood smear are actually cytotoxic, COB+ T lymphocytes, which are reacting to the viral infection. As a result, patients often have leukocytosis with a lymphocytic
predominance. Cytomegalovirus (CMV) is another cause of infectious mononucleosis and an acute HIV infection may present with a similar syndrome. Clinically, the syndrome commonly presents with fever, lymphadenopathy (typically involving the posterior
cervical lymph nodes), and hepatosplenomegaly along with pharyngitis. Patients with infectious mononucleosis will typically test positive on the Monospot test, which detects heterophile antibodies through the agglutination of sheep or horse erythrocytes. Treatment
is supportive and patients should be counseled to avoid any contact sports until the splenomegaly has resolved due to the risk of splenic rupture. If amoxicillin is administered for treatment of a misdiagnosed pharyngitis thought to be from Streptococcus pyogenes,
patients may develop a generalized maculopapular rash.

Incorrect Answers: A, B, D, and E.

While the EBV virus infects B lymphocytes through CD21 (Choice A), these are not the cell type that appear atypical on peripheral blood smear.

T lymphocytes can be broken into three groups. CDS+ T lymphocytes are cytotoxic and recognize major histocompatibility complex (MHC)-1 on the surface of antigen presenting cells. They are responsible for the direct killing of infected cells. CD4+ T lymphocytes
(Choice B) recognize MHC-11 on the surface of antigen presenting cells. They assist B lymphocytes in making antibodies and create a cytokine milieu, which recruits macrophages and leukocytes to the site of infection. Regulatory T lymphocytes (Choice D), regulate
the adaptive immune response to preserve self-tolerance and prevent autoimmunity. Neither CD4+ T lymphocytes nor regulatory T lymphocytes take on an atypical appearance during EBV infection.

Natural killer cells (Choice E) are a component of the innate immune system. They directly cause the apoptosis of virally infected and neoplastic cells through the use of perforin and granzymes. They are activated by nonspecific activation signals such as the
absence of MHC-I or the presence of pathogen associated molecular proteins. They do not take on an atypical appearance during EBV infection.

Educational Objective: Infectious mononucleosis is a viral illness caused often by infection with Epstein-Barr virus (EBV). While EBV infects B lymphocytes through CD21, the atypical lymphocytes seen on peripheral blood smear are cytotoxic CDS+ T lymphocytes
reacting to the viral infection.

r r ,r: ta P r ti"
Previous Next Score Report Lab Values Calculator Help Pause
Exam Section 2: Item 44 of 50 National Boa.rd pf M·11dlcal Exi\llllners®
Comprehensive tsas1c science Serr-Assessment

✓ 44. A 75-year-old man with alcoholism is brought to the emergency department 30 minutes after he began vomiting blood. He has a history of portal hypertension. Abdominal examination shows spider angiomata. Which of the following is the most likely cause
of this patient's hematemesis?

A}Anaslomosis bel\Yeen t11e s1a11:1erior and inferior mesenteris •�eins


B } Aortoesophageal fistula
G}Inflammation of 1110 distal esopl1ag1a1s
D} Retrograde blood flow between tributaries of the left gastric to the azygos veins
Ii}Sl11a1nling of blood lrom the portal para1a1mbilisal veins to tl1e s1a1perior epigastricveins
Correct Answer: D.

Cirrhosis can present with edema, ascites, increased bilirubin, jaundice, spider angiomata, and sequelae of portal hypertension such as esophageal varices, splenomegaly, caput medusae, and rectal varices. It typically occurs in patients with preceding conditions
such as alcohol abuse or chronic hepatitis. In the setting of portal hypertensidn, there is the potential for retrograde blood flow between tributaries of the left gastric and azygos veins (portosystemic anastomosis}. This results in the formation of esophageal varices,
which are dilated submucosal blood vessels. Acutely bleeding esophageal varices typically present with hematemesis, and treatment requires supportive therapy with intravenous fluids and transfusions, along with attempts to stop the bleeding through endoscopic
guided sclerosis or banding of the esophageal varices. A transjugular intrahepatic portal shunt can be used in the prevention of variceal bleeding by creating a shunt between the portal vein and hepatic vein, which decreases portal pressure by shunting blood to the
systemic circulation, bypassing the liver.

Incorrect Answers: A, B, C, and E.

Anastomosis between the superior and inferior mesenteric veins (Choice A} is a normal anatomic variant. The inferior mesenteric vein normally drains into the splenic vein, which in turn combines with the superior mesent.eric vein to form the portal vein.

An aortoesophageal fistula (Choice B} is a life-threatening abnormal communication between the aorta and esophagus, leading to gastrointestinal bleeding and hematemesis. It can be caused by a thoracic aortic aneurysm, foreign body such as an aortic graft,
esophageal malignancy, or postoperative complication. It causes massive upper gastrointestinal hemorrhage and is often fatal. It is not associated with portal hypertension.

Inflammation of the distal esophagus (Choice C} such as with chronic reflux esophagitis can lead to tissue metaplasia and portends an increased risk of esophageal adenocarcinoma. It would be an unlikely cause of large volume hematemesis.

Shunting of blood from the portal paraumbilical veins to the superior epigastric veins (Choice E} of the anterior abdominal wall is also a potential consequence of portal hypertension. This presents as engorged superficial abdominal wall veins referred to as caput
medusae. These do not present with hematemesis.

Educational Objective: Portal hypertension is a consequence of cirrhosis and can lead to the formation of esophageal varices secondary to retrograde flow through the portosystemic anastomosis between the left gastric and azygos veins. Varices are dilated
submucosal blood vessels prone to bleeding and may cause massive gastrointestinal bleeding and hematemesis.

r
Previous
r
Next
,
Score Report

Lab Values
P
Calculator
r
Help
r
Pause
Exam Section 2: Item 45 of 50 National Board of Medical Examiners®
Comprehensive sas1c Science Serr-Assessment

✓ 45. A 10-year-old boy with spinal muscular atrophy is brought to the emergency department because of a 2-week history of nausea and vomiting. He also has had decreased appetite during this period. One week prior to his illness, his entire family had similar
symptoms which improved after a few days. Physical examination shows decreased skin turgor, dry mucous membranes, and diffuse muscle weakness and atrophy. Laboratory studies show:
Serum
Na + 140 mEq/L (N=138-145)
K+ 4 mEq/L (N=3.�.7)
c1- 105 rnl;q/l (N=95-105)
HCO3- 18 mEq/L (N=22-28)
Urea nitrogen 18 mg/dL (N=S-18)
Glucose 30 mg/dL (N=60-100)
Urine
pH 5.0 (N=4.6-8.0)
Specific gravity 1.020 (N=1.003-1.030)
Glucose none
Protein trace
Ketones 4+
RBC 0/hpf
WBC 0/hpf

Which of the following biochemical cycles in the liver is most likely providing this patient's brain with an energy source at this time?

---8)A ) Alanine-glucose cycle


Camitine-acylcarnitine cycle
C) Lactic acid cycle
D) Tricarboxylic acid cycle
E ) Urea cycle
Correct Answer: B.
Spinal muscular atrophy is a group of autosomal recessive disorders that lead to the degeneration of motor neurons and result in progressive hypotonia and areflexia. Patients may exhibit difficulty with mobility and frequently experience respiratory distress and
difficulty feeding. The brain primarily utilizes glucose for energy, and during the fasting state is highly dependent on glycogenolysis and hepatic gluconeogenesis. In the initial fasting state, glycogen is broken down in the liver to assist in the maintenance of blood
glucose levels. Glycogen stores are typically depleted after 1 day of fasting. Following this, blood glucose is maintained through the breakdown of muscle and fat to provide substrates for hepatic gluconeogenesis. After approximately 3 days of fasting, fatty acids
become the primary source for energy through the production of ketone bodies. This patient's laboratory values, including profound hypoglycemia, decreased bicarbonate, and ketonuria, are suggestive of starvation ketoacidosis in the setting of acute illness and
prolonged fasting from persistent nausea and vomiting. Carnitine and acylcarnitine are components of the camitine shuttle and serve to transport acyl-CoA into the mitochondrial matrix to allow for the beta-oxidation of fatty acids. During the initial step of the shuttle,
carnitine palmitoy1transferase I transfers the acyl group from fatty acyl-CoA to carnitine, forming acylcarnitine. Acylcamitine is then transported across the inner mitochondrial membrane by camitine-acylcarnitine translocase, a step which is susceptible to negative
feedback by increased malonyl-CoA. In the final step, camitine and fatty acyl-CoA are liberated by carnitine palmitoyltransferase II on the inner mitochondrial membrane. This allows beta-oxidation to proceed within the mitochondrial matrix, which will support the
continued production of ketone bodies to provide the brain with an energy source during prolonged fasting.
Incorrect Answers: A. C, D, and E.
The alanine-glucose cycle (Choice A), also known as the Cahill cycle, utilizes alanine aminotransferase (ALT) to facilitate the transfer of alanine from skeletal muscle to the liver, where it can be used for gluconeogenesis, while also allowing skeletal muscle to rid
it,self of nitrogenous waste.
The lactic acid cycle (Choice C), also known as the Cori cycle, transports lactate produced by skeletal muscle to the liver, where it can be used for glucone-ogenesis. The lactic acid cycle is primarily active in the context of muscular exercise, rather than during the
fasting state. Ketoacidosis secondary to fasting Is a more likely etiology of metabolic acidosis than lactic acidosis In this patient.

Tricarboxylic acid cycle (Choice D) utilizes acetyl-CoA to produce NADH, FADH2, and GTP. The citric acid cycle relies on the glycolytic production of pyruvate and is dependent on the supply of glucose. This is not the primary source of energy for the brain in a
prolonged fasting state.
The urea cycle (Choice E) aids in the excretion of nitrogenous waste by converting ammonia to urea. The urea cycle does not produce glucose or other sources of energy for the brain.
Educational Objective: During a prolonged fasting state (greater than 3 days), the brain is dependent upon ketone bodies as its primary source of energy. Beta-oxidation, which requires the camitine-acylcarnitine cycle to transfer acy1 groups from the cytoplasm to
the matrix, allows for the production of ketone bodies. This manifests on laboratory evaluation with hypoglycemia, metabolic acidosis, and ketonuria.
Exam S-ectlon 2: Item 46 of 50 National Board of Medical Examiners®
Compreltenslve Basic Sctence Sen-Assessment

46. A 45-year-old man comes to the physician becaruse of a 6-month history of progressive shortness of breath with exertion. He has a history of frequent inosebleeds since adolescence. He is 178 cm (5 ft 10 in) tall and weighs 79 kg (175 lb); BMI is 25 kg/m2.
Physical examination shows the findings in the photographs. Inhaled albuterol does not improve his symptoms. Which of the following is the most likely cause of this patient's condition?

A ) Atrial septal defect


B; Hyta18F\dAb,v,1
C) Pulmonary arteriovenous shunting
D) Reactive airway disease
E ) Thromboembolism
Correct Answer: C.

Hereditary hemorrhagic telangiectasia (HHT ) is an autosomal dominant disorder of the vasculature that is characterized by arteriovenous malformations (AVMs) and telangiectasias, which are small vascular malformations which appear as bright red, blanching
lesions of the skin and mucous membranes (as seen in the oral photograph). There are several gene mrutations associated with the disorder that result in abnormal formation of capillaries and the manifestations of the disease. Patients classically present with
recurrent epistaxis, hematuria, and gastrointestinal bleeding. Cerebral AVMs may cause hemorrhagic stroke. Pulmonary AVMs can lead to embolic stroke as venous blood (and a potential embolus) is able to bypass the pulmonary capillary bed and enter the
systemic arterial circulation. Pulmonary AVMs also result in pulmonary arteriovenous shunting with subsequent chronic hypoxemia, which can lead to hypertrophic osteoarthropathy (digital clubbing) as seen in the photograph of the fingers. HHT is associated with
pulmonary arterial hypertension and iron deficiency anemia secondary to chronic blood loss.

Incorrect Answers: A, B, D, and E.

Atrial septa! defect (Choice A) is a common congenital malformation of the interatrial septum that results in a left-to-right shunt with abnormal flow of blood from the left atrium to the right atrium. If the atrial septal defect remains uncorrected, it can result in the
development of Eisenmenger syndrome secondary to prolonged pulmonary vasculature remodeling resulting in pulmonary arterial hypertension and shunt reversal leading to cyanosis. It is not associated with recurrent bleeding or mucocutaneous lesions.

Hypertension (Choice B) is a leading cause of cardiovascular mortality. Chronic hypertension can result in left ventricular hypertrophy and ultimately left-sided heart failure. Left-sided heart failure is the most common cause of right-sided heart failure. Hypertension
is not associated with mucocutaneous lesions.

Reactive airway disease (Choice D) is a term used to describe conditions which result in airway sensitivity to physical, chemical, or pharmacolo9ic stimuli (e9, asthma, COPD). Chronic, undia9nosed reactive airway disease may result in hypertrophic
osteoarthropathy. Symptoms would be expected to improve with albuterol.

Thromboembolism (Choice E) is a complication of Behcet syndrome, which is an autoimmune disorder characterized by small and large vessel vasculitis. It often presents with fever, weight loss, arthritis, enthesitis, and thrombosis or aneurysm secondary to
vascular inflammation. Physical examination may reveal recurrent oral aphthous ulcers, genital ulcers, and various skin lesions. The mucocutaneous lesions in this case are more consistent with a diagnosis of HHT.

Educational Objective: Hereditary hemorrhagic telangiectasia is an inherited disorder of vascular format iion that results in arteriovenous malformations and telangiectasias throughout the body. It is characterized by recurrent bleeding. Pulmonary arteriovenous
shunting can result in chronic hypoxemia.

r r , fr!i r- ,.. ,,.


Previous Next Score Report Lab Values Calculator Help Pause
Exam S-ectlon 2: Item 47 of 50 National Board pf Medical Examiners®
Comprellenslve Baste Science Serr-Assessment

✓ 47. A 66-year-old woman comes to the physician for a follow-up examination 3 months after she underwent a total colectomy for ulcerative colitis. She subsequently required an ileostomy. She feels generally well and has resumed a nearly normal diet. Her
temperature is 37°C (98.6 ° F), pulse is 92/min, and blood pressure is 100/60 mm Hg. Physical examination shows mildly decreased skin turgor and a well-healed ileostomy site. Laboratory studies show:
Serum
Na + 136 mEq/l
K+ 3 mEq/L
c1- 114 mEq/L
HCO3- 14 mEq/L
Urea nitrogen 32 mg/dl
Creatinine 1 mg/dl
Arterial blood gas analysis

on room air:
pH 7.32
Pco2 30 mm Hg
Po2 80 mm Hg

Which of the following best describes this patient's acid-base disturbance?

A ) Anion gap metabolic acidosis, partially compensated


B ) Anion gap metabolic acidosis, uncompensated
C)- Chro, 1h, r:esr •r-a1Ary alKalosis, partially sompensated
0)- Ch,o,1i1. Ftts!'"--� ,.uKalosis, unsompensated
E ) Non-anion gap metabolic acidosis, partially compensated
F ) Non-anion gap metabolic acidosis, uncompensated
Correct Answer: E.

This patient's laboratory and arterial blood gas evaluation demonstrates a non-anion gap metabolic acidosis with partial compensation. Non-anion gap metabolic acidosis is identified by a pH less than 7.35 (acidemia), with decreased PCO2 and HCO3-. The
respiratory system compensates for the acidosis by increasing minute ventilation, which eliminates carbon dioxide, an acid. This results in decreased PCO2 and increased pH back to approximately normal range. This patient's persistently decreased pH suggests
+
that respiratory compensation is only partial. Metabolic acidosis describes a derangement involving excess acid and is subdivided into anion gap and non-anion gap categories. Anion gap ([Na ] - ([Cll + [HCO37) calculates unmeasured anions in serum whi: ch are
contributing to acidity, whereas non-anion gap acidosis generally results from excess supply of protons or chloride, retention of protons or chloride by the kidney, or failure to secrete bicarbonate buffers. Non-anion gap acidosis is often a hyperchloremic metabolic
acidosis, as seen in this patient. Causes of non-anion gap acidosis include infusions of normal saline, Addison disease, renal tubular acidosis, gastrointestinal losses of bicarbonate (eg, diarrhea), spironolactone, and acetazolamide use. Chloride retention or
oversupply occurs in nearly all of these etiologies. In the case of diarrhea or gastrointestinal disturbance, bicarbonate secreted in the small intestine is normally reclaimed in the large intestine in exchange for chloride ions. If the colon cannot reclaim bicarbonate,
such as following colectomy or during diarrheal illness, the level of serum chloride will remain increased and serum bicarbonate decreased.

Incorrect Answers: A, B, C, D, and F.

Anion gap metabolic acidosis, partially compensatedl (Choice A) and anion gap metabolic acidosis, uncompensated (Choice B) would present with an increased anion gap, which is not demonstrated on this patient's laboratory evaluation. Causes of anion gap
metabolic acidosis arise from increased intake or generation of or decreased metabolism or excretion of organic acids, including uremia, ketoacidosis, lactic acidosis, cyanide or carbon monoxide toxicity, methanol, ethylene glycol, or propylene glycol toxicity, iron
overdose, or salicylate overdose (late).

Chronic respiratory alkalosis, partially compensated (Choice C), or chronic respiratory alkalosis, uncompensated (Choice D), are not demonstrated on this patient's laboratory analysis, and would be marked by alkalotic pH (greater than 7.45). Respiratory alkalosis
results from hyperventilation and is seen in cases of panic attacks, early salicylate intoxication, pulmonary embolism, high altitudes (hypoxemia), and early asthma exacerbations without significant obstruction or ventilation-perfusion mismatch.

Non-anion gap metabolic acidosis, uncompensated (Choice F) is incorrect, as Winter's formula demonstrates partial compensation. Determining what the partial pressure of carbon dioxide should be, given bicarbonate level, is as follows: PCO2 "' 1.5 x [HCO37 +8,
:t: 2. Therefore, 1.5 x 14 mEq/L + 8 "'29 mm Hg ± 2. If the respiratory system is appropriately compensating, the partial pressure of carbon dioxide should be between 27 and 31 mm Hg. It is 30 mm Hg, yet t.he pH is slightly acidic, indicating partial compensation.

Educational Objective: Non-anion gap metabolic acidosis is identified by a pH less than 7.35 (acidemia), with decreased PCO2 and HCo3-. The respiratory system compensates for the acidosis by increasing minute ventilation, which eliminates carbon dioxide, an
acid. Decreased levels of carbon dioxide with persistent acidic pH suggests partial compensation.

r r , fc; P ,-. ,,.


se
Va es C ao e
Exam Section 2: Item 48 of SO National Board pfMedical Examiner,;®
Comprehensive Basic Science Self-Assessment

✓ 48. A 65-year-old man who is a veteran comes to the office because of a 3-month history of increasingly severe pain of his low back. He has no history of major medical illness. Physical examination shows point tenderness over the spine at the level of LS.
Laboratory studies show:
Hemoglobin 9.8 g/dL
Hematocrit 28%
Serum
Ca2• 11.2 mg/dL
Creatinine 2.2 mg/dL
lgG 8000 mg/dL (N=650-1500)

X-rays of the spine show a compression fracture at LS and four lytic lesions in the iliac crest. Examination of a bone marrow biopsy specimen confirms the diagnosis and the presence of a chromosomal translocation. Treatment with a chemotherapeutic
regimen, including a proteasome inhibitor. is started. Which of the following is most likely to occur in the affected cells as a result of this therapy?

A ) Decreased calcium release


8 ) Decreased genomic instability
C ) Decreased ubiquitination
D ) Increased protein degradation
E ) Induction of apoptosis

Correct Answer: E.

This patient's presenting findings of bone pain, anemia, hypercalcemia, renal dysfunction, hypergammaglobulinemia, and lytic bone lesions are consistent with a diagnosis of multiple myeloma. Multiple myeloma is a malignancy caused by the neoplastic proliferation
of plasma cells. Neoplastic plasma cells overproduce monoclonal immunoglobulin and light or heavy chains, which may result in acute renal failure. Patients also commonly present with constitutional symptoms of fatigue and weight loss, and hepatosplenomegaly,
with symptoms of hypercalcemla (eg, abdominal cramping, kidney stones, psychiatric disturbance), symptoms of anemia (eg, palror, light-headedness, dyspnea on exertion), or with opportunistic Infections secondary to immune dysfunction. Further workup Is
recommended with complete blood count, complete metabolic panel, urinalysis, urine and serum protein electrophoresis, and imaging to determine the extent of maligna ncy. The diagnosis Is confirmed with bone marrow biopsy. As with many hematologic and
lmmunoprolilerative malignancies, the Initiation of chemotherapy results in the rapid Induction of apoptosls In malignant cells. Proteasome Inhibitors, such as bortezomlb. result In Impaired proteasomal activity, especially In myeloma cells. Due to the increased
production of proteins by plasma cells, they depend on the ability to degrade unfolded or misfolded proteins to limit the stresses placed on the cell by such, which is the primary physiologic function of the proteasome. When the cells are unable to clear misfolded or
abnormal proteins, the induction of apoptosis occurs due to the response to unfolded protein. Patients initiating chemotherapy should be monitored for signs of tumor lysis syndrome, including hyperkalemia, hyperphosphatemia, hypocalcemia, and hyperuricemia.

Incorrect Answers: A, 8, C, and D.


Decreased calcium release (Choice A) from bone may occur after inijiation of chemotherapy but results as a secondary effect of decreased bone lysis in the setting of myeloma, rather than as a result of plasma cell apoptosis. Rather, lysis of malignant cells can
result in increased intracellular calcium release, one of the features of tumor lysis syndrome. Additionally, calcium influx into the cell is a pathway that can lead to apoptosis.

Increased, rather than decreased genomic instability (Choice 8), would result from some chemotherapeutic agents used against multiple myeloma. Chemotherapeutic agents such as cyclopho.sphamide, melphalan, and platinum-based agents, function by cross­
linking DNA, leading to increased genomic instability and induction of apoptosis.

Decreased ubiquitination (Choice C) will alter the processing of intracellular proteins by the proteasome-ubiquitin system. Proteasome inhibitor chemotherapeutic agents, such as bortezomib, achieve their therapeutic effect by impairing proteasome function and
causing the toxic accumulation of intracellular proteins and apoptotic factors, rather than by decreasing ubiquilination.

Increased protein degradation (Choice D) is not the mechanism of action of proteasome inhibitors. Proteasome inhibitors achieve their cytotoxic effects by decreasing protein degradation.

Educational Objective: Multiple myeloma is a malignant plasma cell neoplasm that presents with fatigue, weight loss, bone pain, tnepatosplenomegaly, immune deficiency, anemia, hypercalcemia. renal dysfunction, hypergammaglobulinemia, and lytic bone lesions.
Proteasome inhibitors function by inducing apoptosis of neoplastic plasma cells as misfolded proteins accumulate. Patients undergoing induction of chemotherapy should be monitored for tumor iysis syndrome.

r r , € F r r
Prevlous Next Score Report Lab Values Calculator Help Pause
Exam Section 2: Item 49 of 50 National Board pf Mqdlcal Examiners®
Comprehensive sas1c SClence Self-Assessment

✓ 49. A 58-year-old woman comes to the office for a follow-up examination 2 days after she noticed a lump in her left breast on self-examination. Menopause occurred 6 years ago. Physical examination shows a 1-cm, poorly circumscribed, nodular mass in the
left periareolar tissue. Mammography shows an ill-defined density suspicious for malignancy. A needle biopsy specimen is nondiagnostic. Microscopic examination of the excised mass shows a large dilated space lined by glandular epithelium and
surrounded by a dense lymphopnasmacytic inflammatory infiltrate; the associated lumen is filled with lipid-laden macrophages and amorphous debris. Which of the following is the most likely diagnosis?

A}Acute mastilis
B}Fibi:oadeAoma
C) Fibrocystic changes
D) lntraductal carcinoma
E) Mammary duct ectasia

Correct Answer: E.

Mammary duct ectasia is caused by widening of a lactiferous duct with cdncomitant fibrosis and inflammation, often near the nipple. It frequently occurs in perimenopausal and postmenopausal women. When ectatic ducts become blocked, they may become
nodular and can be felt as a mass, as in this patient. Other symptoms include nipple discharge, nipple retraction, and pain. On mammography, subareolar branching densities can be mistaken for spiculated neoplasms and calcifications may be present. On
histopathologic examination, mammary duct ectasia is marked by a dilated duct lined with glandular epithelium and surrounded by inflammatory cells. Lipid-laden macrophages result from lipid penetrating the duct wall and being engulfed by macrophages. Lipid is
usually contained within the duct. Duct ectasia is not inherently malignant, nor does it increase a patient's risk for developing breast carcinoma. However, the presence of a malignancy may cause the original blockage to form and thus mammary duct ectasia could
be an initial presentation of malignancy. When this is not the case, duct ectasia typically resolves with conservative management.

Incorrect Answers: A, B, C, and D.

Acute mastitis (Choice A) presents with erythema and induration of the breast in an area of tender, warm skin. It classically occurs in breastfeeding mothers as a consequence of microtrauma to the nipple leading to inoculation from cutaneous or oral flora.
Staphylococcus aureus is a common pathogen. It is uncommon in postmenopausal women and does not classically present with a breast lump.
Fibroadenoma (Choice 8) presents as a unilateral, nontender, mobile, rubbery mass in the breast, often in a younger woman. Ultrasound or mammography demonstrates a well-circumscribed, round tumor and needle biopsy is often unnecessary.

Fibrocystic changes of the breast (Choice C) are a benign and cyclic etiology of breast pain that presents with focal masses or nodules. They are seen primarily in premenopausal women as a manifestation of fluctuating hormones during the menstrual cycle.

lntraductal carcinoma (Choice 0), or ductal carcinoma in situ, is a breast cancer with ductal differentiation which has not yet invaded the basement membrane. Most cases of intraductal carcinoma in situ are discovered by mammography and patients do not usually
have any breast-related symptoms or physical examination findings.

Educational Objective: Mammary duct ectasia often occurs in perimenopausal women when a lactiferous duct becomes blocked and accumulating lipid and debris cause dilation to occur.

r r , fc!f F r ff'
Previous Next Score Report Lab Values Calculator Help Pause
Exam Sectlon 2: Item 50 of 50 National Board pf Mf!dlcal Examiners®
Comprehensive 15as1c science SelT•Assessment

✓ 50. An 1814-g (4-lb) female newborn is delivered in the hospital at 24 weeks' gestation to a 25-year-old primigravid woman via a spontaneous vaginal delivery. Examination of the newborn shows no peripheral cyanosis. There is a grade 3/6, continuous
murmur over the left midclavicular, second intercostal space. At the age of 2 days, the newborn develops respiratory distress. Echocardiography shows a failure of a vascular structure to close. The embryologic origin of this anomaly is most likely which of
the following aortic arches?

A ) First
B) Second
C ) Fourth
(;)�F-if:tl=I
E ) Sixth

Correct Answer: E.

This patient has a moderate to large patent ductus arteriosus (PDA) as evidenced by a continuous murmur and respiratory distress. In utero, oxygenated blood is obtained from the placenta, not from the lungs, and the ductus arteriosus (DA) allows the shunting of
oxygenated blood from the pulmonary artery directly to the aorta while bypassing the fetal lungs, which are filled with amniotic fluid. At birth, closure of the DA occurs by the release of prostaglandins and increased arterial oxygen tension. A DA that fails to close is
called a PDA, and if large enough, can lead to a profound left-to-right shunt, with consequent heart failure, respiratory distress, and pulmonary hypertension. PDAs that are small often go undetected until adulthood because the left-to-right shunt is not significant
enough to cause symptoms. The DA arises from the sixth aortic arch, which also gives rise to the proximal right pulmonary artery.

Incorrect Answers: A, B, C, and D.

First (Choice A) and second (Choice B) aortic arch structures include the maxillary artery, a part of the external carotid artery, the ascending pharyngeal artery, and the stapedial artery.

Fourth (Choice C) aortic arch structures include the right subclavian artery and the aortic arch from the origin of the left carotid artery to the beginning of the DA.

Fifth (Choice D) aortic arch structures regress and do not contribute to adult cardiovascular structures.

Educational Objective: The sixth aortic arch gives rise to the ductus arteriosus, a structure that allows shunting of placental oxygenated blood from the pulmonary artery to the systemic circulation during development in utero but closes shortly after birth. Persistence
of this structure is called a patent ductus arteriosus (PDA). Symptoms are dependent upon the size of the PDA and the degree of left-to-right shunting, with severe shunts causing right-sided heart failure and pulmonary hypertension.

r
Previous
r
}�ext
,-
Score �eport

Lab Values
F
Calculator
r
Help
If>
Pause
_
Exam Section 3: Item 1 of SO National Board pf Mqdlcal Examiners®
Comprehensive tsas1c science Serr-Assessment

y 1. The higher incidence of birth defects in children born to women more than 35 years old may be the result of environmental influences on chromosomes in developmentally arrested oocytes at which of the following stages of cell division?

A ) Prophase, meiosis I
S} Prophase, meiesis II
G} Metaphase, meiesis I
• D) Metaphase, meiosis II
e} Telephase, meiesis I

Correct Answer: A.

In the normal development of the oocY1e, women are born with a complement of follicles that develop in utero and arrest in prophase of meiosis I. Upon reaching menarche, some of the follicles mature during the menstrual cycle prior to being released during
ovulation. In response to the hormonal stimulation from pituitary follicle-stimulating hormone (FSH), the oocytes that may be ovulated advance from prophase of meiosis I to metaphase of meiosis II. If fertilization occurs, the oocyte advances through the remainder
of metaphase II to completion. In prophase of meiosis I, the chromosomes remain dormant in a paired tetrad (four total chromosomes: duplicated maternal and paternal chromosomes). As meiosis I advances through metaphase, anaphase, and telophase, the
recombined, crossed-over tetrads nonnally divide into two separate cells, where they subsequently arrest in metaphase II. Normal progression through meiosis II then leads to a single haploid chromosome in each of the four daughter cells. Nondisjunclion during
prophase I results in the daughter cells receiving an uneven complement of chromosomes. A nondisjunction event in meiosis I followed by a normal meiosis II results in four gametes, two of which lack a maternal or paternal chromosome, and two of which contain
both a ll)atemal and paternal chromosome. In the instance of normal separation in meiosis I followed by a nondisjunction event of meiosis II there are two chromosomes (both from the same parent) within one gamete, with the other gamete lacking the
chromosome. In the case of a 35-year-old woman, oocytes maturing and ovulated at that time would have remained arrested in prophase I for the lifetime of the patient until that point.

Incorrect Answers: B, C, D, and E.

Prophase, meiosis II (Choice B); metaphase, meiosis I (Choice C); and telophase, meiosis I (Choice E) occur in oocytes during the maturation process within a single cycle of follicle maturation. They are not arrest points of the cell division and genetic
recombination process.

Metaphase, meiosis II (Choice D) is the phase in which oocytes arrest once ovulated. Upon fertilization, the oocytes advance through the remainder of meiosis II to form haploid gametes.

Educational Objective: Oocytes remain arrested in prophase of meiosis I until the follicle matures in preparation for ovulation. As oocytes mature, they arrest again in metaphase II until fertilization. Nondisjunction in either meiosis I or II can result in the inheritance of
an abnormal number of chromosomes leading to disorders such as Down syndrome (trisomy 21).

r
Prevlous
r
Next
,-
Score Report

Lab Values
P
Calculator
r
Help
ttt
Pause
_ , _ _ _ .
Exam Section 3: Item 2 of 50 National Boa.rd pf M11dlcal Examiners®
Comprehensive 8aslc Science Serr-Assessment

✓ 2. A 50-year-old man collapses and dies while running to catch an airplane. A photograph of his aortic valve is shown. Which of the following best describes the lesion?

A ) Calcified congenital bicuspid valve


B ) Infective endocarditis
C) Myxomatous degeneration
D) Postin,flammatory valve stenosis

Correct Answer: A.

Calcified congenital bicuspid valve describes the lesion depicted in the gross photograph. The normal aortic valve is a tri-leaflet valve, but a bicuspid valve is found in between 0.5% to 2% of the population. It can be inherited or sporadic. Bicuspid valves, as a
result of their geometry_ and abnormal leaflet excursion, are subjected to more turbulent and shear forces, and become calcified much earlier than tri-leaflet valves as a result of chronic inflammation. Bicuspid valves are more prone to the development of early
aortic stenosis or aortic regurgitation. Abnormal valve architecture may present a nidus for infection; hence, infective endocarditis may be more frequently seen. The symptoms of aortic stenosis generally include lightheadedness or syncope with exertion, angina
pectoris, and dyspnea. Auscultation findings include a crescendo-decrescendo systolic murmur best heard at the upper right sternal border, and pulsus parvus et tardus (weak and delayed) may be noted on examination of peripheral pulses. Due to the chronic
increased after1oad from a fixed obstruction by the valve, left ventricular hypertrophy and resultant diastolic dysfunction can occur. If early aortic stenosis occurs, the patient may be a candidate for aortic valve replacement. Sudden cardiac death (SCD) may occur
in patients with severe aortic stenosis or severe aortic regurgitation.

Incorrect Answers: B, C, and D.

Infective endocarditis (Choice B) occurs at higher rates in patients with bicuspid aortic valves, but this patient's presentation with SCD is not consistent with endocarditis, which is diagnosed using the Duke criteria. Criteria include positive blood cultures, vascular
and immunologic phenomena such as Roth spots, Osler nodes, and Janeway lesions, and evidence of valvular vegetation on imaging. The photograph demonstrates a calcified, stenotic valve without vegetation.

Myxomatous degeneration (Choice C) describes progressive disarray of connective tissue within valve leaflets. It often affects the mitral and aortic valves and predisposes to aortic regurgitation in the latter. On gross examination, the leaflets typically appear
thickened.

Postinflammatory valve stenosis (Choice D) describes changes to the mitral or aortic valve seen after rheumatic heart disease. Gross examination reveals fibrosis, retraction of the leaflets, and fusion of the cusps.

Educational Objective: Bicuspid aortic valves occur in about 1 % of the population and predispose to early valvular pathology such as aortic stenosis or regurgitation. When severe, these pathologies can predispose to sudden cardiac death.

r
Previous
r
Next
,
Score fteport
fr3
Lab Values
P
Calculator
r-
Help
If"
Pause
Exam Section 3: Item 3 of 50 National Board pf M41dlcal Examiners®
Comprehensive Basic SClence Self-Assessment

✓ 3. A 2-month-old boy is brought to the physician because of a ?-clay history of hoarseness. His mother has AIDS and used cocaine throughout her pregnancy. Physical examination shows no abnormalities. A fiberoptic laryngoscopic examination shows a
nodule on the left vocal cord. Which of the following viruses is the most likely cause of this nodule?

A ) Cytomegalovirus
B ) Herpes simplex virus
G}-1,W
D ) Human papillomavirus
et- \laricella zoster virus
Correct Answer: D.

Human papillomavirus (HPV) is a common human pathogen that infects epithelial cells. There are multiple strains which may be low- or high-risk for the development of squamous cell carcinoma, which result in the development of a variety of lesions. Anogenital
manifestations are common (eg, cervical intraepithelial neoplasia, condylomata acuminata). Non-anogenital manifestations include verruca vulgaris, plantar warts, and mucosa! lesions of the oropharynx, respiratory epithelium, and larynx. Laryngeal papillomatosis
frequenUy affects the vocal cords and presents as a vocal cord nodule. Symptoms include dysphonia (hoarseness) and may progress to dyspnea, chronic cough, dysphagia, globus sensation, and airway obstruction if the lesion becomes large enough. Risk factors
for clinically significant HPV infection include immunodeficiency and injury to the epithelium from another cause (eg, trauma, coinfection). HPV is primarily transmitted through genital contact. In children, the most common vector for HPV infection is transmission
from an affected mother during labor and transit through the birth canal, though the presence of these lesions may also raise concern for sexual assault in the correct clinical context. Treatment focuses on maintaining a patent airway, surgical removal of
symptomatic lesions, and preserving voice quality.

Incorrect Answers: A, B, C, and E.

CytomegalQvirus (Choice A), also known as human herpesvirus-5 (HHV-5), can be transmitted through multiple modes, including sexual contact, urine, respiratory droplets, and across the placenta. It can cause a variety of presentations, including mononucleosis in
immunocompetent patients, along with retinitis, infectious esophagitis, and pneumonia in immunocompromised patients. It is not associated with laryngeal papillomatosis.
Herpes simplex virus (Choice B) is typically transmitted through contact with infected skin or mucosa! surfaces, and can present with a variety of symptoms, includi111g genital herpes, herpes labialis, herpetic whitlow, encephalitis, esophagitis, and keratoconjunctivitis.
Though the presence of HSV can result in more aggressive laryngeal papillomatosis, the causative agent is HPV.

HIV (Choice C) is a retrovirus that infects human T cells, resulting in immunosuppres.sion. The patient in this case may have acquired HIV infection through vertical transmission from the mother. HIV does not directly cause laryngeal papillomatosis, rather, an
impaired immune system can result in more severe HPV lesions.

Varicella-zoster virus (Choice E), also known as human herpesvirus-3 (HHV-3), is transmitted via respiratory secretions and direct contact with infected skin lesions. It causes the disease chickenpox in immunocompetent children and can cause encephalitis and
pneumonia in immunosuppressed patients. The classic presentation of chickenpox is a pruritic, asynchronous rash with fluid-filled vesicles that crust over. It is not associated with vocal cord lesions.

Educational Objective: Vocal cord nodules in the setting of immunodeficiency are most likely caused by human papillomavirus (HPV), which infects epithelial cells and is responsible for a variety of skin and mucocutaneous lesions.

r r , fr!i F r ff'
Previous Next Score Report Lab Values Calculator Help Pause
Exam Section 3: Item 4 of 50 National Boa.rd pf M·11dlcal Exi\llllners®
Comprehensive tsas1c science Serr-Assessment

✓ 4. A 16-year-old boy has a 6-month history of type 1 diabetes mellitus and requires daily insulin injections. His blood glucose control has been poor recently, which the physician suspects may be related to the patient's reluctance to self-administer insulin.
Which of the following questions is most appropriate to broach this issue with the patient?

A) "Giving yourself insulin injections can be difficult. What's it like for you?"
B ) "I get the feeling you have not been taking your insulin regularly. Would you like me to arrange some training for you by our nurses?"
C) "It's been my experience that most poorly controlled diabetics have trouble giving themselves insulin. Do you have this problem?"
D) "You are clearly having difficulty with insulin injections. Would you like to arrange for a family member to give you your shots?"
E) "You seem to have missed your shots. How often has this happened?"
Correct Answer: A.

A physician who suspects that a patient is not adhering to treatment should take a nonjudgmental and curious stance and ask open-ended questions to elicit the patient's reasons for not adhering. This physician is nonjudgmental in acknowledging that taking insulin
can be hard and assuming that the patient is experiencing an external barrier to adherence rather than inferring or implying that the patient is at fault. Using nonjudgmental, open-ended questioning can improve therapeutic alliance, help the physician and patient
problem-solve around the nonadherence, and increase the patient's motivation to adhere. Motivational interviewing similarly uses open-ended questions to encourage patients to explore their reasons to change or maintai:n behavior and increases a patient's
readiness to make or maintain a behavioral change such as adhering to insulin.

Incorrect Answers: B , C, D, and E.

Immediately proposing solutions to the problem such as arranging training (Choice B) or help from family members (Choice D) presumes knowledge of the underlying barrier to adherence. Alternatively, open-ended questions serve to identify the specific barriers.
Choices B and D also assume that the patient has not been self-administering insulin and subtly infer that the patient is at fault, which may injure therapeutic alliance.

Calling this patient a "poorly controlled diabetic" (Choice C) may be interpreted by the patient as judgmental. In this statement, the physician is placing the patient as a whole person into a category or stereotype rather than maintaining a narrow focus on the specific
behavior of taking insulin. This type of statement may damage the therapeutic alliance and promote defensiveness rather than an open discussion about the factors contributing to this patient's inability to adhere.

Asking how frequently the patient has missed his shots (Choice E) may be appropriate later in the discussion after the physician has confirmed that the patient has had difficulty with his insulin shots and explores the patient's challenges in adhering. Beginning the
conversation with an open-ended question will avoid an accusatory tone and allow the physician to investigate this patient's specific barriers to adherence.

Educational Objective: Effective physician responses to a patient's suspected nonadherence to treatment include assuming the patient has experienced barriers to adherence and asking open-ended questions to explore these reasons. This approach can improve
therapeutic alliance and allow for targeted problem-solving.

r
Previous
r
Next
,
Score Report

Lab Values
P
Calculator
r
Help
r
Pause
Exam Section 3: Item 5 of SO National Board of Medical Examiners®
Comprehensive tsaslc Science Self-Assessment


'iii

Time

5. The diagram depicts ai cardiac murmur heard in a healthy 92-year-old man. Which of the following age-related processes is responsible for the murmur?

A ) Calcification of the mitral valve


B ) Enlargement of the mitral valve annulus
C) Fibrosis and calcification of the aortic valve
D) Myxomatous degeneration of the aortic valve cusp
E ) Myxomatous degeneration of the mitral valve
Correct Answer: C.

Fibrosis and calcification of the aortic valve is responsible for the murmur depicted, which shows a systolic crescendo-decrescendo murmur of aortic stenosis. Calcification of the aortic valve is a pathologic consequence of mechanical stresses on heart valves, and
results from repetitive microtrauma from the opening and closing of valve leaflets with associated chronic inflammation. Many people will develop some degree of aortic valve stenosis (AS) over time, but structural abnormalities of the valve, such as a bicuspid aortic
valve, alter the biomechanics of valve opening and closing and increase the likelihood of earlier calcification and resultant stenosis. The severity of AS is graded on echocardiography through the measurement of valve area, peak velocity, and mean pressure
gradient across the valve. While many patients may be asymptomatic, those with severe AS may complain of fatigue, shortness of breath, cough, diminished exercise tolerance, angina, or syncope with exertion. Examination findings include a crescendo­
decrescendo systolic murmur best heard at the upper right sternal border, and pulsus parvus et tardus (weak and delayed) may be noted on examination of peripheral pulses. Due to the chronic increased after1oad from a fixed obstruction by the valve, left
ventricular hypertrophy and resultant diastolic dysfunction can occur. Severe symptomatic aortic stenosis is an indication for valve replacement.
Incorrect Answers: A, B, D, and E.

Calcification of the mitral valve (Choice A) can result in mitral stenosis, classically heard as an opening snap, followed by a diastolic rumble that is loudest over the cardiac apex and radiates to the axilla. Mitra! stenosis is commonly seen as a manifestation of
rheumatic heart disease.

Enlargement of the mitral valve annulus (Choice B) causes mitral regurgitation, which is characterized by a holosystolic murmur best heard at the apex that radiates to the axilla with the patient in the left lateral decubitus position. The murmur will get louder with
handgrip or squatting maneuvers.
Myxomatous degeneration of the aortic valve cusp (Choice D) or of the mitral valve (Choice E) can be primary or secondarily related to an underlying connective tissue disorder (eg, Ehlers-Danlos syndrome, Marfan syndrome). Myxomatous degeneration increases
the risk for regurgitant munmurs of the involved valve.

Educational Objective: Calcification and fibrosis of the aortic valve occurs in most people as they age as a result of chronic mechanical stress and can sometimes result in aortic stenosis, which presents with a systolic crescendo-decrescendo murmur heard best at
the right upper sternal border, with radiation to the carotid. Symptoms depend on the severity of aortic valve stenosis (AS) and are graded by echocardiography.

r r ,,. '1!i P r trt


Previous Next Score Report Lab Values Calculator Help Pause
Exam Section 3: Item 6 of 50 National Boa.rd pf M·11dlcal Examiners®
Comprehensive saslc Science Self-Assessment

✓ 6. A 65-year-old man with diabetes mellitus develops a neuropathy of the cranial nerve indicated by the arrow in the normal brain stem. Which of the following is the most likely associated functional deficit?

________________________________________________________
A ) Complete ptosis of the ipsilateral eye
B ) Inability to abduct the iRSilateral....,
eye
C) lpsilateral sensorineural deafness
D) Lack of tear formation in the ipsilateral eye
E ) Paralysis of ipsilateral jaw muscles

Correct Answer: B.

The abducens nerve (cranial nerve VI) arises from the mid-pons and exits the brainstem ventrally at the pontomedullary junction. It ascends steeply through the subarachnoid space across the clivus before proceeding through Dorella's canal to the cavernous
sinus. It exits the cavernous sinus through the superior orbital fissure before reaching its target, the lateral rectus muscle, in the lateral orbit. The most common etiologies of acute abducens nerve palsy in adults include microvascular ischemia (secondary to
diabetes, hypertension, and atherosclerotic cardiovascular disease), idiopathic etiologies, and trauma. The abducens nerve is fragile and vulnerable to injury as it crosses the clivus by any mechanism causing downward traction, such as brain herniation.
Abducens nerve palsy presents with binocular horizontal diplopia and an inability to abduct the ipsilateral eye. Treatment includes systemic workup for vasculopathic risk factors and patching to reduce diplopia. Abducens nerve palsy secondary to microvascular
disease may resolve spontaneously over several months.

Incorrect Answers: A, C, D, and E.

Complete ptosis of the ipsilateral eye (Choice A) can occur as a component of oculomotor nerve (cranial nerve Ill) palsy, or in association with Horner syndrome, myasthenia gravis, trauma, or age-related dehiscence of the aponeurosis of the levator palpebrae
superioris muscle. The abducens nerve is not responsible for innervation of the muscles of the upper eyelid.

lpsilateral sensorineural deafness (Choice C) suggests pathology of the vestibulocochlear nerve (cranial nerve VIII), which may be caused by microvascular ischemia, multiple sclerosis, compression secondary to acoustic neuroma, trauma, or infection.

Lack of tear formation in the ipsilateral eye (Choice D) may be a feature of facial nerve palsy, otherwise known as Bell palsy. The facial nerve (cranial nerve VII) provides parasympathetic innervation to the lacrimal gland. However, in this setting, excessive
1

tearing may aiso occur due to decreased blinking and inefficiency of the lacrimal pump.

Paralysis of ipsilateral jaw muscles (Choice E) may result from injury to the third branch of the trigeminal nerve (cranial nerve V), also known as the mandibular nerve.

Educational Objective: The abducens nerve exits the brainstem ventrally at the pontomedullary junction. The abducens nerve innervates the lateral rectus muscle of the eye. Abducens nerve palsy may result from microvascular ischemia or trauma and presents
with binocular horizontal diplopia and ipsilateral abduction deficits of the affected eye.

r
Previous
r
Next
,
Score Report

Lab Values
P
Calc;:ulator
r-
Help
r
Pause
_ .
Exam Section 3: Item 7 of SO National Board of Medical Examiners®
Comprehensive tsas1c Science Self-Assessment

✓ 7. A 2-year-old boy has had recurrent bacterial infections with markedly increased absolute neutrophil counts in the blood (30,000-150,000/mm3). Studies of neutrophils show:
Test Substance Patient (% Control)
Bacterial killing
Normal opsonized patient serum 80±20
Patient opsonized normal serum 70±15
Phagocytosis
Normal opsonized patient serum 85±20
Patient opsonized normal serum 60±20
Chemotaxis
Normal patient C5a 90±15
Patient patient C5a 10±5

Which of the following is the most likely diagnosis?

A ) Chronic granulomatous disease


B ) Leukocyte adhesion deficiency
C) Myeloperoxidase deficiency
D) NADPH oxidase deficiency
E ) Neutrophil-specific granule deficiency
Correct Answer: B.

Leukocyte adhesion deficiency (LAD) results from a defect in the attachment of leukocytes to the vascular endothelium, which consequently results in the impaired recruitment and migration to sites of extravascular inflammation or infection. LAD type 1 is due to a
dysfunctional LFA-1 integrin (CD18) protein on the leukocyte surface, which does not allow for the normal attachment of neutrophils to the vascular endothelium. It is typically characterized by recurrent bacterial infections, impaired wound healing, a delayed
detachment of the umbilical cord after birth, and complete lack of neutrophils at sites of infections with no pus. LAD type 3 is due to dysfunctional integrin response to chemoattractants (such as C5a, IL-8, leukotriene B4), and presents with a similar clinical
syndrome to LAD type 1. The actions of leukocyte phagocytosis and bacterial killing are not impaired. Laboratory studies in patients with LAD will show increased absolute neutrophil counts in the blood due to failure to marginate. Neutrophils from the patient's
serum will perform similarly to controls with respect to phagocytosis and bacterial killing. In this case, the neutrophil response to C5a is diminished relative to the control, which suggests an impaired chemotactic and adhesive response and a diagnosis of LAD.

Incorrect Answers: A, C, D, and E.

Chronic granulomatous disease (Choice A) results from a defect or deficiency in the NADPH oxidase complex (Choice D). This impairs the production of reactive oxygen species and the respiratory burst in neutrophils, increasing a patient's susceptibility to catalase
positive pathogens. Diagnosis is made by an abnormal dihydrorhodamine test, or a nitroblue tetrazolium reduction test. Decreased bacterial killing compared to control would be expected with opsonized patient serum.

Myeloperoxidase deficiency (Choice C) is an autosomal recessive immune disorder caused by mutations in the MPO gene on chromosome 17. It is associated with fungal infections.

Neutrophil-specific granule deficiency (Choice E) is a rare primary immunodeficiency characterized by recurrent bacterial and fungal infections, pulmonary abscesses, and mastoiditis. It is due to the defective maturation of neutrophils and absent production of
substances such as lactoferrin and defensins. Peripheral blood smear reveals neutrophils with absent granules and bilobed nuclei. Bacterial killing is impaired.

Educational Objective: Leukocyte adhesion deficiency is a group of disorders characterized by impaired leukocyte adhesion to the vascular endothelium in response to infection. Leukocytes retain the ability to phagocytose and eliminate foreign pathogens but are
unable to migrate to sites of infection or inflammation in the extravascular space.

r
Previous
r
Next
,
Score Report
fr!i
Lab Values
P
Calculator
r
Help
trt
Pause
Exam Section 3: Item 8 of 50 National Board of Medlcal Examiners®
Comprehensive 1:1as1c Sctence Self-Assessment

✓ 8. A 36-year-old man comes to the physician's office without an appointment and demands to be seen immediately. He has had a sore throat for 24 hours and explains his problem is more important than the other patients' because he is a vocalist. He is
extremely upset when the receptionist explains he will need to wait for 1 hour to see the physician. He talks with her constantly while he waits. He says, "My wife and I split like 3 months ago. We have a 6-month-old boy, but she was just so into her
pregnancy, she had no time for me. And she doesn't really appreciate my music. I need full support from my lady. Especially when my band's just about to make it big." Which of the following is the most likely personality disorder?

A ) Antisocial
B ) Borderline
C) Compulsive
D) Dependent
E ) Histrionic
F Narcissistic
G) Paranoid
H) Schizoid

Correct Answer: F.

Narcissistic personality disorder, a cluster B disorder (the dramatic or emotional cluster), is characterized by fragile self-esteem and compensatory arrogant, self-aggrandizing behavior to gain admiration, sometimes at others' expense. For example, a patient with
narcissistic personality disorder may berate their physician for making a small mistake, which unconsciously inflates the patient's ego. This patient also demonstrates this entitled attitude, expecting special treatment from both his wife and doctor's office. He expects
the receptionist to be interested in learning about his life, and he likely hopes she will verbalize approval of his grandiose perspective. First-line treatment for narcissistic personality disorder is psychotherapy.

Incorrect Answers: A, B, C, D, E, G, and H.

Antisocial personality disorder (Choice A) is a cluster B personality disorder that arises from a deficit in empathy, resulting in pervasive violations of others' rights, aggression, and a hostile and manipulative attitude toward others. These patients may also be
chronically bored and hence sensation-seeking, leading to a high rate of substance abuse and gambling. Though this patient self-promotes at the expense of others, he does not demonstrate the aggression or violations of basic human rights that would be expected
in antisocial personality disorder.

Borderline personality disorder (Choice B) is a cluster B personality disorder that features an unstable sense of self and tumultuous relationships. Likely due to a combination of genetics and emotional invalidation during childhood and maturation, patients with
borderline personality disorder unconsciously learn to make impulsive and dramatic gestures, including self-harm, to obtain emotional fulfilment from others. This patient fulfils his emotional needs through gaining others' admiration rather than impulsively self­
harming.

Compulsive personality disorder (Choice C), or obsessive-compulsive personality disorder, is a cluster C personality disorder, the anxious or fearful cluster. These patients demonstrate a pattern of inflexible orderliness and perfectionism as well as an intense fear of
making mistakes. The arrogant and self-aggrandizing behavior of this patient would be atypical of obsessive-compulsive personality disorder.

Dependent personality disorder (Choice D), a cluster C personality disorder, presents with an excessive need to be cared for by others that manifests as severe separation anxiety and clinging behavior.

Histrionic personality disorder (Choice E), a cluster B personality disorder, is characterized by theatrical, superficial expressions of emotion that unconsciously serve to garner attention from others to fulfill emotional needs. These patients may dress in a seductive
way for the same purpose. Patients with histrionic personality disorder do not generally demonstrate arrogant, entitled behaviors.

Paranoid personality disorder (Choice G) is a cluster A personality disorder characterized by pervasive mistrust of others, viewing others as possessing malicious intent. These patients tend to blame others for their own shortcomings. This patient instead relies on
others' admiration.

�chizoid personality disorder (Choice H), a cluster A personality disorder, manifests as extreme social detachment and a cold, restricted affect.
Educational Objective: Narcissistic personality disorder is characterized by a fragile self-esteem and compensatory arrogant, self-aggrandizing behavior to gain admiration. In interactions with physicians, these patients may demand special treatment.

r
Previous
r
Next
,
Score Report
fc3
Lab Values
P
Calculator
r
Help
,,.
Pause
Exam Section 3: Item 9 of SO National Boa.rd of Medical Exi!Jlllners®
Comprehensive 1:1as1c Science Serr-Assessment

✓ 9. In biosynthesis of thyroid hormones, iodide is concentrated by the thyroid gland and oxidized by thyroidal peroxidase to iodine. Iodine iodinates amino acid residues within the thyroglobulin molecule. Which of the following amino acids undergoes iodination?

A ) Histidine
B ) Phenylalanine
C) Praline
D) Tryptophan
E ) Tyrosine

Correct Answer: E.

Thyroid hormone is produced in the follicular cells of the thyroid gland. Thyroid-stimulating hormone is produced in the anterior pituitary and signals the production of thyroid hormone b y the thyroid. Tyrosine molecules are assembled into protein-containing chains
known as thyroglobulin. Concurrently, iodine is transported into the follicular cells and is linked to the tyrosine residues in thyroglobulin by thyroid peroxidase, forming thyroxine (T4) or triiodothyronine (T3). The T4 and T3 molecules are then cleaved from
thyroglobulin. These thyroid hormones are released into the bloodstream, where they act upon target tissues in the periphery.

Incorrect Answers: A, B, C, and D.

Histidine (Choice A) is an amino acid that is a precursor to the biologically important molecule histamine. Histamine is released from mast cells and contributes to vascular permeability. It is key in allergic responses. It also functions in the central nervous system to
promote wakefulness and causes release of gastric acid in the stomach.

Phenylalanine (Choice B) is an amino acid that can be converted into multiple neurotransmitters. Inability to metabolize phenylalanine due to genetic defects in the enzyme phenylalanine hydroxylase leads to deposition in central nervous system tissues resulting in
intellectual disability and seizures.

Praline (Choice C) is an amino acid that is important for the formation of collagen. It is essential for maintaining the connective tissue integrity. Vitamin C deficiency prevents hydroxylation of praline, leading to scurvy.

Tryptophan (Choice 0) is an amino acid that cannot be synthesized in humans, making it an essential amino acid. It is a biochemical precursor for serotonin, melatonin, and niacin.

Educational Objective: Thyroid hormone is produced via the iodination of the tyrosine residues within thyroglobulin. Thyroxine has four iodine atoms attached, while triiodothyronine exhibits three attached iodine atoms.

r r , � r-:- r r
Previous Next Score Re_port Lab Values Calculator H!!_IP Pause
Exam Secllon 3: Item 10 of 50 National Board pf Medical Examiners®
Comprehensive saslc Science Serr-Assessment

✓ 10. A 56-year-old man comes to the physician for a routine examination. His stool is positive for occult blood. Colonoscopy shows a pedunculated grow1h that is coarsely lobulated and 1.4 cm in d.iameter. The excised lesion is covered by colonic epithelium
and composed of branching glands lined by tall hyperchromatic cells. Some areas appear mildly dysplastic, but there is no evidence of invasion ot the stalk. Which of the following oncogenes is most likely to be activated in this lesion?

A) erb
B ) fos
C) K-ras
D) p53 (TP53)
E ) Rb
F ) src
Correct Answer: C.

Neoplasms of the colon (benign or malignant) have their molecular pathogenetic roots in the progressive accumulation of mutations that lead to disordered or dysregulated cell growth, loss of control of the cell cycle, evasion of immune detection or destruction,
angiogenesis, and loss of DNA damage repair capability. Colonic neoplasia often begins with the formation of a polyp, a small growth that may be flat, sessile, or pedunculated (as in this case). Histologically, polyps may be adenomatous (neoplastic, glandular,
tubular, tubulovillous, or villous as in this case), serrated (premalignant), hamartomatous (disorganized growth of native tissue), or hyperplastic. Adenomatous polyps typically occur due to chromosomal instability; most precursor mutations involve the APC gene, a
tumor suppressor. When both copies of APC are mutated, increased cellular proliferation may occur. Inherited mutations of APC are common in familial adenomatous polyposis syndrome. A second mutation that follows often involves the K-ras oncogene. K-ras
codes for a signaling protein involved in MAP-kinase and Pl3-K pathways that promote cell growth, cell differentiation, and cell survival. Only a single mutated allele in oncogenes (in contrast to the requirement for two mutated alleles in tumor suppressor genes) is
needed to promote neoplasia. When constitutively active, uncontrolled growth and division can result, reflected grossly as an adenomatous polyp. Additiional mutations in p53 (a tumor suppressor) that follow mutations in K-ras may permit sufficient loss of cell cycle
control resulting in malignant transformation. This pathway (APC, K-ras, p53 mutations generally in sequence) accounts for 80% to 85% of sporadic cases of colorectal carcinoma. The other 15% of cases are due to microsatellite instability from DNA mismatch
repair, including both sporadic mutations and inherited Lynch syndrome.

Incorrect Answers: A, B, D, E, and F.

Erb (Choice A) is a signaling molecule involved in the epidermal growth factor receptor pathway, which involves multiple downstream cascades including Ras-Raf-MAP-kinase, and Pl3-K. This pathway controls cell growth and differentiation an.d is constitutively
active in many cancers including breast, lung, and in some cases, colorectal. It is less frequently involved in the pathogenesis of colorectal cancer.

Fos (Choice B) is an oncogene that promotes cell growth and survival. Constitutive mutations are seen in osteosarcoma, and endometrial and breast carcinoma. It has been identified in mutations a ssociated with colorectal carcinoma but is less frequently involved
in the pathogenesis of colorectal cancer.

p53 (TP53) (Choice D) and Rb (Choice E) are tumc:ir suppressor genes, not oncogenes. Mutations in p53 and Rb are associated with multiple cancers, Li-Fraumeni syndrome, osteosarcoma, and retinoblastoma.

Src (Choice F) is involved in tyrosine kinase pathways related to cell growth, survival, and angiogenesis. Overexpression or constitutive activation has been noted in colorectal, breast, and prostate cancer, but it is not as commonly involved in the pathogenesis of
colorectal cancer as APC, K-ras, and p53.

Educational Objective: Mutations in APC, K-ras, and p53 promote disordered cell growth and division, leading to malignant transformation in colorectal cancer. K-ras is an oncogene promoting cell division and survival, whereas APC and p53 are tumor suppressors.

r r , ta F r r-
Previous Next Score Report Lab Values Calculator Help Pause
Exam Section 3: Item 11 of 50 National Boa.rd pf Mt1dlcal Examiners®
Comprehensive saslc Science Self-Assessment

✓ 11. A 27-year-old man has hepatomegaly and mild jaundice. Serum alkaline phosphatase and y-glutamyltransferase activities are increased. Which of the following properties of these enzymes allows them to be used to detect defects in bile flow?

A ) Present in bile ductules


B ) Present on the basal surface of hepatocytes
C) Released from the gallbladder
D ) Required for bile salt synthesis
E ) Required for bilirubin conjugation
F ) Required for heme catabolism
Correct Answer: A.

Alkaline phosphatase and y-glutamyltransferase are enzymes present in bile ductules. Serum levels are increased in disorders of the biliary tract due to defects in bile flow. An increase in serum alkaline phosphatase, total bilirubin (especially with a conjugated or
direct bilirubin predominance), and y-glutamyltransferase is termed a cholestatic liver injury pattern. Al�aline phosphatases are isoenzymes that catalyze the hydrolysis of organic phosphate esters. They are found on the outer membrane of numerous cell types.
The majority of serum alkaline phosphatase is produced by the liver and bone. The small intestine may produce a small detectable amount. In the liver, alkaline phosphatase is found on the canalicular membrane of hepatocytes. Increased serum alkaline
phosphatase is associated with cholestasis, infiltrative disease, and bone disease. Increased y-glutamyltransferase levels are associated with disorders of the liver, biliary tract, and pancreas, as well as alcohol ingestion.

Incorrect Answers: B, C, D, E, and F.

Present on the basal surface of hepatocytes (Choice B) is incorrect as alkaline phosphatase and y-glutamyltransferase are located on the canalicular membrane (apical surface) of hepatocytes. The basal surface interfaces with the sinusoids.

Bile is the principle substance released from the gallbladder (Choice C). Bile aids in digestion, neutralizes gastric acid, promotes fat absorption, and is the primary means of excreting cholesterol and bilirubin. Alkaline phosphatase and y-glutamyltransferase are
membrane-bound enzymes, which are not excreted into bile.

Required for bile salt synthesis (Choice D) describes the enzyme cholesterol 7a-hydroxylase, which catalyzes the initial step in synthesizing bile salts from cholesterol. Deficiencies in this enzyme increase the risk of cholesterol gallstone formation.

Required for bilirubin conjugation (Choice E) describes the enzyme glucuronyltransferase, which conjugates indirect bilirubin with glucuronic acid to make it soluble in water and suitable for excretion into the bile (direct bilirubin).

Required for heme catabolism (Choice F) describes the enzymes heme oxygenase and biliverdin reductase, which convert heme to biliverdin and reduce biliverdin to unconjugated bilirubin, respectively. Heme oxygenase is expressed by almost all cell types.
Biliverdin reductase is found in cells of the reticuloendothelial system (spleen, liver, lymph nodes).

Educational Objective: Alkaline phosphatase and y-glutamyltransferase are enzymes located on the canalicular membrane of hepatocytes. Increased serum levels of these enzymes are commonly associated with disorders of the biliary tract.

r
Previous
r
Next
,
Score Report

Lab Values
P
Cal�ulator
r-
Help
r
Pause
_ .
Exam Section 3: Item 12 of 50 National Board pf Medical Examiners®
Comprehensive Basic SClence Serr-Assessment

y 12. A 62-year-old man has had petechiae and easy bruising for the past month. He has a 2-year history of alcoholic cirrhosis with portal hypertension. Laboratory studies show:
Hemoglobin 10 g/dL
Hematocrit 30%
Leukocyte count 4000/mm3
Neutrophils 65%
Lymphocytes 30%
Monocytes 4%
Eosinophils 1%
Basophils 0%
Reticulocyte count 8% of red ceUs
Mean corpuscular volume 102 µm3
Platelet count 68,000/mm3
Prothrombin time 16 sec
Partial thromboplastin time
28 sec
(activated)
Serum
Fibrinogen 200 mg/dL (N=200-400 mg/dL)
Fibrin split products <10 (N s 10)

Which of the following is the most likely cause of the pancytopenia?

A �r,inaied iAl�a•Jascular coagulatioA


B ) Hypersplenism
C � I ..
D) Vitamin B1 (thiamine) deficiency
E ) Vitamin C deficiency
Correct Answer: B.

Hypersplenism is defined as the presence of one or more cytopenias in the setting of splenomegaly, which is a common complication of cirrhosis and portal hypertension. Other causes of splenomegaly and potential hypersplenism include lymphoma, splenic
sequestration from sickle cell disease, malaria, mononucleosis, and connective tissue disorders. Splenomegaly is associated with increased splenic activity and removal of cells from the blood with the potential development of decreased cell counts in one or more
cell lines. Pancytopenia, as seen on this patient's laboratory studies, refers to a decrease in all three cell lines (red blood cells, white blood cells, and platelets). Symptoms of thrombocytopenia include abnormal bleeding or bruising, petechiae, ecchymoses, and
mucosa! bleeding (eg, uterine bleeding, epistaxis), along with persistent •oozing• at puncture and operative sites. Anemia may present with fatigue, light-headedness, and dyspnea. Leukopenia increases susceptibility to infection. Treatment for hypersplenism
requires management of the underlying cause (eg, transjugular intrahepatic portosystemic shunt [TIPS] for portal hypertension), supportive therapy with potential transfusions for the cytopenias, and, if severe or refractory, splenectomy and/or liver transplantation.

Incorrect Answers: A. C, D, and E.

Disseminated intravascular coagulation (Choice A) is a systemic disorder characterized by the widespread activation of the clotting cascade resulting in microthrombi formation, thrombocytopenia, and bleeding complications due to consumption of platelets and
clotting factors. It can be provoked by trauma, sepsis, malignancy, pancreatitis, exposure to toxins, and multiple other etiologies. Laboratory studies reveal thrombocytopenia, prolonged prothrombin time (PT) and activated partial thromboplastin time (aPTT),
decreased fibrinogen levels, decreased platelet cournt, and increased O-dimer levels (fibrin split products).

Iron deficiency (Choice C) is a common cause of microcytic anemia. It may result from chronic bleeding or insufficient dietary intake of iron. Iron deficiency does not cause a decrease in leukocyte or platelet cell lines.

Vitamin B1 (thiamine) deficiency (Choice D) results from dietary insufficiency and is associated with chronic alcohol use disorder. T hiamine is an essential cofactor for pyruvate dehydrogenase. Deficiency may result in anemia, Wernicke-Korsakoff syndrome, or
heart failure (beriberi). If severe, pancytopenia may be seen due to the decreased production of cells from the bone marrow. A decreased reticulocyte index would be expected.

Vitamin C deficiency (Choice E) causes the constellation of symptoms known as scurvy, which is characterized by petechiae, perifollicular hemorrhage, bruising, poor wound healing, and corkscrew-shaped hairs. Vitamin C deficiency results from dietary
insufficiency, which is rare in resource-rich settings, and is not associated with pancytopenia.

Educational Objective: Hypersplenism is defined as the presence of one or more cytopenias in the setting of splenomegaly, which is a common complication of cirrhosis and portal hypertension. Laboratory studies will indicate normal or hyperproliferation of bone
marrow precursor cells (a normal or increased reliculocyte index) and normal markers of coagulation cascade activation (normal PT, aPTT, fibrinogen, fibrin split products).

r
Previous
r
Next
,
Score Report

Lab Values
F
Calculator

Help
r
Pause
Exam Section 3: Item 13 of 50 National Board pf Mqdlcal Examiners®
Comprehensive Basic Science Self-Assessment

✓ 13. A47-year-old woman comes to the physician because she wants to "do something about my wrinkles." Physical examination shows fine wrinkles in photodamaged skin on the face. Treatment with tretinoin is started. Collagen synthesis will most likely be
increased in this patient by which of the following mechanisms?

A ) Activation of nuclear gene transcription


B) Decreased cAMP production
G}- DisplaGed vitamlR Afrom Gelil,Jlar stores
c+ IRGreased sebum proouGlioR
E ) Protection of keratinocytes from UVB irradiation
Correct Answer: A.

Tretinoin, a Vitamin A analog, exerts its effects by binding to retinoic acid receptors, which are transcription factors in the nucleus of the fibroblast. All-trans-retinoic acid (ATRA) is another name for tretinoin. This medication is also used systemically for the treatment
of acute promyelocytic leukemia. In the case of tretinoin, binding to its receptor causes activation of the nuclear gene transcription of collagen mRNA. Once the mRNA exits the nucleus, it is transported to the rough endoplasmic reticulum where it undergoes
translation into preprocollagen and post-translational processing. The praline and lysine residues of preprocollagen are hydroxylated, which requires vitamin C as a cofactor, and then glycosylated. They form a triple helix called procollagen which is exocytosed into
the intracellular space. There, each end of procollagen is trimmed creating tropocollagen. Many strands of tropocollagen then crosslink, which requires copper as a cofactor, and form the final collagen fiber. An increase in collagen leads to a plumper dermis, which
is visibly perceived as a more youthful and less wrinkled appearance.

Incorrect Answers: B, C, D, and E.

cAMP is a signaling molecule involved in many intracellular pathways. However, decreased cAMP production (Choice 8) does not play a role in the mechanism by which tretinoin increases collagen production.

While tretinoin and other retinoids are vitamin A derivatives, they do not displace vitamin A from cellular stores (Choice C).

Increased sebum productio,n (Choice D) occurs in response to androgen stimulation of sebaceous glands. Sebum, an oily substance, does not have any effect on collagen production. Sebum decreases the dryness of the skin but does not affect the presence of
wrinkles or photodamage.

Protection of keratinocytes from UVB irradiation (Choice E) will decrease the risk of thymidine dimer formation.-Thymidine dimers are caused by covalent cross-linking between two adjacent thymidine nucleotides. If uncorrected, they can cause misreading during
replication or transcription of DNA. Thymidine dimer formation increases the risk of skin cancer but is not related to collagen production.

Educational Objective: Tretinoin and other retinoids bind transcription factors in the nucleus called retinoic acid receptors. In the case of the dermal fibroblast, binding of these factors increases transcription of collagen mRNA.

r
Previous
r
Next
,-
Score Report

Lab Values
F
Calculator
r
Help
,,.
Pause
Exam Sectlon 3: Item 14 of 50 National Board pf M11dlcal Examiners®
Comprehensive i,aa1c: sc:ien" SelT•A&&eaament

✓ 14. An investigator attempting to develop a vaccine for gonorrhea produces an attenuated organism that he administers to female rats as an inhaled aerosol. Swabs of vaginal secretions show a significant increase in antibodies specific for gonorrheal
antigens. Which of the following mechanisms best accounts for this immune response in the genital tract?

A ) Fecal-oral transfer of attenuated organisms


B ) Synthesis of lgG by circulating lymphocytes exposed to antigens 1in the lung
C) Trafficking of lgA-producing lymphocytes to seed all mucosal sites
D) Transport of attenuated Neisseria gonorrhoeae from the lung to the vaginal mucosa
E ) Transport of circulating lgM across the vaginal epithelium

Correct Answer: C.

Trafficking of lgA-producing lymphocytes to seed all mucosa! sites is the most likely mechanism accounting for the immune response to mucosal exposure to attenuated N. gonorrhoeae. Attenuated organisms are non-viable organisms that still possess the antigens
required to induce the formation of specific antibodies against the pathogen via the actions of the adaptive immune system. Plasma cells, which are terminally differentiated B lymphocytes, produce immunoglobulins in large quantities after antigenic stimulation.
Each B lymphocyte possesses a unique B lymphocyte receptor created during V(D)J rearrangement in the bone marrow that will bind to a specific antigen. Antigens binding to the B lymphocyte receptors activate B lymphocytes to recruit helper T lymphocytes,
which aid in the proliferation and differentiation of B lymphocytes into immunoglobulin-producing plasma cells. Additionally, B lymphocytes present antigens via major histocompatibility complex-II to CD4+ T lymphocytes, which aid in adaptive immunity. Pathogens
that are primarily introduced into mucosal sites are more likely to stimulate the production of lgA immunoglobulins as these are the primary immunoglobulins involved with mucosal immunity. In this study, gonorrhea antigens introduced to the respiratory tract through
an inhaled aerosol stimulate the activation of lgA-producing plasma cells. These lgA-producing plasma cells, along with memory B lymphocytes, are then trafficked to other mucosal sites such as the genital tract, which explains the presence of gonorrheal
antibodies present in vaginal secretions.

Incorrect Answers: A, B, D, and E.

Fecal-oral transfer of attenuated organisms (Choice A) would be unlikely to induce such a robust immune response in this instance. While there are some vaccines that are given orally; including the rotavirus, cholera, and typhoid vaccines, adequate delivery is
hindered by the acidic environment of the stomach. Delivery mechanisms would require that this and other physiologic barriers be accounted for during development.

Synthesis of lgG by circulating lymphocytes exposed to antigens in the lung (Choice 8) could occur but would be unlikely to result in significant mucosal immunity against gonorrhea. lgA is the predominant immunoglobulin class found on mucosal surfaces including
the genitourinary tract.

Transport of attenuated N. gonorrhoeae from the lung to the vaginal mucosa (Choice D) is unlikely to occur. Following inhalation, attenuated bacteria would be unlikely to travel through the systemic circulation as they lack pathogenic capabilities, although they
would be able to stimulate antibody production through interaction with B lymphocytes, helper T lymphocytes, and macrophages in the lung.

Transport of circulating lgM across the vaginal epithelium (Choice E) does not likely account for the immune response in the genital tract While lgM is found on mucosal surfaces, lgA remains the predominant immunoglobulin class in these areas.

Educational Objective: Attenuated organisms can be used in vaccines to induce the production of specific antibodies. Organisms that primarily affect and are introduced to mucosa! surfaces are likely to stimulate the activation of lgA-producing plasma cells and
memory B lymphocytes, which are then trafficked to other mucosal sites such as the genital tract.

r
Previous
r
Next
,
Score Report

Lab Values
P
Calculator
r
Help
trt
Pause
Exam Section 3: Item 15 of 50 National Board pf M-idlcal Examiners®
Comprehensive 1:1as1c science Self-Assessment

✓ NONSMOKERS SMOKERS

Diastolic blood pressure Cholesterol quintile, Diastolic blood pressure Cholesterol quintile,
quintile, (mm Hg) (mg/dl) quintile, (mm Hg) (mg/dl.)

15. The graphs show the combined effect of total serum cholesterol concentration and diastolic blood pressure on age-adjusted coronary artery disease death rates for both smokers and nonsmokers. Two persons, X and Y, have the following data:
Smokes Diastolic Blood Pressure Total Serum Cholesterol
X no 72 mm Hg 180 mg/dl
Y yes 95 mm Hg 250 mg/dl

Compared with X, which of the following is the relative risk of dying from coronary artery disease for Y?

A) 2
B) 6
C) 12
D) 30
E ) 55
Correct Answer: C.

Relative risk (RR) describes the difference in likelihood of the occurrence of a particular disease outcome between two groups of patients with or without a particular exposure. In this example, the combined risk of death from coronary artery disease as evidenced
by diastolic blood pressure and cholesterol is compared between groups of smokers and nonsmokers. Calculations of relative risk are commonly performed in cohort studies. RR is calculated by dividing the fraction of patients with a positive exposure and who
developed disease (a) amongst all patients who were exposed (a +b) by the fraction of patients with a negative exposure and who developed disease (c) amongst all patients who were not exposed (c +d). RR thus equals (a / (a + b)) / (c / (c +d)). In this case, the
data is presented as the fraction of the cohort (per 10,000), meaning that the sums a +b and c +d are already accounted for. The numerator in this case (deaths from coronary artery disease in smokers with diastolic blood pressure of 92+mm Hg and total serum
cholesterol of 245+mg/dl) is 60 deaths per 10,000 persons. The denominator in this case (deaths from coronary artery disease in nonsmokers with diastolic blood pressure of less than 76 mm Hg and total serum cholesterol less than182 mg/dl) is 5 deaths per
10,000 persons. The RR is calculated as follows: RR= (60 deaths) / (5 deaths)= 12. RR values greater than 1.0 indicate an incre ·ased risk of developing disease in association with the exposure, whereas values less than 1.0 indicate a reduced risk of developing
disease, and RR equal to 1.0 indicates that the disease outcome and the exposure are not related.Incorrect Answers: A, B, D, and E.

2 (Choice A) reflects the RR of death from coronary artery disease among smokers with diastolic blood pressure of 92+ mm Hg and total serum cholesterol of 245+ mg/dl (60 deaths) as compared to nonsmokers with diastolic blood pressure of 92+ mm Hg and
total serum cholesterol of 245+mg/dl (30 deaths).

6 (Choice B) reflects the RR of death from coronary artery disease among smokers with diastolic blood pressure of 92+mm Hg and total serum cholesterol of 245+mg/dl (60 deaths) as compared to smokers with diastolic blood pressure of less than 76 mm Hg and
total serum cholesterol of less than 182 mg/dl (10 deaths).

30 (Choice D) and 55 (Choice E) cannot be computed without summing data points to create composite numerators, which is not necessary given the already calculated representation of data as relative to a composite population (eg, per 10,000).

Educational Objective: Relative risk (RR) describes the risk of developing disease in a group with a particular exposure divided by the risk of developing disease in a group without the exposure. RR is calculated as (a I (a + b)) / (c / (c + d)).

r
Previous
r
Next
,,.
Score Report
frj
Lab Values
F
Calculator
r
Help
,,.
Pause
Exam Section 3: Item 16 of 50 National Board of Medical Examiners®
Comprehensive tsas1c Science Self-Assessment

✓ 16. A 36-year-old man who is heterozygous for an LDL-receptor deficiency has a total serum cholesterol concentration of 330 mg/dL. After taking a drug that inhibits cholesterol synthesis, his total serum cholesterol concentration decreases to 200 mg/dL.
Which of the following proteins would be upregulated as a result of drug therapy?

A ) Cholesterol acyltransferase
B ) Hepatic lipase
C) LDL receptor
D) Lipoprotein lipase
E ) Lysosomal cholesterol esterase
Correct Answer: C .

LDL receptors would be upregulated as a result of therapy with a statin medication. Stalins inhibit the enzyme HMG-CoA reductase, which catalyzes the rate limiting step in cholesterol biosynthesis, thereby resulting in reduced intrahepatic levels of cholesterol.
Reduced intrahepatic cholesterol levels lead to increased expression of LDL receptors on the surface of hepatocytes with consequent reduction of circulating levels of LDL. As LDL is implicated in the pathogenesis of atherosclerosis, reducing circulating LDL levels
has a positive impact on cardiovascular health. This patient with LDL-receptor deficiency due to a heterozygous mutation likely has familial hypercholesterolemia. The disorder is caused not only by mutations in the LDL receptor gene, but also by mutations of the
PCSK9 or APOB genes. Defects in these genes lead to the decreased clearance of cholesterol by the liver and increased levels of cholesterol in the serum.

Incorrect Answers: A, B, D, and E.

Cholesterol acyltransferase (Choice A) is an enzyme that converts cholesterol to cholesteryl ester to be incorporated into HDL and LDL particles. Deficiency of this enzyme causes anemia, corneal opacities, and renal disease.

Hepatic lipase (Choice B) is a hepatic enzyme that plays a role in the conversion of intermediate-density lipoprotein (IDL) to LDL. It does not affect LDL receptor expression direcUy.

Lipoprotein lipase (LPL) (Choice D) exists primarily as an extracellular enzyme on the surface of vascular endothelial cells (not present in the liver) and acts to hydrolyze circulating triglycerides into their respective parts, monoacylglycerol and fatty acids, which are
stored in adipose tissue. Its action is not affected by statin medications, but fibrate medications increase the activity of LPL and result in decreased serum triglyceride levels.

Lysosomal cholesterol esterase (Choice E) hydrolyzes cholesterol and sterol esters to release fatty acids in the lysosome. Deficiency leads to the accumulation of mucolipids and mucopolysaccharides and causes a type of lysosomal storage disorder known as
cholesteryl ester storage disease, which results in hepatomegaly and cirrhosis.

Educational Objective: Stalin medications inhibit HMG-CoA reductase, the enzyme that catalyzes the rate-limiting step in cholesterol synthesis. This results in decreased intrahepatic cholesterol levels and consequently leads to increased expression of LDL
receptors on hepatocytes, which further reduces circulating levels of LDL.

r r , fr!i P r trt
Previous Next Score Report Lab Values Calculator Help Pause
Exam Section 3: Item 17 of 50 National Board pf Mqdlcal Examiners®
Comprehensive Basic Science Self-Assessment

y 17. During a genome project, an experimental animal is found to have a DNA sequence with si'milarities to a growth hormone receptor. Which of the following peptide segments shown is the transmembrane segment?

A
Ala LeuHisAsp Asn T yr Lys P ro Glu Phe TyrAsn Asp Asp SerTrp Val Glu Phe Glu

�ffiG�Glu�r�GluWrGlu�uA�H•G��sSer�uG�A��s�
B
Asp Asp Ser Gly Arg Thr Ser CysGly Cys Glu Ala Cys Thr Ser�uCysAla GluLeu

Lys His Ala Ser LysG�Ser Thr Arg HisThrG�Gin Ala Asn Arg Arg Arg Lys His �u

Asp Tyr Asn Asp Asp G lu lie Asp lie Asp Asp Thr Asp Glu ThrGlu Glu Ser Asp Thr

Leu Val �u lie Phe G� Met lie Gly Val Val lie Val Val LeuLeuVal Leu Leu lie Phe

Gly Asp Glu Lys Leu Ala �u �u �u Asp G In P ro G luSer �uThr Ala Ala Ala Gly

Lye SerG��uAla GluGin Arg LeuSer CysGluThrSerlie Pro LysAsp TyrThrSer

lie Phe �uTyr

A)
B)
C)
D)
E)
F)
Correct Answer: E.

Amino acids contain an amino group, carboxy group, and side chain that may be nonpolar, polar, acidic, or basic. Nonpolar amino acids include glycine, alanine, valine, leucine, isoleucine, methionine, phenylalanine, proline, and tryptophan. Polar amino acids
include serine, threonine, asparagine, cysteine, tyrosine, and glutamine. These amino acids have a polar side chain without a net charge. Amino acids with basic side chains include lysine, arginine, and histidine. Amino acids with an acidic side chain include
aspartate and glutamate. Transmembrane proteins such as peptide hormone receptors (eg, growth hormone receptor) have hydrophilic extracellular and intracellular domains, and hydrophobic transmembrane domains. In tum, the structure of transmembrane
domains is composed of multiple nonpolar amino acids. At times, these domains may fold to form alpha helices. In the peptide sequence shown, sequence E consists of primarily nonpolar amino acids, which would demonstrate affinity with the hydrophobic
phospholipid bilayer, likely comprising the transmembrane domain.

Incorrect Answers: A, B, C, and D.

Sequences A through D (Choices A, B, C, and D) each include a mixture of polar, acidic, and basic amino acids in addition to occasional (but few) nonpolar amino acids. Each of these domains would have strong hydrophilic or cross-linking characteristics and would
demonstrate considerably less affinity for the hydrophobic phospholipid bilayer. These domains are more likely extracellular or cytosolic.

Educational Objective: Transmembrane proteins such as peptide hormone receptors (eg, growth hormone receptor) have hydrophilic extracellular and intracellular domains, and hydrophobic transmembrane domains. In tum, the structure of transmembrane
domains in humans is composed of multiple nonpolar amino acids that have affinity with the hydrophobic, nonpolar phospholipid bilayer.

r
Previous
r
Next
,
Score Report

Lab Values
F
Calculator
r
Help
,,.
Pause
Exam Secllon 3: Item 18 of 50 Nallonal Board pf Medical Examiners®
Comprehensive Basic Science Serr-Assessment

✓ 18. A 43-year-old woman comes to the physician because of a 6-month history of altered consciousness. During the interview, she stops talking in mid-sentence, turns her head to the right, and extends and stiffens her right upper extremity. She has a blank
look and does not respond to any questions. She then has some repetitive lip smacking and picking movements of the hands. The episode lasts approximately 30 seconds. She slowly returns to her normal state during the next 4 to 5 minutes. The most
likely diagnosis is which of the following types of seizures?

A ) Absence
B ) Complex partial
C) Generalized tonic-clonic
D) Simple partial motor
E ) Simple partial sensory
Correct Answer: B.

Complex partial seizures are also called focal onset impaired awareness seizures. As this name suggests, complex partial seizures affect a specific (focal) region of the brain and therefore produce motor or sensory symptoms that are localized to one or a few areas
of the body. Oral and manual automatisms, as in this patient, are suggestive of a temporal lobe origin. Upper limb dystonia as in this patient is also common in temporal lobe seizures. Patients may experience visual, auditory, or sensory auras preceding the seizure.
This patient was unresponsive during the seizure (impaired awareness) and confused for a few minutes postictally, which is typical for complex partial seizures (as opposed to intact consciousness during simple seizures and more prolonged postictal confusion after
generalized seizures). Complex partial seizures demonstrate localized rhythmic activity with generalized slowing on electroencephalogram (EEG). Treatment requires anti-epileptic medication.

Incorrect Answers: A, C, D, and E.

Absence seizures (Choice A) are characterized by a transient blank stare and 3-Hz spike-and-wave complexes on EEG without postictal confusion. This patient had postictal confusion as well as motor symptoms, making absence seizures an unlikely explanation.

Generalized tonic-clonic seizures (Choice C) are characterized by a sudden loss of consciousness followed by a stiffening all of the muscles of the arms, legs, chest, and back (tonic phase), progressing to jerking or twitching of all of the muscles (clonic phase).
After the seizure, postictal confusion is common. This patient had focal rather than generalized motor symptoms.

Simple partial motor seizures (Choice D) can present with similar motor symptoms localized to a particular area of the body as in this patient. However, impaired consciousness and postictal confusion would be inconsistent with a simple partial seizure (sensory or
motor).

Simple partial sensory seizures (Choice E) typically present with sensory symptoms (eg, numbness, paresthesias, pain) in a focal area of the body. Impaired consciousness and postictal confusion would be inconsistent with a simple partial seizure (sensory or
motor).

Educational Objective: Complex partial seizures present with motor or sensory symptoms localized to a particular area of the body combined with impaired consciousness and postictal confusion.

r
Prev1ous
r
Next
,
Score Report
� P
Lab Values Calculator
r
Help
r-
Pause
_
Exam Section 3: Item 19 of 50 National Boa.rd pf M11dlcal Examiners®
Comprehensive t1a&1c science serr-A1111eument

y 19. A 50-year-old woman from Egypt comes to the physician because of painless blood in her urine for 2 weeks. Urinalysis is within the reference range. Urine cytologic findings show malignant squamous epithelial cells. Which of the following is the strongest
predisposing ris ,k factor for this type of tumor?

A) Chronic pyelonephritis
B ) Cigarette smoking
G) Oiabetic nephropathy
C�El«:e&sive consumption of caffeine
Ii) bong lei:m use of phenasetin
F ) Schistosoma haematobium infection
Correct Answer: F.

Infection with Schistosoma haematobium, a trematode, is a risk factor for the development of squamous cell carcinoma of the bladder. In the United States, urothelial (transitional) cell carcinoma of the bladder is a more common variant, accounting for 90% of new
bladder cancer diagnoses, but in Africa and the Middle East, where Schistosoma haematobium is endemic, squamous cell carcinoma predominates. The organism causes chronic inflammation in the bladder wall, which eventually induces atypia followed by
neoplasia. Regardless of the subtype, bladder cancer classically presents with painless hematuria, which may be grossly visible or microscopic, and occasionally dysuria or urinary frequency. Among all patients with painless gross hematuria, the incidence of
bladder cancer is 10% to 20%. If the disease becomes advanced or metastatic, systemic signs such as fatigue, weight loss, and anorexia may be present. Urine cytology demonstrating malignant cells is highly specific for bladder cancer, though this finding is not
always present. Cystoscopy with biopsy is the gold standard for the diagnosis of bladder cancer.

Incorrect Answers: A, B, C, 0, and E.

Chronic kidney disease has many etiologies but may be caused by chronic pyelonephritis (Choice A), diabetic nephropathy (Choice C), or analgesic nephropathy. Chronic kidney disease affects the function of the renal glomeruli and nephrons rather than the lower
urinary tract. It is not a risk factor for squamous cell carcinoma of the bladder, though may increase the risk of renal cell carcinoma.

Cigarette smoking (Choice 8) is a strong risk factor for transitional cell carcinoma of the urinary tract. Carcinogens are excreted in the urine and cause exposure to the mucosa! surfaces of the upper and lower urinary tract. Cigarette smoking is also a risk factor for
squamous cell carcinoma of the urinary tract, but in a patient from an area where Schistosoma haematobium is endemic, this is more likely to be the strongest predisposing factor.

Excessive consumption of caffeine (Choice 0) has not been proven to be a risk factor for human cancers. In studies of the effect of caffeine consumption on bladder cancer, there was a slight increase in the incidence of bladder cancer in caffeine drinkers, but it was
not dose dependent, calling into question whether the relationship was causal.

Phenacetin (Choice E) exposure is a key risk factor predisposing to the development of transitional cell carcinoma of the urinary tract, along with other factors such as smoking, aniline dyes, and cyclophosphamide. Transitional cell carcinoma is the most common
malignancy of the urinary tract, although this patient demonstrates cytologic findings concerning for squamous cell carcinoma.

Educational Objective: Infection with Schistosoma haematobium, which is endemic to Africa and the Middle East, is a risk factor for squamous cell carcinoma of the bladder. Patients with painless hematuria should be evaluated for bladder cancer with urinalysis,
cytology, and cystoscopy.

r
Previous
r
Next

Score Report
fr!i
Lab Values
P
Calculator

Help
tr'
Pause
Exam Section 3: Item 20 of 50 National Board pf M11dlcal Examiners®
Comprehensive 8aslc Science Serr-Assessment

✓ 20. A 45-year-old woman with chronic myelogenous leukemia comes to the physician for a follow-up examination. Physical examination shows pallor and bleeding gums. Laboratory studies show a leukocyte count of 90,000/mm 3 (60% blasts) and a platelet
count of 27,000/mm 3. A drug regimen of mitoxantrone, vinblastine, and thioguanine is initiated, with no improvement. During the last cycle of scheduled treatment, a sample of the blast cells is harvested, and the intracellular concentrations of mitoxantrone
and vinblastine in these blast cells are found to be undetectable. An increase in which of the following mechanisms best explains this observation?

A ) Drug efflux pump P-glycoprotein activity in leukemic cells


B ) Drug metabolism by the hepatic CYP450 system
C) Drug metabolism by novel heme enzymes in leukemic blast cells
D) Production of sphingomyelin by the leukemic cells
E) Renal excretion of the drugs
F ) Thiopurine methyltransferase activity in the leukemic cells
Correct Answer: A.
Drug efflux pump P-glycoprotein activity in leukemic cells accounts for the undetectable levels of mitoxantrone and vinblastine. Chronic myelogenous leukemia (CML) is defined by the presence of the Philadelphia chromosome, a translocation between
chromosomes 9 and 22, which causes constitutive activation of the ABL1 kinase. Patients demonstrate significant leukocytosis, with levels often exceeding 100,000 cells/mm3. Basophilia and eosinophilia are often present. Treatment is typically with tyrosine kinase
inhibitors (TKls) imatinib or dasatinib, but CML blast crisis may require additional chemotherapeutic agents. Leukemic cell resistance to chemotherapy can occur through a variety of mechanisms, but one common mechanism is the expression of transporter proteins
that pump drugs out of the cells. Several examples include the ABC and MOR class of efflux pumps, which utilize ATP to shuttle drugs out of the cell against a concentration gradient. The P-glycoprotein efflux pump belongs to the ABC class of transporters. Both
vinblastine and mitoxantrone must be intracellular to exert antineoplastic effects, so efflux of these proteins results in intracellular concentration below the levels required to exert their chemotherapeutic effects.
Incorrect Answers: B, C, D, E, and F.
Drug metabolism by the hepatic CYP450 system (Choice B) does not account for the lack of vinblastine or mitoxantrone within leukemic cells. Vinblastine is metabolized by CYP2D6 and CYP3A4 while mitoxantrone is metabolized by BCRP/ABCG2.
Drug metabolism by novel heme enzymes in leukemic blast cells (Choice C) does not explain reduced intracellular drug levels. Heme oxygenase-1 is an inducible protein in CML cells that degrades heme and confers resistance to TKls. This is a potential
therapeutic target in CML.
Production of sphingomyelin by the leukemic cells (Choice D) is accomplished by sphingomyelin synthase, which converts ceramide to diacylglycerol (DAG), a mitogenic compound that favors cellular proliferation. This pathway is a potential target for treatment of
CML but does not account for low intracellular levels of vinblastine or mitoxantrone.
Renal excretion of the drugs (Choice E) is not the correct answer. While a small amount of mitoxantrone is renally excreted, vinblastine is primarily excreted in bile and stool.
Thiopurine methyltransferase (TMPT) activity in the leukemic cells (Choice F) does not account for reduced intracellular drug levels. TPMT metabolizes a class of medications called thiopurines. While this class of medication is used in some pediatric ALL or AML
regimens, neither mitoxantrone nor vinblastine belong to this class of medication.
Educational Objective: The active efflux of chemotherapy agents out of the cell is one mechanism by which leukemic cells develop resistance to specific drugs. This manifests as low or absent intracellular levels of these chemotherapy agents despite adequate
dosing.

r
Previous
r
Next
,
Score Report

Lab Values
F
Calculator
r
Help
,,..
Pause
Exam Secllon 3: Item 21 of 50 National Board pf M41dlcal Exftllllners®
Comprehensive Da&lt science Serr-Assessment

✓ 21. An 8-year-old girl is brought to the physician by her parents because of abdominal pain for 3 days. She is at the 90th percentile for height and the 80th percentile for weight. Breast, pubic hair, and axillary hair development is Tanner stage 3. Pelvic
examination shows a firm, smooth, right ovarian mass. Ultrasonography of the abdomen confirms the presence of an 8-cm-diameter ovarian mass. Which of the following types o,f ovarian lesions is most likely in this patient?

A ) Cystadenocarcinoma
B ) Dermoid cyst
C) Embryonal rhabdomyosarcoma
D ) F ibroma
E ) Granulosa cell tumor
F) Krukenberg tumor
G) Papillary carcinoma
Correct Answer: E.

Granulosa cell tumors are a type of malignant sex-cord stromal tumor. They generally occur in women in their sixth decade of life, though can occur at any age, including during childhood. Their presentation is marked by the effect of their functional production of
hormones (eg, estro,gen). Thus, it may present with precocious puberty, as in this patient, or vaginal bleeding in younger or premenstrual women. Precocious puberty is suspected whem gins younger than eight years old or boys younger than nine years old develop
secondary sexual characteristics. Tanner stage 3 characteristics, including thickening of pubic and axillary hair and breast enlargement, are typically seen in females ages 11 to 13. The presence of these findings at a significantly earlier age suggests precocious
puberty and warrants evaluation. Granulosa cell tumors are typically indolent and may not be detected until large or advanced. On histology, granulosa cell tumors demonstrate Call-Exner bodies, which are granulosa cells arranged around eosinophilic fluid,
resembling ovarian follicles. Treatment is through surgical excision, and if the tumor is early stage, prognosis is generally favorable. In the postmenopausal patient, granulosa cell tumors often present with postmenopausal vaginal bleedin91, which should prompt
investigation.

Incorrect Answers: A, B, C, D, F, and G.

Cystadenocarcinoma (Choice A) is the most common ovarian malignant neoplasm. They are generally seen in women of advanced age and those having a genetic predisposition to malignancy (eg, BRCA, hereditary nonpolyposis colorectal malignancy).

Dermoid cyst (Choice B), also known as a mature cystic teratoma, is a benign tumor of the ovary. They are common germ-cell tumors often seen in women age 20 to 40 years. They contain tissue derived from all three germ layers and as a result, often contain
differentiated structures such as hair, bone, or teeth. They may be large, and cause pain due to rapid enlargement or associated ovarian torsion.
Sarcoma botryoides is a variant of embryonal rhabdomyosarcoma (Choice C) and presents in young gins with a grape-like, clear, polypoid mass protruding from the vagina. It is not a cause of precocious puberty.

Ovarian fibroma (Choice D) may present as a palpable adnexal mass. However, this entity does not secrete estrogen and is unlikely to lead to precocious puberty.

Krukenberg tumor (Choice F) is an ovarian tumor that has metastasized from another site, classically gastric adenocarcinoma. Histopathologic evaluation shows ample mucin-secreting cells. As many are derived from gastric mucosa, they are not a hormonally
active tumor and do not lead to symptoms of increased estrogen.

Papillary carcinoma (Choice G) is a histologic variant of breast cancer in which tumor cells organize into frond-like structures with fibrovascular cores that protrude into the ductal lumen. While breast cancer is respondent to estrogen, it does not secrete it and would
not be expected to cause precocious puberty. Papillary carcinoma is generally not a histologic pattern seen in sex-cord stromal tumors.

Educational Objective: Granulosa cell tumors are a malignant sex-cord stromal tumor in women that produce female sex hormones. In children and young women, they often result in precocious puberty or premenstrual vaginal bleeding.

r
Previous
,.
Next
,,.
Score Report
@
Lab Values
P
Calculator

Help
1ft'
Pause
Exam Secllon 3: Item 22 of 50 Nallonal Board pf Medical Examiners®
Comprehensive Basic Science Serr-Assessment

✓ 22. A 22-year-old woman of Irish descent is brought to the emergency department because of a 1-hour history of severe pain and coldness of her left leg. She has an IQ of 80, and she dropped out of high school in the tenth grade. Her parents and two older
siblings have normal intelligence and have no history of these symptoms. Ophthalmologic examination shows a partially dislocated lens in the right eye. Physical examination shows mottling and loss of pulses in the left lower extremity and foot.
Arteriography of the left lower extremity shows thrombosis of the femoral artery. This patient most likely has a metabolic disorder involving which of the following amino acids?

A ) Glycine
B ) Homocyst(e)ine
C) Leucine
D) Phenylalanine
E ) Tyrosine
Correct Answer: B.

Homocystinuria is an autosomal recessive disorder that may vary in etiology and pathophysiology but resul�s in excess homocysteine. Homocysteine is normally metabolized by cystathionine synthase to become cystathionine or methionine synthase to become
methionine. Cystathionine synthase deficiency, methionine synthase deficiency, or abnormal affinity of cystathionine synthase for pyridoxine may all cause excess homocysteine accumulation. Homocystinuria is characterized by increased levels of homocysteine in
the urine, intellectual disability, lens subluxation or dislocation (typically downward), osteoporosis, marfanoid habitus, thrombosis, and atherosclerosis that increases the risk of peripheral arterial disease, ischemic stroke, and coronary artery disease.

Incorrect Answers: A, C, D, and E.

Glycine (Choice A) accumulation can result from the autosomal recessive disorder glycine encephalopathy and is characterized by seizures, intellectual disability, lethargy, and hypotonia. Thrombosis and lens subluxation are not characteristic of glycine
encephalopathy.

Maple syrup urine disease is caused by the abnormal degradation of isoleucine, leucine, and valine (Choice C) and is characterized by severe neurologic defects, intellectual disability, vomiting, and poor feeding. The urine characteristically has an odor similar to
maple syrup. It is not associated with thrombosis or atherosclerosis.

Phenylketonuria is an autosomal recessive disorder that occurs secondary to a deficiency in phenylalanine hydroxylase, which results in an excess of phenylalanine (Choice D). It can also present with intellectual disability, but is additionally characterized by
seizures, eczema, and a musty body odor.

Alkaptonuria is an autosomal recessive congenital deficiency that prevents the degradation of tyrosine (Choice E) to fumarate via homogentisate oxidase. It is characterized by a blue-black discoloration of connective tissue, sclerae, and urine.

Educational Objective: Homocystinuria is an autosomal recessive disorder that results in excess homocysteine and is characterized by increased levels of homocysteine in urine, intellectual disability, lens subluxation or dislocation, osteoporosis, marfanoid habitus,
thrombosis, and atherosclerosis that increases the risk of peripheral arterial disease, ischemic stroke, and coronary artery disease.

r
Prev1ous
r
Next
,
Score Report
� P
Lab Values Calculator
r
Help
r-
Pause
_
Exam Section 3: Item 23 of 50 National Board of Medical Examiners®
Comprehensive saslc Science Serr-Assessment

✓ 23. A 68-year-old man comes to the physician because of a 1-month history of not being able to sustain an erection for sexual intercourse, although he has no difficulty during masturbation. His wife of 40 years died 2 years ago after a prolonged illness. He
began dating recently. He has a multinodular goiter but takes no medications. His serum thyroid-stimulating hormone concentration is 4.0 µU/ml, and serum testosterone concentration is within the reference range. He does not have any symptoms of
major depressive disorder. Which of the following pairs of additional findings is most likely to be found on history taking?

Libido Nocturnal Erections


A) Decreased decreased
B) Decreased normal
C) Normal decreased
D) Normal normal
Correct Answer: D.

Male erectile disorder, or erectile dysfunction, is a common condition, especially in older males. Erectile dysfunction can be caused by vascular, neurologic, hormonal, and/or psychological dysfunction. Physicians should ask about the patient's erectile function
during masturbation and while sleeping; if the patient continues to have erections while masturbating or sleeping (nocturnal), the erectile dysfunction is more likely due to a psychogenic etiology rather than a vascular, neurologic, or hormonal etiology. This patient
had likely adapted psychologically to having sexual intercourse with his late wife such that sexual dysfunction with new partners may be related to adapting to the new sexual partner or to performance anxiety. In the absence of depression, low testosterone, or
thyroid dysfunction, patients are likely to possess a normal libido. The treatment depends on the cause. Patients with a psychological etiology respond best to psychotherapy such as cognitive-behavioral therapy.

Incorrect Answers: A, B, and C.

Libido and nocturnal erections can both be decreased (Choice A) in patients with hormonal causes of erectile dysfunction. Hypogonadism and hypothyroidism are common culprits.

Decreased libido with normal nocturnal erections (Choice B) can result from multiple problems: medications (eg, selective serotonin reuptake inhibitors, antiandrogens), alcoholism, recreational drugs, depression, relationship stress, fatigue, and systemic illness.
This patient does not possess these risk factors for low libido.

Decreased nocturnal erections with normal libido (Choice C) would likely be caused by neurological or cardiovascular problems due to nerve damage or endothelial dysfunction respectively. Neurological causes include stroke, trauma, and neuropathy (eg, from
longstanding diabetes). This patient continues to experience nocturnal erections, indicating that his erectile neurovascular physiology is functioning.

Educational Objective: Patients with erectile dysfunction related to psychological factors will continue to experience erections during masturbation and at night. In the absence of depression or relationship stress, libido is likely to also be normal.

r
Previous
r
Next
,,.
Score Report

Lab Values
P
Calculator
r-
Help
r
Pause
Exam Sectlon 3: Item 24 of 50 National Board pf M11dlcal Examiners®
Comprehensive i,aa1c: sc:ien" SelT•A&&eaament

✓ 24. Following blockage of a cerebral artery, the neurons that die from hypoxia release a neurotransmitter that promotes excessive calcium influx, lysis, and cell death in nearby undamaged neurons. Which of the following substances is the primary cause of
this cascade of excitotoxicity?

A ) Acetylcholine
B ) y-Aminobutyric acid
C) Glutamate
D ) Glycine
E ) Norepinephrine

Correct Answer: C.

Excitotoxicity describes the phenomenon in which neurons endure additional damage or cell death beyond an initial inciting insult such as a traumatic brain injury, stroke, spinal cord injury, neurodegenerative disease, or withdrawal from y-aminobutyric acid (GABA)
receptor agonists (eg, alcohol, benzodiazepines). In response to damage to the central nervous system (CNS), glutamate may be released rapidly into the synaptic clefts. When such concentrations persist at high levels, neuronal apoptosis ensues. Unchecked
excitatory stimulus from glutamate results in excessive stimulation of the neuron, frequent firing of action potentials, and calcium i'nflux into the cell with resultant activation of phospholipases and proteases. Calcium influx is believed to play a key role in neuronal
apoptosis through the activation of intracellular enzymes including calpain and through increased mitochondrial membrane permeability. Such dysregulation can lead to activation of caspases leading to apoptosis. Damage to subcellular organelles, membrane lipids,
and DNA can also trigger the intrinsic apoptotic cascade. Excitotoxicity can occur from excess glutamate, as in this example, or from N-methyl-D-asparlic acid (NMDA). Excitotoxicity can be further exacerbated by concomitant dysregulation of glucose;
hypoglycemia in the setting of glutamate excitotoxicity can further damage 1he neurons and worsen the extent of neuronal loss.

Incorrect Answers: A, B, D, and E.

Acetylcholine (Choic:8 A) is a neurotransmitter that has mixed excitatory and inhibitory activities. Active in the neuromuscular junction, autonomic nervous system, and brain, acetylcholine is an excitatory neurotransmitter in the brain, where it mediates attention,
arousal, memory, and motivation. Acetylcholine deficits underlie dementia..Acetylcholine is not involved in excitotoxicity.

y-Aminobutyric acid (GABA) (Choice 8) is the primary inhibitory neurotransmitter in the CNS. GABA binds to both ionotropic and metabotropic GABA receptor subtypes and hyperpolarizes neurons, decreasing the frequency of action potentials.

Glycine (Choice D) is an amino acid and a primarily inhibitory neurotransmitter in the CNS that binds to ionotropic receptors to enable chloride influx, which hyperpolarizes neurons. As an inhibitory neurotransmitter, glycine is not involved in excitotoxicity.

Norepinephrine (Choice E) is a catecholamine hormone that functions as an excitatory neurotransmitter in the CNS. In the brain, norepinephrine mediates alertness and attention and reduces neuroinflammation. Norepinephrine is not involved in excitotoxicity.

Educational Objective: Glutamate is associated with excitotoxicity. lschemic, traumatic, or substance-induced insults to the central nervous system can cause excitotoxicity, leading to neuronal loss via apoptosis.

r
Previous
r
Next
,
Score Report

Lab Values
P
Calculator
r
Help
trt
Pause
Exam Section 3: Item 25 of 50 National Board pf Medical Exi!Jlllners®
Comprehensive Basic Science Serr-Assessment

)( 25. While digging in the forest, an archaeologist finds the skeleton of a child in a shallow grave. Examination of the skull shows frontal bossing; there is delayed eruption of the teeth and abnormal enamel formation of those teeth that are erupted. The spine and
arm bones :show no abnormalities. The tibias show marked bowing. Which of the following was the most likely cause of this child's skeletal defects?

A ) Osteitis deformans (Paget disease)


• B) Osteogenesis imperfecta
C ) Rheumatic fever
D) Tuberculosis
E) Vitamin B1 (thiamine) deficiency
F ) Vitamin D deficiency
Correct Answer: F.
Childhood vitamin D deficiency (Rickets) presents with multiple skeletal deformities including frontal bossing, nodules at the costochondral junctions of the anterior chest, and delayed closure of the growth plates due to impaired bone mineralization. Children will
demonstrate varus bowing of the tibia as well as bowing of the femur. Children will have hypoplastic teeth and multiple caries as mineralization of dentin is impaired. Vitamin D deficiency in children commonly comes from low exposure to UV radiation and low
dietary vitamin D intake (particularly from low vitamin D in breast milk). Fat malabsorption syndromes such as celiac disease or cystic fibrosis can impair absorption of vitamin D in the gastrointestinal tract.
Incorrect Answers: A, B, C, D, and E.
Osteitis deformans (Choice A), also known as Paget disease, is a disorder of osteoclast and osteoblast activity marked by the formation of disorganized, woven bone instead of lamellar bone. This disease may present with pathologic fractures, lytic or sclerotic
lesions on x-rays, and cranial nerve dysfunction from compression within the neural foramina of the skull base.
Osteogenesis imperfecta (Choice B) is caused by mutations in collagen genes, commonly COL 1A 1 and COL 1A2. These mutation·s cause decreased collagen synthesis and impaired bone matrix development leading to multiple fractures throughout life. A classic
examination finding is thin, translucent sclera that appear blue due to the reflection of light from underlying choroidal veins.
Rheumatic fever (Choice C) is a late complication of untreated Group A Streptococcus infection characterized by carditis, subcutaneous nodules, erythema marginatum, Sydenham chorea, and migratory polyarthritis. The etiology is related to immune cross­
reactivity due to molecular homology between streptococcal M protein and human cardiac myosin proteins .. It does not involve skeletal pathology.
Tuberculosis (Choice D) is a slow-growing organism transmitted via respiratory secretions. It presents with primary, latent, and reactivation patterns. In primary disease, patients present with subacute fevers, weight loss, night sweats, cough, and malaise. It can
cause osteomyelitis; however, ii does not cause symmetric tibial bowing.
Vitamin 81 (thiamine) deficiency (Choice E) leads to impaired glycolysis with resultant decreased production of adenosine triphosphate and damage to tissues with high energy requirements (eg, heart, brain). The clinical features of thiamine deficiency are variable.
Dry beriberi presents as peripheral neuropathy with mental status change. Wet beriberi refers to the development of a dilated cardiomyopathy leading to heart failure. Wemicke encephalopathy classically presents as confusion, ataxia, and ophthalmoplegia.
Educational Objective: Childhood vitamin D deficiency, referred to as rickets, leads to decreased mineralization of the bones often presenting with resultant bowing of the long bones.

r
Previous
r
Next
,
Score Report
fr§ r-:-
Lab Values Calculator
r
Help
r
Pause
Exam Section 3: Item 26 of 50 National Boa.rd of Medical Examiners®
Comprehensive 1:Jas1c Science Serr-Assessment

✓ 26. A previously healthy 42-year-old Asian woman is brought to the emergency department because of a 24-hour history of nausea, vomiting, and progressive lethargy. She has smoked 1 pack of cigarettes daily for 25 years and drinks four glasses of wine
daily. She uses high-dose acetaminophen daily for headaches. She does not use illicit drugs. She is 155 cm (5 ft 1 in) tall and weighs 50 kg (110 lb); BMI is 2 1 kg/m2. She is responsive to painful stimuli. Initial laboratory studies show increased hepatic
aminotransferase. Which of the following effects of alcohol most likely contributed to this patient's condition?

A}- Deer-eased generation ofA1acetyl p benroqiJinoneimine


B ) Increased glucuronidation
G}- Increased hepatic glulathione stores
D) Increased sulfation
E ) Induction of cytochrome P450

Correct Answer: E.

This patient presents with acetaminophen overdose in the setting of chronic alcohol consumption. When taken at therapeutic doses, acetaminophen is safely metabolized through phase II conjugations, including glucuronidation and sulfation. At high doses,
saturation of phase II metabolic pathways leads to excess acetaminophen being metabolized by cytochrome P450-mediated reactions to N-acetyl-p-benzoquinoneimine (NAPQI), which has strong oxidizing properties and is directly hepatotoxic. Chronic ethanol
ingestion leads to the induction of cytochrome P450, Chronic alcoholics are therefore at an increased risk of hepatotoxicity when consuming high doses of acetaminophen due to the increased production of NAPQI. There is no definite increased risk of
hepatotoxicity in association with either acute alcohol intoxication or in patients with chronic alcohol consumption who take normal therapeutic doses of acetaminophen.

Incorrect Answers: A, B, C, and D.

Decreased generation of N-acetyl-p-benzoquinoneimine (NAPQI) (Choice A) would not be observed in the setting of acetaminophen toxicity. NAPQI is a toxic metabolic byproduct of acetaminophen. Through the induction of cytochrome P450, chronic alcohol
ingestion leads to an increased generation of NAPQI.

Increased glucuronidation and sulfation (Choices B and D) are observed in the setting of acute acetaminophen overdose due to tlhe large quantity of acetaminophen ingested, but this is not a direct effect of alcohol. Glucuronidation and sulfation reactions convert
acetaminophen into harmless metabolites. Once glucuronidation and sulfation pathways are overwhelmed, cytochrome P450 converts excess acetaminophen into the toxic metabolite NAPQI. Ethanol does not directly lead to the increased glucuronidation or
sulfation of acetaminophen.

Increased hepatic glutathione stores (Choice C) are not observed in this context. Glutathione helps to safely metabolize acetaminophen; increased hepatic stores of glutathione would therefore help prevent hepatotoxicity. Chronic alcohol ingestion does not lead to
increased levels of glutathione.

Educational Objective: Chronic ethanol ingestion leads to the induction of cytochrome P450. Cytochrome P450 is responsible for the generation of the toxic acetaminophen metabolite NAPQI. Patients with a history of chronic ethanol ingestion are at an increased
risk of hepatotoxicity when consuming high doses of acetaminophen.

r r ,: tr5 P r ti"
e_revi9us
__ �&__l?cC?niJ!BP!1Jt Ll!b V11�1!_�_C11lc_1,1lato_
r
__ _ _l:i_e Jp____ie_a1,1.se
_ _ _
Exam Section 3: Item 27 of 50 National Board pf M-idlcal Examiners®
Comprehensive 1:1as1c science Self-Assessment

✓ 27. An investigational drug (Drug X) under development for the treatment of hypertension is being tested in phase 1 clinical trials. A significant number of subjects develop dose-dependent increases in serum AST and ALT activities. The investigators
hypothesize that this effect is due to oxidative stress. Assuming that biopsy samples of the liver can be obtained from the subjects, which of the following would be the best measure of hepatic oxidative stress caused by Drug X?

A ) Concentration of mRNA for C-reactive protein


B) Detection of nitrated proteins in hepatic cell membranes
C) Microsomal cytochrome P450 concentration
D ) Rate of state 3 mitochondrial respiration
E) Ratio of GSH:GSSG

Correct Answer: E.

Glutathione is a primary hepatic antioxidant. Reactive oxygen species, such as superoxide and hydroxyl radicals, possess unpaired electrons (free radicals) that damage tissues through oxidation reactions. Glutathione contains a sulfur atom that is capable of
reducing reactive oxygen species and free radicals, neutralizing these molecules before they can cause DNA strand breaks, lipid peroxidation, and protein damage. Glutathione exists primarily in a reduced form, with a hydrogenated sulfur group (GSH). Two
molecules of GSH can be oxidized in order to form glutathione disulfide, GSSG. The ratio of GSH to GSSG therefore reflects the degree of oxidation of the liver's supply of available, reduced glutathione. Hepatic metabolism of drugs that produce oxidative stress
through the production of free radicals will deplete hepatic stores of GSH, leading to a decreased ratio of GSH to GSSG.

Incorrect Answers: A, B, C, and D.

Concentration of mRNA for C-reactive protein (Choice A) is expected to be altered during inflammatory states. C-reactive protein is an acute phase reactant that is produced in states of inflammation and infection by the liver in response to inter1eukin-6.

Detection of nitrated proteins in hepatic cell membranes (Choice 8) is also a result of hepatic oxidative stress due to production of peroxynitrite. However, nitrated cell membrane proteins are only one type of oxidation product, whereas glutathione is involved
ubiquitously in the neutralization of free radicals and reactive oxygen species, making it a more sensitive marker.

Microsomal cytochrome P450 concentration (Choice C) will be altered by factors that induce P450, such as the use of alcohol, phenytoin, or rifampin. Microsomes are laboratory artifacts composed of fragments of endoplasmic reticulum and are not present in vivo.

Rate of state 3 mitochondrial respiration (Choice D) does not play a primary role in the hepatic metabolism of oxidizing agents, however increased mitochondrial activity may occasionally result in the production of oxidizing agents.

Educational Objective: Glutathione is a primary hepatic antioxidant and neutralizes oxidizing compounds by absorbing electrons at its sulfur atom. This reaction oxidizes two molecules of reduced glutathione, GSH, to glutathione disulfide, GSSG. Hepatic oxidizing
stress is most directly reflected by a depletion of GSH, leading to a reduction in the ratio of GSH to GSSG.

r
Previous
r
Next
,,.
Score Report
frj F
Lab Values Calculator
r
Help
,,.
Pause
Exam Section 3: Item 28 of 50 National Board pf Medical Exi!Jlllners®
Comprehensive Basic Science Serr-Assessment

'� .t7.f •
)( 28. A 32-year-old woman comes to the physician because of a 3-month history of fevers, weakness, and night sweats. Physical examination shows enlarged axillary lymph nodes. A photomicrograph of tissue obtained on
biopsy of an affected node is shown. A stain for acid-fast bacilli is negative. Which of the following is most likely involved in the pathogenesis of the .se findings?

A ) Activation of alternate complement pathway '


�·
�-fl.

B ) Streptococcal infection of the region drained by the lymph node


C) Type I (immediate) hypersensitivity reaction
D) Type Ill (immune complex-mediated) hypersensitivity reaction
E ) Type IV (delayed) hypersensitivity reaction

Correct Answer: E.

Type IV (delayed) hypersensitivity reaction is the correct answer. This patient with fevers, night sweats, and lymphadenopathy with a lymph node biopsy showing noncaseating granulomas likely has :sarcoidosis, a disease with myriad manifestations that most
commonly involves ihe lungs, eyes, skin, and central nervous system. It is characterized by granuloma formation in the absence of pathogenic microbes such as fungi or mycobacterium. The noncaseating epithelioid granulomas, which are the hallmark of
sarcoidosis, are !nought to contain an antigen at their core, with surrounding macrophages that form multinucleated giant cells. The dense lymphocytic inflammatory infiltrate that surrounds these granulomas contains many helper CD4+ T lymphocytes, cytotoxic
CDS+ T lymphocytes, B lymphocytes, and fibroblasts. Granuloma formation is often a consequence of type IV hypersensitivity reactions, which involve the exposure of a sensitized individual to an antigen, followed 24 to 72 hours later by an immune reaction to the
antigen. The response is primarily cellular rather than mediated by antibodies, and the cells involved are macrophages and Th1 helper T cells. Thus, a type IV hypersensitivity reaction is consistent with granuloma formation in sarcoidosis.

Incorrect Answers: A, B, C, and D.

Activation of alternate complement pathway (Choice A) does not occur in sarcoidosis and has no role in the formation of granulomas. The alternate complement pathway has been implicated in the pathogenesis of atypical hemolytic uremic syndrome, among
other diseases such as lupus nephritis.

Streptococcal infection of the region drained by the lymph node (Choice B) would not likely present with granulomas on lymph node biopsy. Bacterial infections commonly lead to a reactive lymph node profile with infiltration of the node by neutrophils.

Type I (immediate) hypersensitivity reaction (Choice C) is lgE-mediated and is the type of reaction that occurs in allergies and anaphylaxis. lgE antibodies are produced in response to an initial exposure to an antigen. With subsequent exposure, lgE antibodies
cross-link on mast cells and basophils, resulting in immediate degranulation of these cells with the release of histamine, leukotriene, and prostaglandins.

Type Ill (immune complex-mediated) hypersensitivity reaction (Choice D) is a result of immune complex (antigen-antibody) deposition in tissues resulting in a local inflammatory response and complement fixation with subsequent damage to tissues. Common
diseases in which this plays a role include polyarteritis nodosa, systemic lupus erythematosus, and serum sickness.

Educational Objectiye: Sarcoidosis is characterized by the development of non-caseating granulomas, which is achieved via a type IV hypersensitivity reaction to an unknown antigen.

r
Previous
r
Next
,
Score Report
fr§
Lab Values
r-:-
Calculator
r
Help
r
Pause
Exam Section 3: Item 29 of 50 National Board pf Medical Exi!J'lllners®
Comprehensive 1:Jas1c Science Sert-Assessment

✓ 29. A 20-year-old woman comes to the physician because her menstrual period is 1 week late. Menses had occurred at regular 28-day intervals. She had unprotected sexual intercourse with her boyfriend 20 days ago. A urine pregnancy test result is positive.
Which of the following best describes the stage of development of the embryo at this time?

A }-The sytetropheblast is present. b1o1t the synsylletrepheblast has not yet foi:med
a}- Gastrnlatien is semplete, b1o1t there are enly twe germ layers
C) The neural plate is present, but the neural tube is not yet complete
Q}- Plasental develepment is somplete, but the embrye is resistant te teratogens
E ) The sclerotome cells have begun migrating, but the somites have not yet formed
Correct Answer: C.

This patient's embryo has been developing for at most 20 days. At this time, the embryo has progressed through the stages of a blastocyst (week one), bilaminar disc (week two), trilaminar disc, and gastrulation has occurred or is occurring. Three germ layers are
present or are being formed, and the primitive streak (epiblast invagination), notochord, and neural plate are present. Progressing forward through the fourth to eighth weeks of embryogenesis, development will include formation of the neural tube, neuroectoderrn,
primitive organs, and limb buds.

Incorrect Answers: A, B, D, and E.

The cytotrophoblast is present but the syncytiotrophoblast has not yet formed (Choice A) is incorrect as by the third to fourth weel!c of gestation, the syncytiotrophoblast has formed. The syncytiotrophoblast begins to form initially during blastocyst implantation which
occurs in week one.

Gastrulation is complete, but there are only two germ layers (Choice B) is incorrect, as gastrulation is recognized as the process that generates three germ layers (the ectoderrn, mesoderm, and endoderm). It occurs during the third week.

Placental development is complete, but the embryo is resistant to teratogens (Choice D) is incorrect, as the placenta develops over several weeks to months. The embryo is highly sensitive to teratogens at this time. Once organogenesis and cell differentiation are
complete in the second and third trimester, sensitivity to teratogens decreases. The early first trimester is when the embryo is most sensitive.

The sclerotome cells have begun migrating, but the somites have not yet formed (Choice E) is incorrect as a sclerotome is a subdivision of a somite. Semites are segmental mesodermal derivatives oriented axially and adjacent to the neural tube. They subdivide
into sclerotomes, myotomes, and dermatomes that migrate, giving rise to the yertebrae, rib cage, tendons and cartilage, skeletal muscle, and some skin.

Educational Objective: In the third week of embryogenesis, gastrulation has occurred, and the neural plate is present. The neural tube will form during week four. The embryo is highly sensitive to teratogens at this time.

r r ,: ta P r ti"
Previous Next Score Report Lab Values Calculator Help Pause
Exam Sectlon 3: Item 30 of 50 National Board pf Medical Examiners®
Comprehensive isaslc Science Self-Assessment

✓ 30. A healthy 70-year-old woman has participated in a longitudinal study of the effects of aging on performance during pulmonary function tests for the past 50 years. She has undergone a complete set of tests, including arterial blood gas analyses, every 5
years. Which of the following sets of changes best represents this woman now compared with her results at the age of 20 years?

Alveolar-arterial Po 2
Residual Volume Arterial Poz Difference
A) t t !
B) t l t
G} Me shange t .!.

(;)� Ille shange t ¾


E} t t .1.

�} •
.1. ¼ •
.1.

Correct Answer: B.

Normal lung function declines with age, with an expected increase in residual volume, decreased arterial PO 2, and an increased alveolar-arterial PO 2 difference (A-a gradient). These changes result from structural, physiological, and immunological alterations
that occur with aging. Osteoporosis and kyphosis of the vertebral column alter the mechanics of respiratory function, and calcification of the rib cage results in a stiffened chest wall with decreased compliance. Residual volume (volume of air remaining at the end
of forced expiration) increases with age. The expansion of the lungs at the end of expiration is a balance between an outward force from the chest wall and an inward force from the lungs. Decreased chest wall compliance results in a greater outward force, while
loss of lung elastic recoil results in a reduced inward force. Arterial PO2 decreases with age due to a decreased responsiveness of the respiratory control center to hypoxemia and hypercarbia, which would normally stimulate an increase in minute ventilation.
The A-a gradient has been also shown to increase with age, which may be due to the loss of alveolar surface area for gas exchange due to persistent wear and progressive damage to the alveolar-capillary interface along with reduced regeneration secondary to
pneumocyte cell senescence, and chronic damage to the small airways over time.

Incorrect Answers: A, C, D., E, and F.

Choices A, D, E, and F are incorrect as the A-a gradient increases with age, which is secondary to alveolar surface dysfunction from progressive damage over time, along with small airways degradation, an� reduced repair mechanisms.

Choices C, D, E, and F are incorrect as residual volume commonly increases with age. The most common cause of a decreased residual volume is a restrictive lung disease, which includes diagnoses such as interstitial lung disease, obesity hypoventilation
syndrome, and neuromuscular disorders (eg, amyotrophic lateral sclerosis).

Educational Objective: Agi111g results in expected changes to respiratory mechanics and a decline in pulmonary function over time. A stiff chest wall decreased responsiveness to normal drivers of respiration (hypoxemia and hypercarbia), and loss of alveolar
surface area for gas exchange result in an increased residual volume, decreased arterial PO 2, and an increased A-a gradient over time.

r
Previous
r
}�ext
,
Score �eport
tr§
Lab Values
F
C alculator
r
Help
,,.
Pause
_ .
Exam Section 3: Item 31 of 50 National Board of Medical Exi!Jlllners®
Comprehensive Basic Science Serr-Assessment

✓ 31. A study is conducted to assess the relationship between angiotensin gene polymorphisms (I versus D alleles) and the development of progressive renal disease due to lgA nephropathy. A total of 250 patients are enrolled: 125 patients with progressive
renal disease due to lgA nephropathy (Group X) and 125 patients with lgA nephropathy with stable renal disease (Group Y). Results show that 67 patients in Group X have the D allele, whereas 12 patients in Group Y have the D allele. Which of the
following tests is most appropriate to determine whether there is a significant difference in the prevalence of the D allele between these two groups?

A ) Analysis of variance
B ) Chi-square test
C) Linear regression
D) Nonparametric regression
E ) Student's t-test
Correct Answer:. B.

Statistical tests can be employed to assess whether a difference in values exists. Often, these values are means, though they need not be. Many tests have been derived to assess differences in numerical, quantitative, categorical, and qualitative datasets.
Choosing the appropriate test is important in order to appropriately draw conclusions from data. In this case, comparing allele frequency across two groups is a comparison of categorical values (the presence or absence of the allele in question), which is best
assessed by a chi-square test. Chi-square tests assess for statistically significant differences between two or more categorical values (eg, proportions, percentages). It is commonly used for larger samples and is also able to compare multiple values (eg, three
different experimental groups). In this study, assessing the difference between the percentage of patients having the allele between the two arms is a categorical comparison.

Incorrect Answers: A, C, D, and E.

Analysis of variance (Choice A) is a statistical test that compares means between three or more groups. In this case, the comparison is of a percentage, not a mean, and only two groups are analyzed.

Linear regression (Choice C) and nonparametric regression (Choice D) model the relationship between dependent and independent variables in a linear fashion or without beginning with a predetermined form but instead deriving the trend from the data,
respectively. It is not used to compare means. It is more frequently used to predict the dependent variable from an independent variable assumed to behave in a similar way to the data used to construct the regression.

Student's t-test (Choice E) also simply referred to as a f-test, calculates differences between two means. For example, comparing the resting heart rates between athletes and non-athletes to assess for statistically significant differences.

Educational Objective: Chi-square tests assess for statistically significant differences between two or more categorical values (eg, proportions, percentages).

r r , fr!f r::- r r
Previous Next Score Re ort Lab Values Calculator Hel Pause
Exam Secllon 3: Item 32 of 50 Nallonal Board pf Medical Examiners®
Comprehensive Basic Science Serr-Assessment

✓ 32. A44-year-old woman comes to the physician because of a 3-week history of severe headache. She is 157 cm (5 ft 2 in) tall and weighs 86 kg (190 lb); BMI is 35 kg/m2. Funduscopic examination shows papilledema. An MRI of the brain shows no
abnormalities. A lumbar puncture is done. Cerebrospinal fluid (CSF) analysis shows an opening pressure of 32 cm Hf). In addition to recommendations regarding dietary changes and weight loss, treatment with acetazolamide is begun. This drug will
most likely improve the patient's symptoms by which of the following mechanisms?

A ) Decreased CSF production by the choroid plexus


B) Decreased venous pressure around the arachnoid granulations
C) Increased permeability of tight junctions (zonulae occludentes) between endothelial cells
D) Increased sodium permeability of the ependyma
E ) Increased water transport by astrocyte end-foot processes
Correct Answer: A.

Acetazolamide is a carbonic anhydrase inhibitor that acts as a diuretic and indirectly decreases CSF production by the choroid plexus. Pseudotumor cerebri, or idiopathic intracranial hypertension, is an idiopathic increase in intracranial pressure (ICP) that typically
occurs in obese women of childbearing age and presents with headaches that may be postural, transient visual deficits due to pressure on the optic nerve, and nausea/vomiting (symptoms of increased ICP). Papilledema is common on fundoscopic examination.
MRI is typically unremarkable, and lumbar puncture is diagnostic if increased opening pressure is demonstrated. Treatment includes weight loss and acetazolamide, serial lumbar punctures (which decrease ICP), or placement of a CSF shunt in refractory cases.

Incorrect Answers: B, C, D, and E.

Decreased venous pressure around the arachnoid granulations (Choice B) would lower ICP by promoting venous drainage down a hydrostatic pressure gradient. However, treatments for pseudotumor cerebri, which features an idiopathic etiology of increased ICP,
do not utilize this mechanism.

Increased permeability of tight junctions (zonulae occludentes) between endothelial cells (Choice C) would lead to increased diffusion of substances from the blood into the CSF, as these tight junctions form the blood-CSF barrier. However, this mechanism would
not address increased ICP and might actually increase it. It is not a mechanism employed in pseudotumor cerebri treatment.

Increased sodium permeability of the ependyma (Choice D) would cause increased sodium diffusion across the ependyma (the permeable lining of the ventricles that separates the CSF system and brain parenchyma). Increased sodium in the brain parenchyma
would not decrease ICP (and may worsen headaches as increased. brain sodium levels are proposed as a migraine mechanism). In turn, increased CSF sodium concentration could potentially increase ICP as water follows an osmolar gradient.

Increased water transport by astrocyte end-foot processes (Choice E) could occur by manipulating the expression of aquaporin channels in both the choroid plexus and ependymal cells. Aquaporins play a role in the pathogenesis of hydrocephalus and are potential
therapeutic targets. There are no pseudotumor cerebri medications that utilize this mechanism, and it is not known whether this mechanism could help in pseudotumor cerebri.

Educational Objective: Pseudotumor cerebri is an idiopathic disorder of increased ICP that is most common in obese women of childbearing age that presents with headache and physical examination findings of increased ICP. Acetazolamide can decrease CSF
production by the choroid plexus, thereby decreasing ICP.

r
Prev1ous
r
Next
,
Score Report
� P
Lab Values Calculator
r
Help
r-
Pause
_
Exam Section 3: Item 33 of 50 National Board pf Medical Examiners®
Comprehensive Basic Science Self-Assessment


a�
�-
• �#

� ·�-
-� ,: .
-�- ,
'!'<

33. A 42-year-old man has an autoimmune disorder resulting in proximal muscle weakness of the lower extremities. Arrows in the photomicrograph shown indicate membranes that contain high concentrations of channels that are targeted by this condition.
Which of the following is the most likely diagnosis?

A ) Multiple sclerosis
B ) Myasthenia gravis
C) Myasthenic (Lambert-Eaton) syndrome
D ) Periodic paralysis
E ) Tetany
Correct Answer: C.

Lambert-Eaton myasthenic syndrome is an autoimmune condition in which autoantibodies directed against voltage-gated calcium channels in the presynaptic neuron decrease the release of acetylcholine (ACh). Limited ACh release limits muscle cell depolarization
and results in proximal muscle weakness. Additional symptoms include hyporeflexia, constipation, and autonomic dysregulation. With use, muscle strength improves due to the gradual accumulation of ACh in the synapse with repeated presynapti c depolarization.
The diagnosis can be confirmed with testing for anti-voltage gated calcium channel antibodies. In many cases, Lambert-Eaton myasthenic syndrome is a paraneoplastic syndrome associated with small cell lung carcinoma. The electron micrograph demonstrates
arrows pointing to the cell membrane of the presynaptic neuron terminal. T he rounded structure enve:loped by this membrane is known as the synaptic bouton. Within the bouton, preformed vesicles containing neurotransmitter are present. Directly distal to this, the
undulating surface of the sarcolemma can be recognized.

Incorrect Answers: A, B, D, and E.

Multiple sclerosis (Choice A) is an autoimmune demyelinating disease that typically presents in Caucasian females in their fourth decade of life. It is characterized by central nervous system demyelination of white matter in both the brain and the spinal cord. This
leads to weakness, hyperreflexia, and spasticity. Optic neuritis is a classic presenting diagnosis. Acute flares are treated with high-dose steroids.

Myasthenia gravis (Choice B) is a disease sinnilar to Lambert-Eaton myasthenic syndrome except autoantibodies are produced against ACh receptors of the neuromuscular junction. In myasthenia gravis, weakness becomes more profound with increased muscle
use and bulbar weakness is common.
Periodic paralysis (Choice D) is a genetic disease that leads to malfunctions in the ion channels of skeletal muscle. Electrolyte dysregulation, hyperthyroidism, cold, heat, stress, and physical activity can all precipitate episodes of paralysis in affected persons. It
does not involve neurotransmitter release at the neuromuscular junction.

Tetany {Choice E) refers to the involuntary and persistent contraction of muscles. Causes include electrolyte derangements, and classically, Clostridium tetani toxicity, which inhibits GABA release.

Educational Objective: Lambert-Eaton myasthenic syndrome is caused by antibodies directed against voltage-gated calcium channels, which decrease the release of acetylcholine at the neuromuscular junction. This leads to decreased reflexes, muscle weakness
that improves with repeated use, and autonomic instability. It is frequently a para neoplastic syndrome .

r r , tr?J F ,- r
Previous Next Score Report Lab Values Calculator Help Pause
Exam Section 3: Item 34 of 50 National Boa.rd pf Medical Examiners®
Comprehensive 1:1as1c Science Serr-Assessment

✓ 34. A 6-year-old boy is brought to the emergency department because of coughing, wheezing, and rapid breathing for 6 hours. He was diagnosed with an upper respiratory tract infection 2 days ago. His temperature is 37.2° C (99° F), pul.se is 120/min,
respirations are 44/min, and blood pressure is 90/60 mm Hg. lnspirat:ory and expiratory wheezes are heard throughout the lung fields. There is decreased tactile fremitus. Which of the following is the most likely diagnosis?

A) Asthma
S }-Atelectasis
C) Bronchitis
C} ldl-ft.&ided heart failure
e }-Pneumococcal pneumonia
Correct Answer: A.
The patient is likely experiencing an asthma exacerbation. Asthma is characterized by reversible obstruction of the bronchi secondary to hyperreactivity and airway inflammation. Patients present with episodes of wheezing, dry cough, and dyspnea. Physical
examination often reveals tachycardia, tachypnea, diffuse wheezes (rhonchi), and prolonged expiration relative to inspiration. Decreased tactile fremitus may be noted due to air trapping which decreases lung density (leading to reduced transmission of vibrations
through the lung parenchyma to the body wall). Chest radiographs are typically normal. Pulmonary function tests may be normal between symptomatic exacerbations, but during such episodes demonstrate an obstructive pattern with increased residual volume and
total lung capacity. Viral upper respiratory tract infection is a common trigger of acute asthma exacerbation in patients under 12 years of age. Acute management includes supplemental oxygen, inhaled short-acting 13-adrenergic agonists, and systemic
corticosteroids.
Incorrect Answers: B, C, D, and E.
Atelectasis (Choice 8) refers to the partial or complete collapse or incomplete expansion of pulmonary parenchyma. There are multiple type.s of atelectasis, including obstructive, passive, compre.ssive, cicatricial, and adhesive. It is most common in patients who are
immobile and/or in the postsurgical state, who are unable to produce adequate tidal volume to expand the lungs. Signs include decreased breath sounds and dullness to percussion in the affected region.
Acute bronchitis (Choice C) is a respiratory tract infection involving inflammation of the bronchi. It is most commonly caused py viruses such as influenza A and B, parainfluenza, respiratory syncytial virus, coronavirus, adenovirus, rhinovirus, and human
metapneumovirus. It typically presents with cough, fever, and sore throat. It can be challenging to distinguish from an acute asthma exacerbation, especially if there is no established history of asthma. Differentiating features include time course (asthma
exacerbation is typically acute on the order of hours while symptoms of acute bronchitis are present for days) and the presence of fever.
Left-sided heart failure (Choice D) presents with shortness of breath, dyspnea on exertion, orthopnea, and production of frothy sputum, and may be exacerbated by a viral infection in patients with a predisposing cardiac condition or as a sequela of viral myocarditis.
Physical examination findings include tachycardia, tachypnea, crackles (rales) on auscultation of the lungs, and an S3 or S4 gallop on cardiac auscultation. Pulmonary congestion and edema can sometimes result in wheezes (cardiac wheeze). There may be an
associated pleural effusion which would result in decreased tactile fremitus in the region on palpation. Asthma is the more likely diagnosis in a young patient with no known prior cardiac disease.
Pneumococcal pneumonia (Choice E) is a lung infection caused by Streptococcal pneumoniae. Pneumonia classically presents with fever, dyspnea, productive cough, and pleuritic chest pain. Physical examination reveals decreased breath sounds in the affected,
consolidated lung. The increased lung density in the region of consolidation results in increased transmission of vibrations to the body wall and increased tactile fremitus on palpation.
Educational Objective: Asthma exacerbations present acutely with symptoms developing over minutes to hours. Diffuse wheezing is typically appreciated on examination. Viral upper respiratory tract infections are the most common trigger in children.

r
Previous
r
Next
,
Score Report
� F
Lab Values Calculator
,:-.
Help
ff'
Pause
Exam Secllon 3: Item 35 of 50 Nallonal Board pf Medical Examiners®
Comprehensive Basic Science Serr-Assessment

✓ 35. An 11-year-old boy receives succinylcholine and sevoflurane for anesthesia during surgical repair of a hernia. During the procedure, the patient develops muscle rigidity, tachycardia, hyperkalemia, hypocalcemia, and hyperthermia. This patient most likely
carries a variant form of the gene coding for which of the following?

A ) N-Acetyltransferase
B ) Cytochrome P450 206
C ) Cytochrome P450 3A4
D ) Glutathione synthetase
E) Ryanodine receptor

Correct Answer: E.

Malignant hyperthermia (MH) is caused by an autosomal dominant genetic defect in the ryanodine receptor of muscle cells, leading to increased calcium release from the sarcoplasmic reticulum with exposure to triggering agents (eg, succinylcholine, volatile
anesthetics). This leads to sustained muscle contraction with increased oxygen demand and carbon dioxide production. Physical examination and vital signs will demonstrate muscle contractures (eg, masseter muscle rigidity), fever, and tachycardia, while
laboratory evaluation will show increases in CO2 and lactic acidosis. Risk factors include a family history of malignant hyperthermia, being born in the upper Midwest of the United States, and certain musculoskeletal disorders, such as multi-minicore myopathy.
Treatment is with dantrolene, which acts as a ryanodine receptor antagonist, as well as external cooling and correction of electrolyte and acid-base abnormalities. Mortality is high, ranging from 5% to 30% with treatment.

Incorrect Answers: A, B, C, and D.

N-Acetyltransferase (NAT) (Choice A) transfers acetyl groups from acetyl-CoA to proteins, including serotonin, and is important in phase II hepatic metabolism of drugs. Polymorphisms in the genes that encode the NAT enzymes cause differences in speed of
metabolism of medications between individuals but do not cause MH.

Cytochrome P450 206 (Choice B) plays an important role in the metabolism and activation of many medications through hydroxylation, demethylation, and dealkylation. Variant genes lead to rapid elimination of medications or slower than normal elimination, but
they do not cause MH.

Cytochrome P450 3A4 (Choice C) plays an important role in the metabolism and activation of many medications through oxidation and hydroxylation. Most variant genes do not cause significant differences in elimination between patients, although a rare few cause
decreased metabolism.

Glutathione synthetase (Choice D) is involved in the synthesis of glutathione, which acts as an antioxidant. Deficiency can present with jaundice due to chronic hemolytic anemia and metabolic acidosis. It does not present with MH.

Educational Objective: Malignant hyperthermia is caused by a genetic defect in the ryanodine receptor and presents with muscle rigidity, tachycardia, fever, lactic acidosis, and hypercarbia when the patient is exposed to succinylcholine or volatile anesthetics. Risk
factors include a family history, being born in the upper Midwest of the United States, and certain musculoskeletal disorders. Dantrolene is the treatment of choice.

r
Prev1ous
r
Next
,
Score Report
� P
Lab Values Calculator
r
Help
r-
Pause
_
Exam Section 3: Item 36 of 50 National Board pf Mqdlcal Examiners®
Comprehensive Basic Science Self-Assessment

✓ 36. A 50-year-old man comes to the physician because of a 2-week history of progressive shortness of breath while climbing stairs to his office every morning. He reports no other problems, but he is concerned because his father had a major myocardial
infarction at the age of 52 years. His pulse is 110/min and regular, respirations are 16/min, and blood pressure is 135/95 mm Hg. The lungs are clear to auscultation. Cardiac examination shows normal heart sounds with a physiologic split of S 2. Stress
echocardiography shows hypokinesis of the posterior left ventricle with increasing activity levels. Which of the following is the most likely cause of the posterior left ventricular findings in this patient?

A ) Disruption of the sympathetic nerves to the left ventricle


g )- Extravascular compressioA of the ooroAary arteries
C) Increased left ventricular end-diastolic pressure
0� IAcreased myocardial oxygeA c0As11mplion
E ) Stenosis of the right coronary artery

Correct Answer: E.

Stenosis of the right coronary artery (RCA) most likely accounts for this patient's posterior left ventricle hypokinesis on stress echocardiography. The coronary arteries supply oxygenated blood to the myocardium and originate just distal to the aortic valve at the right
and left coronary ostia, which give rise to the RCA and left coronary artery (LCA), respectively. The RCA in right-dominant individuals gives rise to several branches, including the sinoatrial nodal artery, the posterior descending artery (PDA), and the right marginal
branch. In left-dominant individuals, the PDA arises from the left circumflex artery, which is a branch of the LCA. The RCA descends along the right side of the heart in the coronary groove between the right atrium and right ventricle. It wraps around the base of the
heart, moving posteriorly and inferiorly. It supplies blood to the right atrium, right ventricle, and the inferior aspect of the heart apex. Stenosis of one or more parts of the coronary arteries is termed coronary artery disease (CAD) and is usually a result of
atherosclerotic plaque buildup over many years. Depending on the degree of stenosis, the myocardium may not receive sufficient oxygen to meet its metabolic demands during times of increased work, such as exercise. This mismatch in supply and demand of
oxygen results in myocardial ischemia and can cause angina. If myocardial ischemia is severe enough and not reversed, which occurs in cases of plaque rupture, it results in myocardial infarction. Many individuals with CAD will have angina! chest pain when they
exercise, but symptoms abate with.adequate rest or the use of nitroglycerin, which decreases the myocardial oxygen demand and decreases the supply-demand mismatch. Exercise stress tests, including stress echocardiogram, are employed in an attempt to
identify areas of decreased perfusion. In this instance, hypokinesis on echocardiogram indicates a flow-limiting stenosis of one of the coronary arteries (greater than 70% stenosis). As most people are right-dominant, the induced hypokinesis of the posterior left
ventricle on stress echocardiography likely indicates stenosis in the right coronary artery but would need to be confirmed by coronary angiography.

Incorrect Answers: A, B, C, and D.

Disruption of the sympathetic nerves to the left ventricle (Choice A) is postulated to be a critical abnormality in Takotsubo cardiomyopathy, also known as stress cardiomyopathy. Disruption results in significant left ventricular wall motion abnormalities primarily at the
apex and a reduced left ventricular ejection fraction. In most patients, this is reversible over time.

Extravascular compression of the coronary arteries (Choice 8) is a normal phenomenon during the cardiac cycle and occurs during systole. The coronary arteries perfuse the myocardium during diastole. This would not result in hypokinesis of the left ventricle.

Increased left ventricular end-diastolic pressure (Choice C) occurs in patients with diastolic dysfunction, which is often a long-term sequela of chronic hypertension. The ventricular cavity becomes stiff and loses its elasticity as a result of chronic remodeling with
consequent impaired diastolic filling and increased end-diastolic pressure.

Increased myocardial oxygen consumption (Choice D) occurs normally in states requiring increased cardiac output, such as exercise, high-output heart failure, or sepsis. Hypokinesis of the posterior left ventricle implies inadequate perfusion and therefore
inadequate oxygenation of 1his part of the myocardium due to a flow-limiting stenosis of a coronary artery.

Educational Objective: In a majority of people, the RCA supplies the posterior left ventricle via tlhe PDA. In patients with CAD and a flow-limiting stenosis, stress echocardiography can demonstrate hypokinesis of the region of myocardium supplied by the affected
artery.

r
Previous
r
Next
,-
Score Report
� F
Lab Values Calculator
r
Help
,,.
Pause
Exam Section 3: Item 37 of 50 Natlonal Board of Mqdlcal Examiners®
Comprehensive Basic Sctence Serr-Assessment

✓ 37. An investigator is studying Helicobacter pylori strains isolated from the same patient over several years. The immune response to proteins produced by these strains is observed. It is found that the original H. pylori isolate from the patient expresses one
protein recognized by the patient's antibodies, but subsequent isolates do not express this protein. Sequencing of the gene encoding the protein from the original and subsequent isolates is done. It is found that subsequent isolates have nine consecutive
cytidine residues, whereas the original isolate has eight The results are shown.

Original isolate:
,., ACC CCC CCC ACT CM ATT GM CCT AGC ...
... Thr Pro Pro THr Gin lie Glu Pro Ser ...

Subsequent isolates:
... ACC CCC CCC CAC TCA AAT TGA ACC TAG
... Thr Pro Pro His Ser Asn STOP

Which of the following mechanisms best explains this genetic change?

A ) Crossing over
B ) DNA excision repair
C) Slipped-strand mispairing
D ) Thymidine dimer formation
E ) Transposon insertion
Correct Answer: C.
Slipped-strand mispairing is a type of mutation that may occur during DNA replication. In an area of repetitive nucleotides, such as the series of eight cytidine residues in this case, DNA polymerase slips and either inserts one or more additional nucleotides or
removes one or more additional nucleotides by mistake. Here, the addition of one cytidine residue has led to a frameshift mutation in which the base pair reading frame has been shifted over by one, leading to a completely different amino acid sequence during
translation and an early stop codon. In this case, the frameshift mutation caused by slipped-strand mispairing has led to a dysfunctional, truncated protein. If a slipped-strand mutation led to the insertion or deletion of nucleotides in groups of three, it would cause
the addition or omission of amino acids to the original sequence as the reading frame would be preserved. Since antibody recognition of proteins is highly specific, this truncated protein seen in this case does not bind to the same antibody that the wild-type protein
previously did.
Incorrect Answers: A, B, D, and E.
Crossing over (Choice A) is part of the recombination process, which takes place in meiosis. It is the process by which two chromatids exchange a small portion of genetic material and separate, or unlink, two traits previously present on the s_ame chromatid. The
total genetic material on each chromosome remains the same, so it would not explain the addition of a nucleotide base pair.
DNA excision repair (Choice B) is a proofreading process by which mutated nucleotides (nucleotide excision repair) or bases (base excision repair) are removed. This process leads to the removal of nucleotides rather than the insertion.
Thymidine dimer formation (Choice D) is a mutation caused by ultraviolet radiation, which leads to covalent cross-linking between two adjacent thymidine nucleotides to create a dimer. If uncorrected, thymidine dimers can cause misreading during replication or
transcription. However, thymidine dimer formation does not lead to nucleotide insertion.
Transposon insertion (Choice E) occurs when a segment of double-stranded DNA moves position within or across chromosomes or between plasmid and chromosome. Transposons may contain genes that confer survival advantages to the host, for example, when
the transfer of genes yields antibiotic resistance to a bacterium. However, it would not explain the addition of a single nucleotide into a strand of DNA, as in this case.
Educational Objective: Slipped-strand mispairing occurs at regions of repetitive nucleotides within the DNA due to the slipping of DNA polymerase. When nucleotides are not added or deleted in multiples of three, a frameshift mutation occurs, altering the sequence
of amino acids in the final protein.

r r , ta e r "'
Previous Next Score Report Lab Values Calculator Help Pause
Exam Section 3: Item 38 of 50 National Board pf M41dlcal Examiners®
Comprehensive Basic SClence Self-Assessment

✓ 38. An 18-month-old girl is brought to the physician because of failure to thrive and frequent wet diapers. She is at the 3rd percentile for length and weight. Physical examination shows dehydration, decreased muscle tone, and bowing of the legs. Slit lamp
examination shows crystals in the corneas. Serum studies show hypophosphatemia, hypokalemia, and a nonanion gap metabolic acidosis. Urine studies show a pH of 5, glucosuria, phosphaturia, and generalized aminoaciduriia. Which of the following is
the most likely cause of the metabolic acidosis in this patient?

A} Desi:eased sarbei::iis anhydrase asti>.<ity in the distal tubule


B } Decreased carbornic anhydrase activity in the proximal tubule
G} Desreased hydrogen ion sesi:etien in the distal tubule
D} Decreased hydrogen ion secretion in the proximal tubule
Ii} Desi:eased sodium bisartionaui reabsorption in the distal tubule
F } Decreased sodium bicarbonate reabsorption in the proximal tubule

Correct Answer: F.

Cystinosis is a rare lysosomal storage disease that results in increased levels of cystine and its accumulation in tissues, which can lead to severe organ dysfunction. Cystine is normally stored in lysosomes, but in cystinosis, failure of storage leads to crystal
formation and deposition in tissues as its concentration increases. Deposition in the kidney can result in dysfunction of the proximal convoluted tubule (nephropathic cystinosis}, causing renal Fanconi syndrome, which is characterized by impaired reabsorption in the
proximal tubule of the kidn.ey leading to glycosuria, phosphaturia, and aminoaciduria. As well, the reabsorption of bicarbonate is impaired in the proximal convoluted tubule, which manifests with a non-anion gap metabolic acidosis and hypokalemia (renal tubular
acidosis type 2). This increases the risk for hypophosphatemic rickets, which may explain the patient's leg bowing. Additional signs and symptoms of cystinosis occur secondary to cystine crystal deposition in tissues, including failure to thrive, polyuria, dehydration,
myopathy, gastrointestinal dysmotility, hepatomegaly, splenomegaly, and visual disturbances.

Incorrect Answers: A, B, C, D, and E.

Carbonic anhydrase (Choices A and B} is an enzyme that converts CO2 to bicarbonate to maintain acid-base balance. Decreased activity of carbonic anhydrase in the proximal or distal tubule would cause decreased bicarbonate formation and result in a non-anion
gap metabolic acidosis, but would not affect glucose, phosphate, and amino acid balance.

Hydrogen ion secretion (Choices C and D} occurs in the distal tubule, not the proximal tubule. Impaired hydrogen ion secretion would result in a non-anion gap metabolic acidosis (renal tubular acidosis type 1 ), but would not result in glucosuria, phosphaturia, or
aminoaciduria.

Cystinosis affects reabsorption in the proximal tubule (renal Fanconi syndrome), not the distal tubule (Choice E}.

Educational Objective: Cys1inosis is a lysosomal storage disease that results in increased levels of cystine and its accumulation in tissues, which can lead to severe organ dysfunction. It is characterized by the impaired reabsorption in the proximal tubules of the
kidney (renal Fanconi syndrome), leading to losses of sodium, potassium, glucose, amino acids, phosphate, and bicarbonate. The loss of bicarbonate in the urine leads to a non-anion gap metabolic acidosis (renal tubular acidosis type 2}.

r r , fr§
Lab Values
F
Calculator
r
Help
ff'
Pause
Previous Next Score Report
Exam Secllon 3: Item 39 of 50 National Board pf Mqdlcal Exftllllners®
Comprehensive aas1c: scienc:e Serr-Assessment

't 39. A study is conducted to evaluate the effectiveness of cough medication in children. A total of 120 patients under the age of 12 years who have been diagnosed with a viral upper respiratory tract infection are randomly selected to receive dextromethorphan
or no treatment. The participants were recruited from several emergency departments in an urban area. Parents complete a rating scale for their children's cough at baseline and 1 day after receiving dextromethorphan or nothing. The results show a
statistically significant improvement in cough scores for patients receiving dextromethorphan (p=0.04) compared with children receiving no treatment. Which of the following is the most significant limitation to the internal validity of this study?

A ) Lack of blinding between groups


B ) Limited number of participants
C ) Marginal statistical significance
D) Study location limited to only emergency departments
E ) Subjective nature of the survey instrument

Correct Answer: A.

Internal validity describes the degree to which a study establishes or supports a faithful conclusion about cause and effect-in other words, was the design of the study sufficient enough to support the conclusion that was drawn. Sufficient internal validity should limit
the degree to which alternative explanations of cause and effect can be reasonably drawn. Internal validity thus refers to whether the treatment in question (or variable being tested) makes a difference, and whether the experiment was designed in a way to actually
support the claim. Common threats and limitations to internal validity include many types of selection bias and confounding, alterations in the characteristics of the study participants over the course of the study, experimental mortality, observer biases, Hawthorne
effect, and Berkson bias. In this case, the tack of blinding between groups could theoretically cause parents to draw conclusions that may be based on expected results as opposed to true outcomes (eg, "My child received the actual medicine and is doing better.").

Incorrect Answers: B, C, D, and E.

Limited number of participants (Choice 8) often limits statistical power, the ability of the study to detect a difference if one exists. While the actual number required is calculated by effect size and beta level, there is no indication that this study tacks power.

Marginal statistical significance (Choice C) implies that since the p value in the study was close to the designated alpha level, that the study may be less statistically significant than one with a greater difference between the p value and alpha level. The study
demonstrated a p value less than the alpha level; it is therefore considered statistically significant. Such a statement would be equally true for a p value of 0.04, 0.03, 0.02, 0.01, or less.

Study location limited to only emergency departments (Choice D) limits external validity, not internal. The population that presents to an emergency department may be fundamentally different in some way from children treated at home, or by pediatricians in the
outpatient setting.

Subjective nature of the survey instrument (Choice E) does not limit internal validity as long as the study is otherwise appropriately conducted. Many studies require subjective assessments even when such are quantitated, for example, pain scales are commonly
used to analyze the effectiveness of analgesics, though each patient experiences pain differently. Further, the study, as provided, does not describe the rating scale for the parental judgement of cough. It may be objective (eg, number of times coughing per hour) or
subjective (eg, my child seems to be more comfortable).

Educational Objective: Internal validity describes the degree to which a study establishes or supports a faithful conclusion about cause and effect-in other words, was the design of the study sufficient enough to support the conclusion that was drawn. Common
threats and limitations to internal validity include many types of selection bias and confounding, alterations in the characteristics of the study participants over the course of the study, experimental mortality, observer biases, Hawthorne effect, and Berkson bias.

r
Previous
r
Next
,,,
Score Report

Lab Values
P
Calculator
r
Help
1ft'
Pause
Exam Section 3: Item 40 of 50 National Board pf Mqdlcal Examiners®
Comprehensive Basic Sctence Serr-Assessment

✓ 40. A 64-year-old man with a 25-year history of alcoholism is brought to the emergency department by his wife because of a 1-day history of confusion. He is disoriented and disheveled. His pulse is 110/min, respirations are 20/min, and blood pressure is
100/64 mrm Hg. Physical examination shows signs of dehydration and jaundice, and spider angiomata over the face and chest. There is a flapping, up-and-down motion of the hands when the upper extremities are outstretched horizontally. Abdominal
examination shows distention with bulging flanks and shifting dullness. In addition to other appropriate pharmacotherapy, administration of oral neomycin is begun. Which of the following primary mechanisms of action is most likely to occur in this patient
as a result of this drug treatment?

A}- Binding of ammooia and other hepatisally cleared !Gxins in the g11t
8}- Blosl<ade of new protein synthesis by the liver
G} Bloskade of t\•Jo SYGGessive steps iA the metabolismoffolis asid
D) Increased endotoxin production where bacterial overgrowth occurs in the gut
E ) Killing of bacteria in the gut that generate ammonia

Correct Answer: E.
Killing of bacteria in the gut that generate ammonia is the mechanism of action by which neomycin helps to treat hepatic encephalopathy (HE). Neomycin is an aminoglycoside antibiotic with activity against Gram-negative aerobes that was used frequently in the
past for the treatment of HE. It works by inhibiting the 30S subunit of bacterial ribosomes to prevent protein translation. Ammonia is a waste product generated by bacteria in the gut. In its nonionized form, ammonia is readily absorbed by the gut and into the
systemic circulation. In cirrhosis, normal mechanisms of detoxification and processing of ammonia via the urea cycle are impaired, and high levels of ammonia play a role in the dysfunction of neurons seen in HE. Killing Gram-negative gut flora reduces the amount
of ammonia produced. Neomycin is no longer a preferred therapy for HE as it is absorbed readily and has side effects including acute kidney injury and ototoxicity. Rifaximin is currently the antibiotic of choice for the adjunctive treatment of hepatic encephalopathy,
which also targets colonic ammoniagenic bacteria.
Incorrect Answers: A, B, C, and D.
Binding of ammonia and other hepatically cleared toxins in the gut (Choice A) is accomplished by the action of AST-120, an oral microspherical carbon that binds ammonia directly. It is not currently in clinical use.
Blockade of new protein synthesis by the liver (Choice B) is not a mechanism of action of currently used medications for the treatment of hepatic encephalopathy. Many proteins are synthesized in the liver, including coagulation factors and albumin. These are not
thought to play a role in HE.
Blockade of two successive steps in the metabolism of folic acid (Choice C) is the function of trimethoprim-sulfamethoxazole (TMP-SMX). This pathway is required for the synthesis of nucleic acids, which are necessary for DNA replication in bacteria. This
medication is occasionally used for prophylaxis against spontaneous bacterial peritonitis, not HE.
Increased endotoxin production where bacterial overgrowth occurs in the gut (Choice D) describes one action of alcohol. Alcohol ingestion may increase endotoxin production by gut bacteria and also contribute to the breakdown of the mucosa! barrier with
subsequent translocation of bacteria across the lumen and into the blood stream.
Educational Objective: Neomycin is an antibiotic that kills Gram-negative gut flora, thereby reducing ammonia production. It is not commonly used in current clinical practice given its numerous side effects and has largely been supplanted by rifaximin, another
antibiotic that targets colonic ammoniagenic bacteria.

r r , ta F r "'
Previous Next Score Report Lab Values Calculator Help Pause
Exam Sectlon 3: Item 41 of 50 National Board pf M11dlcal Examiners®
Comprehensive i,aa1c: sc:ien" SelT•A&&eaament

✓ 41. A 6-year-old girl is brought to the physician for a follow-up examination. She has been receiving treatment with potassium citrate monohydrate since the diagnosis of renal tubular acidosis was made at the age of 2 years. Physical examination shows no
abnormalities. Serum studies show:
Na + 142 mEq/L
K+ 3.5 mEq/L
c1- 115 mEq/L
HCO3- 18 mEq/L
Urea nitrogen 9 mg/dl
Creatinine 0.9 mg/dl

A defect in renal ammoniagenesis is suspected. Which of the following substrates is the most likely source of ammonia production in this patient?

A}-Crealinine
B ) Glutamine
C}-Glycine
Q}beucine
E ) Urea
Correct Answer: B.

Production of ammonia, or ammoniagenesis, takes place in the proximal convoluted tubule of the renal nephron. Ammonia, NH 3, is able to accept a proton (H + ) to become ammonium, NH4+, and thus aids in the renal excretion of acid into the urine. Unlike other
compounds, the majority of the ammonia in the nephron lumen comes from renal tubular cell production rather than glomerular filtration. The amino acid glutamine is the primary substrate for the production of ammonia. Glutamine is converted into a-ketoglutarate
and two molecules of ammonium through the sequential enzyme reactions of glutaminase and glutamate dehydrogenase. These ammonium molecules are then secreted into the lumen of the proximal convoluted tubule via a sodium-ammonium exchanger.

There are three main types of renal tubular acidosis (RTA), all of which prevent the kidneys from either absorbing HCo 3- or excreting titratable acid in the form of NH4+. In Type 1, or distal RTA, there is impaired secretion of protons by the alpha intercalated cells of
the collecting tubule. Type 2, or proximal RTA, is caused by impaired HCo3 - resorption in the proximal convoluted tubule. In Type 4 RTA, decreased levels of aldosterone result in decreased NH4+ excretion. Types 1 and 2 are most common in the pediatric
population and can be inherited or acquired. Any cause of decreased ammonia production will compromise the kidney's ability to excrete acid and lead to a metabolic acidosis.

Incorrect Answers: A, C, D, and E.

Creatinine (Choice A) is a byproduct of creatine, which is produced by muscle cells. Creatinine is excreted by the kidney and used clinically as an indicator of renal function, but it is not a substrate for ammoniagenesis.

Glycine (Choice C) and leucine (Choice D) are both amino acids. Neither of their side chains have amine groups, and thus cannot be used as a substrate for ammoniagenesis. Asparagine and glutamine are the two amino acids capable of contributing an amine
group to a metabolic reaction.

Urea (Choice E) is the end product of the urea cycle. This metabolic process primarily takes place in the liver and converts ammonia to urea, which can be safely excreted. Urea is not involved in the production of ammonia.

Educational Objective: Ammoniagenesis takes place in the proximal convoluted tubule using glutamine as the primary substrate. Ammoniagenesis is critical to the process of acid excretion in the urine and is impaired in renal tubular acidosis.

r
Previous
r
Next
,
Score Report

Lab Values
P
Calculator
r
Help
trt
Pause
Exam Sectlon 3: Item 42 of 50 National Boa.rd pf Medical Examiners®
Comprehensive 1:Saslc Sctence Self-Assessment

✓ 42. A 38-year-old man with a 3-year history of type 2 diabetes mellitus comes to the physician for a follow-up examination. In addition to taking an oral antihyperglycemic agent, he has tried contro-lling his condition with diet modifications and exercise. He is
188 cm (6 ft 2 in) iall and weighs 113 kg (250 lb); BMI is 32 kg/m2. Physical examinaiion shows no other abnormaliiies. His hemoglobin A 1 cis 10%. The physician recommends initiation of insulin injeciions io obtain better control over the paiieni's blood
glucose concentration. The patient responds, "I know that insulin would help control my blood sugar. But a lot of people in my family have diabetes, and insulin made them really sick at times.• This patient is most likely at which of the following stages of
change regarding insulin administration?

A ) Precontemplation
B) Contemplation
C) Preparation
D) Action
E ) Maintenance
Correct Answer: 8.

In this scenario, the patient is aware of his poorly controlled blood glucose and insulin's potential benefits. He is considering initiating insulin, but he is indecisive about whether he is ready to take action due to concern about side effects, which is consistent with the
contemplation stage. The stages of behavioral change are used to define a patient's readiness to change a health-related behavior such as treatment adherence, substance use, diet, or exercise halbits. In sequential order, the stages of behavioral change are
precontemplation, contemplation, preparation, action, maintenance, and termination. Physicians aim to move patients through these stages over time with an interview technique called motivational interviewing. Motivational interviewing involves using open-ended,
non-judgmental questions to help the patient explore their reasons for wanting to change or maintain the habit.

Incorrect Answers: A, C, D, and E.

Patients in the precontemplation stage (Choice A) are not interested in changing their habit and may not see the benefit in making a change. This patient is aware of the potential benefit of insulin and is considering starting it, which means he has surpassed the
precontemplation stage.

Patients in the preparation stage (Choice C) have committed to making a change and are ready to discuss strategies and resources to help them make the change. For example, a patient may research information about a low-carbohydrate diet to improve his
eating habits. This patient expresses knowledge of insulin's potential health benefits but is not yet committed to starting this medication.

In the action stage (Choice D), patients start to take steps to change their behavior. For example, a patient may begin a low-carbohydrate diet to improve his unhealthy eating habits.

After making the change, the patient must maintain the change (Choice E) and strategize how to cope with temptation to return to the old habit. For example, a patient may join a support group to assist with maintaining their improved behavior choice.

Educational Objective: The contemplation stage of behavioral change refers to ambivalence about changing a behavior, in which the patient expresses knowledge of the benefit of changing their behavior but is not yet prepared to make plans to change. Motivational
interviewing is a technique used by physicians to move patients through the stages of behavioral change.

r ,,.
Prev!�!!�_!'-IJ!��o_i:e
.
,
Report
,e P
l,�b Y.a!_ue�_C_aJ�!!lato_r
r, "'
_ H!!IP�1,1.se
_ _ _ _ _
Exam Sectlon 3: Item 43 of 50 National Board pf Medical Examiners®
Comprehensive isaslc Science Self-Assessment

✓ 43. A 28-year-old woman comes to the physician because of a 4-day history of palpitations, severe neck pain, fatigue, and malaise. Her pulse is 120/min and regular. Physical examination shows a diffusely tender, mildly enlarged thyroid gland. There is no
exophthalmos. Serum studies show a thyroid-stimulating hormone concentration of 0.01 µU/mL. Which of the following is the most likely diagnosis?

A) Factitious thyrotoxicosis
g� Graves disease
C) Subacute granulomatous thyroiditis
D) Thyroid abscess
E ) Toxic multinodular goiter

Correct Answer: C.

This patient's presenting findings of subacute fatigue, malaise, neck pain, palpitations, and an enlarged and tender thyroid are highly suggestive of subacute granulomatous thyroiditis, also known as subacute (de Quervain) thyroiditis. Subacute granulomatous
thyroiditis is a self-limited inflammatory condition of the thyroid gland that often follows an acute viral illness. Presenting findings are typically suggestive of clinical hyperthyroidism and may include diaphoresis, palpitations, neck pain, dysphagia, fever, tachycardia,
and a decreased serum concentration of thyroid-stimulating hormone (TSH). Patients may eventually become hypothyroid or euthyroid following the acute inflammatory, hyperthyroid phase. Subacute granulomatous thyroiditis is distinguished from other forms of
hyperthyroidism by the presence of a painful, tender thyroid, which is highly suggestive of the diagnosis. It is also characterized by the absence of iodine uptake on a radionuclide scan, and by the presence of multinucleated giant cells on fine needle aspiration.

Incorrect Answers: A, 8, D, and E.

Factitious thyrotoxicosis (Choice A) is due to the deliberate ingestion of excessive quantities of ·thyroid hormone, such as levothyroxine. Factitious thyrotoxicosis can be distinguished from subacute granulomatous thyroiditis and Graves disease by the absence of
suggestive clinical findings, including goiter, thyroid swelling, pain or tenderness, orbital involvement, or pretibial myxedema.

Graves Disease (Choice B) is the most common cause of hyperthyroidism and is caused by an autoantibody that activates TSH receptors on the thyroid. It presents with symptoms of hyperthyroidism, pretibial myxedema, and thyroid ophthalmopathy, which can
cause diplopia, proptosis, and restrictive strabismus. The thyroid is not typically painful or tender.

Thyroid abscess (Choice D) results from a bacterial infection that often begins in the pharynx. There may be associated systemic thyrotoxicosis, as well as a painful, tender thyroid, which may display localized erythema and fluctuance.

Toxic multinodular goiter (Choice E) is also a common cause of hyperthyroidism and is secondary to thyroid hormone-secreting nodules that are active independently of thyroid-stimulating hormone. Patients present with symptoms of hyperthyroidism, but the thyroid
is not typically painful or tender.

Educational Objective: Subacute granulomatous thyroiditis, also known as subacute (de Quervain) thyroiditis, is a self-limited thyroiditis that often follows an acute viral illness and that presents with symptoms of hyperthyroidism. A painful, tender thyroid is highly
suggestive of the diagnosis..

r
Previous
r
Next
,
Score Re ort

Lab Values
F
Calculator
r
Hel
,,.
Pause
Exam Section 3: Item 44 of 50 Natlonal Board pf Medlcal Examiners®
Comprehensive Basic Sctence Sen-Assessment

44. A 42-year-old man comes to the physician because of a 1-week history of a diffuse, painless rash on his back. He first noticed the rash after he spent a week sunbathing at the beach. He says the rash does not itch. A photograph of the rash is shown.
Which of the following is the most likely causal organism?

A) Malassezia furfur
B ) Staphylococcus aureus
C) Streptococcus pyogenes (group A)
D) Treponema pal/idum
E ) Trichophyton rubrum
Correct Answer: A.
nnea versicolor is a superficial infection of the epidermis caused by Ma/assezia furfur. Clinically, it is characterized by hypopigmented and hyperpigmented macules and patches with a fine scale. The infection has a predilection for sites with increased sebum
production such as the chest and back. Patients commonly notice the rash after spending time outdoors, as Malassezia furfur inhibits tanning of the affected areas, making them more noticeable when compared to the unaffected neighboring skin. A potassium
hydroxide preparation of a skin scraping can be helpful and will show fungal spores and short hyphae along the edges of epidermal cells. Tinea versicolor may be treated using topical antifungal preparations including topical ketoconazole or selenium sulfide
preparations.
Incorrect Answers: B, C, D, and E.
Staphylococcus aureus (Choice B) is a common cause of cutaneous infections including impetigo, cellulitis, folliculitis, and furunculosis. It can also produce an exotoxin leading to toxic shock syndrome. Whil!e toxic shock syndrome can cause an erythematous rash,
the patient also classically exhibits fever, vital sign inistability, and signs of end-organ dysfunction related to the underlying systemic shock.
Streptococcus pyogenes (group A) (Choice C) is a Gram-positive bacterium that can result in cellulitis and pharyngitis. Additional conditions related to Streptococcus pyogenes include scarlet fever, necrotizing fasciitis, glomerulonephritis, and rheumatic fever.

rreponema pal/idum (Choice D) is the causative spirochete of syphilis, which presents in multiple stages with varying symptoms, including primary with a painless chancre, secondary with fever, lymphadenopathy, and condylomata lata, and tertiary with tabes
dorsalis, aortitis, and gummas.
T:richophyton rubrum (Choice E) is a dermatophyte (skin fungus) that can cause superficial infection on any part of the skin, but most commonly affects the trunk or extremities (tinea corporis), groin (tinea cruris), foot (tinea pedis), scalp (tinea capitis), and nail (tinea
unguium). Tinea versicolor, in contrast, is caused by the yeast Malassezia furfur.

Educational Objective: Tinea versicolor is a superficial infection of the epidermis caused by Malassezia furfur. Patients develop hypopigmented or hyperpigmented patches on the chest and back, which are more conspicuous after sun exposure.

r r , � F ,.. r
Previous Next Score Report Lab Values Calculator Help Pause
Exam Sectlon 3: Item 45 of 50 National Board pf Medical Examiners®
Comprehensive isaslc Science Self-Assessment

y 45. A 60-year-old woman is receiving cisplatin therapy for advanced transitional cell bladder cancer. She develops paresthesias, and the medication is stopped. The medication likely damaged the largest cells in the region
labeled "B" in the drawing of the spinal cord shown. The damage to these cells would most likely lead to which of the following motor signs?

A ) Babinski sign
B ) Clonus
G}- Fibrillations

Ii� MusGle atrophy


Correct Answer: D.

This patient's dorsal root ganglion cells (labeled "B") have likely been damaged by cisplatin, resulting in hyporeflexia due to decreased sensation. Dorsal root ganglia are present at each spinal cord level and contain the cell bodies of first-order sensory neurons.
Each sensory neuron begins with sensory receptors in the skin, tendon, joint, or muscle. The axon travels medially, with the cell body extending into'the dorsal root ganglion, and the axon continuing on to enter the spinal cord, ultimately synapsing with the second­
order neuron in the gray matter of the spinal cord. Symptoms resulting from dorsal root ganglion damage include paresthesias (as in this patient), numbness (resulting in this patient's hyporeflexia), loss of proprioception, abnormal temperature sensation, and
decreased vibration sensation. Patients with decreased sensation may be unable to sense and therefore respond to a reflex hammer, representing a disruption in the reflex pathway and manifesting as hyporeflexia. Cisplatin and other platinum compounds used
for chemotherapy typically cause symmetrical, predominantly sensory peripheral neuropathy. Treatment is symptomatic, and the neuropathy is sometimes reversible after stopping cisplatin.

Incorrect Answers: A, B, C, and E.

The Babinski sign (Choice A), or extensor plantar response, is an upper motor neuron sign that indicates damage to the corticospinal tract, a descending motor tract: located in the central nervous system. The Babinski sign is present when the lateral part of the
plantar surface of the foot is firmly stroked and the hallux extends (moves rostrally) while the other toes abduct. This patient has peripheral sensory nervous system damage rather than central motor nervous system damage.

Clonus (Choice B) is a rhythmic stretch reflex that occurs with extreme hyperreflexia, representing an upper motor neuron sign. This patient has peripheral sensory nervous system damage rather than central motor nervous system damage.

Fibrillations (Choice C) are spontaneous electropotentials in muscles that result from motor neuron damage. This patient demonstrates sensory nerve damage instead.

Muscle atrophy (Choice E) typically results from peripheral motor nerve damage and consequent chronic denervation of the muscle. This patient demonstrates sensory nerve damage instead.

Educational Objective: Cisplatin chemotherapy can damage dorsal root ganglia. Patients can present with a symmetric peripheral sensory neuropathy and consequent hyporeflexia due to disruption of the reflex pathway.

r
Previous
r
}�ext
,
Score �eport
tr§
Lab Values
F
Calculator
r
Help
,,.
Pause
_ .
Exam Section 3: Item 46 of 50 National Board pf Medical Exi!J'lllners®
Comprehensive 1:Jas1c Science Sert-Assessment

✓ 46. A 35-year-old woman comes to the emergency department 3 hours after the sudden onset of severe right-sided back pain. She has a history of recurrent urinary tract infections. Her most recent infection was 1 month ago; she was successfully treated for
Klebsiel/a pneumoniae with antibiotics. The patient is not taking any medications. She says that she has been eating more fast food and drinking less water during the past month because of a busy schedule. She is sexually active and uses a spermicidal
° °
cream for contraception. Her temperature is 37 C (98.6 F), pulse is 100/min, respirations are 14/min, and blood pressure is 138/66 mm Hg. Physical examination shows marked tenderness to palpation over the right flank. Urinalysis shows:
pH 8
Protein 1+
RBC 30-35/hpf
WBC 20-25/hpf

An excess of which of the following is the most likely underlying cause of this patient's flank pain?

A ) Calcium oxalate
B ) Cystine
C) Magnesium ammonium phosphate
D) Potassium citrate
Ii}- UriGasid
Correct Answer: C.

This patient's presenting findings of acute back pain, tachycardia, hematuria, pyuria, proteinuria, and mild alkalization of urine are suggestive of nephrolithiasis with a magnesium ammonium phosphate calculus. Magnesium ammonium phosphate, also known as
struvite, is a less common cause of nephrolithiasis than calcium oxalate. Struvite calculi occur in the setting of infections caused by urease-positive microbial pathogens, such as Klebs;ella or Proteus. Urease catalyzes the conversion of urinary urea to ammonia
which results in urine alkalinization. The solubility of struvite is decreased under alkaline conditions and precipitates as a crystal. Struvite crystals have an orthorhombic configuration under light microscopy. Struvite calculi are radiopaque and can be potentially
identified by a radiograph or CT scan. Struvite calculi may adopt a branchirng morphology that molds itself to the shape of the collecting system. Given their large size and often ramified structure, it is rare for struvite calculi to pass through the urinary tract and
surgical removal is often necessary, along with treatment of the underlying infection.

Incorrect Answers: A, B, D, and E.

Calcium oxalate (Choice A) is the most common etiology of nephrolithiasis. Citric acid reduces the formation of calcium oxalate crystals by forming soluble complexes with calcium ions, and therefore, calcium oxalate calculi tend to form in the setting of
hypocitraturia. Calcium oxalate crystals have an octahedral morphology under light microscopy and form radiopaque calculi. Patients can be treated with thiazide diuretics, a low-sodium diet, and supplementation with potassium citrate (Choice D), which increases
the urinary concentration of citric acid.

Cystine (Choice B) precipitates as a urinary calculus in the setting of increased urinary cystine concentration in patients with a hereditary defect of the renal proximal convoluted tubule that results in decreased reabsorption of cystine, omithine, lysine, and arginine.
Cystine crystals are hexagonal and form radiolucent calculi. The urinary cyanide-nitroprusside test can be used to confirm the diagnosis.

Uric acid (Choice E) calculi form most commonly in patients with hyperuricemia (eg, gout), in the setting of rapid cellular turnover due to malignancy, or in the setting of dehydration. Uric acid crystals have a rhomboid configuration and form radiolucent calculi. Given
the absence of these risk factors, the patient's history is more suggestive of a struvite calculus.

Educational Objective: Magnesium ammonium phosphate, or struvite, forms renal calculi in the setting of a urinary tract infection with urease-positive organisms, such as Klebsiella or Proteus. Urease catalyzes the conversion of urinary urea to ammonia and carbon
dioxide, which results in urine alkalinization. The solubility of struvite is decreased under alkaline conditions and precipitates as a crystal. Struvite crystals exhibit an octahedral morphology and form calculi that are radiopaque, large, and may be branching. Struvite
calculi often require surgical removal.

r r ,: ta P r tl"-
Prev1ous Next Score Report Lab Values Calculator Help Pause
Exam 5-ectlon 3: Item 47 of 50 National Board pf Medical Examiners®
Comprehensive Basic SClence Serr-Assessment

✓ 47. A 68-year-old woman is brought to the emergency department 3 hours after falling onto her right wrist while working in her yard. Examination of the right wrist shows swelling and ecchymoses. Passive flexion of
the right wrist exacerbates the pain. An x-ray of the right wrist is shown. Which of the following is ·the most likely cause of these findings?

A ) Avascular necrosis of the scaphoid


Bl Av n 1e ulrn1r .;lyloid process
Cl D1 o fi " proximal phalanx
D) Disruption of the radioulnar joint
---------------------------------------------------------------'
E ) Fracture of the distal radius

Correct Answer: E.

The x-ray demonstrates a fracture of the distal radius. In this fracture, the carpal bones remain attacheol to the distal radius fragment by the radioscaphocapitate ligament and accessory carpal ligaments that maintain position of the carpal bones on the distal
radius. Generally, there are two common fracture patterns involving the distal radius. A volar displacement of the distal fragment is termed a Smith fracture, which occurs when axially loading a flexed wrist, while a Colles fracture is more common and results from
axially loading the wrist in extension. Colles fractures exhibit a dorsally displaced distal radius fragment. Both fracture patterns are common in older persons, especially those predisposed to osteoporosis {eg, women) as in this patient. Classic presentations
include pain in the affected wrist, swelling, ecchymosis, and limited passive and active range of motion. These fractures can cause acute median nerve entrapment. In the long-term, abrasion, attenuation, and rupture of extensor tendons that glide over the
fracture site may occur.

Incorrect Answers: A, B, C, and D.

Avascular necrosis of the scaphoid {Choice A) can occur in the setting of a displaced scaphoid fracture that fails to heal property. The blood supply for the scaphoid is oriented in a distal to proximal direction. If the fracture extends through the scaphoid such that
the proximal pole is separate from the distal pole, the proximal pole will not have sufficient blood supply and may subsequently necrose.

Avulsion of the ulnar styloid process (Choice B) can sometimes occur along with distal radius fractures. This injury usually does not cause secondary complications; however, it can be associated with disruption of the ligaments that form the distal radioulnar joint.

Dislocation of the first proximal phalanx (Choice C) is a common injury of the thumb. The joint between the proximal phalanx and the metacarpal of the thumb includes a sesamoid bone that can be mistaken for a fracture fragment.
Disruption of the radioulnar joint (Choice D) can occur in forearm injuries. It is represented on radiographs by widening of the space between the distal radius and the distal ulna. This joint is made up of the dorsal and volar radioulnar ligaments. These ligaments
attach to the ulnar styloid process and the margin of the radial articular surface contributing to stability of the wrist.

Educational Objective: Distal radius fractures are a common injury particularly in patients with osteoporosis. Smith fractures {volar displacement) and Colles fractures (dorsal displacement) are common fracture patterns. Complications include acute carpal tunnel
syndrome and delayed tendon rupture.
Exam Section 3: Item 48 of SO National Board pf M· 11dlcal Exftn:llners®
Comprehensive Da&lc: Sc:tenee serr-Alisessment

y 48. A 56-year-old man with esophageal varices who is awaiting liver transplantation undergoes placement of a transjugular portosystemic shunt During the procedure, a catheter with a stent is threaded through the right internal jugular vein and down through
the vena cava to the liver. To relieve the portal hypertension via this route, the stent should be placed between a branch of the portal vein and which of the following additional veins?

A ) Esophageal

• C ) Left gastric
1:>)- SpleAiS
Ii t S1iperior meseAteris
Correct Answer: B.

Portal hypertension (PH) is often a consequence of liver cirrhosis but may also occur in patients with schistosomiasis or portal vein thrombosis. The portal venous system describes a series of interconnected veins that drain blood from the colon, small intestines,
spleen, liver, and stomach. While this blood eventually makes its way to the systemic circulation via the inferior vena cava, it must first pass through the liver. In cirrhosis, obliteration of the hepatic sinusoids through progressive fibrosis increases the resistance to
blood flow through the liver, which is transmitted to the portal venous system causing PH. Anatomically, the portal vein starts at the confluence of the left gastric, splenic, and superior mesenteric veins. Manifestations of PH include esophageal varices, gastric
varices, caput medusae, hemorrhoids, splenomegaly, and ascites. The hepatic veins are not part of the portal venous system and act to drain blood from the liver into the inferior vena cava. In patients with cirrhosis who have ascites that is refractory to diuretic
therapy or life-limiting complications of PH such as bleeding varices, the placement of a transjugular intrahepatic portosystemic shunt (TIPS) is an option for relieving portal hypertension. In this procedure, a stent is placed between the portal vein and the hepatic
vein, effectively allowing blood to bypass the liver. One common side effect of TIPS is more frequent and severe episodes of hepatic encephalopathy, which is a drawback to this procedure.

Incorrect Answers: A, C, D, and E.

Esophageal veins (Choice A), if inferior, drain into the left gastric vein and then into the portal vein. They may become engorged in portal hypertension resulting in esophageal varices (EVs). These are usually treated endoscopically by band ligation. Patients with
cirrhosis must be screened for EVs, as the risk of life-threatening upper gastrointestinal bleeding is high.

The left gastric vein (Choice C) drains into the portal vein. Transmission of portal pressure through this vein leads to esophageal and gastric varices.

The splenic vein (Choice D) drains into the portal vein. In cases of splenic vein occlusion, stents may be placed in the vein, but this does not relieve PH.

The superior mesenteric vein (Choice E) drains blood from the small intestines into the portal vein. Stents are not placed here to relieve PH.

Educational Objective: Portal hypertension (PH) is a frequent complication of cirrhosis. Placement of a transjugular intrahepatic portosystemic shunt (TIPS) between the portal vein and the hepatic vein reliably relieves increased portal pressures but predisposes to
more frequent bouts of hepatic encephalopathy.

" ,. , @ F' � If'


Previous Next Score Report Lab Values Calculator Help Pause
Exam Section 3: Item 49 of 50 National Board pf Medical Examiners®
Comprehensive Basic Science Sen-Assessment

✓ 49. A 56-year-old man comes to the physician because of a 3-month history of intermittent regurgitation of material with an acidic taste. He is unable to sleep and has missed 3 days of work during the past month because of the symptom. Various non­
pharmacologic measures, including elevating the head of his bed and decreasing his dietary fat intake and portion size, have resulted in only mild improvement. Physical examination shows no abnormalities. Esophageal endoscopy shows severe
erosions; there are no other structural abnormalities. The most appropriate pharmacotherapy for this patient is a drug with which of the following mechanisms of action on the parietal cells?

A ) Agonism of cholecystokinin B (CCKz) receptor


B) Agonism of prostaglandin E receptor 3 (EP3) receptor
C ) Antagonism of histamine-2 (Hz) receptor
0) Antagonism of muscarinic (M 3) acetylcholine receptor

----------
E ) Irreversible inhibition of H +-1< + ATPase

Correct Answer: E.

Irreversible inhibition of H +-1< + ATPase is the mechanism by which proton-pump inhibitors (PPls) exert their effect on gastric acid production. Acid reflux in the esophagus causes mucosa! irritation and inflammation, which can present as mucosa! erythema and
multiple erosions (reflux esophagitis). The entire clinical spectrum of disease is known as gastroesophageal reflux disease (GERO). Over time, if GERO remains untreated, metaplasia can occur leading to the development of Barrett esophagus, a state of pre­
malignancy marked by intestinal metaplasia of the distal esophagus. Untreated, this can lead to esophageal adenocarcinoma. Parietal cells are the acid secreting cells of the stomach and are found primarily in the gastric body. These cells utilize water and carbon
dioxide to create carbonic acid, which dissociates into bicarbonate and hydrogen ions. Bicarbonate is exchanged with chloride and enters the blood stream while hydrogen ions are secreted into the gastric lumen in exchange for potassium via the H+-K+ ATPase
pump. Chloride is also secreted into the lumen. The accumulation of hydrogen ions results in an osmotic gradient that pulls water into the lumen to help make acidic gastric chyme. PPls irreversibly inhibit the H+-K + ATPase in gastric parietal cells and thereby limit
the secretion of hydrogen ions, which results in an increased gastric pH, The reduction of stomach acid production allows for the neutralization of stomach contents, limiting the inflammatory and erosive sequelae of GERO in the distal esophagus, and allowing for
the appropriate healing of previous esophagitis and/or Barrett esophagus.

Incorrect Answers: A, B, C, and 0.

+ +
Agonism of cholecystokinin B (CCK2) receptor (Choice A) is normally achieved by the action of gastrin from G cells and CCK from the duodenum. Stimulation of this receptor leads to activation of the H -K ATPase pump and increased gastric acid secretion.

Agonism of prostaglandin E receptor 3 (EP3) receptor (Choice B) inhibits the H+-K+ ATPase pump and therefore decreases gastric acid secretion, and while there are medications in development to target this pathway, they are not currently in clinical use. 0111 the
contrary, non-steroidal anti-inflammatory drugs (NSAIOs) inhibit COX-1 and COX-2, thereby reducing the amount of prostaglandin available to activate the receptor which can have a deleterious effect on the stomach and esophageal mucosa.

Antagonism of histamine-2 (H2) receptors (Choice C) describes the mechanism of action of the H2-receptor antagonist class of medication such as cimetidine and famotidine. H 2 receptors on the surface of parietal cells are activated by histamine released from
e111terochromaffin-like cells and work to augment the action of the H+/� ATPase pump. H2-receptor antagonists block this signaling pathway and reduce acid secretion, but not directly. They are considered second line medications in GERO and esophagitis.

Antagonism of muscarinic (M3 ) acetylcholine receptors (Choice 0) is achieved by drugs that block the action of acetylcholine, such as atropine. Parasympathetic stimulation via action of the vagal nerve acts on muscarinic receptors, which in turn increases the
activity of the H+-K+ ATPase pump through the IP3/Ca2+ signaling pathway. Blocking this pathway reduces acid secretion, but not directly, and atropine is not used readily for this purpose in clinical practice.

Educational Objective: PPls reduce acid secretion by irreversibly binding to the H +-K + ATPase pump. They rapidly and effectively increase gastric pH and are first-line therapy for GERO and erosive esophagitis.

r r , irE F' ,,. �


Previous Next Score Report Lab Values Calculator Help Pause
Exam Section 3: Item 50 of 50 National Board pf Mqdlcal Examiners®
Comprehensive Basic Science Self-Assessment

✓ 50. A 46-year-old woman comes to the office because of a 3-month history of fever, malaise, and easy bruising. Her mother and maternal grandmother died of breast cancer at the ages of 49 and 52 years, respectively. Her temperature is 38° C (100.4 °F),
pulse is 115/min, respirations are 16/min, and blood pressure is 125/90 mm Hg. Physical examination shows numerous large ecchymoses over the trunk and lower extremities. Laboratory studies show an increased leukocyte count with immature
segmented neutrophi.ls. Genetic testing shows the presence of the Philadelphia (Ph 1) chromosome. Which of the following best explains this patient's current condition?

A}-ABL. activatigngfthe p23 tymgr Sllppressgr gene


B ) Fusion protein influence on the cell cycle
C}- Pgint mYtatign in the .fl.Sb gncggene
O� Pgint mYtatign intheBCR gene
E ) Underexpression of the p53 tumor suppressor gene
Correct Answer: B.

Fusion protein influence on the cell cycle accounts for this patient's presentation, which is consistent with chronic myelogenous leukemia (CML). The Philadelphia (Ph) chromosome is created by a translocation between chromosomes 9 and 22, leading to
constitutive activation of the ABL 1 tyrosine kinase. Activation of the ABL 1 tyrosine kinase leads to subsequent activation of the JAK/STAT and Ras/MAPK/ERK pathways, with the downstream consequences of increased cellular proliferation, inhibition of normal
checkpoint inhibition, and resistance to apoptosis. While the Ph chromosome is required for the diagnosis of CML, it can also be found in acute lymphoblastic leukemia (ALL) and acute myelocytic leukemia (AML). Typical laboratory findings in CML include
hyperleukocytosis, with an increase in nearly all of the cell lines. Basophilia and eosinophilia are particularly characteristic. Leukostasis, a condition in which hyperleukocytosis leads to organ damage from occlusion of capillaries by malignant, non-distensible cells,
is uncommon in CML because malignant cells are differentiated and pliable. This is in contrast to conditions such as AML and ALL in which circulating blasts, which are large and not particularly pliable, easily lodge in capillaries. Treatment of CML is with tyrosine
kinase inhibitors such as imatinib or dasatinib. They are remarkably effective and often result in complete molecular response within several months of initiation.

Incorrect Answers: A, C, D, and E.

ABL activation of the p23 tumor suppressor gene (Choice A) is not the defining feature of the Philadelphia chromosome. Activation of a tumor suppressor gene would result in inhibition of tumor growth, as opposed to cellular proliferation.

Point mutation in the ABL oncogene (Choice C) does not define the Philadelphia chromosome. In CML, the ABL oncogene is constitutively activated by the translocation of ABL next to the BCR gene. Point mutations do not play a role.

Point mutation in the BCR gene (Choice D) does not occur in the Philadelphia chromosome. In CML, the BCR gene is a wild-type gene that, when placed next to the ABL1 tyrosine kinase gene, leads to constitutive activation of ABL 1.

Underexpression of the p53 tumor suppressor gene (Choice E) is a common finding in many solid tumors and is not specific to any particular malignancy.

Educational Objective: CML is defined by the presence of the Philadelphia chromosome, which occurs secondary to a translocation of chromosomes 9 and 22, placing the tyrosine kinase ABL 1 next to BCR, leading to constitutive activation of ABL 1. This results in
activation of multiple downstream pathways and causes unregulated cellular proliferation of white blood cells.

r
Previous
r
Next
,-
Score Report
tr§
Lab Values
F
Calculator
r
Help
,,.
Pause
Exam Section 4: Item 1 of 50 National Boa.rd pf M11dlcal Examiners®
Comprehensive 8aslc Science Serr-Assessment

)( 1. A 40-year-old woman has easy bruising and menorrhagia. Platelet count is 100,000/mm 3, and bleeding time is prolonged. Factor Vlll (antihemophilic factor) concentration is decreased, and platelet adhesion to collagen is abnormal. Which of the following is
the most likely diagnosis?

• A ) Hemophilia A
B ) Hemophilia B
C) Immune thrombocytopenic purpura
D) Ingestion of aspirin
E ) Systemic lupus erythematosus
F ) Thrombotic thrombocytopenic purpura
G) von Willebrand disease
Correct Answer: G.

von Willebrand disease is one of the most common hereditary bleeding disorders and is due to quantitative or qualitative abnormality of von Willebrand factor (vWF), which binds platelets and subendothelial collagen in primary hemostasis. Impaired platelet
adhesion to the subendothelial lining, mediated by vWF, leads to a prolonged bleeding time. von Willebrand factor also transports factor VIII in plasma, which degrades rapidly when unbound. Factor VIII is a critical component of the intrinsic coagulation pathway
and decreased levels lead to a prolonged partial thromboplastin time (PTT). von Willebrand disease is inherited in an autosomal dominant pattern. It can present with epistaxis, gingival bleeding, petechiae, easy bruising, and menorrhagia. Additional diagnostic
testing can be performed with the ristocetin cofactor assay, which requires functional vWF for platelet aggregation to occur. Desmopressin can be used for treatment by promoting release of additional vWF stored in endothelial cells.

Incorrect Answers: A, B, C, D, E, and F.

Hemophilia A (Choice A) and Hemophilia B (Choice B), X-linked coagulopathies, arise due to a genetic deficiency in factor Vlll or IX synthesis or activity, respectively. Patients often present with easy bruising and hemorrhage involving muscles and joints.
Hemophilia does not cause platelet dysfunction and would not cause an increased bleeding time.

Immune thrombocytopenic purpura (Choice C) is an acquired disorder of immune-mediated platelet destruction. It classically presents with petechiae and purpura, and prolonged bleeding time on laboratory analysis. Platelet levels are typically less than
100,000/mm3 . It does not affect factor VIII.

Ingestion of aspirin (Choice D) prolongs bleeding time by irreversibly inhibiting platelet activation through covalent binding to the cyclooxygenase enzyme. Aspirin does not affect factor VIII levels.

Systemic lupus erythematosus (SLE) (Choice E) is an autoimmune disorder with a wide range of presentations. The most common cause of thrombocytopenia in patients with SLE is immune thrombocytopenic purpura. There may also be a concomitant
autoimmune hemolytic anemia.

Thrombotic thrombocytopenic purpura (Choice F) is a thrombotic microangiopathy due to inhibition or deficiency of ADAMTS13, which is a metalloprotease that degrades vWF. Impaired ADAMTS13 activity leads to the accumulation of large vWF multimers and
increased platelet adhesion/activation, forming platelet microthrombi. The intrinsic coagulation pathway is not activated.

Educational Objective: von Willebrand disease is a hereditary bleeding disorder affecting both platelet adhesion and intrinsic coagulation pathway function. Factor VIII levels are often decreased as vWF binds and stabilizes it in the plasma.

r r , rcE P r- If"
Previous Next Score Report Lab Values Calculator Help Pause
Exam Secllon 4: Item 2 of 50 Nallonal Board pf Medical Examiners®
Comprehensive Basic Science Serr-Assessment

✓ 2. A 14-year-old boy is brought to the physician's office because of decreased appetite and abdominal pain over the past 3 weeks. His mother says that during the same time he has withdrawn from everyone and sleeps constantly. He describes his symptoms
vaguely. Physical examination is normal. During further history-taking, it is most critical for the physician to obtain information about which of the following?

A ) Developmental history
B ) Family history of affective disorders
C) Orientation to time, place, and person
D) School history
E ) Suicidal ideation or attempts

Correct Answer: E.

The physician should conduct a thorough suicide risk assessment including questions about suicidal ideation or attempts in this adolescent patient demonstrating neurovegetative symptoms concerning for depression. Symptoms of major depressive disorder (MOD)
typically include 2 or more weeks of depressed mood, anhedonia, guilt or worthlessness, difficulty concentrating, suicidal thoughts, and/or neurovegetative symptoms (decreased energy, sleep disturbance, appetite disturbance). These symptoms disrupt everyday
functioning. Compared to adults, children are more likely to present with vague somatic symptoms such as abdominal pain (as in this patient) and irritable mood rather than depressed mood. Suicide is the most serious and acute risk of patients with MOD, so
physicians should ask detailed questions about patients' suicidal ideation and recent or remote suicide attempts.

Incorrect Answers: A, B, C, and D.

Developmental history (Choice A), family history of affective disorders (Choice B), and school history (Choice D) are important parts of a thorough psychiatric history in children but are not as important as doing a thorough suicide risk assessment. Some behavioral
disorders can be related to developmental delays (as revealed by a developmental history). Gathering a family history of affective disorders can assist in assessing a patient's risk of developing an affective disorder and guide treatment decisions (eg, choosing an
antidepressant that was effective in the patient's mother). Obtaining a school history can inform the physician about the child's intellectual and social functioning and screen for mental disorders such as intellectual disability and attention-deficit/hyperactivity disorder.
None of these histories directly influence a patient's acute suicide risk.

Orientation to time, place, and person (Choice C) would likely be normal and therefore unhelpful to evaluate in this patient. This patient is young with a normal physical examination, making dementia or delirium unlikely. His social withdrawal implies that he was
�reviously socially functioning, so it is unlikely he has a severe developmental delay that would cause this degree of cognitive impairment. Depression is also unlikely to cause cognitive impairment this severe.

Educational Objective: Major depressive disorder in children and adolescents can manifest with vague somatic symptoms in conjunction with decreased energy, increased or decreased sleep, and increased or decreased appetite. Suicide is the most acute and
serious risk of major depressive disorder and should be assessed regularly in patients with suspected major depressive disorder.

r
Prev1ous
r
Next
,
Score Report
� P
Lab Values Calculator
r
Help
r-
Pause
_
Exam Sectlon 4: Item 3 of 50 National Board pf Mf!dlcal Examiners®
Comprehensive 15as1c science SelT•Assessment

y 3. Which of the following best explains why deoxygenated blood can carry more carbon dioxide for a given Pco 2 than oxygenated blood?

A ) Deoxyhemoglobin does not bind to 2,3-bisphosphoglycerate as efficiently as oxyhemoglobin


B ) Oeoxyhemoglobin has a lower capacity to form carbamino compounds than oxyhemoglobin
C) Deoxyhemoglobin has a lower pKa than oxyhemoglobin
D) Oeoxyhemoglobin is a better buffer of hydrogen ions than oxyhemoglobin
E ) Oxygen and carbon dioxide compete for the same binding site in hemoglobin
F) Oxyhemoglobin binds nitric oxide with a higher affinity than deoxyhemoglobin
Correct Answer: D.

Oeoxyhemoglobin is a better buffer of hydrogen ions than oxyhemoglobin, which explains why deoxygenated blood can carry more carbon dioxide for a given Pco2 than oxygenated blood. This is known as the Haldane effect. The hemoglobin tetramer exists in two
primary conformations, the taut form, which has a reduced affinity for oxygen, and the relaxed form, which has an increased affinity for oxygen. As hemoglobin molecules accept oxygen from the lungs and deliver it to tissues, they undergo repeated conformational
change between the taut (when unloaded) and relaxed (when loaded) form. At the level of the alveoli, there is a high partial pressure of oxygen that favors the loading of oxygen onto hemoglobin and unloading of carbon dioxide, while in the periphery there is a
lower partial pressure of oxygen that favors the unloading of oxygen from hemoglobin. Carbon dioxide produced by cellular respiration must be transported back to the alveoli for removal. It can travel in the blood dissolved in plasma (-10%), bound to NH2 groups
on hemoglobin (-10%), or as bicarbonate (-80%). When hemoglobin unloads oxygen in the tissues and undergoes conformational change, it is more capable of accepting hydrogen ions. The cytosolic increase in pH from the removal of hydrogen ions that are now
bound to hemoglobin favors the formation of bicarbonate from carbon dioxide and water. Bicarbonate is a highly soluble form of carbon dioxide and therefore, in the setting of deoxyhemoglobin, more carbon dioxide can be carried in the blood for any given Pco2.

Incorrect Answers: A, B, C, E, and F.

Oeoxyhemoglobin does not bind to 2,3-bisphosphoglycerate (2,3-BPG) as efficiently as oxyhemoglobin (Choice A) is incorrect. 2,3-BPG binds to deoxyhemoglobin with high affinity and promotes further oxygen unloading in the tissues.

Oeoxyhemoglobin has a lower capacity to form carbamino compounds than oxyhemoglobin (Choice B) is incorrect, as deoxyhemoglobin has a higher affinity for carbon dioxide, which facilitates the removal of carbon dioxide from peripheral tissues.

Oeoxyhemoglobin has a lower pKa than oxyhemoglobin (Choice C) is incorrect because the taut form of hemoglobin, which is deoxygenated, has a higher pKa than oxyhemoglobin.

Oxygen and carbon dioxide compete for the same binding site in hemoglobin (Choice E) is not true. They bind at different sites.

Oxyhemoglobin binds nitric oxide (NO) with a higher affinity than deoxyhemoglobin (Choice F) is true but is not the correct answer, as it does not account for the differences in carbon dioxide concentrations between oxyhemoglobin and deoxyhemoglobin.

Educational Objective: More carbon dioxide is carried in the blood at any given Pco2 in the presence of deoxyhemogtobin because the taut form (unloaded) of hemoglobin is an excellent buffer of hydrogen ions, resulting in a higher concentration of bicarbonate, the
soluble transport form of carbon dioxide.

r
Previous
r
}�ext
,-
Score �eport

Lab Values
F
Calculator
r
Help
If>
Pause
_
Exam Section 4: Item 4 of 50 National Board pf Mqdlcal Examiners®
Comprehensive Basic Sctence Self-Assessment

✓ 4. A 55-year-old man has a cough, hemoptysis, and dyspnea. He has smoked 2 packs of cigarettes daily for the past 30 years. Examination shows bilateral conjunctiva! edema, distention of sublingual veins, and edema of the upper extremities. Which of the
following additional findings is most likely to be present on examination?

r
A ) Bisferiens carotid pulsation
B ) Increased jugular venous pressure
C) Pansystolic apical murmur
D) S3
E ) S4

Correct Answer: B.

The patient is presenting with physical examination findings suggestive of superior vena cava (SVC) syndrome, which is caused by obstruction of the SVC and subsequent venous congestion of the head, neck, and upper extremities. Obstruction of the SVC is often
due to extrinsic compression by a malignant mediastinal mass, especially lung cancer, non-Hodgkin lymphoma, and metastatic disease. Nonmalignant etiologies include venous thrombus formation, which may be more prevalent in patients with indwelling
intravascular devices (eg, central venous catheters, PICC lines, port systems, pacemakers, defibrillator leads). Physical examination findings include swelling of the face and neck, increased jugular venous pressure, upper extremity swelling, distended chest vein
collaterals, and conjunctiva! edema. Treatment depends on the underlying etiology. Options for bypassing the obstruction include endovascular recanalization and open surgical repair.

Incorrect Answers: A, C, D, and E.

Bisferiens carotid pulsation (Choice A) is associated with aortic regurgitation, combined aortic regurgitation and stenosis, and less commonly hypertrophic cardiomyopathy. The arterial waveform has a double systolic peak, with an initial sharp, shortened peak
followed by a lower amplitude, broader peak. The examination findings in this case are more suggestive of SVC syndrome.

A pansystolic apical murmur (Choice C) is most characteristic of mitral regurgitation, which is best heard in the left fourth or fifth intercostal space along the midclavicular line and radiates to the left axilla. It is commonly associated with mitral valve prolapse and prior
myocardial infarction. SVC syndrome does not cause mitral regurgitation.

s3 gallop (Choice D) is a heart sound that occurs in early diastole and is associated with increased ventricular filling pressures. It can typically be appreciated in mitral regurgitation, congestive heart failure, and dilated cardiomyopathy.
S4 gallop (Choice E) is a heart sound that occurs in late diastole during atrial contraction. It is associated with increased atrial pressure and ventricular hypertrophy as the left atrium attempts to overcome a stiff and noncompliant left ventricle. It is not typically
associated with SVC syndrome.

Educational Objective: Obstruction of the superior vena cava results in superior vena cava syndrome, which is characterized by venous congestion of the head, neck, and upper extremities. Physical examination findings reflect increased venous pressure in these
regions.

r
Previous
r
Next
,
Score Report

Lab Values
F
Calculator
r
Help
,,.
Pause
Exam Sectlon 4: Item 5 of 50 National Board pf M11dlcal Examiners®
Comprehensive i,aa1c: sc:ien" SelT•A&&eaament

y 5. A 57-year old man has a hemoglobin concentration of 18.5 g/dl. A peripheral blood smear is shown. These findings are most consistent with which of the following disorders?

A ) Agnogenic myeloid metaplasia


B) Chronic obstructive pulmonary disease
C) Hereditary hemochromatosis
D) Hypersplenism
• E ) Myelodysplastic syndrome

Correct Answer: B.

Chronic obstructive pulmonary disease (COPD) is an obstructive lung disease characterized by decreased lung function from a combination of features of chronic bronchitis and/or emphysema, resulting in airway obstruction on expiration. COPD commonly
presents with chronic hypoxemia, which triggers increased erythropoietin (EPO) production by the kidneys. EPO induces the proliferation and maturation of erythroid progenitor cells in the bone marrow and results in secondary polycythemia. Polycythemia refers
to an increased absolute red blood cell mass in the body. It is reflected by an increased hemoglobin concentration and an increased density of normochromic, normocytic erythrocytes oo peripheral blood smear. Other common causes of secondary polycythemia
are obstructive sleep apnea, obesity hypoventilation syndrome, living at high altitude, and performance-enhancing drugs such as exogeneous erythropoietin, testosterone, and anabolic steroids.

Incorrect Answers: A, C, D, and E.

Agnogenic myeloid metaplasia (Choice A), also known as primary myelofibrosis, is a chronic myeloproliferative disorder characteri,zed by the progressive fibrosis and ablation of the bone marrow. It is commonly associated with JAK2 mutations. Peripheral blood
smear typically reveals leukoerythroblastosis (immature myeloid cells and nucleated red blood cells) along with teardrop-shaped erythrocytes.

Hereditary hemochromatosis (Choice C) is an inherited disorder characteri'2ed by the abnormal accumulation of iron in the body. It typically presents with liver failure, diabetes, arthritis, heart failure, darkening of the skin, and gonadal atrophy. Diagnostic studies
may include liver biopsy, which commonly demonstrates excess iron deposition seen in hepatocytes on Prussian blue stain.

Hypersplenism (Choice D) is defined as the presence of one or more cytopenias in the setting of splenomegaly. Peripheral blood smear may reveal leukopenia, anemia, thrombocytopenia, or a combination of the three.

Myelodysplastic syndrome (Choice E) refers to a heterogeneous group of hematologic neoplasms in which a malignant, pluripotent progenitor cell leads to progressive dysplasia of the bone marrow and cytopenia in one or more cell lines. Peripheral blood smear
may show leukopenia, anemia, thrombocytopenia, or a combination, as well as immature cell forms (blasts) dependent on the specific type of myelodysplastic syndrome. Some types carry a risk of transformation into acute myeloid leukemia.

Educational Objective: COPD is a common cause of secondary polycythemia resulting from chronic hypoxemia. Polycythemia presents with increased hemoglobin concentration and an increased density of normochromic, normocytic erythrocytes on peripheral
blood smear.

r
Previous
r
Next
,
Score Report

Lab Values
P
Calculator
r
Help
trt
Pause
Exam Section 4: Item 6 of 5D National Board pf Medical Exilllllners®
Comprehem,lve t1a&1c sc1enee serr•Al!&e&&ment

✓ 6. One hundred patients discharged from the hospital with a diagnosis of proximal deep venous thrombosis were followed for the development of venous ulceration for up to 8 years (DVT group). They were compared to a group of 200 patients who were
discharged from the hospital with a diagnosis of pneumonia (the control group). At the end of 5 years, 25 patients in the DVT group and 10 patients in the control group developed venous ulceration. Which of the following is the attributable (excess) risk per
one hundred patients for development of venous ulceration following deep venous thrombosis over 5 years?

A) 1
B) 2
C) 2.4
D) 2.5
E) 10
__F...,) 20
G) 24
H) 25
Correct Answer: F.

Attributable risk (AR) describes the risk of developing disease that can be attributed to the exposure as compared to the risk that exists without exposure. AR is calculated as the difference between disease incidence among those with the exposure and those
without the exposure. In biostatistics, the lowercase letters a, b, c, and d are frequently used to represent numbers of patients exposed to a risk, receiving an intervention, or having a disease. The number of persons having the exposure or receiving the intervention
who have the disease or condition in question is represented by •a•. The number of persons having the exposure or receiving the intervention who do not have the disease or condition in question is represented by "b". The number of persons who do not have the
exposure or who do not receive the intervention in question and who do have the disease or condition is represented by "c". The number of persons who do not have the exposure or who do not receive the intervention in question and who do not have the disease
is represented by "d". By this convention, the incidence of those with the exposure is computed by determining the number of pati.ents with a positive exposure and who developed disease (a) and dividing it by the total number of patients who were exposed (a+ b).
The incidence of those without the exposure who developed disease is computed by determining the number of patients without a positive exposure who developed disease (c) and dividing it by the total number of patients who were not exposed (c +d). AR is
defined as AR= (a/ (a+ b))-(c/ (c+ d)). In this scenario, AR= (25/ 100)- (10/ 200)= (25/ 100)-(5/ 100)= 20 per 100 patients.

Incorrect Answers: A, B, C, D, E, G, and H.


1 (Choice A) and 10 (Choice E), represent the incidence, in the study period, of ulcerations in the control group per 20, and per 200 patients, respectively. The incidence rate equals the number of new cases divided by the number of persons at risk.

2 (Choice B) represents the attributab(e risk per 10 patients, not per 100 patients.

2.4 (Choice C) and 24 (Choice G) are not easily computed from the numbers available.

2.5 (Choice D) represents the ratio of DVT to control group patients who developed ulceration. The entire sample should be considered, not just those in the control group. Additionally, the control group contained twice as many patients as the DVT group, for which
data should be adjusted.
25 (Choice H) represents the number of affected persons in the exposure group (DVT group) and reflects the incidence over the study period per 100 patients.

Educational Objective: Attributable risk (AR) describes the risk of developing disease that can be attributed to the exposure as compared to the risk that exists without exposure. AR is calculated as the difference between disease incidence among those with the
exposure and those without the exposure.

r
Previous
r
Next
,
Score Report

Lab Values
r-
Calculator
"
Help
r
Pause
Exam Section 4: Item 7 of 50 National Boa.rd of Medical Examiners®
Comprehensive 1:1as1c Science Serr-Assessment

✓ 7. An 80-year-old man comes to the physician because of bleeding gums and bruises under his skin. He lives by himself and follows a diet low in vegetables and fruits. His prothrombin time is normal. Which of the following enzymes is most likely
compromised in this clinical scenario?

A ) 2,3-Epoxide reductase
8 ) Glucose 6-phosphate dehydrogenase
C) Hexokinase
D) Methionine synthase
E ) Prolyl hydroxylase
F ) Pyruvate carboxylase
G) Pyruvate dehydrogenase
Correct Answer: E.

Vitamin C (ascorbic acid) is an antioxidant, facilitator of iron absorption, and coenzyme in the synthesis of collagen via prolyl hydroxylase and neurotransmitters via dopamine hydroxylase. Deficiency in vitamin C leads to scurvy, which presents with signs and
symptoms of impaired collagen synthesis including swollen, bleeding gums, easy bruising and bleeding (eg, hemarthrosis), petechiae, impaired wound healing, and short, fragile, curly hair. It should not be confused with hemophilia, as it does not result in a
deficiency of factors. Instead, the collagen and connective tissue deficiency weakens blood vessel walls resulting in easy bruising and bleeding. Collagen is synthesized by fibroblasts and begins in the rough endoplasmic reticulum with translation of collagen chains,
which are glycine and proline rich. Prolyl hydroxylase, in a reaction requiring vitamin C, hydroxylates proline and lysine residues. This step, along with glycosylation, forms alpha-chains through disulfide bridging plus hydrogen bonding. Procollagen is exocytosed,
where it forms tropocollagen after removal of the terminal ends. It is cross-linked extracellularly in a reaction that requires copper. In this case, the patient's bleeding gums, bruising, and diet limited in vitamin C plus normal prothrombin time suggests a diagnosis of
scurvy due to impaired collagen synthesis.

Incorrect Answers: A, 8, C, D, F, and G.

2,3-epoxide reductase (Choice A) is an enzyme involved in hepatic synthesis of clotting factors and involves vitamin K as a coenzyme. This patient's prothrombin time is normal, suggesting appropriate hepatic synthesis of coagulation factors and sufficient vitamin
K.
Glucose-6-phosphate dehydrogenase (Choice B) is an enzyme involved in the metabolism of nicotinamide adenine dinucleotide phosphate. In deficiency, it presents with hemolytic anemia in response to oxidant stressors. Deficiency is generally inherited.

Hexokinase (Choice C) is an enzyme that catalyzes the addition of phosphate to six-carbon sugars (eg, glucose). It requires adenosine triphosphate (ATP).

Methionine synthase (Choice D) catalyzes the creation of methionine from homocysteine in a vitamin 8 12 (cobalamin}-dependent reaction. Vitamin 8 12 (cobalamin) deficiency is common in vegans and patients with malabsorption (eg, sprue), pernicious anemia,
and Crohn disease. Vitamin 8 12 (cobalamin) deficiency presents with megaloblastic anemia and subacute combined degeneration of the spinal cord.

Pyruvate carboxylase (Choice F) converts pyruvate to oxaloacetate in an ATP-dependent reaction. It requires biotin (vitamin 87). Symptoms include dermatitis and enteritis, not demonstrated by this patient.

Pyruvate dehydrogenase (Choice G) is a multi-subunit enzyme complex that converts pyruvate to acetyl-CoA. It requires vitamin 81 (thiamine) among other 8 vitamins for catalysis.

Educational Objective: Vitamin C (ascorbic acid) is an antioxidant and coenzyme in the synthesis of collagen via prolyl hydroxylase. Deficiency is common in persons having diets poor in fruits and vegetables, and results in scurvy, which presents with blood vessel
fragility (easy bruising, petechiae), impaired wound healing, and disordered hair growth.

r
Previous
r
Next
,
Score Report

Lab Values
P
Calculator
r-
Help
r
Pause .
Exam Section 4: Item 8 of 50 National Board of Mqdlcal Examiners®
Comprehensive aas1c Sctence Serr-Assessment

✓ 8. A 40-year-old woman comes to the physician because of a 2-month history of increasing redness around her nose and cheeks, and pimples around her mouth. She appears very upset and tells the physician, "I look like a teenaged Santa Claus." Which of
the following initial responses by the physician is most appropriate?

A ) "Can you tell me how often you go out in the sun and what kind of protection you use?"
S}- "Don't worry.. Let me assw:e you this is notaser:iOYs condition."
C "It's upsetting when ou have a skin roblem on our face."
Q}- "Maybealittle. At least you are keeping your senseof humor about the situation."
�t- "You really do look rather odd. I can understand v,•hy yo1,1 are so upset."
Correct Answer: C.
Emotional validation can strengthen the therapeutic alliance and relieve patients' distress. There are methods of emotional validation: being present (actively listening and indicating understanding via nonverbal cues such as nodding), accurate reflection (restating a
patient's words), mindreading (inferring the emotional content behind the patient's words), and normalizing (recognizing the patient's reaction as a response any other person would have). This physician chose to infer this patient's emotion from her statement and
nonverbal cues.
Incorrect Answers: A, B, D, and E.
Asking about the patient's clinical risk factors for this skin condition (Choice A) fails to validate the patient's emotions. This response may be perceived as emotionally distant and would not strengthen the physician-patient relationship or relieve patient distress.
Reassuring the patient that her skin condition is not serious (Choice B) may be appropriate later in the conversation. However, this patient is less concerned now about the seriousness of the condition and more concerned about how others will perceive the
condition. The physician should initially focus on the patient's concerns that are causing the most distress.
Physicians should not critique a patient's aesthetic appearance (Choices D and E), which is unrelated to the physician's role to treat medical illness and would likely hurt the patient's feelings. Commenting on a patient's appearance in general, even if the physician
has made a positive observation, can be perceived as unprofessional and outside of a physician's scope. The praise and validation in the second parts of Choices D and E, respectively, may be appropriate at some point in the conversation.
Educational Objective: Emotional validation, including inferring a patient's underlying emotion, can strengthen the therapeutic alliance and relieve the patient's distress.

r
Prevlo11s
r
lllext
,
Sc11re Report
rrE
Lab Values
F'
Calcula�or
,.
Hel1>
1ft
Pause
_ _ _ _ _
Exam Section 4: Item 9 of 50 National Board of Mqdlcal Examiners®
Comprehensive aas1c Sctence Serr-Assessment

✓ 9. A40-year-old woman lhas had hypercalcemia for 1 year and recently passed a renal calculus. Serum parathyroid honnone and calcium concentrations are increased, and serum phosphate concentration is decreased. Parathyroid hormone most likely
causes an increase in the serum calcium concentration by which of the following mechanisms?

A}-Desi:eased d09r:adation of 25 hydmxyGholesalsiferol


S}-Direst astion on intestine to insrease salsil,lrn absorption
G}-Direst aslion on intestineto insrease rnagnesiYrn absorption
D ) Increased synthesis of 25-hydroxycholecalciferol
E ) Inhibition of calcitonin production
F ) Stimulation of 1,25-dihydroxycholecalciferol production

Correct Answer: F.

Parathyroid hormone (PTH) stimulates 1a-hydroxylase in the kidney, which leads to the conversion of stored 25-hydroxycholecalciferol to active 1,25-dihydroxycholecalciferol. Once produced, 1,25-dihydroxycholecalciferol stimulates the increased absorption of
calcium and phosphate from the intestine. PTH also has a role in the regulation of calcium and phosphate by stimulating osteoclastic bone reabsorption and distal convoluted tubular calcium reabsorption and phosphate excretion in the nephron. Primary
hyperparathyroidism results in hypercalcemia and hypophosphatemia secondary to the aberrant increased production of PTH. The most common cause of primary hyperparathyroidism is a parathyroid adenoma. Common symptoms of primary hyperparathyroidism
are secondary to the patient's resultant hypercalcemia and include recurrent nephrolithiasis, bone pain from osseous resorption, polyuria, dehydration, constipation, and psychiatric disturbances.

Incorrect Answers. A, B, C, D, and E.

PTH has no direct effect upon either the degradation (Choice A) or the synthesis (Choice D) of 25-hydroxycholecalciferol. 25-hydroxycholecalciferol is produced and stored in the liver through the hydroxylation of cholecalciferol by the enzyme cholecalciferol 25-
hydroxylase. It is an inactive form of vitamin D and does not play an active role in the absorption of calcium and phosphate from the intestines.

Direct action on intestine to increase calcium absorption (Choice B) is a characteristic of 1,25-dihydroxycholecalciferol, the active form of vitamin D. While PTH does increase intestinal absorption of calcium, this is primarily an indirect effect that is mediated by
vitamin D.

Direct action on intestine to increase magnesium absorption (Choice C) is not a characteristic of PTH. Magnesium is absorbed in the small intestine. Its absorption is influenced primarily by gastrointestinal transit time and by the quantity of magnesium ingested,
rather than by endocrine regulation.

Inhibition of calcitonin production (Choice E) is not a direct effect of PTH. Calcitonin is produced by thyroid parafollicular cells. Its actions generally oppose the effects of PTH, and include decreasing .bone resorption of calcium, which results in decreased serum
calcium concentration.

Educational Objective: Parathyroid hormone (PTH) stimulates 1a-hydroxylase in the kidney, which leads to the conversion of 25-hydroxycholecalciferol to active 1,25-dihydroxycholecalciferol (vitamin D). The active form of vitamin D then stimulates the increased
absorption of calcium and phosphate from the intestines. PTH regulates calcium and phosphate concentrations by stimulating osteoclastic bone reabsorption and distal convoluted tubular calcium reabsorption and phosphate excretion in the nephron.

r
Previous
r
Next
,
Score Re ort
fa
Lab Values
P
Calculator
r
Hel
"'
Pause
Exam Section 4: Item 10 of 50 National Board pf Mqdlcal Examiners®
Comprehensive Basic Sctence Self-Assessment

✓ 10. In a 3-week-old knockout mouse, a leg bone reaches only half the length of that of a similar-aged wild-type mouse. The epiphyseal plates in the bone of the transgenic animal are significantly narrower than in the bone of the wild-type animal. The knockout
mouse most lilkely has a deficiency of which of the following?

A) Calcitonin
B) Cortisol
C) Insulin-like growth factor I
D) Platelet-derived growth factor
E) Thyroxine (T4)
Correct Answer: C.
The hypothalamus-pituitary-somatotropic axis is the endocrine regulatory feedback loop that is responsible for human growth regulation. In this axis, the pituitary gland secretes growth hormone (GH, somatotropin) which stimulates insulin-like growth factor-1 (IGF-
1, somatomedin C) production in the liver and peripheral tissues. IGF-1 binds its receptor, a tyrosine kinase-based receptor, that stimulates cell growth, proliferation, and growth of the axial and appendicular skeleton. Deficiency of the GH receptor (Laron syndrome)
leads to short stature from insensitivity to GH. Conversely, a GH-secreting pituitary adenoma can cause gigantism or acromegaly through excessive, dysregulated stimulation of GH receptors, leading to chronic elevations in IGF-1. Gigantism results from excess
IGF-1 signaling before fusion of the growth plates, and results in enlargement of both long and flat bones, whereas acromegaly occurs after fusion, leading only to expansion of flat bones and tissues.

Incorrect Answers: A, 8, D, and E.

Calcitonin (Choice A) is a peptide hormone produced by the parafollicular cells of the thyroid gland. Calcitonin is a regulator of calcium homeostasis and its primary function is to reduce blood levels of calcium. It inhibits osteoclast activity and also inhibits renal
tubular resorption of calcium and phosphate.

Cortisol (Choice 8) is a steroid hormone which is produced in the zona fasciculata of the adrenal cortex. Cortisol production is cyclic with the human sleep-wake cycle and is a key regulator of glucose metabolism with higher levels of cortis.ol leading to
gluconeogenesis. It also has important effects on the central nervous system relating to stress and mood regulation, and it is a powerful anti-inflammatory molecule as it inhibits production of tumor necrosis factor alpha.

Platelet-derived growth factor (Choice D) is secreted by platelets and stimulates cellular proliferation and angiogenesis. It does not have a primary role in bone growth.

Thyroxine (T4) (Choice E) is the main hormone produced in the thyroid gland. It is the primary regulator of the basal metabolic rate. Congenital deficiency of thyroid production can lead' to cretinism which is a syndrome of small stature, enlarged tongue, and
intellectual disability. It stimulates bone growth, however, does not have as significant of a role as IGF-1 on axial and appendicular skeletal growth.

Educational Objective: Insulin-like growth factor-1 (IGF-1) is regulated by growth hormone and is produced predominantly in the liver. Decreased IGF-1 production leads to decreased axial and appendicular skeletal growth.

r
Previous
r
Next
,
Score Report

Lab Values
F
Calculator
r
Help
,,.
Pause
Exam Section 4: Item 11 of 50 Natlonal Board of Mqdlcal Examiners®
Comprehensive Basic Sctence Serr-Assessment

� \ I /I . I ,. '\ I

11. A 60-year-old man develops palpitations over a 6-week period during1 which he has been drinking approximately 8 cups of coffee per day. A rhythm strip from his ECG is shown. Which of the following is most likely responsible for the palpitations in this
man?

A ) Second-degree atrioventricular block


B ) Sinoatrial exit block
C) Su raventricular remature beats
C} Third degroe atrio•.•entricbllar elock
li}- l/-0nlricular tachycardia
Correct Answer: C.
Supraventricular premature beats account for this patient's palpitations and are apparent on the rhythm strip at beat three and beat seven, followed by compensatory pauses and resumption of sinus rhythm. Normal cardiac conduction starts with an electrical
impulse in the sinoatrial (SA) node located in the upper right atrium, which is depicted on the rhythm strip as the P wave. The electrical impulse is conducted throughout the atria, causing coordinated contraction of the right and left atria wit!h active filling of the
ventricles. The electrical impulse reaches the atrioventricular (AV) node located in the interatrial septum. Conduction is temporarily slowed in the AV node to allow for complete active and passive filling of the ventricles (PR interval), followed by conduction of the
electrical impulse down the His-Purkinje system in the interventricular septum, then to the right and left bundles, and finally to the myocardial cells of the ventricles. This results in contraction of the right and left ventricles (QRS complex) with expulsion of blood into
the pulmonary and systemic circulations, respectively. The ventricles subsequently repolarize (T wave) and relax to allow filling for the next cardiac cycle. A premature supraventricular beat, also called a premature atrial contraction (PAC), occurs when an electrical
impulse is generated at a location in the atria outside the SA node. If a P wave is visualized. it will have an abnormal appearance, but typically the P wave cannot be seen as it is hidden in the T wave from the prior beat. The impulse travels through the AV node and
conduction system, so the QRS complex appears similar to the other sinus beats. There is a compensatory pause following a PAC because the ectopic beat causes the SA node to become temporarily refractory. PACs are often seen in patients with states of high
adrenergic drive, such as during the consumption of caffeine in this patient but are usually harmless.
Incorrect Answers: A, B, D, and E.
Second-degree atrioventricular (AV) block (Choice A) has two presentations: type I and type II. Second-degree AV block type I demonstrates a prolonged PR interval that progressively increases followed by a non-conducted P wave (no QRS), with resumption of
normal conduction. It is usually benign. Second-degree AV block type II is identified by a prolonged PR interval that does not progressively increase with intermittent non-conducted P waves. Treatment usually involves placement of a pacemaker for type II.
Sinoatrial exit block (Choice B) implies failure of the electrical impulse to leave the SA node. This manifests on the rhythm strip as occasional absent P waves. There are no premature beats.
Third-degree atrioventricular block (Choice D) occurs when no sinus impulses are transmitted through the AV conduction pathway resulting in complete dissociation between the P waves and the QRS complexes. On the rhythm strip, there will be a constant interval
between P waves and a different constant interval between QRS complexes, which will demonstrate no relationship to one another. Treatment usually requires a pacemaker.
Ventricular tachycardia (VT) (Choice E) occurs when an abnormal electrical focus or a re-entry circuit causes rapid contraction of the ventricles. The general morphology will demonstrate a wide QRS complex tachycardia. There are monomorphic and polymorphic
forms of VT. Treatment depends on the duration, type, and symptomatology.
Educational Objective: Premature atrial contractions (PACs) occur when an ectopic atrial focus generates a premature impulse that travels down the normal conduction pathway, resulting in a narrow QRS (ventricular contraction). As the sinoatrial (SA) node
becomes temporarily refractory, there is a compensatory pause following a PAC. PACs are a common cause of palpitations, and typically occur in states of adrenergic tone (eg, caffeine).

r r , ta e r "'
Previous Next Score Report Lab Values Calculator Help Pause
Exam Section 4: Item 12 of 50 National Board pf Mqdlcal Examiners®
Comprehensive sas1c SClence Self-Assessment

✓ 12. In an experimental design evaluating assays for the bactericidal effect of inflammatory cells, which of the following compounds is considered the most efficient in the killing of bacteria by neutrophils?

A ) Bactericidal permeability increasing protein


B ) Hypochlorite
C) Lactoferrin
D) Lysozyme
E ) Major basic protein
Correct Answer: B.

Hypochlorite is important in the efficient killing of bacteria by neutrophils as it is a component of the "respiratory burst," a process employed by neutrophils that results in rapid bactericidal activity. The respiratory burst describes a process by which neutrophils and
phagocytes utilize oxygen to produce free radicals, hypochlorite, and peroxides, all of which are toxic to rapidly dividing bacterial cells. The process begins with oxygen free radical generation via the action of NADPH oxidase, with additional conversion to hydrogen
peroxide by superoxide dismutase. The addition of a chloride ion to hydrogen peroxide by myeloperoxidase creates hypochlorite. The actions of superoxide anions, hydrogen peroxide, and hypochlorite result in rapid and efficient bactericidal activity.

Incorrect Answers: A, C, D, and E

Bactericidal permeability increasing proteins (Choice A) are released by neutrophils in response to infections with both Gram-positive and Gram-negative bacteria, but their role has been best described in Gram-negative infections. They bind to the bacterial
lipopolysaccharide wall (LPS), where they increase bacterial membrane permeability and contribute to opsonization.

Lactoferrin (Choice C) scavenges and sequesters iron, thereby preventing bacterial pathogens from using iron, an important component of many necessary oxidation-reduction reactions. It has bacteriostatic properties and does not directly act to k.ill bacteria.

Lysozyme (Choice D) is a bactericidal enzyme that hydrolyzes the bonds between layers of the Gram-positive bacterial peptidoglycan wall. It reduces the integrity of this structure and indirectly contributes to bacterial death.

Major basic protein (Choice E) is contained primarily within eosinophilic granules and is released in response to infection with helminths. While it also plays a role in atopy, it is not known to play a major role in bactericidal activity.

Educational Objective: Hypochlorite is produced in neutrophils during a process known as the respiratory burst. Its release results in rapid and efficient bactericidal activity.

r r , fc!f F r ff'
Previous Next Score Report Lab Values Calculator Help Pause
Exam Section 4: Item 13 of 50 National Board of Medical Examiners®
Comprehensive i:saslc Science Serr-Assessment

✓ 13. A 35-year-old woman is brought to the emergency department comatose after she sustained multiple injuries in a motor vehicle collision. During the next few days, she develops sepsis and respiratory insufficiency,
and then she dies. A photomicrograph of her lungs obtained at autopsy is shown. If this patient had recovered, regeneration of the alveolar epithelium would have been accomplished via hyperplasia of which of the
following cell types?

A ) Alveolar capillary endothelial cell


B ) Alveolar macrophage
C ) Chondrocyte
D) Ciliated columnar epithelial cell
E ) Club cell
F ) Goblet cell
G) Kulchitsky cell
H ) Squamous epithelial cell
I ) Type I pneumocyte
J ) Type II pneumocyte

Correct Answer: J.

The patient likely suffered from acute respiratory distress syndrome (ARDS) secondary to sepsis. ARDS is characterized by acute onset, diffuse, inflammatory lung injury and bilateral pulmonary infiltrates on chest imaging leading to respiratory failure. There are
numerous etiologies, including sepsis (especia'lly in the setting of Gram-negative bacteria due to macrophage overactivation by lipopolysaccharide), pneumonia, chemical pneumonitis (eg, aspiration), pulmonary contusion, acute pancreatitis, trauma,
transfusions, medications, and amniotic fluid embolism. Alveolar damage and inflammation lead to increased permeability of the alveolar-capillary interface. The initial (acute or exudative) phase is characterized by diffuse alveolar damage, a leukocytic infiltrate,
and a protein•ric-tl exudate which coats the alveoll In a thick hyaline membrane, as seen in the photomicrograph. The sec-0nd (proliferative or organiz.ing) phase begins around one week after the initial injury, with repair of the damaged interface and proliferation of
fibroblasts. Repair is accomplished via hyperplasia of type II pneumocytes and alveolar septa! fibrosis, which can be seen on histology. Type II pneumocytes are cuboidal cells which are the main producers of pullmonary surfactant. They also differentiate into
type I pneumocytes to replace damaged cells, :as type I pneumocytes are unable to self-replicate.

Incorrect Answers: A, B, C, D, E, F, G, H, and I.

Alveolar capillary endothelial cells (Choice A) form a thin, single-cell layer between the vascular lumen and the basement membrane of the alveolar-capillary interface. They are damaged in ARDS. Endothelial cell hyperplasia is needed to restore the endothelial
layer but would not regenerate the epithelial layer of the interface.

Alveolar macrophages (Choice B) are responsible for phagocytosis of infectious, toxic, and foreign particles. Activation in ARDS contributes to inflammatory injury.

Chondrocytes (Choice C) are the only cell types found in cartilage, where they produce collagen and proteoglycans to form the extracellular matrix. They are not involved in the organizing phase of ARDS.

Ciliated columnar epithelial cells (Choice D) form the epithelial lining of the fallopian tubes and parts of the respiratory tract (nasal cavity, trachea, bronchi, and bronchioles). The action of cilia clears mucus and foreign particles. They are not a component of the
alveolar epithelium.

Club cells (Choice E) are non-ciliated exocrine cells of the bronchiolar epithelium, which secrete glycosaminoglycans to protect the epithelial lining. They are not part of the alveolar-capillary interface.

Goblet cells (Choice F) are columnar epithelial cells of the respiratory, reproductive, and gastrointestinal tracts, which secrete mucins. They are located proximally in the respiratory tract.

Kulchitsky cells (Choice G) are enteroendocrine cells that reside in the gastrointestinal epithelium. They secrete serotonin.
Squamous epithelial cells (Choice H) may form simple or stratified squamous epithelium. Stratified sciuamous epithelium is found lining the oral cavity, esopihagus, and vagina. Simple squamous epithelium lines the alveoli, vasculature, and lymphatic vessels.
The simple squamous epithelial cells of the alveoli are called type I pneumocytes.

Type I pneumocytes (Choice I} form the majority of the alveolar epithelium. The thin, single layer of cells is ideal for diffusion of carbon dioxide and oxygen between the alveoli and the pulmonary capillary endothelium. Type I pneumocytes are unable to replicate.
When damaged, proliferation of type II pneumocytes with differentiation into type I pneumocytes regenerates the alveolar interface.

Educational Objective: Acute respiratory distress syndrome (ARDS) is characterized by diffuse damage to the alveoli and a proteinaceous exudate, which forms a thick hyaline membrane around the alveolar epithelium. Differentiation of type II pneumocytes into
type I pneumocytes is the primary mechanism of repair.

r r , � F r r
Exam Section 4: Item 14 of 50 National Board pf Medical Examiners®
Comprehensive Basic Science Self-ASsessment

✓ 14. A 36-year-old man comes to the physician after he noticed a dark lesion on his back. Physical examination shows a dark, irregularly shaped, raised region on the back. He has a history of frequent sun exposure at the beach. A biopsy specimen of the
region confirms the diagnosis of a melanoma. Which of the following embryonic tissues most likely formed this tumor?

A ) Dermatome
B ) Lateral plate mesoderm
C) Neural crest
D) Nonneural ectoderm
E ) Prechordal plate mesoderm

Correct Answer: C.

Melanoma is a tumor of melanocytes, which are dendritic cells that reside in the basal layer of the epidermis and produce melanin that is distributed among the surrounding keratinocytes. Melanocytes are derived from the neural crest cells, which begin at the dorsal
neural tube. During embryologic development they migrate from dorsal to ventral, and then to the epidermis. Melanocytes also migrate to the inner ear, nervous system, and eye. Failure of melanocytes to migrate leads lo pigmentary abnormalities. For example, a
while forelock, or a white patch of hair usually on the frontal hairline, occurs when the melanocytes do not fully migrate cephalad. Melanoma is a neoplastic proliferation of melanocytes and is likely to be present when a lesion demonstrates asymmetry, irregular
appearing borders, variable coloration, a diameter greater than 6 mm, and rapid evolution in characteristics. Melanoma has the ability to rapidly invade and metastasize, which carries a poor prognosis when diagnosed late.

Incorrect Answers: A, B, D, and E.

A dermalome (Choice A) is one of the components of the embryonic somile, a paired segment of paraxial mesoderm. When fully developed, the dermatome is an area of skin whose afferent fibers travel within a single spinal nerve. Herpes zoster infection is often
dermatomal because the virus replicates in the dorsal root ganglion of a single spinal nerve. Melanocytes do not follow dermatomes.

The mesoderm is one of the three primary germ layers of the embryo. The mesoderm can be divided into three segments, moving medially lo laterally: the paraxial mesoderm, the intermediate mesoderm, and the lateral plate mesoderm (Choice B). The prechordal
plate mesoderm (Choice E) is localed at the cephalic tip of the nolochord. Langerhans cells, which are dendritic antigen-presenting cells also found in the epidermis, are derived from mesoderm as is the dermis. Other mesoderm derivatives include the muscle,
bone, connective tissue, cardiovascular structures, lymphatics, kidneys, and gonads.

The ectoderm is also one of the three primary germ layers of the embryo. The keratinocytes of the epidermis are derived from nonneural ectoderm (Choice D), but this is not the origin of melanocytes.

Educational Objective: Melanocytes are dendrilic cells located in the epidermis basal layer and are derived from neural crest cells. Melanoma is a neoplastic proliferation of melanocytes.

r r , tr!1 F· r "'
Previous Next Score Re ort Lab Values Calculator Hel Pause
Exam Sectlon 4: Item 15 of 50 National Board pf Medical Examiners®
Comprehensive isaslc Science Self-Assessment

y 15. A 31-year-old woman is brought to the emergency department by her husband because of difficulty breathing and severe muscle weakness for 10 minutes. She has a 3-year history of myasthenia gravis treated with neostigmine. The husband reports that
she doubled her dosage 2 days ago because she was feeling extraordinarily weak, but her weakness has increased since then. Which of the following events is the most likely cause of the increased muscle weakness in this patient?

A ) Autoimmune hyperactivation of nicotinic receptors


El) Autoimmune inactivation of muscarinic r-eceptors
C) Desensitization of nicotinic receptors
C1 Excessive degrada:tion of acet-ylcholine
E} Hypersensitization of muscarinic receptors
�} Jn&uffiGieAU&lease of acetylcholine
Correct Answer: C.
Myasthenia gravis is an autoimmune disorder in which the body makes antibodies against nicotinic acetylcholine receptors at the postsynaptic membrane of the neuromuscular junction, rendering them unresponsive to acetylcholine. It commonly presents with
ptosis, diplopia, dysphagia, and muscle weakness, which are worse with repetitive activity and later in the day. Neostigmine is an acetylcholinesterase inhibitor, which decreases the breakdown of acetylcholine in the neuromuscular synaptic junction and increases
its duration of action. It is often used in the treatment of myasthenia gravis, as it allows the concentration of acetylcholine in the neuromuscular junction to increase, maximizing the receptor binding and increasing the likelihood of muscular depolarization with a
neural impulse. Complications of neostigmine include bradyarrhythmias and bronchoconstriction. Overdoses of neostigmine can lead to a cholinergic crisis with respiratory muscle weakness, paralysis, diarrhea, and changes in vision due to desensitization of the
nicotinic receptors by prolonged exposure to acetylcholine. Treatment includes the administration of atropine.
Incorrect Answers: A, B, D, E, and F.
Autoimmune hyperactivation of nicotinic receptors (Choice A) would lead to muscle contraction followed by weakness, hypertension, tachycardia, mydriasis, urinary retention, and diaphoresis. She has no other symptoms associated with this condition, and a
secondary primary autoimmune disorder is a less likely cause of her symptoms than desensitization of nicotinic receptors in the setting of high doses of neostigmine.
Autoimmune inactivation of muscarinic receptors (Choice 8) would lead to mydriasis, bronchodilation, dry skin, increased body temperature, tachycardia, urinary retention, and constipation. Muscarinic receptors do not play a role in skeletal muscle activation, and
their inactivation would not cause progressive muscle weakness.
Excessive degradation of acetylcholine (Choice D) would lead to muscle weakness, but neostigmine and other acetylcholinesterase inhibitors decrease the rate of acetylcholine metabolism. This would be an unlikely mechanism of this patient's weakness in the
setting of her recently increased dosing of neostigmine.
Hypersensitization of muscarinic receptors (Choice E) would lead to miosis, bronchoconstriction, bradycardia, urinary incontinence, and diarrhea. Muscarinic receptors do not play a role in skeletal muscle activation, and their hypersensitization would not cause
weakness.
Insufficient release of acetylcholine (Choice F) would lead to worsening muscle weakness due to an insufficient concentration of acetytcholine in the neuromuscular synaptic junction. Neostigmine increases the amount of acetylcholine in the junction through
affecting the metabolism of acetylcholine. It does not directly affect the release of acetylcholine, making this an unlikely etiology.
Educational Objective: Neo,stigmine is a potent acetylcholinesterase inhibitor used in the treatment of myasthenia gravis. Increased doses can lead to a cholinergic crisis with muscular weakness, including weakness of the respiratory muscles, due to desensitization
of the nicotinic receptors secondary to prolonged exposure to high levels of acety1choline.

r
Previous
r
}�ext
,
Score �eport
tr§
Lab Values
F
c alculator
r
Help
,,.
Pause
. .
Exam Section 4: Item 16 of 50 National Board pf Mqdlcal Examiners®
Comprehensive Basic Science Self-Assessment

✓ 16. A 25-year-old woman at 38 weeks' gestation undergoes an oxytocin stress test to evaluate the integrity of the placental circulation. Based on the results, a full-scale induction of labor with oxytocin is started. Treatment with oxytocin is most likely to activate
which of the following substances in this patient's uterine smooth muscle?

A) Adrenergic nerve terminals


B) Cholinergic nerve terminals
C) Ligand-gated calcium channels
D) Na +-K + ATPase
E) Voltage-gated chloride channels
Correct Answer: C.
Oxytocin is commonly used to promote uterine contractions during labor. Oxytocin receptors are expressed by the uterine myometrial smooth muscle cells and myoepithelial cells of the mammary gland. The oxytocin receptor (OTR) belongs to the rhodopsin-type
group of G-protein-coupled receptors. Binding of oxytocin to the OTR triggers a signaling cascade which results in activation of ligandi)ated calcium channels, leading to increased calcium influx into the uterine smooth muscle cells. The resultant increase in
intracellular calcium leads to increased calcium-calmodulin complex activation of myosin light chain kinase (MLCK). Myosin light chains in muscle cells are then phosphorylated by MLCK, leading to cross-bridge formation between myosin heads and actin filaments
to generate smooth muscle· contraction.

Incorrect Answers: A, B, D, and E.

Adrenergic nerve terminals (Choice A) in smooth muscle cells are coupled to signal transduction pathways that activate adenylyt cyclase and result in increased intracellular cAMP concentrations. Cyclic AMP inhibits the activity of myosin light chain kinase, leading
to smooth muscle relaxation.

Cholinergic nerve terminals (Choice B), also called muscarinic receptors, are found on uterine smooth muscle cells. Multiple subtypes of the receptor are present with different signal responses. Oxytocin does not activate these receptor types.

Na +_K + ATPase (Choice D) is an essential transmembrane enzyme that transports three Na+ ions out of the cell in exchange for two K + ions into the cell. Increased Na +-K + ATPase activation indirectly leads to an increased activity of the Na+.ca2+ antiporter,
which removes Ca2+ from the cell. Decreased intracellular Ca2+ results in decreased smooth muscle contractility.
Voltage-gated chloride channels (Choice E) are involved in the regulation of the transmembrane potential in smooth muscle cells. Oxytocin does not act on these channels.

Educational Objective: Oxytocin stimulates uterine contractions by acting on the oxytocin receptor of uterine smooth muscle cells, which results in the activation of ligandi)ated calcium channels. The consequent increased intracellular calcium concentration
promotes uterine smooth muscle cell contraction.

r
Previous
r
Next
,
Score Report

Lab Values
F
Calculator
r
Help
,,.
Pause
Exam Section 4: Item 17 of 50 National Board pf M-idlcal Examiners®
Comprehensive 1:1as1c science Self-Assessment

✓ 17. A 73-year-old man with polymicrobial sepsis begins treatment with a variety of antibiotics, including gentamicin (current dose indicated in table). Three days later, serum gentamicin concentrations are 8.8 µg/ml (peak) and 2.5 µg/ml (trough). Which of the
following dose regimens is most appropriate for achieving a peak serum gentamicin concentration between 4 and 10 µg/mL and a trough concentration of less than 2 µg/ml?

Dosage
Intravenous Administration (mg) Time Between Each Dose (hours)
(Current= 80 mg) (Current= 8 hours)
A) 60 6
B) 80 6
C) 80 12
D) 120 8
E) 120 12
Correct Answer: C.

Increasing the time between doses is the most effective way to achieve a trough concentration of less than 2 µg/ml while maintaining a peak between 4 and 10 µg/ml. In this patient, gentamicin, an aminoglycoside antibiotic, is dosed intravenously for sepsis.
Peak and trough concentrations are obtained 3 days into the course. The half-life of the medication can be estimated assuming steady state kinetics. At time = zero, the peak is 8.8 µg/ml; after one half-life, the concentration would be 4.4 µg/ml, and after two
half-lives, the concentration would be 2.2 µg/ml. The trough is 2.5 µg/ml at 8 hours; therefore, the half-life of gentamicin in this patient is approximately 4 hours (half-life = hours/approximate number of half-lives passed). Steady state concentration generally
requires four to five half-lives, which would have been reached on the first day of administration in this patient. The peak concentration of a drug in serum occurs immediately after bolus dosing, as the entire amount of administered drug is contained within serum
before distribution to tissues. The trough concentration of a drug occurs immediately prior to the next dose and reflects the nadir of serum concentration, which is a function of redistribution into tissues plus drug metabolism. In this example, the peak
concentration is 8.8 µg/ml, which is within the target range of 4 to 10 µg/ml. Increasing the dose would result in an increase in the peak concentration, while decreasing the dose would decrease the peak concentration. The dosed amount falls within the peak
parameters and should not be changed. Trough concentration is a function of distribution plus metabolism. First-order metabolism results in a 50% reduction in drug concentration per half-life. From a peak of 8.8 µg/ml, one half-life (approximately 4 hours) would
yield a serum concentration of 4.4 µg/rnL, a second half-life (approximately 8 hours) would yield a serum concentration of 2.2 µg/ml, still greater than the target trough of less than 2 µg/ml. A third half-life (approximately 12 hours) would yield a serum
concentration of 1.1 µg/ml, which is at the desired trough level. Therefore, maintaining the dose but extending the interval is the appropriate method of adjusting serum gentamicin concentration in this patient.

Incorrect Answers: A, B, D, and E.

Choice A !owe.rs the dose, which might lower the peak concentration beneath 4 µg/ml, and decreases the dosing interval, which might raise the trough concentration depending on the new peak.

Choice B maintains the dose but decreases the dosing interval to between one and two half-lives. The trough concentration in this case would be between 2.2 and 4.4 µg/ml, above the target trough.

Choices D and E increase the dose, which would raise the peak concentration, while maintaining or lengthening the dosing interval, which would maintain or lower the trough; however, the increased peak may exceed 10 µg/ml.

Educational Objective: The peak concentration of an intravenous drug in serum occurs immediately after bolus dosing, as the entire amount of newly administered drug is contained within serum. The trough concentration of a drug occurs immediately prior to the
next dose and reflects the nadir of serum concentration, a function of redistribution into tissues plus any metabolism of the drug.

r
Previous
r
Next
,,.
Score Report
frj F
Lab Values Calculator
r
Help
,,.
Pause
Exam Section 4: Item 18 of 50 National Boa.rd pf M11dlcal Examiners®
Comprenenslve 8aslc Science Serr-Assessment

✓ 18. A 17-year-old boy with chronic renal insufficiency undergoes a unilateral nephrectomy. A photograph of the resected kidney is shown. Which of the following pathologic processes is most likely
present in the kidney?

A)-Acute glomerulonephritis
li}Acute tubular necrosis
G)-Angiomyolipoma
C}-Autosomal recessive polycystic kidney disease
ti)- Flbromuscular dysplasia
--------------------------------------------------------
- -

F ) Hydronephrosis
Gt Nephroblastoma
l
l,i} Papillary necrosis SPECIMEN DATE
l )- Renal vein thrombosis
J ) Staghorn calculus
Correct Answer: F.

Congenital urinary tract abnonnalities include, but are not limited to, unilateral renal agenesis, fused kidneys, ureteral stricture and stenosis, duplex collecting system, posterior urethral valves, bladder agenesis, bladder exstrophy, hypo- or epispadias, and
urethral strictures. Regardless of the etiology of malformation, ureteral obstruction and vesicoureteral reflux are common complications. Chronic reflux can lead to hydroureter and hydronephrosis, characterized by dilation of the renal pelvis and calyces resulting
in compression atrophy of the renal parenchyma and eventual renal insufficiency and failure. Hydronephrosis can be graded in severity, in which the most severe cases compress the parenchyma resulting in gross distbrtion of renal architecture.

Incorrect Answers: A, B, C, D, E, G, H, I, and J.


Acute glomerulonephritis (Choice A) refers to a variety of glomerular diseases, including nephritic and nephrotic syndromes. Nephritic syndromes typically present with acute renal failure associated with hematuria, red blood cell urine casts, and hypertension.
Nephrotic syndrome- typically presents with excessive proteinuria, hyperiipidemia, hypoalbuminemia, and edema. Diagnosis is usually made with renal biopsy; gross dilation of the renal pelvis and calyces are inconsistent with the diagnosis.

Acute tubular necrosis (Choice B) occurs following an ischemic or nephrotoxic insult to the kidneys, which results in necrosis of the tubular epithelium. Granular, muddy brown casts are typical on urinalysis. It would not cause gross distortion of the pelvic and
calyceal architecture.

Angiomyolipomas (Choice C) are tumors derived from perivascular epitheli.oid cells. While typically benign, they can be associated with tuberous sclerosis. They present as a mass, which can be composed of smooth muscle, adipocyte, and epithelioid cells.

Autosomal recessive polycystic kidney disease (Choice D) is caused by genetic mutations that result in the development of multiple cysts in the kidney due to structural abnormalities of the renal tubules. Cysts may fonn in infancy, childhood, or adulthood, and
eventually compress adjacent nonnal renal parenchyma.

Fibromuscular dysplasia (Choice E) is a non-atherosclerotic angiopathy of medium-sized arteries with scattered nodular aneurysms that can lead to renal artery stenosis and secondary hypertension due to excessive production of renin and angiotensin. It
commonly presents as refractory hypertension in a young patient with no medical comorbidities or abnonnal laboratory findings.

Nephroblastoma (Choice G) is the most common renal malignancy in childhood due to mutations in tumor suppressor genes WT1 or WT2. It is characterized by a large, often palpable, unilateral flank mass and hematuria.

Renal papillary necrosis (Choice H) occurs following ischemic, inflammatory, infectious, or toxin-mediated damage to the renal papilla and describes the sloughing and loss of the papillae including substructures such as the distal collecting tubule. RPN can be
triggered by infections, diabetes mellitus, sickle cell disease, or nonsteroidal anti-inflammatory drugs.

Renal vein thrombosis (Choice I) occurs most commonly in patients who have an underlying thrombophilic condition and can present with flank pain, hematuria, acute kidney injury, or may be asymptomatic. It would not distort intrarenal ar,chitecture like that seen
in hydronephrosis.

Staghorn calculi (Choice J) are struvite stones, which can grow rapidly and can be seen on gross pathology as large stones that fill the intrarenal collecting system.

Educational Objective: Congenital urinary tract abnonnalities can be complicated by ureteral obstruction and vesicoureteral reflux. Chronic reflux can lead to hydronephrosis, characterized by dilation of the renal pelvis and calyces resulting in compression
atrophy of the renal parenchyma and eventual renal insufficiency and failure.

r
Previous
r
Next
,
Score fteport

Lab Values
F
Calculator
r-
Help
If"
Pause
Exam Section 4: Item 19 of 50 National Board pf Medical Exi!Jlllners®
Comprehensive Basic Science Serr-Assessment

✓ 19. A researclher in a pharmaceutical company designs a new protease inhibitor that inhibits replication of HIV in T lymphocytes in culture. In subsequent assays, which of the following findings is most likely to indicate that the compound is working specifically
as a protease inhibitor?

A )-The drug pF8>Jenl& integralisn gf pm11iral OMAints lhe hsst gensme


B ) The RNA is partially reverse transcribed into proviral DNA
C) There is a lack of a mature core
D) Transcription from the HIV promoter is blocked
Ii)- The >Jirus dses ngt bind lg C04 in the pF8sence gf thedrug
Correct Answer: C.
Lack of a mature core would indicate that the new protease inhibitor is specific to that function. HIV drugs are separated into several classes and each of them work via a different mechanism to prevent viral replication. Drugs of disparate mechanisms are combined
to inhibit HIV replication at multiple steps, thereby preventing the development of resistance. HIV infection and replication can be broken into seven primary steps: binding, fusion, reverse transcription, integration, replication, assembly, and budding. After HIV binds
to the surface of CD4 cells via interaction of its specific glycoproteins with CD4 receptors such as CCR5, the virus fuses with the cell membrane and releases its contents into the cytoplasm. HIV RNIA is reverse transcribed by reverse transcriptase to create HIV
DNA, followed by incorporation of this DNA into the CD4 cell genome. Once integrated, HIV hijacks the cellular machinery to produce the HIV genome and all the requisite proteins to form a new HIV viral particle. The new virus then buds off of the CD4 cell and is
released to infect other lymphocytes. However, the viral particles are immature as they contain long proteins that have not yet been cleaved, a task accomplished by HIV protease. Following protein cleavage, the HIV viral particle core has reached maturity.
Protease inhibitors block this step but have no effect on the preceding steps of HIV replication. As such, a pure protease inhibitor would allow for normal replication and budding of the HIV virus, but the virus would remain immature and non-infectious as a result of
failure to undergo the final step of protein cleavage.
Incorrect Answers: A, B, D, and E.
The drug prevents integration of proviral DNA into the host genome (Choice A) is not correct, as protease inhibitors do not inhibit this step of HIV replication. Inhibition of that step is accomplished by integrase inhibitors.
The RNA is partially reverse transcribed into proviral DNA (Choice B) is also not correct, as inhibition of this step is accomplished by non-nucleoside reverse transcriptase inhibitors (NNRTls) and nucleoside reverse transcriptase inhibitors (NRTls).
Transcription from the HIV promoter is blocked (Choice D) by investigational therapies that target the trans-activator of transcription (Tat) protein. These drugs have not yet been routinely incorporated into highly active antiretroviral therapy regimens.
The virus does not bind to CD4 in the presence of the drug (Choice E) is a characteristic of CCR5 antagonists. This prevents viral entry into the CD4 cell.
Educational Objective: HIV attaches to CD4 cells, enters via fusion, reverse transcribes its genome which is then integrated into the lymphocyte genome, and hijacks cellular machinery to produce new viral particles that require protease activity to become fully
mature. Protease inhibitors block this last step. They therefore allow for the preceding steps of viral replication but block the formation of a mature viral core from immature precursors.

r
Previous
r
Next
,
Score Report
fr§
Lab Values
r-:-
Calculator
r
Help
r
Pause
Exam Section 4: Item 20 of 50 National Board pf Medical Examiners®
Comprellenslve Basic Science Serr-Assessment

✓ 20. A 42-year-old woman comes to the physician because of a 3-week history of numbness of the fingers on the left hand. Neurologic examination shows loss of touch graphesthesia and loss of two-point discrimination in the left hand. Tl'lis patient most likely
has a lesion in which of the following locations of the hemispheres shown?

A)
B)
C)
D)
e,
J;: }­
G)­
W}-
t }-­
J }--
Correct Answer: B.

Choice B reflects the right anterolateral parietal lobe, representing the primary sensory cortex in the postcentral gyrus. This brain region mediates sensation in the left (contralateral) distal upper extremity. Lesions of this area result in deficits in all sensory domains
including the loss of sensation of fine touch, pressure, proprioception, vibration, and pain. These deficits can also manifest on physical examination as loss of two-point discrimination, agraphesthesia, and astereognosia. This brain area is supplied by the middle
cerebral artery (MCA). Cerebral infarctions (strokes) occur due to ischemic or hemorrhagic loss of blood supply to the brain. Approximately 80% to 85% of strokes are ischemic, commonly arising from thromboembolic disease (eg, middle cerebral artery occlusion
from a thrombus), whereas 15% to 20% of strokes are hemorrhagic due to blood vessel rupture (eg, hypertension-related intraparenchymal hemorrhage from a perforating artery). Risk factors for stroke include smoking, hypertension, diabetes, carotid or intracranial
atherosclerotic disease, history of hypercoagulability, atrial fibrillation, and advanced age. Classically, strokes manifest as a neurologic deficit related to the affected part of the brain.

Incorrect Answers: A. C, D, E, F, G, H, I, and J.

Choices A and J identify the posterior, superior temporal gyrus, an area that in the dominant hemisphere makes up one portion of the Wemicke area, a brain region involved in the understanding of language. In a stroke, lesions involving this area can result in
Wernicke aphasia. In the non-dominant hemisphere, this area is involved in discriminating the pitch of sound. This region is supplied by the MCA.

Choices C and H reflect the superolateral aspect of the posterior frontal lobes (precentral gyrus). This region contains the primary motor cortex and mediates motor function of the upper extremity and trunk. Strokes involving this area present with contralateral upper
extremity and trunk weakness. The MCA supplies this territory.

Choices D and G reflect the posterior frontal lobes immediately anterior to the primary motor cortex, an area that represents the lateral premotor cortex. The lateral premotor cortex mediates the intention to initiate a movement in response to an external cue. Signals
from the premotor cortex are then transmitted to the primary motor cortex, which initiates the movement. The lateral premotor cortex is supplied by the MCA.
Choices E and F reflect the posterior, inferior frontal lobes. In the dominant hemisphere, this brain region represents the Broca area. In a stroke, lesions involving this area can result in Broca aphasia. In the non-dominant hemisphere, this brain region may be
involved in cognitive control mechanisms such as inappropriate response suppression. This area is supplied by the MCA.

Choice I reflects the left anterolateral parietal lobe, representing the primary sensory cortex in the postcentral gyrus. This brain region mediates sensation in the right (contralateral) distal upper extremity. This patient presents with sensory deficits of the left upper
extremity.

Educational Objective: The lateral aspect of the postcentral gyrus mediates sensation of the contralateral upper extremity. Strokes affecting this region can result in sensory deficits of the contralateral hand and/or arm.

r r ,. frE P ,,. r
Exam Section 4: Item 21 of 50 National Board pf Medical Examiners®
Comprehensive 1:1as1c Science Self-Assessment

✓ 21. A 35-year-old woman with sickle cell trait is brought to the emergency department because of severe left flank pain for 1 hour. She has a history of frequent headaches. Her blood pressure is 150/90 mm Hg. Pressure on the costophrenic angle causes
pain. Urinalysis shows a few RBCs and numerous neutrophils. A CT scan of the abdomen shows obstruction and dilation of the left ureter and blunting of several renal papillary pyramids. Which of the following is the most likely diagnosis?

A ) Acute glomerulonephritis
B ) Acute tubular necrosis
G} Diabetis Aephmpathy
D} Papillary lr:aAsili1:mal sell sarsiAoma
E ) Renal papillary necrosis

Correct Answer: E.
Renal papillary necrosis (RPN) occurs following ischemic, inflammatory, infectious, or toxin-mediated damage to the renal papilla and describes the sloughing and loss of the papillae including substructures such as the distal collecting tubule. RPN can be triggered
by infections (eg, acute pyelonephritis), diabetes, sickle cell disease, or nonsteroidal anti-inflammatory drugs (NSAIDs). In sickle cell disease, renal papillary necrosis is common and occurs secondary to renal medullary ischemia and infarction. It typically presents
with hematuria and acute flank pain. It can also be complicated by urinary tract obstruction and/or infection secondary to necrotic tissue sloughing into the renal collecting system and ureters. It characteristically presents with gross hematuria and proteinuria on
urinalysis.
Incorrect Answers: A, B, C, and D.
Acute glomerulonephritis (Choice A) refers to a variety of glomerular diseases, including nephritic and nephrotic syndromes. Nephritic syndromes typically present with acute renal failure with associated hematuria, red blood cell urine casts, and hypertension.
Nephrotic syndrome typically presents with excessive proteinuria (greater than 3 g/day) hyperiipidemia, hypoalbuminemia, and edema. It would not typically cause ureteral obstruction.
Acute tubular necrosis (Choice B) typically occurs following an ischemic or nephrotoxic insult to the kidneys, which results in necrosis of the tubular epithelium. Granular, muddy brown casts are typical on urinalysis. It would be less likely to cause ureteral
obstruction.
Diabetic nephropathy (Choice C) is a chronic process that occurs following nonenzymatic glycosylation of the glomerular basement membrane and efferent arterioles, characteristically presenting as Kimmelstiel-Wilson lesions on light microscopy. It progresses over
time in patients with diabetes mellitus, initially beginning as microalbuminuria, which can subsequently lead to macroalbuminuria and then end-stage renal disease.
Papillary transitional cell carcinoma (Choice D) typically presents with painless, gross hematuria. It is associated with smoking, aniline dyes, and cyclophosphamide.
Educational Objective: Renal papillary necrosis (RPN) occurs following ischemic, inflammatory, infectious, or toxin-mediated damage to the renal papilla and describes the sloughing and loss of the papillae into the renal collecting system. In sickle cell disease, RPN
presents with hematuria, acute flank pain, and can be complicated by urinary tract obstruction.

r
Previous
r
Next
,.
Score Report
� F
Lab Values Calculator

Help
,,,
Pause
Exam Sectlon 4: Item 22 of 50 National Board of Medical Examiners®
Comprehensive isaslc Science Self-Assessment

✓ 22. A 68-year-old man wi:th dementia, Alzheimer type, has improvement in memory and cognition with donepezil therapy. Which of the following is the most likely mechanism of this beneficial effect?

A ) Blockade of choline reuptake into presynaptic terminals


B} Qlsska<:le gf effosts 9R the RisetiRisresepters QR mussle
G} ERhaRsed iRflux of salGium iRte the presyT'laptiG lermiRal
C} Pre•.•enti9R gf aGe(¥1GheliRe release frem subsequeRtastieR petentials
E ) Prolongation of acetylcholine effects in the synaptic cleft

Correct Answer: E.

Donepezil is a cholinesterase inhibitor, which prolongs acetylcholine effects in the synaptic cleft. Alzheimer dementia is the most prevalent dementia, presenting with progressive cognitive decline that begins with short-tenn memory impairment, progresses to
apraxia and language abnormalities, and culminates in behavioral and personality changes preventing the patient from performing basic activities of daily living. Alzheimer dementia is postulated to result from decreased cholinergic signaling in the cortex and basal
forebrain. Acetylcholinesterase nonnally hydrolyzes acetylcholine in synaptic clefts. Donepezil non-competitively and reversibly inhibits acetylcholinesterase activity and thereby increases the amount of synaptic acetylcholine available for neurotransmission. Though
donepezil may slow the rate of cognitive decline and modesUy improve functionality, donepezil and the other cholinesterase inhibitors are not curative.

Incorrect Answers: A, B, C, and D.

Blockade of choline reuptake into presynaptic terminals (Choice A) will lead to decreased synthesis of acetylcholine, as reuptake of choline is the rate-limiting step rn acetylcholine synthesis. Therefore, this mechanism of action may exacerbate acetylcholine deficits
and would not be helpful as an Alzheimer dementia medication.

Blockade of effects on the nicotinic receptors on muscle (Choice B) is the mechanism of action of muscle relaxants used in anesthesia such as succinylcholine and atracurium. Alzheimer dementia arises from central-not peripheral-acetylcholine deficiency and
would not be affected by medications with this mechanism of action.

Enhanced influx of calcium into the presynaptic tenninal (Choice C) would increase presynaptic neurotransmitter release from vesicles via exocytosis into the synaptic cleft. Botulism and tetanus cause dysfunction of this mechanism of presynaptic exocytosis. No
Alzheimer dementia medications utilize this mechanism.

Prevention of acetylcholine release from subsequent action potentials (Choice D) may occur in botulism. Further decreasing synaptic acetylcholine in Alzheimer dementia may Wt?rsen cognitive function.

Educational Objective: Cognitive impainnent in Alzheimer dementia is related to decreased acetylcholine signaling in the cortex and basal forebrain. Donepezil and other cholinesterase inhibitors decrease degradation of acetylcholine and therefore increase the
amount of acety1choline available for synaptic transmission.

r r , tr§
Lab Values
F
Calculator
r
Hel
,,.
Pause
Previous Next Score Re ort
Exam Section 4: Item 23 of 50 National Board pf Mqdlcal Examiners®
Comprehensive sas1c SClence Self-Assessment

✓ 23. The graph shows the relationship between serum leptin concentration and BMI, expressed as (weight in kg)/ (height in meters)2 for eight patients labeled A through H. The dashed lines indicate the upper and C

A B C
0
lower limits of normal. Which of the following patients most likely has primary leptin deficiency?

---·.-----·'----
'E
A ) Patient A �
B ) Patient B
a D .:' Normal ..: E
C ) Patient C � ---.-• - - - -,-• - - -
D) Patient D
.:; F G H
E ) Patient E (/)

F ) Patient F 2
BMI (kg/m )
G) Patient G
H) Patient H
Correct Answer: H.

Leptin, a peptide hormone, is produced by adipocytes and enterocytes. In the fed state or settings of energy excess (eg, obesity), leptin production is upregulated and acts on hypothalamic nuclei to mediate feeding behavior by reducing the sensation of hunger
and increasing satiety. Counterregulatory to leptin is ghrelin, which is produced in states of low energy balance and stimulates hunger and anabolism. In disease states, mutations of leptin resulting in inactivation or decreased production, or decreased receptor
response to binding by leptin (analogous to type 2 diabetes mellitus) have been associated with obesity. Such derangement results in obesity marked by an increased BMI, reduced desire to exercise, and increased appetite in spite of sufficient caloric intake. In
the graph above, patient H demonstrates an increased BMI and a low serum level of leptin, suggesting a primary leptin deficiency. Primary deficiencies describe an absent or sub-functional protein, which prevents or limits the downstream effect of that protein.

Incorrect Answers: A, B, C, D, E, F, and G.

Choices A, B, and C reflect patients with increased serum leptin, which would not be seen in the case of a primary leptin deficiency. In a primary deficiency, leptin protein would be absent, mutated, or degraded early due to misfolding. Choice C in particular may
reflect a suboptimal response by the hypothalamus to leptin as it is marked by both obesity and increased leptin level.

Choice D and Choice E reflect patients with a normal leptin level with a low BMI or a normal leptin level with a high BMI, respectively. These cases may reflect polymorphisms in leptin generation or response but would not reflect a primary deficiency.

Choices F and G may reflect primary leptin deficiency as the leptin level is low, however a secondary deficit is theoretically present given the low or normal associated BMI. Leptin deficiency should lead to increased BMI promoting feeding behavior and limited
physical activity.

Educational Objective: Leptin is produced by adipocytes and enterocytes in the fed state, and via action on hypothalamic nuclei, mediates feeding behavior by reducing the sensation of hunger and increasing satiety. Primary leptin deficiencies result in low levels
of leptin with increased BMI, a consequence of increased appetite and reduced exercise in the setting of already sufficient energy balance.

r r , fc!f F r ff'
Previous Next Score Report Lab Values Calculator Help Pause
Exam Section 4: Item 24 of 50 National Board of Mqdlcal Examiners®
Comprehensive aas1c Sctence Serr-Assessment

✓ 24. A 50-year-old man is brought to the physician 2 days after the onset of double vision and drowsiness. Three weeks ago, he began using over-the-counter cimetidine for dyspepsia. He has epilepsy treated with phenytoin for the past 2 years. Physical
examination shows nystagmus and an ataxic gait. Which of the following effects of cimetidine on phenytoin is the most likely cause of the adverse effects in this patient?

A}-Altered binding tof)lasma f)mteins


S}-Altered binding to tissue reseptors
G}-Altered distribulioA to asti•Je reseptors
D) Inhibition of metabolism
�}- Inhibition of renal eKGretion

Correct Answer: D.

The cytochrome P-450 (CYP450) superfamily of oxidative enzymes is responsible for the majority of phase I metabolism of drugs. Cimetidine is a potent CYP450 inhibitor, which results in the inhibition of metabolism of multiple drugs, including phenytoin. Phenytoin
is an anticonvulsant agent with neurotoxic effects at increased serum levels. The neurotoxic effects can range from mild nystagmus to ataxia, slurred speech, lethargy, coma, and death. Other potent CYP450 inhibitors include tricyclic antidepressants,
fluoroquinolones, ketoconazole, amiodarone, quinidine, proton pump inhibitors, diltiazem, isoniazid, chloramphenicol, erythromycin, sulfonamides, metronidazole, ritonavir, and grapefruit juice. In the presence of a CYP450 inhibitor, phenytoin is unable to be
metabolized appropriately, resulting in a progressively increasing concentration of phenytoin, which results in the development of neurotoxic symptoms.

Incorrect Answers: A, B, C, and E.

Altered binding to plasma proteins (Choice A) affects the concentration of physiologically active phenytoin in the serum as the majority of drug is bound to albumin. Patients with hypoalbuminemia have a higher risk of toxicity due to a decreased concentration of
bound phenytoin and increased free, physiologically active drug. This is not the mechanism by which cimetidine increases the activity of phenytoin.

Altered binding to tissue receptors (Choice B) is not associated with cimetidine use. Phenytoin acts on voltage-gated sodium channels. It binds preferentially to channels in the inactive state, blocking them from converting to the active state.

Altered distribution to active receptors (Choice C) occurs with activated charcoal administration in cases of known toxic phenytoin ingestion. Activated charcoal binds phenytoin in the gastrointestinal tract to prevent absorption and distribution to receptors.

Inhibition of renal excretion (Choice E) does not significantly affect phenytoin levels as it is primarily cleared by the liver.

Educational Objective: Cimetidine can cause significant drug-drug interactions through the inhibition of CYP450 metabolism, resulting in the increased concentration of drugs typically metabolized by the CYP450 system. For example, phenytoin concentration
increases with the concomitant administration of cimetidine, increasing the risk for abnormally high phenytoin levels and associated neurotoxicity.

r r , ta e r "'
Previous Next Score Report Lab Values Calculator Help Pause
Exam Section 4: Item 25 of 50 National Board pf Mqdlcal Examiners®
Comprehensive tsas1c science Serr-Assessment

✓ 25. A 59-year-old man comes to the physician because of a 3-month history of progressive difficulty swallowing; he has had a 6.8-kg (15-lb} weight loss during this period. He has smoked 1 pack of cigarettes daily for 40 years and consumes four to six beers
daily. He is cachectic. Bilateral wheezes are heard on auscultation of the chest. A chest x-ray shows enlargement of the paratracheal and hilar lymph nodes. Subsequent endoscopy shows a 6-cm mass in the mid esophagus. The mass is centrally
ulcerated with an elevation of the surrounding mucosa! rim. Examination of a biopsy specimen of the mass is most likely to show neoplastic cells with which of the following?

A } Acinar formation
B } Foci of keratinization
C } lntracytoplasmic pigment
D} Neuroendocrine granules
E } Smooth muscle differentiation
Correct Answer: B.

Esophageal cancer typically presents with progressive dysphagia due to the mass effect of the tumor. Initially, the mass is small enough to only obstruct the passage of large, solid foods. As the mass enlarges, passage of all solids becomes affected, along with
some or all liquids. Nonspecific signs of malignancy, such as weight loss, are also seen. Direct extension of the malignancy or reactive lymphadenopathy surrounding the neighboring structures may also cause symptoms. For example, enlargement of paratracheal
and hilar lymph nodes may cause whe.ezing through airway narrowing, and involvement of the recurrent laryngeal nerve may cause hoarseness. The two most common subtypes of esophageal cancer are squamous cell carcinoma and adenocarcinoma. Risk
factors for squamous cell carcinoma including smoking, alcohol use, hot liquids, and the presence of strictures. The upper two thirds of the esophagus are more likely to be affected. In contrast, chronic gastroesophageal reflux disease, Barrett esophagus, and
obesity increase the risk of adenocarcinoma, which is more likely to affect the distal third of the esophagus. In this patient, squamous cell carcinoma is more likely given his significant alcohol and smoking history and location of the mass in the mid esophagus.
Squamous cell carcinoma, regardless of the organ involved, is derived from cells which produce keratin. When squamous cells become aberrant, they continue producing keratin but in small foci called keratin pearls. In contrast, adenocarcinoma is derived from
glandular epithelium and thus forms gland-like structures, even when malignant.

Incorrect Answers: A, C, D, and E.

Acinar formation (Choice A) is a feature of adenocarcinoma. An acinus is a small cavity within a gland. Because adenocarcinomas are derived from glandular epithelium, in malignant form the cells may create gland and duct-like structures.

lntracytoplasmic pigment (Choice C} may be seen in neoplastic melanocytes. Malignant melanoma is a neoplasm of melanocytes. While melanoma can develop in the oral cavity, the esophagus is an unlikely place for primary melanoma to form. Melanoma
m.etastases commonly spread to the skin, lung, brain, and liver, but not the esophagus.

Neuroendocrine granules (Choice D} are seen in tumors derived from neuroendocrine cells such as neuroblastoma, small cell lung cancer, carcinoid tumor, and Merkel cell carcinoma. They are not a feature of squamous cell carcinoma or adenocarcinoma.
Carcinoid tumors are often seen in the gastrointestinal tract, but the esophagus is rarely involved.

Smooth muscle differentiation (Choice E} is a feature of rhabdomyosarcoma or leiomyomata. These are not typically seen in the esophagus.

Educational Objective: Squamous cell carcinoma and adenocarcinoma are the two most common subtypes of esophageal cancer. Carcinoma derived from squamous cells tend to produce foci of keratin, which is one method of differentiating them from
adenocarcinoma.

r
Prevlous
r
Next
,.
Score Report

Lab Values
P
Calculator
r
Help
ttt
Pause
_ . _ _ _ .
Exam Section 4: Item 26 of 50 National Boa.rd pf M11dlcal Examiners®
Comprehensive 8aslc Science Serr-Assessment

✓ 26. A 25-year-old primigravid woman at 12 weeks' gestation comes to the physician 1 hour after she passed a small amount of tissue vaginally at home. Gross examination of this tissue shows an obvious fetus. Microscopic examination of tissue obtained via
dilatation and curettage shows chorionic villi with focal edema and trophoblastic proliferation. Which of the following is the most likely diagnosis?

A} Choriosa�sinoma
B )- Complete hydalidiform mole

--'
C) Partial hydatidiform mole
C} Placenta assrota
E) Placental site trophoblastic tumor

Correct Answer: C.

Hydatidiform moles are either caused by the fertilization of an empty egg by a single sperm followed by duplication of the paternal DNA resulting in 46 chromosomes (complete) or the dual fertilization of a normal ovum resulting in 69 chromosomes (partial).
Maternal symptoms include hyperemesis gravidarum or persistent nausea and vomiting related to abnormally increased human chorionic gonadotropin (hCG) levels. They can also cause first trimester vaginal bleeding, early preeclampsia, and uterine size greater
than expected for dates. Complete moles contain no fetal tissue, while partial moles contain some fetal tissue, but neither are viable pregnancies and no fetal heartbeat will be found on Doppler evaluation. Pelvic examination may demonstrate cystic molar clusters
in the vagina, and laboratory evaluation will reveal abnormally increased 13-hCG levels, with greater increases associated with complete molar pregnancies. Ultrasound of the pelvis will typically show bilateral multilocular ovarian cysts (theca-lutein cysts) and
echogenic structures in the uterus in the setting of a complete molar pregnancy but may only demonstrate fetal parts in a partial molar pregnancy. Molar pregnancies are treated with dilation and curettage, and histology will reveal hypertrophy of the chorionic villi
with focal edema and trophoblastic hyperplasia, as seen in this patient.

Incorrect Answers: A, 8, D, and E.

Choriocarcinoma (Choice A) can develop during or after a regular or molar pregnancy due to the malignant proliferation of trophoblastic tissue. It typically presents with marked increases in 13-hCG and associated vaginal bleeding, as well as shortness of breath and
hemoptysis due to a propensity for metastases to the lungs. Histologically, there will be proliferation of syncytiotrophoblast, cytotrophoblast, and intermediate trophoblast cells, with an absence of chorionic villi. Choriocarcinoma is excluded given the presence of
chorionic villi histologically.

Complete hydatidiform mole (Choice 8) would present similarly to a partial hydatidiform mole with vaginal bleeding and severely increased hCG. However, the presence of fetal tissue excludes this diagnosis.

Placenta accreta (Choice D) refers to the abnormal attachment of the placenta to the myometrium, rather than just the endometrium. It most commonly presents with postpartum hemorrhage due to retained products of conception. Histology will show chorionic villi
directly attached to the myometrium, unlike the histology seen in this patient.

Placental site trophoblastic tumor (Choice E) is also a form of gestational trophoblastic disease, with symptoms including vaginal bleeding. There will be no villi on histologic examination, but there will be trophoblastic cells with irregular nuclei and eosinophilic
cytoplasm, extracellular fibrous deposition, and invasion of blood vessels. This histology is not seen in this patient.

Educational Objective: Molar pregnancies are due to the fertilization of an empty egg by a single sperm followed by duplication of the paternal DNA resulting in 46 chromosomes (complete) or the dual fertilization of a normal ovum resulting in 69 chromosomes
(partial). Complete moles contain no fetal tissue, while partial moles contain some fetal tissue, although neither are viable. Histology will reveal hypertrophy of the chorionic villi with focal edema and trophoblastic hyperplasia.

r
Previous
r
Next
,
Score Report

Lab Values
F
Calculator
r-
Help
If"
Pause
Exam Sectlon 4: Item 27 of 50 National Board of Medical Examiners®
Comprehensive isaslc Science Self-Assessment

✓ 27. A 2½-year-old boy is admitted to the hospital because of generalized weakness, repeated episodes of vomiting, and coma after several days of reduced food intake because of a minor febrile illness. He is treated successfully with intravenous glucose.
After 13 hours of fasting when the child is well, his serum glucose, 13-hydroxybutyrate, and free carnitine concentrations decrease markedly, and serum concentrations of long-chain acylcarnitines increase. Triglycerides containing only medium-chain fatty
acids are fed, and the serum 13-hydroxybutyrate concentration increases to the reference range. Which of the following disorders of fatty acid metabolism is the most likely diagnosis?

A ) Carnitine-acylcarnitine translocase deficiency


Ii)- Dietary camitine deficiency
C)- f'.atty acy.1 Ce.A. synthetase deficiency
0� Medium chain acyl CG.A. dehydmgenase deficiency
E)- Short chain acyl CG.A. dehydmgenase deficiency
Correct Answer: A.

Carnitine-acylcarnitine translocase functions in the transport of camitine and camitine-fatty acid complexes from the cytoplasm into the mitochondrial matrix as part of the carnitine shuttle system of the mitochondrial membrane. Short and medium-chain fatty acids
can enter the mitochondria directly, but longer-chain fatty acids require transport via the camitine shuttle. Carnitine-acylcamitine translocase deficiency disrupts this transport, leading to impaired and decreased fatty acid metabolism, which results in the
accumulation of long-chain fatty acids, particularly in muscle and liver. It is a rare autosomal recessive disorder and onset of symptoms usually occurs in infancy or early childhood. Patients may present with encephalopathy, vomiting, hypoketotic hypoglycemia,
hyperammonemia, muscle weakness, skeletal myopathy, cardiomyopathy, and ventricular dysrhythmias. Symptoms can be triggered by illness or fasting, and treatment involves the administration of a low-fat, high-carbohydrate formula and carnitine
supplementation. Medium-chain fatty acids should only be administered as part of the patient's diet, as these do not require the camitine shuttle.

Incorrect Answers: B, C, D, and E.

Dietary camitine supplements are needed for patients with primary carnitine deficiency (Choice B), a rare disorder of insufficient carnitine that is characterized by poor feeding, irritability, hypoglycemia, and failure to thrive.

Fatty acy1 CoA synthetase deficiency (Choice C) is characterized by the impaired beta-oxidation of long-chain fatty acids. Fatty acyl CoA synthetase catalyzes the formation of fatty acy1 CoA from fatty acids and coenzyme A, which is required prior to the
transportation of fatty acids into the mitochondrial matrix by the camitine shuttle. Deficiency of this enzyme results in the accumulation of long chain fatty acids, which can lead to adrenoleukodystrophy, seizures, coma, and death.

Medium-chain acyl CoA dehydrogenase deficiency (Choice D) is an autosomal recessive disorder characterized by the inability to break down medium-chain fatty acids into acetyl-CoA, leading to the accumulation of medium-chain fatty acyl camitines and
hypoketotic hypoglycemia. Signs and symptoms may include vomiting, liver dysfunction, seizures, and lethargy or coma.

Short-chain acyl CoA dehydrogenase deficiency (Choice E) is typically asymptomatic and requires no treatment.

Educational Objective: Long-chain and very long-chain fatty acids require carnitine-acy1camitine translocase to enter the mitochondrial matrix for beta-oxidation. Patients with deficiency may present with encephalopathy, vomiting, hypoketotic hypoglycemia,
hyperammonemia, muscle weakness, skeletal myopathy, cardiomyopathy, and ventricular dysrhythmias. Symptoms can be triggered by illness or fasting, and treatment involves the administration of a low-fat (medium-chain fatty acids), high-carbohydrate formula
and camitine supplementation.

r
Previous
r
Next
,
Score Re ort

Lab Values
F
Calculator
r
Hel
,,.
Pause
Exam Section 4: Item 28 of 50 National Board of Mqdlcal Examiners®
Comprehensive aas1c Sctence Serr-Assessment

✓ 28. A 52-year-old man comes to the physician because of a 5-day history of headaches and muscle weakness. His blood pressure is 175/95 mm Hg. Physical examination shows no other abnormalities. Laboratory studies show:
Serum
Na+ 142 mEq/L
K+ 2 mEq/L
HC03- 33 mEq/L
Creatinine 1.4 mg/dL
Cortisol <2 µg/dL
Arterial blood gas analysis on room air:
pH 7.45
Pco 2 48 mm Hg

Plasma renin activity is undetectable. Which of the following is the most likely cause of this patient's hypertension?

A}-Addison disease
Iii}- Chronic kidney disease
G}- Cl,ishing disease
D) Hyperaldosteronism
E ) Pheochromocytoma
Correct Answer: D.

Aldosterone is produced by the zona glomerulosa of the adrenal cortex and plays an important role in the maintenance of blood volume and sodium homeostasis, primarily through the renin-angiotensin-aldosterone pathway. Renin is produced by the
juxtaglomerular cells of the kidney as a response to decreased blood flow, decreased sodium delivery to the macula densa, or increased sympathetic tone. Renin converts angiotensinogen to angiotensin I, which is converted to angiotensin II by angiotensin­
converting enzyme (ACE). Angiotensin II subsequently signals the adrenal gland to secrete aldosterone. Aldosterone promotes the reabsorption of sodium ions in exchange for potassium and hydrogen ion secretion in the distal convoluted tubule and collecting
ducts, causing the indirect reabsorption of water, which results in the development of hypertension. High levels of aldosterone provide negative feedback to renin, decreasing its production. Primary hyperaldosteronism presents with hypertension, metabolic
alkalosis, hypokalemia, and an increased aldosterone-to-renin ratio due to the independent secretion of aldosterone, most commonly from an adrenal adenoma or bilateral adrenal hyperplasia.

Incorrect Answers: A, B, C, and E.

Addison disease (Choice A) or primary adrenal insufficiency is most commonly due to autoimmune destruction or tuberculosis. It presents with hypotension, hyperkalemia, metabolic acidosis, and skin hyperpigmentation secondary to a deficiency of aldosterone and
cortisol. It does not describe this patient's clinical condition.

Chronic kidney disease (Choice B) presents with elevated creatinine, as well as hyperkalemia due to the inability to secrete potassium, and metabolic acidosis due to the inability to reabsorb bicarbonate. It would not cause hypokalemia and metabolic alkalosis, as
in this patient.

Cushing disease (Choice C) occurs due to elevated cortisol levels and presents with truncal weight gain, striae, muscle weakness, and fatty deposits around the face and neck. Vital signs will demonstrate hypertension, and labs will reveal hypokalemia and
hyperglycemia. This patient's cortisol levels are low.

Pheochromocytoma (Choice E) presents with episodes of increased blood pressure, headache, and palpitations, most commonly due to a tumor of the adrenal medulla. It does not cause hypokalemia or metabolic alkalosis, and the symptoms would be episodic,
unlike in this patient.

Educational Objective: Primary hyperaldosteronism presents with hypertension, hypokalemia, metabolic alkalosis, and an increased aldosterone-to-renin ratio due to the independent secretion of aldosterone. It is most commonly associated with an adrenal
adenoma or bilateral adrenal hyperplasia.

r r , fa P r "'
Previous Next Score Report Lab Values Calculator Help Pause
Exam Section 4: Item 29 of 50 National Board pf Medical Examiners®
Comprehensive 1::1as1c sc:1ence sen-Assessment

✓ 29. A 35-year-old homeless man is brought to the emergency department by police after they found him slumped on a bench. His pulse is 40/min, respirations are 10/min, and blood pressure is 100/60 mm Hg. He is stuporous but can be aroused when shaken
mildly. Physical examination shows constricted pupils. This patient is most likely abusing which of the following substances?

A ) Alcohol
B ) Barbiturates
C ) Benzodiazepines
D) Marijuana
E ) Opioids

Correct Answer: E.

This patient is likely intoxicated with opioids, which are central nervous system (CNS) depressants used as analgesics and recreational drugs. Opiates act throughout the CNS and peripheral nervous system and interact with several neurotransmitter systems,
leading to diverse effects. Opiate intoxication causes euphoria (due to interaction with dopamine), altered mental status, sedation, bradycardia and hypotension, depressed respiratory drive, and constricted pupils. Miosis is a distinctive finding that is less common in
intoxication with other CNS depressants and is caused by direct opiate receptor activity in brain areas responsible for pupillary control. Opiates also act on receptors within the enteric nervous system, reducing gut motility and causing constipation.

Incorrect Answers: A, B, C, and D.

Alcohol (Choice A) is a CNS depressant that can cause slurred speech, emotional !ability, behavioral disinhibition, ataxia, and memory lapses (similar to benzodiazepine intoxication). Alcohol intoxication typically results in tachycardia, not bradycardia. In severe
intoxication, stupor and respiratory depression can occur.

Barbiturates (Choice B) are CNS depressants with similar toxidromes to alcohol and benzodiazepines. Bradycardia can occur, and the risk of dangerous respiratory depression is higher in barbiturates than benzodiazepines or alcohol. Pupils are typically of normal
size or dilated in barbiturate intoxication, and physical examination may reveal nystagmus.

Benzodiazepines (Choice C) are CNS depressants with a toxidrome of slurred speech, ataxia, and altered mental status. Co-ingestion with an opiate more commonly causes respiratory depression than benzodiazepine intoxication alone. Nystagmus may be seen,
and pupillary constriction would be atypical.

Marijuana (Choice D) is a hallucinogen that causes euphoria, paranoia, hallucinations, cognitive slowing, cdnjunctival injection, and dilated pupils. Tachycardia and hypertension are likely to be seen as opposed to this patient's bradycardia and hypotension.

Educational Objective: Opiate intoxication causes symptoms of central nervous system depression including sedation, respiratory depression, bradycardia, and hypotension. Miosis differentiates opiate intoxication from other CNS depressant toxidromes.

r
Prev1ous
r
Next

Score Report
tr!i
Lab Values
P
Calculator
r
Help
trt-
Pause
Exam Section 4: Item 30 of SO National Board pf M�dlcal Exftllllners®
Comprehensive sas1e science serr-As!l8ssment

y 30. A 22-year-old man is brought to the emergency department 1 hour after being involved in a motor vehicle collision. Physical examination shows deep lacerations to the right lower extremity. Which of the following functional capabilities of the involved
muscles will be decreased by fibrotic scar formation at the sites of injury?

A) Formation of glycogen deposits


B) Maintenance of prolonged tension
C) Passive stretching
D) Relaxation after contraction
Correct Answer: C.
Skeletal muscle cells (myocytes) convert energy stored as adenosine triphosphate into mechanical force via myosin motor fiber interaction with thin actin filaments. Myosin-actin interactions are regulated by troponins and calcium ions from the extracellular space
and sarcoplasmic reticulum. Muscle injury and healing, particularly when muscle fibers are completely severed, is characterized by fibrosis and scarring within the muscle. Muscle fibers do not regenerate once destroyed. Areas of injured muscle are repaired by
fibroblasts and myofibroblasts with collagen, forming a scar. Scarring of muscle with collagen will lead to decreased muscle movement as well as decreased range of motion of the nearby joints. In contrast to intact myofibrils with actin and myosin filaments that can
slide past each other, collagen fibers are fixed in position and do not stretch or deform significantly. Muscle injuries, such as ischemia or crush, can lead to near1y complete replacement of muscle with scar tissue resulting in joint contractures, limited passive
stretching, and limited mobility.
Incorrect Answers: A, B, and D.
Formation of glycogen deposits (Choice A) within a particular myocyte is not affected by the presence of scar tissue. Glycogen is a form of carbohydrate energy storage. Glycogen is synthesized during states of energy availability in both the liver and in skeletal
myocytes. Glycogen stores are utilized during starvation states or high-energy exercise.
Maintenance of prolonged tension (Choice B) is commonly needed in tissues that are comprised of smooth muscle that have basal tonic contraction. For example, the smooth muscle of the vasculature is required to have a basal force output. Myosin cross-bridges
known as latch-bridges cycle at a slow rate which facilitates reduced energy requirements with sustained contraction. Skeletal muscle generally cycles much more rapidly.
Relaxation after contraction (Choice D) is impaired in myotonic states and is characterized by a prolonged depolarization of the muscle fibers that occurs when the sarcolemma does not halt calcium release. Myotonia is a characteristic clinical finding in disorders
such as myotonic dystrophy and myotonia congenita, which are genetic disorders. It is not necessarily related to traumatic muscle injury.
Educational Objective: After injury to muscle, there is limited regeneration capacity. Damaged muscle is often replaced by fibrotic scar, which leads to stiffness, reduced passive stretching, and joint contractures.

r
Previous
r
Next
,,.
Score Report

Lab Values
F
Calculator

Help
trt
Pause .
Exam Section 4: Item 31 of 50 National Boa.rd of Medical Exi!J'lllners®
Comprehensive 1:Jas1c Science Sert-Assessment

✓ 31. A study is conducted to assess the prevalence and incidence of obesity in a group of middle-aged men. A total of 900 men between the ages of 40 and 50 years are chosen to participate. The table shows the number of participants at baseline and at the
follow-up study 5 years later for each BMI range.
Number of Participants
BMI (kg/m2) Baseline 5 Years Later
Normal(< 25) 300 225
Overweight(between 25 and 30) 500 525
Obese(� 30) 100 150

The prevalence of obesity in this study population after 5 years is which of the following?

A ) 50/100
B) 50/500
C) 50/800
D) 50/900
E ) 150/375
F ) 150/525
G) 150/750
H) 150/900

Correct Answer:. H.

Prevalence is defined as the number of cases of a disease divided by the total number of persons in the at-risk population. It describes the percentage or fraction of those who are affected within those who might be affected. Point prevalence is the prevalence of a
disease at a specific point in time. This distinction can be useful as the prevalence of a disease may change over time. In this study, the prevalence of obesity at the start of the study(100 obese persons/900 middle-aged men at risk) is less than the prevalence of
obesity after five years(150 obese persons/900 middle-aged men at risk).

Incorrect Answers: A, B, C, D, E, F, and G.

50/100(Choice A), calculates the percentage increase in prevalence over time. At year 5, 150 persons were identified as obese. 50/100 represents a 50% increase in obesity, calculated as 150 obese persons - 100 obese persons / 100 obese persons.

50/500(Choice 1B), calculates the ratio of the number of people newly diagnosed with obesity at time= 5 years to the number of persons who were overweight at the start of the study.
50/800(Choice C), represents another important epidemiologic measure that can be contrasted with prevalence, incidence. Incidence is the number of new cases over the total population at risk over a specified period of time. The cumulative incidence of new
diagnoses of obesity in this population is 50 (the number of new cases occurring during the study). Dividing this by 800(the total number of persons at risk at the beginning of the study period) reflects the incidence.
50/900(Choice D), calculates the ratio of the number of people newly diagnosed with obesity at time= 5 years to the total number of participants in the study.

150/375(Choice E), calculates the percentage of obese persons at time= 5 years within the group of persons having a normal or obese BMI at that time.

150/525(Choice F), calculates the ratio of the number of obese persons at time= 5 years to the number of persons who were overweight at that time.

150/750(Choice G), calculates the ratio of the number of obese persons at time= 5 years to the number of persons who were normal or overweight at that time.

Educational Objective: Prevalence is the number of cases of a disease at a specific point in time divided by the population at risk. Incidence refers to the number of new cases of a disease over a period of time.

r
Previous
r
Next
,:
Score Report
tr5
Lab Values
P
Calculator
r
Help
ti"
Pause
Exam Section 4: Item 32 of 50 National Board pf Medical Examiners®
Comprehensive Basic Science Self-Assessment

✓ 32. A 55-year-old man comes to the physician because of a 2-month history of hoarseness and difficulty swallowing. He underwent an operation to resect a lobe of his lung because of lung cancer 1 year ago. Physical examination shows ptosis of the left
eyelid and miosis of the left pupil. There is diminished flushing and sweating on the left half of the face. This patient most likely has disordered function of neurons at which of the following labeled locations in the photograph of a cross section of the spinal
cord?

Left Right
A}-­
B)
G} -­
O}-­
E)
�} --
Correct Answer: B.
This patient likely has a lesion of the ciliospinal center of Budge (labeled "B"), which includes spinal cord levels CB to T2 of the intermediolateral cell column (also known as the intermediate horn). This patient demonstrates Horner syndrome, which classically
presents with the triad of ipsilateral miosis, ptosis, and anhidrosis. Homer Syndrome can arise from lesions along a sympathetic pathway. This pathway starts. in the hypothalamus and descends ipsilaterally to the first synapse in the ciliospinal center of Budge.
Second-order neurons exit the spinal cord and enter the cervical sympathetic chain (in close proximity to the pulmonary apex and subclavian artery), where they then ascend to the superior cervical ganglion. Some• third-order neurons branch from this ganglion to
innervate the sweat glands and vasculature of the face, while others ascend along the internal carotid! artery, eventually joining with the oculomotor nerve to enter the orbit and innervate the pupil and eyelid smooth muscle. Miosis and ptosis occur as a result of
denervation of the sympathetic supply to the pupil and superior tarsal muscle respectively, while anhidrosis happens due to denervation of the sweat glands. Lesions distal to the branching of the nerves innervating the sweat glands and vasculature of the face may
exclude facial anhidrosis. Etiologies of Homer syndrome can include intracranial pathology (eg, stroke, trauma, multiple sclerosis) as well as diseases that compress the nerves along the sympathetic pathway such as malignancies involving the lung apex,
subclavian artery aneurysm, and internal carotid artery dissection. Diagnostic workup should include head, neck, and/or chest cross-sectional imaging (eg, CT scan, MRI) depending on the suspected etiology. Appropriate therapeutic management is based on the
underlying etiology.
Incorrect Answers: A, C, D, E, and F.
The nucleus dorsalis (Choices A and D), also known as the column of Clarke, is a group of interneurons that assists in conveying proprioceptive information to the cerebellum. This nucleus is not involved in Horner syndrome.
The right-sided ciliospinal center of Budge (Choice E) houses sympathetic fibers innervating the right-sided face and eye. A lesion at this location would cause right-sided Horner syndrome.
The anterior horn (Choices C and F ) is the location of the cell bodies of lower motor neurons. Lesions. of the anterior horn would result in an ipsilateral lower motor neuron pattern of skeletal muscle weakness including hyporeflexia and decreased muscle tone.
Educational Objective: Lesions of the intermediolateral cell column, or intermediate horn, between spinal cord levels CB and T2 can interrupt the sympathetic nerve supply to the ipsilateral face and eye, and cause Homer syndrome. Homer syndrome classically
presents with ipsilateral ptosis, miosis, and anhidrosis.

r r , � F r r
Previous Next Score Re ort Lab Values Calculator Hel Pause
Exam Section 4: Item 33 of 50 National Boll.rd pf M-idlcal Examiners®
Comprehensive tsas1c science sen-ABBessment

✓ 33. A 28-year-old woman, gravida 3, para 2, comes to the physician to request an elective abortion 2 days after she received a positive home pregnancy test result. Her last menstrual period was 6 weeks ago. Ultrasonography shows a uterus consistent in
size with a 6-week gestation. After counseling, she decides to undergo a medication-induced abortion rather than a surgical abortion. An oral medication is administered, and she is instructed to return to the office in 2 days to receive a second medication.
The first drug administered most likely belongs to which of the following classes?

A)- Human chorionic gonadotropin antibodies


B ) Oxytocics
C) Progesterone antagonists
C)- Proslaglandin synlt:lesis inhibitors
e)- Selecli•�e estrogen reGeptor modulators
Correct Answer: C.

Mifepristone is a competitive inhibitor of progesterone and is commonly used along with misoprostol for the elective termination of pregnancy. Mifepristone blocks the effects of progesterone on the uterus, causing dilation of the cervix. It also causes the placenta to
separate from the uterus due to changes in the endometrium and primes the uterine muscles for the action of misoprostol, which is a synthetic prostaglandin that causes uterine contractions. This combination of medications can be used up to 77 days of gestational
age, with mildly less efficacy after 10 weeks. Side effects of this medication regimen include uterine cramping, vaginal bleeding, nausea, vomiting, joint pain, incomplete termination, and rarely pelvic inflammatory disease. Mifepristone should not be used in adrenal
failure, chronic steroid use, porphyria, bleeding diatheses, or ectopic pregnancy.
Incorrect Answers: A, B, D, and E.

Human chorionic gonadotropin (hCG) antibodies (Choice A) would also lead to abortion, as the lack of hCG would cause the corpus luteum to degenerate, resulting in the subsequent loss of progesterone and disruption of the placental-endometrial interface.
However, antibodies are not used to induce medical abortions.

Oxytocics (Choice B) are medications that work similarly to endogenous oxytocin, causing uterine contraction, particularly in the setting of uterine atony. They are commonly used to induce delivery but are not used to induce the elective termination of pregnancy.

Prostaglandin synthesis inhibitors (Choice D) include non-steroidal anti-inflammatory drugs (NSAIDs), COX-2 selective inhibitors, and corticosteroids. Decreased levels of prostaglandins would cause a decrease in uterine contraction, making these medications an
inappropriate choice for medical abortion.

Selective estrogen receptor modulators (Choice E) include tamoxifen and raloxifene, which exhibit different effects on the estrogen receptor in different tissues. They are used to treat breast cancer and postrnenopausal osteoporosis but are not used in the process
of an elective termination of pregnancy.

Educational Objective: Mifepristone is a progesterone antagonist that is commonly used along with misoprostol to induce medical abortion. It causes dilation of the cervix and placental separation, along with the augmentation of uterine contractions caused by
misoprostol.

r
Previous
r
Next

Score Report
ffi
Lab Values
F
Calculator

Help
1ft
Pause
Exam Section 4: Item 34 of 50 National Board pf Medical Examiners®
Comprehensive Basic Science Sen-Assessment

34. A 1-year-old boy is brought to the emergency department because of a 6-day history of temperatures to 39.4° C (103 ° F) and a 2-day history of a severe diaper rash and swelling of his hands and feet. His temperature is now 39.4° C (103 ° F). A photograph
of the genital area is shown. The face and lips appear red and the conjunctivae appear injected. There is bilateral cervical lymphadenopathy. The lungs are clear to auscultation. Cardiac examination shows an S 3 gallop with no murmur. There is edema
and erythema of the hands and feet. Which of the following pathologic findings is most likely in ltlis patient?

A ) Acute arteritis with aneurysms in coronary arteries


B ) Epidermal hyperplasia with epidermal microabscesses and parakeratosis
G ➔ GI l4k>.. � •neGrosis in G81'\'iGi>l lympt:i nod,qs
C \ Gr..n1.1l1 .+" ......111&1'1 ,n cel'\'iGal and tempor.., arterie1
E ) Paracortical lymphoid hyperplasia with eosinophilic intranuclear inclusions in perihilar lymph nodes
Correct Answer: A.

Kawasaki disease is a multisystemic vasculitic disease resulting in acute arteritis with a risk for the development of aneurysms in coronary arteries. In children, the disease is characterized by fever for at least 5 days, bilateral nonexudative conjunctivitis, a
polymorphous rash, lymphadenopathy, erythema and edema of the hands and feet, and erythema affecting the mucous membranes with dry, cracked lips. To diagnose Kawasaki disease, fever for 5 days or more plus four of the remaining criteria are required.
Laboratory evaluation may show thrombocytosis and increased erythrocyte sedimentation rate. Complications of Kawasaki disease include coronary artery aneurysms, and if untreated, the potential for aneurysmal expansion, rupture, or myocardial infarction.
Therefore, children suspected of having Kawasaki disease require immediate evaluation with ECG and echocardiography, immediate treatment, and monitoring for progression or complications. Treatment of Kawasaki disease involves high-dose aspirin and
intravenous immunoglobulin, which is the only widely accepted indication for aspirin use in febrile children due to the risk of Reye syndrome. Kawasaki disease is also known as mucocutaneous lymph node syndrome, is one of the most common childhood
vasculitides, and is a common cause of pediatric heart disease.

Incorrect Answers: B, C, D, and E.

Psoriasis would demonstrate epidermal hyperplasia with epidermal microabscesses and parakeratosis (Choice B) on histopathology. While psoriasis can present with an inverse subtype in which shiny, pink patches form in the inguinal folds, it would not explain the
patient's fever, mucosa! findings, or cervical lymphadenopathy. Psoriasis is associated with cardiovascular disease in adults, but it is not a cause of pediatric heart disease.
Granulomas with caseous necrosis in cervical lymph nodes (Choice C) characterizes granulomatous lymphadenitis, an infection of the lymph nodes. Paracortical lymphoid hyperplasia with eosinophilic intranuclear inclusions in perihilar lymph nodes (Choice E)
c;haracteriz;es viral lymphadenitis caused by cytomegalovirvs (CMV). Kawasaki disease is an inflammatory rather than infectious prQCess and neither bacterial nor viral pathogens are involved.

Granulomatous arteritis in cervical or temporal arteries (Choice D), also referred to as giant cell or temporal arteritis is a vasculitis that affects the large vessels such as the aorta, aortic arch vessels, temporal branch of the external carotid, and vertebral arteries. It is
characterized by headache, temporal tendernes-s, jaw claudication, and fever. Complications include monocular blindness from inflammation involving the retinal branch arteries, and therefore requires the urgent administration of glucocorticoids, ophthalmologic
evaluation, and temporal artery biopsy for diagnosis. It is rarely seen in children.

Educational Objective: Kawasaki disease is characterized by fever for at least 5 days, bilateral nonexudative conjunctivitis, a polymorphous rash, lymphadeniopathy, erythema and edema of the hands and feet, and erythema affecting the mucous membranes with
dry, cracked lips. Coronary artery aneurysms are a potential complication of Kawasaki disease and all children should undergo electrocardiogram and echocardiography for screening and future monitoring, along with treatment involving high-dose aspirin and
intravenous immunoglobulins.
Exam Secllon 4: Item 35 of 50 Nallonal Board pf Medical Examiners®
Comprehensive Basic Science Serr-Assessment

✓ 35. A 21-year-old man comes to the physician because of a lump in his scrotum for 3 months. Physical examination shows a 1-cm nontender nodule in the right scrotum. Transillumination shows a solid, noncystic structure. An operation to remove the testis is
done. After incision of the skin and subcutaneous tissue, the next fascia! layer to be incised in this patient is a direct extension of which of the following abdominal layers?

A ) Dartos muscle
B) External abdominal obli�ue muscle
C ) Internal abdominal oblique muscle
D ) Peritoneum
E ) Transversalis fascia
Correct Answer: B.

From superficial to deep, the layers of the scrotum include the skin, the Dartos muscle and fascia, the external spermatic fascia, the cremaster muscle and fascia, the internal spermatic fascia, and the tunica vaginalis. Each of these layers is derived from a layer of
the abdominal wall because the fetal testes originated in the abdomen. Early in the third trimester the testes begin to migrate through the inguinal canal and into the scrotum, carrying with them the abdominal wall coverings. While the structures are contiguous, they
are named differently in the abdomen and scrotum. In this case, the skin and next layer, the Dartos muscle and fascia (subcutaneous tissue), have been incised. The following layer is the external spermatic fascia which is derived from the external abdominal
oblique muscle aponeurosis.

Incorrect Answers: A, C, D, and E.

The Dartos muscle (Choice A) is derived from Scarpa fascia and is the second layer that would be incised when performing an orchiectomy and has already been incised in this patient.

The internal abdominal oblique muscle (Choice C) will produce the cremasteric muscle and fascia in the scrotum. This layer is deep to the external spermatic fascia.

After migration, the abdominal peritoneum (Choice D) becomes the tunica vaginalis, which is the deepest layer of the scrotum and forms a protective sheath around the testes.

The transversalis fascia (Choice E) will produce the internal spermatic fascia of the scrotum, which is the layer deep to the cremasteric muscle and fascia.

Educational Objective: Each layer of the testes is derived from a layer of the abdominal wall (listed in parentheses). From superficial to deep, the layers of the scrotum include the skin (skin), the dartos muscle and fascia (Scarpa fascia), the external spermatic
fascia (external abdominal oblique muscle and fascia), the cremaster muscle and fascia (internal abdominal oblique muscle), the internal spermatic fascia (transversalis fascia), and tunica vaginalis (peritoneum).

r
Prev1ous
r
Next
,
Score Report
� P
Lab Values Calculator
r
Help
r-
Pause
_
Exam Section 4: Item 36 of 50 National Board of Medical Examiners®
Comprehensive aastc Science Self-Assessment

y 36. A 53-year-old man comes to the physician for a follow-up examination. He began antibiotic therapy 1 month ago for acute bacterial pneumonia. At the time of diagnosis, a chest x-ray showed patchy pneumonic consolidation and a 2.5-cm peripheral nodular
lesion in the right upper lobe. His symptoms have since resolved, and an x-ray today shows absence of the pneumonic consolidation. However, the lesion is still present. A PPD skin test is nonreactive. A wedge resection is done by thoracoscopy. Histologic
examination of the resected tissue shows a solid, well-circumscribed nodule composed predominantly of mature cartilage admixed with small amounts of fibrous tissue and occasional clefts lined by benign respiratory epithelium. Which of the following best
describes this patient's pulmonary lesion?

A) Abssess
Iii) Adenoma
C) Choristoma
D) Hamartoma
Ii) Teratoma
Correct Answer: D.

A hamartoma is a benign, disorganized overgrowth of histologically normal tissue in its native location within the body. In the lungs, they may consist of hyaline cartilage, adipose, respiratory epithelium, and/or smooth muscle cells. Pulmonary hamartomas are
common benign nodules (often presenting as a solitary lung nodule) that are typically asymptomatic and discovered incidentally on imaging. They may occasionally present with hemoptysis, bronchial obstruction, and cough. Radiographic features include a well­
circumscribed nodule with either smooth or lobulated borders. Calcification may be present. These lesions should be further evaluated with CT or PET imaging, with consideration of biopsy to rule out malignancy. Histology will show normal, mature cells in a
disorganized pattern. Surgical resection is curative. Patients without known risk factors for malignancy (eg, smoking history) and without high-risk features on CT or PET imaging may be followed with surveillance CT scans to monitor for growth instead of
undergoing resection.

Incorrect Answers: A, B, C, and E.

Pulmonary abscess (Choice A) typically appears as a cavitary lesion on chest imaging with an air-fluid level. Patients will present with fever, malaise, and cough with sputum production (which is often foul-smelling). Histologic examination, if indicated, would reveal
a necrotic mass of leukocytes and microbes.

Adenoma (Choice 8) describes a benign neoplasm of epithelial cell origin. Adenomas of the respiratory tract and lungs are rare lesions that may have potential for malignant transformation. They are• less common than hamartomas and may present with airway
obstruction or post-obstructive pneumonia.

Choristoma (Choice C) refers to a mass of histologically normal tissue that has developed in an abnormal location. Examples include Meckel diverticulum (ectopic gastric tissue in the distal ileum) and endometriosis (ectopic ovarian tissue in various sites).

Teratoma (Choice E) is a germ cell tumor that contains tissue from one or all three germ cell layers (endoderm, mesoderm, ectoderm), which results from abnormal migration of pluripotent stem cells during development. They are typically benign when mature and
may contain multiple mature tissue types on histologic examination.

Educational Objective: Incidental pulmonary nodules are common findings on imaging. Pulmonary hamartomas are disorganized collections of histologically normal lung tissue, which are commonly discovered as solitary pulmonary nodules. They have a low risk of
transforming into a malignant neoplasm.

r
Previous
,.
Next
"
Score Report
fr!i
Lab Values
P
Calculator

Help
,,..
Pause
Exam Section 4: Item 37 of 50 National Boll.rd pf M-idlcal Examiners®
Comprehensive tsas1c science sen-ABBessment

✓ 37. A previously healthy 32-year-old man is brought to the emergency department because of a 1-day history of intermittent flashing spots and blurred vision, vomiting, confusion, and difficulty walking. He is a painter and says that yesterday he cleaned up a
spilled bottle of paint thinner. Ophthalmologic examination shows dilated pupils with hyperemia of the optic disc and retinal edema. Physical examination shows tachypnea. Serum studies show an anion gap metabolic acidosis. The most appropriate
pharmacotherapy for this patient is most likely to inhibit the activity of which of the following?

A ) Alcohol dehydrogenase
lil}-Aldehyde dehydmgeAase
C) Formaldehyde dehydrogenase
C}- lastate dehydrogeAase
e}- Pyru'.'ate dehydmgeAase
Correct Answer: A.

Methanol is commonly used as paint thinner or as an additive in paint thinner. Methanol can be absorbed into the body via ingestion, inhalation, and skin or eye contact. Once absorbed, methanol can be converted by alcohol dehydrogenase to formaldehyde, which
is then converted by aldehyde dehydrogenase into formic acid, a toxic metabolite that can cause visual disturbances, retinal edema, optic disc hyperemia, gastrointestinal distress (eg, abdominal pain, nausea, vomiting), putaminal hemorrhage, and neurologic
depression (eg, confusion, ataxia). The production of formic acid results in the development of an anion gap metabolic acidosis with a compensatory respiratory alkalosis, manifesting as tachypnea. Treatment requires the administration of fomepizole, which inhibits
alcohol dehydrogenase and blocks the conversion of methanol to its toxic metabolites. Severe methanol toxicity resulting in refractory metabolic acidosis, refractory electrolyte abnormalities, visual disturbances, or deteriorating vital signs may require dialysis.

Incorrect Answers: B, C, D, and E.

Aldehyde dehydrogenase (Choice B) and formaldehyde dehydrogenase (Choice C) are different subtypes of an enzyme catalyzing the second step in metabolic processing of alcohol, methanol, or ethylene glycol following the initial enzymatic conversion by alcohol
dehydrogenase. Formaldehyde dehydrogenase converts formaldehyde into formic acid in methanol metabolism, while aldehyde dehydrogenase converts acetaldehyde into acetate in alcohol metabolism, and glycoaldehyde into glycolic acid in ethylene glycol
metabolism. Disulfiram inhibits some isoforms of aldehyde dehydrogenase but is not used in the acute management of methanol toxicity.

Lactate dehydrogenase (Choice D) is involved in anaerobic metabolism with the conversion of pyruvate to lactate. It has no direct role in methanol metabolism.

Pyruvate dehydrogenase (Choice E) is involved in the conversion of pyruvate to acetyl-CoA to participate in the citric acid cycle for ATP synthesis. It has no direct role in methanol metabolism.

Educational Objective: Methanol can be metabolized to formic acid, a toxic metabolite that can cause visual disturbances, retinal edema, optic disc hyperemia, gastrointestinal distress, putaminal hemorrhage, and neurologic depression. Fomepizole inhibits alcohol
dehydrogenase and thus blocks the conversion of methanol to its toxic metabolites.

r
Previous
r
Next

Score Report
ffi
Lab Values
F
Calculator

Help
1ft
Pause
Exam Secllon 4: Item 38 of 50 Nallonal Board pf Medical Examiners®
Comprehensive Basic Science Serr-Assessment

✓ 38. A 68-year-old woman with a 20-year history of hypertension is brought to the emergency department 30 minutes after the sudden onset of right-sided weakness. Her speech is dysarthric. Physical examination shows drooping of the right side of the face.
Visual field testing shows no deficits. Muscle strength is 4/5 in the right upper extremity and 3/5 in the right lower extremity. Deep tendon reflexes are 2+ in the upper extremities and left lower extremity and 3+ in the right lower extremity. Sensation is intact
throughout. Babinski sign is present on the right. Mental status examination shows no abnormalities. Which of the following is the most likely cause of this patient's condition?

A}- Bor:der zone hypepertusion


B}- embolism of the i:ight anterior GerebraIar:tery
C) Lacunar infarct of the internal capsule
D) Ruptured aneurysm of the circle of Willis
e}-Thrombus of tho left posterior Gerebral ar:tery
Correct Answer: C.

A lacunar infarct of the left internal capsule would explain this patient's contralateral hemiparesis. Lacunar infarcts are small non-cortical infarcts that occur due to the blockage of small penetrating arteries that branch from the large arteries of the Circle of Willis.
Chronic hypertension and diabetes lead fo endothelial damage with subsequent hyaline thickening and sclerosis of the arterial wall, resulting in blockage and ischemic damage. Lacunar infarcts most commonly affect the internal capsule or basal ganglia. The
internal capsule primarily contains the descending corticospinal tract, though also contains axons of various additional pathways, including third-order sensory neurons traveling from the thalamus to the primary sensory cortex. Internal capsule lacunar infarcts can
present with purely motor deficits that cause contralateral hemiparesis, ataxic hemiparesis (contralateral hemiparesis plus ataxia), or clumsy hand-dysarthria (hand weakness plus articulation difficulties). Occasionally, sensory symptoms can accompany the motor
weakness.

Incorrect Answers: A, B, D, and E.

Border zone hypoperfusion (Choice A), which refers to infarcts of brain areas between the major cerebral arteries (eg, anterior, middle, and posterior cerebral arteries), typically occurs in states of systemic hypoperfusion such as hypovolemic shock. Symptoms may
include cortical blindness, stupor, and weakness of the bilateral proximal upper and lower extremities.

Embolism of the right anterior cerebral artery (Choice B) would likely present with left-sided sensory and motor deficits of the lower extremity due to an infarct of the medial primary motor and sensory cortices. In this patient, sensation is intact, and the upper
extremities and face demonstrate weakness, which is inconsistent with an anterior cerebral artery embolism.

Ruptured aneurysm of the circle of Willis (Choice D) typically leads to subarachnoid hemorrhage. Subarachnoid hemorrhages present acutely with severe headache and an absence of focal neurological deficits since the hemorrhage occurs in the subarachnoid
space rather than the brain parenchyma.

Thrombus of the left posterior cerebral artery (Choice E) would affect the primary visual cortex in the left occipital lobe. This would result in right-sided hemianopsia with macular sparing.

Educational Objective: Lacunar infarcts are ischemic strokes that occur in small penetrating arteries, most commonly involving the basal ganglia or internal capsule . Lacunar strokes of the internal capsule typically result in contralate:ral hemiparesis.

r r , � P r r-
Prev1ous Next Score Report L�b Values Calculator Help Pause
_ .
Exam Section 4: Item 39 of 50 National B011.rd pf M11dical Exftll}lners®
Comprehen&lve 11111c: science sen-A&Sessment

39. A 57-year-old man is brought to the emergency department because of a 2-hour history of severe abdominal pain and vomiting. Abdominal examination shows diffuse tenderness. A CT scan of the abdomen and abdominal radiograph are shown; the arrows
indicate an abnormality. The most likely cause of these findings is perforation of which of the following structures?

A ) Duodenum
B}-ESGphagus
C) Ileum
• D) Stomach
Iii} +raAs•,<ei:se coloA

Correct Answer: A.

Peptic ulcer disease describes the presence of ulcers in the stomach or duodenum, which classically present with worsening abdominal pain related to consumption (gastric) or lack of consumption (duodenal) of food. Gastric ulcers can be caused by prolonged or
excess usage of nonsteroidal anti-inflammatory drugs which inhibits cyclooxygenase enzymes in the gastrointestinal tract mucosa, leading to a reduction of prostaglandin secretion and decreased protection of the gastric mucosa. However, the most common cause
of peptic ulcer disease is infection with the bacterium Helicobacter pylori, which accounts for nearly all duodenal ulcers. Persistent inflammation related to a peptic ulcer can result in complications including fibrosis, stricture, hemorrhage, and perforation. Anterior
wall ulcers are associated with a higher risk of perforation, while posterior wall ulcers. demonstrate an increased risk of hemorrhage given their position near the gastroduodenal artery. Perforation commonly presents with acute abdominal pain, peritonitis, fever, and
free air on radiographs and CT. This patient's CT demonstrates air within the retroperitoneum, which indicates the perforation must have occurred in a portion of the bowel that is retroperitoneal. The duodenum's first segment is intraperitoneal, but the second
through fourth segments are retroperitoneal, which is the likely location of the perforated ulcer.

Incorrect Answers: B, C, D, and E.

The esophagus (Choice B) is unlikely to be perforated in this patient. Esophageal perforation is rare and can be iatrogenic secondary to recent instrumentation, such as with an endoscopy, or secondary to forceful vomiting causing spontaneous esophageal rupture.
It can present with dysphagia, sharp chest pain, hematemesis, and subcutaneous emphysema in the chest or neck.

lleal (Choice C) perforations can be related to mesenteric ischemia, inflammatory bowel disease, infections, iatrogenic injury, trauma, or bowel obstructions. It would result in pneumoperitoneum, not retroperitoneal air as the ileum is intraperitoneal.

Stomach perforation (Choice D) can be a complication of gastric peptic ulcer disease, malignancy, trauma, or as an iatrogenic complication during endoscopy. It would result in pneumoperitoneum, not retroperitoneal air given its intraperitomeal location.

Perforations in the transverse colon (Choice E) may be related to diverticulitis, neoplasm, trauma, or iatrogenic injury. Perforated diverticulitis can present with severe abdominal pain, but would produce pneumoperitoneum, not retroperitoneal air as the transverse
colon is intraperitoneal.

Educational Objective: Perforation of a duodenal ulcer is an emergent complication characterized by acute abdominal pain, peritonitis, fever, and free air on radiographs and CT. Given the retroperitoneal location of the duodenal second through fourth segments,
perforation would result in retroperitoneal air, not pneumoperitoneum as in the c�se of intraperitoneal bowel structures.

r
Previous
r
Next
tf:
Score Report
ta F·
Lab Values Calculator
,:,
Help
""
Pause .
Exam Section 4: Item 40 of 50 National Board pf Medical Exi!J'lllners®
Comprehensive 1:Jas1c Science Sert-Assessment

✓ 40. A 3-year-old gi'rl is brought to the physician because of a 1-year history of short stature. She has no history of major medical illness. She is below the 3rd percentile for height and at the 10th percentile for weight. Physical examination shows coarse facial
features and contractures of the large joints. X-rays show dysostosis multiplex. Plasma lysosomal enzyme analysis shows increased �-hexosaminidase, �-glucuronidase, �-galactosidase, and a-fucosidase activities. Which of the following mechanisms is
the most likely cause of the lysosomal enzyme findings in this patient?

A) Abnormal targeting of these enzymes to lysosomes


B) Deficiency of the other Jysosomal enzymes in the cytoplasm
G}- Degi:adatlen of IJ:iese enzymes within the G}'loplasm
D) Degradation of these enzymes within the lysosomes
E ) Storage of these enzymes within the cytoplasm
Correct Answer: A.

Abnormal targeting of these enzymes to lysosomes accounts for this patient's findings, which are consistent with mucolipidosis II (I-cell disease). Mucolipidosis 11, also known as I-cell (inclusion cell) disease, results from accumulation of oligosaccharides, lipids, and
glycosaminoglycans such as heparan sulfate and dermatan sulfate within cells throughout the body. I-cell disease is a type of lysosomal storage disease. It is similar in phenotype and pathophysiology to Hurter syndrome. I-cell disease results from a failure of Golgi­
based post-translational modification of proteolytic enzymes that would typically be targeted to the lysosome. Proteases targeted to the lysosome are tagged with phosphate at the sixth carbon on their mannose residues, forming mannose 6-phosphate. Defective
Golgi N-acety1glucosaminyl-1-phosphotransferase enzymes are unable to join phosphate onto mannose residues. This causes the synthesized enzymes to be abnormally routed into vesicles for exocytosis, instead of to the lysosome. In the absence of these
proteins within the lysosome, normal cellular debris that requires lysosomal degradation accumulates within the cells, causing inclusions seen on light microscopy. The resulting accumulation of such products leads to widespread cellular and organ dysfunction.
Signs and symptoms of I-cell disease include failure to thrive, developmental delay, coarse facial features, restricted skeletal development, hepatosplenomegaly, cardiac structural defects, corneal clouding, and dwarfism. I-cell disease demonstrates autosomal
recessive inheritance; there is no treatment for the condition other than supportive and symptomatic care.

Incorrect Answers: B, C, D, and E.

Deficiency of the other lysosomal enzymes in the cytoplasm (Choice B) is not a feature of I-cell disease, which results from the failure to phosphorylate mannose residues, thereby leading to abnormal trafficking of lysosomal hydrolases. It does not cause deficiency
of these other enzymes.

Degradation of the enzymes within the cytoplasm (Choice C) or within the lysosomes (Choice D) does not occur in I-cell disease. Lysosomal enzymes are made normally but do not undergo the necessary phosphorylation that is required for trafficking to the
lysosome. Instead, these enzymes are expelled from the cell. Abnormal degradation is not a feature of this disease.

Storage of these enzymes within the cytoplasm (Choice E) does not occur. Instead, these lysosomal enzymes are abnormally expelled from the cell entirely through exocytosis; they are not stored in the cytoplasm.

Educational Objective: I-cell disease is an autosomal recessive inherited lysosomal storage disease that results from a defect in N-acetylglucosaminyl-1-phosphotransferase enzymes. This results in the failure of mannose residue phosphorylation of lysosomal
hydrolases, which subsequently leads to their abnormal expulsion from the cell instead of to their normal site of action within the lysosome.

r r ,: ta P r ti"
Previous Next Score Report Lab Values Calculator Help Pause
Exam Section 4: Item 41 of 50 National Boa.rd of M dical Exi!J'lllners®
e
Comprehensive 1:Jas1c Science Sert-Assessment

✓ 41. An 8-year-old girt is brought to the physician by her mother 1 hour after her cat scratched her on her right arm. The mother says that the patient developed bright red streaks along her arm within minutes of being scratched. The scratches were painful, and
the patient was crying when she showed the injury to her mother. Examination of the right forearm shows the surface linear streaks with intact skin, but there is no blood. The appearance of the red streaks most likely indicated which of the following host
responses to this injury?

A ) Bacteria-generated capillary congestion


B ) Complement-activated coagulation

-- C) Histamine-stimulated vasodilation
0) lgE-mediated eosinophil degranulation
E ) Trauma-induced arteriolar hemorrhage

Correct Answer: C.

The body responds to harmful stimuli such as infectious agents, mechanical disruption, and chemical irritants with a localized inflammatory response mediated by the innate immune system. The cardinal signs of inflammation are rubor (redness), calor (heat), tumor
(swelling), dolor (pain), and functio laesa (loss of function). The first step in response to an injury of vascularized tissue is transient reflexive vasoconstriction followed by the release of histamine by mast cells and basophils, which leads to histamine-stimulated
vasodilation of the local arterioles. This leads to increased blood flow in the affected region, which generates the classic rubor of inflammation. Histamine also promotes increased vascular permeability, which allows for the leakage of plasma into the interstitial
tissue resulting in local edema (tumor of inflammation) followed by recruitment and migration of immune cells.

Incorrect Answers: A, B, D, and E.

Bacteria-generated capillary congestion (Choice A) occurs in sepsis due to the vasodilatory and inflammatory effects of bacterial cell components such as lipopolysaccharide. This patient presents immediately following an injury with the classic signs of the initial
inflammatory response, not sepsis.

Complement-activated coagulation (Choice B) results from signaling between coagulation factors and the complement system to achieve hemostasis in areas of localized tissue damage.

lgE-mediated eosinophil degranulation (Choice D) occurs in response to parasitic infections and lgE-mediated allergic reactions. It is not part of the initial inflammato!Y response to injury.

Trauma-induced arteriolar hemorrhage (Choice E) would result in an ecchymosis in the area of injury due to the mechanical disruption of the arterioles and localized bleeding.

Educational Objective: The innate immune system conducts the initial response of the body to tissue injury and infection. Histamine-stimulated vasodilation, primarily released by mast cells and basophils, leads to the development of the classic erythema (rubor)
and edema (tumor) of inflammation.

r r ,: ta F' r ti"
Previous Next Score Report Lab Values Calculator Help Pause
Exam Secllon 4: Item 42 of 50 Nallonal Board pf Medical Examiners®
Comprehensive Basic Science Serr-Assessment

)( 42. A 60-year-old woman dies of a myocardial infarction. Incidental findings on examination at autopsy include multiple 0.5- to 1-cm stool-filled outpouchings of the sigmoid colon alongside the taeniae coli. The outpouchings protrude through the muscularis
propria. Microscopic examination of a lesion shows a thin wall consisting of attenuated colonic mucosa and submucosa. Which of the following mechanisms best explains the formation of these lesions in the colon?

A ) Abnormal development of the muscularis propria


B ) Atrophy of the muscularis propria
G} Hyperplasia of tt:te columnar epithelium
----�
D) Increased intraluminal pressure
�➔ l>leoplaslis proliferation of tt:te solumnar epithelium
Correct Answer: D.

The layers of the gastrointestinal wall, from the luminal surface to the exterior of the bowel, include the mucosa, submucosa, muscularis propria, and serosa. Diverticulosis is characterized by sac-like protrusions of the colonic wall which are hypothesized to develop
secondary to increased intraluminal pressure that predisposes the herniation of mucosa and submucosa through the muscularis propria of the colonic wall. Risk factors for the development of diverticulosis include abnormal colonic motility, high dietary intake of red
meat and low dietary fiber, physical inactivity, and obesity. Complications of diverticular disease include bleeding and diverticulitis.

Incorrect Answers: A, B, C, and E.

Abnormal development (Choice A) or atrophy (Choice 8) of the muscularis propria is rare but may result in diminished or abnormal peristalsis of the gastrointestinal system. Diverticulosis is a common phenomenon and is not related to intrinsic abnormalities of the
muscularis propria.

Hyperplasia of the columnar epithelium (Choice C) results in hyperplastic polyps of the colon, which are most commonly located in the rectosigmoid colon. They are not associated with diverticulosis. They are benign and do not confer an increased risk of
malignancy. Contrarily, neoplastic proliferation of the columnar epithelium (Choice E) results in the formation of adenomatous polyps, which may progress to malignancy. Diverticulosis is not associated with colonic polyps or malignancy.

Educational Objective: The layers of the gastrointestinal wall, include the mucosa, submucosa, muscularis propria, and serosa. Diverticulosis is characterized by sac-like protrusions of the colonic wall, which are hypothesized to develop secondary to increased
intraluminal pressure that predisposes the herniation of mucosa and submucosa through the muscularis propria.

r
Prev1ous
r
Next
,
Score Report
� P
Lab Values Calculator
r
Help
r-
Pause
_
Exam Sectlon 4: Item 43 of 50 National Board pf M11dlcal Examiners®
Comprehensive i,aa1c: sc:ien" SelT•A&&eaament

✓ 43. A 15-year-old girl with acute myelogenous leukemia undergoes a bone marrow transplant procedure. Her 19-year-old brother is the donor. Blood group and human leukocyte antigen (HLA) typing of patient and donor leukocytes are:
Patient Donor
Blood Group AB, Rh-positive A, Rh-positive
HLA-A 2,3 2,3
HLA-B 5, 7 5, 19
HLA-DR 2, 5 2,5

° °
Three weeks later, she develops a temperature of 37.4 C (99.3 F); a diffuse, erythematous rash over most of the body, including the palmar and plantar surfaces; and watery diarrhea. Serum studies show an AST activity of 120 U/L (N=5-55) and ALT
activity of 180 U/L (N=5-45). Which of the following is the most likely diagnosis?

A ) Acute rejection
B ) Cytomegalovirus infection
C) Graft-versus-host disease
D) Hepatitis C
�� Hyperacule Fejection
F ) Leukemia recurrence
G) Serum sickness
Correct Answer: C.

Graft-versus-host disease (GVHD) is the most likely cause of this patient's fever,diarrhea, rash, and increased liver enzymes following a bone marrow transplant. Prior to the grafting of allogeneic stem cells in a bone marrow transplant, recipient native marrow must
be ablated. It is replaced with donor marrow, which recolonizes the marrow cavity. Successful transplantation results in reconstitu1ion of the immune system with the donor's immune cells. GVHD occurs when the donor's immune system recognizes the host's
tissues as foreign and mounts an immune response against them. Manifestations of acute GVHD include rash, ranging from a maculopapular to blistering rash,diarrhea and abdominal pain, and hepatitis with hyperbilirubinemia. Diagnosis should be suspected in
patients with any of these symptoms in the setting of a recent bone marrow transplant. Ideally, donors and recipients should match at all HLA loci. Those receiving transplants from siblings where there is a single mismatch, as in this patient,should ideally receive
high resolution typing of HLA-DRB 1,and should match at seven out of the eight loci. Mismatches of any kind predispose to GVHD,although HLA-A,B,and DR appear to be the most important. Medications used in the treatment of GVHD act either by directly
inhibiting production of T lymphocytes or by suppressing the normal function of these cells. Examples include mycophenolate mofetil, which inhibits the synthesis of nucleotides needed for lymphocyte proliferation,etanercept,a TNF-alpha receptor inhibitor,
pentostatin, a purine analog that inhibits T lymphocyte proliferation, sirolimus, which inhibits mTOR to suppress T lymphocyte proliferation, and anti-thymocyte globulin,which directly targets T lymphocytes.

Incorrect Answers: A,B,D,E, F, and G.

Acute rejection (Choice A) describes failure of the donor bone marrow to graft appropriately as a result of the continued presence of recipient T cells circulating in the blood.

Cytomegalovirus (CMV) infection (Choice B) infection can cause primary infection in immunocompetent hosts with symptoms that are indistinguishable from those of Epstein-Barr virus infection (mononucleosis). Patients who receive bone marrow transplants are
placed on prophylactic medications to prevent CMV infection if the recipient or donor is CMV-positive.

Hepatitis C infection (Choice D) can present in acute and chronic forms. The acute variant typically presents with symptoms of fatigue,nausea,vomiting, jaundice,and right upper quadrant discomfort. If chronic infection ensues it can lead to cirrhosis and end-stage
liver disease.

Hyperacute rejection (Choice E) occurs within minutes following allograft transplantation of solid organs such as a kidney. Following renal transplant,hyperacute rejection appears as immediate mottling and cyanosis of the graft. It occurs secondary to preformed
recipient antibodies reacting to donor antigens (type II hypersensitivity),which activate complement and cause thrombosis of the allograft vasculature.

Leukemia recurrence (Choice F) would manifest with symptoms such as malaise and weight loss. Laboratory evaluation would demonstrate circulating blast cells.

Serum sickness (Choice G) is a type Ill hypersensitivity reaction, defined by the presence of circulating antigen-antibody complexes that deposit in tissues. Medications related to transplant medicine that are known to result in serum sickness include anti-thymocyte
globulin,which this patient did not receive. Additionally, it usually occurs 1 to 2 weeks after initial exposure to the drug. Symptoms include fever, rash, and arthralgias.

Educational Objective: GVHD typically occurs following bone marrow or liver transplantation and manifests with fevers, rash, and hepatitis. The risk for GVHD is higher in patients who receive transplants with a greater degree of HLA mismatch.

r
Previous
r
Next
,
Score Report

Lab Values
P
Calculator
r
Help
trt
Pause
Exam Secllon 4: Item 44 of 50 Nallonal Board pf Medical Examiners®
Comprehensive Basic Science Serr-Assessment

✓ 44. A healthy 21-year-old woman is concerned that her son will develop a familial neurologic disorder. Her older brother died at the age of 45 years after developing restlessness, forgetfulness, incoordination, altered handwriting and speech, and subsequently,
chorea. Autopsy showed atrophy of the basal ganglia. Their mother died at the age of 55 years of a similar disorder. Which of the following best approximates the risk of this woman's son for developing the disorder?

A ) Essentially zero
B) 25%
G}-33-%
D) 50%
iii.}-400¼
Correct Answer: B.

The chance of this woman's son developing Huntington disease (HD) is 25%. HD demonstrates an autosomal dominant pattern of heritability. The woman's mother possessed HD symptoms and died early and therefore presumably possesses one copy of the
mutated gene. This woman has a 50% chance of inheriting the gene from her mother and a 50% chance of passing on this mutated gene to her son (50% >< 50% = 25%). HD is a heritable progressive neurodegenerative disorder typically arising in the third to sixth
decades of life associated with atrophy of the caudate nuclei in the basal ganglia. The disease 1Usually presents with an insidious onset of psychiatric (irritability, depression, psychosis) and cognitive symptoms. Motor symptoms (chorea, hypotonia, dystonia, loss of
voluntary control of movements) typically appear later in the disease course. People with HD typically die 10 to 40 years after disease onset. Chorea, a key HD symptom, can be initially experienced as restlessness. Treatment is symptomatic and can include
tetrabenazine or deutetrabenazine, which target chorea by depleting dopamine stores. HD also demonstrates anticipation, a genetic inheritance phenomenon in which future generations are affected at a younger age or with more severe symptoms.

Incorrect Answers: A, C, D, and E.

The son's risk would be essentially zero (Choice A) if the woman had demonstrated a negative result on genetic testing for HD. Alternatively, if the woman were older than 80 years old without any HD symptoms, the risk that she would develop HD may be low
enough to assume she does not possess the mutated HD gene and that her son's risk is therefore negligible.

Risks of 50% (Choice D) or 100% (Choice E) would signify that either the woman or her son demonstrated the mutated HD gene on genetic testing, respectively. The son's risk would be 50% if the woman had a genetic test indicating confirmed HD. The son's risk
would be 100% if his genetic test were positive.

A risk of 33% (Choice C) would be inconsistent with an autosomal dominant pattern of heritability. When a phenotypic ratio that involves thirds is observed, one of the alleles is likely lethal in utero.

Educational Objective: Huntington disease is a heritable progressive neurodegenerative disorder that presents with psychiatric, cognitive, and motor symptoms including chorea. Huntington disease demonstrates an autosomal dominant pattern of heritability.

r
Prev1ous
r
Next
,
Score Report
� P
Lab Values Calculator
r
Help
r-
Pause
_
Exam Secllon 4: Item 45 of 50 Nallonal Board pf Medical Examiners®
Comprehensive Basic Science Serr-Assessment

✓ 45. A 24-year-old man w1ho is serving overseas in the US Armed Forces is brought to a medical unit 30 minutes after sustaining multiple injuries caused by an explosion that forcibly ejected him from the vehicle he was driving. On arrival, he is conscious and
hemodynamically stable. His respirations are 30/min. Physical examination shows visible shrapnel protruding from the lower sternum. Which of the following structures within the pericardia! sac is most likely damaged in this patient?

A) Ascending aorta
B) Left atrium
C) Left ventricle
D) Right atrium
E ) Right ventricle

Correct Answer: E.

In the typical anatomical orientation, the right ventricle is the most anterior part of the heart. It also forms the inferior (or diaphragmatic) surface of the heart along with the left ventricle. It lies just posterior to the sternum and anterior mediastinum. A penetrating
object entering the inferior sternum would affect the most superficial, anterior structure of the heart in this patient, which is the right ventricle.

Incorrect Answers: A, B, C, and D.

The ascending aorta (Choice A) originates at the left ventricular outflow tract and courses superiorly to the level of the angle of louis before turning posterior and lateral to form the aortic arch. It is located distant from the inferior sternum.

The left atrium (Choice B) is the most posterior part of the heart and lies anterior to the esophagus in the mediastinum. It is less likely to be damaged by a penetrating object because it is deeper in the thoracic cavity, and a penetrating object would have to pass
through multiple anterior structures to reach it.

The left ventricle (Choice C) is located posterior to the right ventricle, and inferolateral to the left atrium. It abuts the left lung and makes up the majority of the left border of the heart.

The right atrium (Choice D) abuts the right lung and makes up the right heart border. It is located posterior to and to the right of the sternum, whereas the right ventricle holds an anterior and inferior position, directly behind the sternum and is thus more susceptible
to penetrating trauma.

Educational Objective: The right ventricle is the most anterior and inferior part of the heart and lies just posterior to the sternum. A penetrating object through the inferior sternum would affect the most superficial, anterior structure of the heart, which is the right
ventricle.

r
Prev1ous
r
Next
,
Score Report
� P
Lab Values Calculator
r
Help
r-
Pause
_
Exam Section 4: Item 46 of 50 National Boa.rd of M dical Exi!J'lllners®
e
Comprehensive 1:Jas1c Science Sert-Assessment

✓ 46. A healthy 28-year-old woman participates in an exercise study. Several physiologic variables are measured as she runs on a treadmill. The ambient room temperature is 75° F. She reaches a steady state that increases her oxygen consumption threefold.
She continues to exercise at that level for 20 minutes. During the first 5 minutes of exercise, the vascular resistance of which of the following is likely to show the greatest increase from restingr values in this volunteer?

A ) Coronary
B) Cutaneous
C) Exercising muscle
D) Pulmonary
E ) Splanchnic
F ) Systemic
Correct Answer:. E.

Splanchnic vascular resistance is likely to show the greatest increase when compared to its resting state. The splanchnic circulation includes the celiac trunk and the superior and inferior mesenteric arteries, which supply blood to the gastrointestinal system.
Adequate splanchnic blood supply is critical to digestion and maintenance of the gastrointestinal mucosal barrier. In the resting state, splanchnic circulation utilizes 25% to 30% of all cardiac output, but this can change drastically after meals (in.creased blood flow) or
during periods of physiologic stress such as exercise or critical illness (decreased blood flow). Changes in splanchnic blood flow are regulated by multiple mechanisms that are grouped into intrinsic, extrinsic, and humeral control. Intrinsic regulatory mechanisms
include vasodilation (increased blood flow) in response to tissue hypoxia or acidosis. Extrinsic control is exerted by the sympathetic and parasympathetic nervous systems, with sympathetic innervation leading to vasoconstriction and parasympathetic innervation
leading to vasodilation. Humeral mechanisms describe a response to the presence of hyperosmolar contents within the lumen of the gastrointestinal tract and stimulate release of nitric oxide to cause vasodilation and aid in digestion. During exercise ,. a greater
degree of cardiac output is required to supply oxygenated blood to the coronary arteries and the skeletal muscles. Action of the sympathetic nervous system leads to increased cardiac output via augmentation of the heart rate, increased splanchnic vascular
resistance, and increased pulmonary, skeletal muscle, coronary, and cutaneous blood flow.

Incorrect Answers: A, B, C, D, and F.

Coronary (Choice A) vascular resistance decreases during intensive exercise leading to increased perfusion of the heart, enabling maintenance of increased cardiac output. Similarly, pulmonary vascular (Choice D) resistance decreases, thereby decreasing
physiologic dead space and improving ventilation-perfusion matching.

Cutaneous (Choice 8) vascular resistance has a variable response to exercise and depends largely on ambient and core temperatures. In general, cutaneous vascular resistance falls to permit vasodilation, allowing for cooling through the skin.

The vascular resistance of exercising muscle (Choice C), which requires the greatest amount of blood flow during periods of intense exercise, decreases to allow for increased perfusion. At first, these changes are mediated by the sympathetic nervous system but
are maintained subsequently by intrinsic regulatory mechanisms (eg, lactate, adenosine).

Systemic (Choice F) vascular resistance generally decreases during exercise. While mean arterial pressure often rises in exercise, this is due to a disproportionately greater increase in cardiac output to compensate for the fall in systemic vascular resistance.

Educational Objective: The splanchnic circulation at rest utilizes a significant percentage of cardiac output. Intensive exercise requires increased cardiac output to meet the metabolic demands of contracting skeletal muscle. Action of the sympathetic nervous
system during exercise leads to increased cardiac output via augmentation of the heart rate, increased splanchnic vascular resistance, and increased pulmonary, skeletal muscle, coronary, and cutaneous blood flow.

r r ,: ta F' r ti"
Previous Next Score Report Lab Values Calculator Help Pause
Exam Section 4: Item 47 of 50 Natlonal Board pf Medical Examiners®
Comprehensive 1:1as1c Science Sen-Assessment

✓ 47. An investigator is conducting a study of the efficacy of certain vitamins in helping to prevent heart disease in women. Eight thousand women, 45 years of age or older, are enrolled in the study. Two thousand women receive vitamin C only, 2000 receive
vitamin E only, 2000 receive both vitamins C and E, and 2000 receive a placebo. The investigator finds that the vitamins are ineffective in helping to prevent heart disease. Which of the following best describes the study design used in this clinical trial?

A) Crossover
B) Cross-sectional
C) Factorial
D) Nested
E) Simple
Correct Answer:. C.

A factorial study design can be used to investigate the interactions between two or more treatments. In this study design, combinations of the treatments are given to different patient groups. It is a fonn of true experimental design. Its advantages include that
researchers may evaluate multiple independent variables simultaneously. Researchers can also evaluate interactions between variables. In this way, factorial design may be more comprehensive in understanding a phenomenon than a study that focuses on a
single factor. In this study design, independent variables are called factors while different values or doses of a factor are referred to as levels.

Incorrect Answers: A, B, D , and E.

In a crossover study design (Choice A) subjects receive multiple treatments in sequence. Crossover designs can be randomized. Crossover studies diminish confounding variables because each patient serves as their own control. Because of this, imbalances in
the randomization are theoretically impossible. Crossover designs also increase statistical power as the number of control patients and the number of treatment patients is increased simultaneously.

A cross-sectional study (Choice B) seeks to identify the prevalence of the condition at a particular point in time. An example of a cross-sectional study would be a single survey of a population inquiring whether patients have coronary artery disease and concurrently
inquiring about activity levels and diet. Thus, the risk factor and the outcomes are measured simultaneously. The study does not follow patients over time. All infonnation is collected at a single time point.

Nested (Choice D) case-control studies are a variation of a standard case-control study in which the cases and controls are drawn from a larger cohort study. In many cases, the nested case-control offers increased statistical efficiency, as only a subset of patients
within the entire cohort is required for data collection and analysis.

Simple trials (Choice E) focus on one or two specific clinical questions and have limited data collection. They are focused on clearly defined outcomes as well as shorter-term clinical effects. The ability to focus on a smaller number of outcomes allows them to be
more cost-effective than traditional randomized controlled triaHs.

Educational Objective: Factorial study design is an interventional study design in which combinations of a number of interventions are trialed concurrenlily, allowing for the investigation of interactions between these interventions as well as for a more complete
understanding of the treatment strategies.

r
Previous
r
Next
,:
Score Report

Lab Values
F
Calculator
,:,
Help
If'
Pause
Exam Section 4: Item 48 of 50 National Boll.rd pf M-idlcal Examiners®
Comprehensive tsas1c science Sert-Assessment

✓ 48. A 26-year-old man comes to the physician because of a 1-week history of temperatures to 38.3°C (101 ° F), chills, and chest and back pain. Two weeks ago, he returned from a trip to Costa Rica, where he had been performing ecologic studies of bats in
their cave habitats. He reports exposure to bat urine and feces during his studies. Today, his temperature is 38.1•c (100.6° F). Physical examination shows no other abnormalities. A complete blood count and serum chemistry studies are within the
reference ranges. A chest x-ray shows bilateral reticulonodular infiltrates. The virulence of the most likely infectious agent in this patient depends on which of the following?

A ) Elaboration of polyribitol phosphate capsule


B ) Growth inside nonactivated macrophages
C) Invasion of host tissues and vasculature
D) Phenotypic switching from yeast to hyphae in the host
E ) Production of a lipopolysaccharide capsule
Correct Answer: B.

Histoplasmosis is an endemic mycosis secondary to Histop/asma capsulatum. Pulmonary histoplasmosis can present in acute, subacute, or chronic phases, generally characterized by fever, malaise, myalgia, cough, and hemoptysis with the associated
development of calcified nodules and mediastinal or hilar lymphadenopathy. It is endemic to the Mississippi and Ohio River valleys in the United States, as well as parts of Central and South America, sub-Saharan Africa, and Southeast Asia. The virulence of
Histop/asma capsulatum is dependent on its growth inside nonactivated macrophages, which protect the fungal cells from detection and elimination. Risk factors for histoplasmosis include exposure to bird or bat droppings. Diagnosis is typically made via the
detection of histoplasma antigen in the serum and/or urine.

Incorrect Answers: A, C, D, and E.

Elaboration of polyribitol phosphate capsule (Choice A) is the major virulence factor of Haemophilus inf/uenzae, which confers resistance to phagocytosis. Haemophilus inffuenzae is a Gram-negative bacterium that can result in mucosa! infections such as
conjunctivitis, otitis media, bronchitis, and pneumonia. It was the leading cause of epiglottitis in children prior to widespread immunization.

Invasion of host tissues and vasculature (Choice C) is the common mechanism by which virulent microbes cause disease, resulting in localized and disseminated infection. Histoplasma capsu/atum virulence, rather, is dependent on its replication within nonactivated
macrophages, as this serves to protect the organism from the immune system.

Phenotypic switching from yeast to hyphae in the host (Choice D) is a characteristic of Candida albicans. It is a dimorphic fungus that switches forms depending on temperature. It can cause systemic infections in immunocompromised patients such as esophagitis
and disseminated candidiasis. It can also cause vulvovaginitis and diaper rash.

Production of a lipopolysaccharide capsule (Choice E) is characteristic of Gram-negative bacteria. Lipopolysaccharide, an endotoxin, is a type of pathogen-associated molecular pattern (PAMP). It is a major virulence factor that is typically detected by CD14 on
macrophages, resulting in the provocation of an exaggerated immune response that results in systemic inflammation and sepsis.

Educational Objective: Histoplasmosis is an endemic fungal infection caused by Histop/asma capsu/atum. The organism is able to evade detection by the immune system through residing and replicating within macrophages, contributing to its virulence.

r r ff: (fj F·
l,ab V:aJ!Jes _C_aJi;ula_tor

Help_ �
1ft
f'.ause
Pre��us_________!!11xt
_ �cpJ11_f{epc,rt
_
Exam Secllon 4: Item 49 of 50 Nallonal Board pf Medical Examiners®
Comprehensive Basic Science Serr-Assessment

✓ 49. A 62-year-old man wiith hypertension comes to the physician because of a 10-day history offatigue and shortness of breath. His temperature is 37.4 •c (99.4°'F), pulse is 102/min, respirations are 18/min, and blood pressure is 110/70 mm Hg. Pulmonary
examination shows diffusely decreased breath sounds and a prolonged expiratory phase. Cardiac examination shows distant heart sounds. There is 1 + pretibial edema ofthe lower extremities. Pulmonary function testing shows:
FEV 1 65% of predicted
FVC 90% of predicted
FEV 1/FVC 60% of predicted

Which of the following is the most likely diagnosis?

A ) Cardiac tamponade
B ) Chronic obstructive pulmonary disease
C ) Congestive heart failure
D) Mitra! stenosis
E ) Pleural effusion
F ) Sarcoidosis
Correct Answer: B.

Chronic obstructive pulmonary disease (COPD) is an obstructive lung disease characterized by decreased lung function from a combination of chronic bronchitis and/or emphysema, resulting in airflow obstruction on expiration. Physical examination reveals
decreased breath sounds due to reduced airflow and a prolonged expiratory phase as the pathologically narrowed airways are further compressed by positive intrathoracic pressure during expiration. Heart sounds may be distant secondary to emphysematous lung
changes (sound is transmitted poorly through air relative to normal lung tissue). Pulmonary abnormalities can be evaluated by pulmonary function testing, which includes spirometry, full-body plethysmography, single-breath diffusing capacity, arterial blood gas
analysis, and pulse oximetry. Spirometry is the most useful in diagnosing obstructive lung disease. The patient breathes into an external device (spirometer) which records the volume and rate of airflow during inspiration and expiration to generate a flow-volume
loop. Obstruction is defined by a decrease in the ratio of the volume of air forcefully expelled in the first second of expiration to the total volume of air expelled during the expiratory phase (FEV1/FVC). The severity of obstruction is graded by the amount of FEV 1
reduction compared to predicted values. In this patient, the decreased FEV1 /FVC (less than 80% of predicted defines obstruction) and the decreased FEV1 suggest moderate obstructive lung disease.

Incorrect Answers: A, C, D, E, and F.

Cardiac tamponade (Choice A) presents with tachycardia, hypotension, jugular venous distention, pulsus paradoxus, and muffled heart sounds, and results from rapid and/or excessive accumulation of fluid within the pericardia! space impairing right ventricular
diastolic filling. Congestive heart failure (Choice C) presents with shortness of breath, orthopnea, paroxysmal nocturnal dyspnea, peripheral edema, jugular venous distention, rales on examination, and an S3 or S4 gallop. Mitra! stenosis (Choice D) is classically
heard as an opening snap followed by a diastolic rumble, loudest over the cardiac apex and radiates to the axilla. If severe enough, it can result in left atrial enlargement, cardiogenic pulmonary edema, and arrhythmias such as atrial fibrillation and flutter. These
conditions are better evaluated by echocardiography than spirometry.

Pleural effusion (Choice E) is an abnormal collection of fluid in the pleural space and may result from multiple underlying conditions including infection such as pneumonia, malignancy such as lung cancer or mesothelioma, and elevated hydrostatic pressure such as
left-sided heart failure. If lung expansion is impaired, a restrictive pattern may be seen on pulmonary function testing (defined by reduced total lung capacity on full-body plethysmography). Pleural effusions are better evaluated with chest imaging and thoracentesis.

Sarcoidosis (Choice F) is a chronic noncaseating granulomatous disease, most commonly presenting with bilateral hilar lymphadenopathy and possible coarse, upper lobe-predominant reticular pulmonary opacities. It most frequently results in a restrictive pattern
on spirometry and plethysmography (defined by reduced total lung capacity) due to loss of lung compliance from destruction of the parenchymal architecture.

Educational Objective: Obstructive lung disease is assessed with spirometry, with the presence of obstruction defined as a reduced FEV1/FVC ratio. Obstructive disease is characterized by decreased breath sounds and a prolonged expiratory phase on
examination. Wheezing may be absent in the setting of reduced air flow.

r r , � P r r-
Prev1ous Next Score Report L�b Values Calculator Help Pause
_ .
Exam Section 4: Item 50 of 50 National Board pf Medical Examiners®
Comprehensive Basic Science Self-Assessment

50. A 55-year-old man comes to the office because of a 7-year history of red, dry, itchy skin over his back and buttocks. Use of emollients and topical corticosteroids has not improved his symptoms. He has no other history of major medical illness. His vital
signs are within normal limits. Physical examination shows the findings in the photograph. Laboratory studies show eosinophilia and increased serum concentrations of lgA and lgE. Malignant transformation of which of the following cell types is the most
likely cause of the findings in this patient?

A ) Blastocytes
B ) Eosinophils
C) Macrophages
D) Mastoid cells
E ) Monocytes
F ) T lymphocytes

Correct Answer: F.
The early stage of mycosis fungoides, the most common form of cutaneous T-cell lymphoma, is characterized by erythematous, wrinkled, scaly patches. If the disease remains untreated, the patches will thicken into plaques and then tumors. The patches classically
involve areas protected from the sun such as the, axillae and buttocks. As skin biopsy can be inconclusive early in the disease, it often takes several years for a diagnosis of mycosis fungoides to be made. It is an indolent lymphoma and has a good prognosis. It is
caused by the infiltration of malignant T lymphocytes Into the epidermis. Histopathologic evaluation with immunohistochemical staining for T lymphocyte specific markers is critical to the diagnosis.
Incorrect Answers: A, B, C, D, and E.
Blastocytes (Choice A) are undifferentiated cells present early In the first stages of embryonic development. They do not play a role in the pathogenesis of mycosis fungoides.
Eosinophils (Choice B) may be increased in primary hypereosinophilia, cutaneous eosinophilic vasculitis (eg, eosinophilic granulomatosis with polyangiitis), and multiple causes of eosinophilic dermatosis (eg, urticaria, drug eruptions, eczema, atopic dermatitis).
They are not the causative cell type in mycosis fungoides.

Macrophages (Choice C) and monocytes (Choice E) are cells of the innate immune system that serve to phagocytose foreign proteins or organisms and also serve as antigen-presenting cells to T-lymphocytes. !Macrophages that reside In the skin are called
Langerhans cells; malignant transformation of Langerhans cells causes Langerhans cell hisliocytosis, not mycosis fungoides.
Mastoid cells (Choice D) describe the air-lined spaces contained within the mastoid region of the temporal bone and are not involved in skin rashes. A similarly named cell, the mast cell, is increased in mastocytosis. Mastocytosis describes a group of disorders
characterized by collections of mast cells in the epidermis and dermis. Mastocytosis often presents as a solitary mastocytoma, a yellow-brown leathery plaque that urticates when scratched. In this case, the examination findings are more consistent with mycosis
fungoides rather than mastocytosis.
Educational Objective: Mycosis fungoides is an indolent form of cutaneous T-cell lymphoma. It is characterized by infiltration of malignant T lymphocytes into the epidermis.

r r , fr!i F � t"
Previous Next Score Report Lab Values Calculator Help Pause

You might also like